Вы находитесь на странице: 1из 339

CLEC'TIC

EDUCATIONAL

CRIES. HEEJJiS.
i

.^'"Yl"^^^^^

'J

^"/^

cC

NEW

PRACTICAL

ARITHMETIC

Revised

Edition

of

the

Practical

Arithmetic

JO^^'E'PH" KA'Y/K/R-,:
Ijjie Professor in
Woodward

College.

YAN
137WALNUT

ANTWERP,
Street,

BRAGG
28

"
Bond

CO.
Street YORK.

CINCINNATI.

NEW

RAY'S

MATHEMATICAL
^_

SERIES,

^f^^f^
^e/^V,

Ray'^s Ray^s Rafs Ray^s Rafs


Ray's Ray^s

Neiv

Primary
Intellectual

Arithmelic. Arithinetico Aritlimetic. Arithmetic.

New
New New

Practical

Elementary
Arithmetic,

Higher
Test Neiv New Plane

Examples. Elemeittary Algeh'a.

Rays Ray^s Ray^s Rafs Ray^s Ray's Ray's

Higher
and

Algebra.
Geometry.

Solid

Geojnetry and Analytic


Elements

Trigonouietry.

Geometry".

of Astronomy.
and

Surveying

Navigatian.

Calculus. i0^d\l\ii^gral\ D"j^'er2ntial

Copyright 1877
BY

Van

Antwerp,

Bragg

"

Co.

EDUCATION

OEfY*

ECLECTIC

PRESS,
BRAGG "

VAN

ANTWERP, CINCINNATI.

CO.

yti tM{

PREFACE.

Changes modes of

in the

methods

of instruction have made

in

our

schools

and
to

in the revise

transactingbusiness
Arithmetic.
on

it necessary

Ray's
No

Practical other work

Arithmetic

ever

had

so

extensive

use

or

spread wide-

popularity. Teachers
and breadth of the land,
are

every-where, throughout
familiar with its pages, and from of

the

length
of

millions the

pupilshave
its forth from In view

gained principles.More

their arithmetical than


ten

knowledge

study of
gone

thousand

editions

it have

the press. of these facts, it has been the constant aim in

making

this

revision to preserve carefully those distinctive features of the former constituted the peculiarphilosophical method of its editions,which learned 1st.

author, viz.:

tion Every principleis clearly explained by an analysisor soluof simple examples, from which Rule is derived. This is a followed by graduated exercises designedto render the pupil familiar with its application. 2d. The is strictly no philosophical; arrangement principleis the pupil is never anticipated; requiredto perform any operation until the principle which it is founded has first been explained. on The changes made fall naturally under two heads: (1) those which of instruction;(2) methods adapt the book better to the advanced those which exhibit present methods of computation in business. In the first place, specialattention is invited to the beauty and of the volume, matter elegance of the typogranhy^^^ The^^ffiyant 961b;"y (iii)

IV

PKEFACE.

the

definition, the
a

solution,

or

the

rule,

is

at

once

clearly

indicated numbered recitation

by

difference

of enhances

type.
the

running
convenience

series of

of the

articles, with
text-book for

paragraphs,
and The for

reference.

analytic
All

solutions obsolete and Cloth

and

written of

operations Weights
and all and

have

been

carefully
such
as

separated.
Beer such Measure
as

Tables

Measures,

Measure,
etc.,
is
are

obsolete The

denominations,

drams,
and extended

roods,
Measures usage,

discarded.
in

Metric with

System
its
now

of

Weights widely
after
A

presented
is

accordance proper

and

assigned

its

place immediately

Decimals. few have

subjects,
been and been its

such

as

Factoring
rewritten,
been

and and

the
in

principles
many

of

tions, Fracthe

entirely
rules have

instances
of

definitions has

simplified.
and
an

The

subject
has novel

age Percentmade
to

much
numerous

expanded,

endeavor many

been and

systematize
features,
with have The teachers for both the

applications;
and

interesting
here be
met

of

subject-matter
time. careful desire The attention
to

classification, will
of Interest

first that

subjects
which

and

Discount demands. the many in Smith the and

received

their their

importance
thanks
are

publishers
whose edition. P.

express and

to

suggestions Especial
for the been
a

corrections is due

embodied M. of W.

present
Mr. In A.

mention valuable wish

Prof,

Morgan

many

features
to

this

revision. the

conclusion,
has
to

publishers
to

reiterate

that

object scientific
with all is

throughout
accuracy;

combine

practical
the

utility with
best has

present

work How

embracing
far this in

methods,
been and

real

improvements.
submitted of education.
to

object
the

secured

again
work

those

engaged

laborious

responsible

Cincinnati,

August,

1877.

TABLE

OF

CONTENTS.

PAGE

The

Arabic

System

of

Notation

The

Koman

System

oe

Notation

20

Addition

22

Subtraction

31

Multiplication Contractions in

39

Multiplication

47

Division Short Division


. .
, .

50

54 59 in

Long

Division Division
.

Contractions General

64

Principles
Numbers

of

Division

67

Compound United

71

States

Money

72 83 Numbers
84 84

Merchants' Reduction

Bills of

Compound

Dry
Rules

Measure for Reduction


. .

87 88

Liquid

Measure

Avoirdupois Long
Square
Solid Time
or

Weight

89 90 90

Measure Measure Cubic Measure

94 96

Measure

(V)

Vi

CONTENTS.

PAGM

Miscellaneous Addition of

Tables

97 Numbers
. . ,

Compound
of

102 106

Subtraction

Compound

Numbers

....

of Compound Numbers Multiplication Division of Compound Numbers

Ill
....

113 115 118

Longitudeand
Factoring To To Find Find

Time

the Greatest the Least

Common

Divisor

....

123
125

Common

Multiple

....

Cancellation

127

Fractions

131 135 137 144 147


. . . . . .

Principles
Reduction Addition of Fractions of Fractions of Fractions

Subtraction

of Fractions Multiplication Compound Fractions Division of Fractions Fractions

149

152 154 157

Complex
Fractional Practice Decimal

Compound

Numbers

159

165

Fractions
Reduction Addition of Decimals of Decimals of Decimals Decimals
.
.

168 175 178 179


180
. . . . .

Subtraction

of Multiplication Division Decimal The

of Decimals

183
186

Compound
System

Numbers
. . . . . .

Metric Measures
Land
or

189 190
192

Measures Measures Table

Length Square Measure of Capacity of Weight


........

of

192
193

of Values

194

CONTENTS.

vii

PAGK

Percentage Formulas for the four


cases

197

of

Percentage.

203 205

of Percentage Applications Mercantile Transactions

206
206

Commission
Trade Discount Loss

208 210
213 214

Profit and Stock

Transactions

Brokerage
Assessments

and

Dividends

.215
216

Stock Stock

Values
Investments

217

Interest

219

Simple
The Formulas

Interest Per Cent Method


cases

221 229
. . . .

Twelve

for the five


Interest

of Interest

237
237

Compound
Annual Partial

Interest

239 241

Payments

Discount Bank True Discount Discount


.

247
247 .256

Exchange

260 261

Exchange Foreign Exchange EnglishMoney French Money German Money Canadian Money
Insurance
Fire and Marine Insurance

Domestic

262
262
'
. .

263 263 263

265 265 267


. .

Life Insurance

viii

CONTENTS.

PAGK

Taxes State United Internal Duties


or

269

and

Local

Taxes
Revenue
.

269

States

273
274
.

Revenue Customs
.... ...

274

Ratio

.276

Principles
Proportion

280

282 285
289
.

Simple Proportion Compound Partnership Bankruptcy


General

Proportion

291

293
"

Average
with Time of

293
. . .

Partnership Equation Average


Involution
,

294

Payments

295
297

298

Evolution
. .
.

300

Square
Cube

Root Root

302 309

Mensuration Measurement Measurement of Surfaces

316 316 323 328

of Solids of Mensuration

Applications
Progressions Arithmetical

331

Progression Progression

331 334

Geometrical

Article
as,
one,
one

1.'

1.

Unit
one

apple,

single thing dollar, one pound.


a

is

of

any

kind;

2. A

Number

consists
men.

of

one

or

more

units

; as,

one,

five,seven computing
4.

cents, nine
treats

3. Arithmetic

of

nimibers,
in two

and

is

the

art

of

by

them.
are

Numbers

expressed
of Notation

ways

first, by words;
of

second, by
5. A numbers 6. Two

characters. is
a

System

method

expressing
the
in

by

characters. of The

systems
Roman.

Notation
Arabic

are

in

use,

Arabic all
our

and. the

system

is used

arithmetical

calculations.

THE

ARABIC

SYSTEM

OF

NOTATION.

2,

1. ten

To

express

numbers,

the

Arabic

Notation

ploys em-

characters, called
0.

figures; namely, 1, 2, 3, 4, 5,

6, 7, 8, 9,
Eemark its characters

1.

"

The appear

Arabic
to

System
been

of

N'otation

is into

so

called

because

have

introduced

Europe (9)

bj' the

10

RAY'S

NEW

PRACTICAL

ARITHMETIC.

Arabians; tut i'cis now


in

acknowledged o-cnv^r.illy
is also

that

they originated
Decimal

India.
Rem.

Not'ition 2^; ^T'ae 'A^f'J^hia


"

called

the

System

and

the

^ClnrvrTio'ii '%,s^m.
Order

'

'

2. The number.

of

figureis

the

place it occupiesin

UNITS

OF

THE

FIRST

ORDER,

OR

UNITS.

3.

1. A One Two Three Four Pive Six

unit unit units units units units units units units

or

singlething
one one one one one one one one more more more more more more more more are

is owe,

written

1. 2. 3.
4.

and and and and and and and and

are

two^

three^ four^ five. six^


seven.

are are are are are are

5. 6. 7. 8. 9.

Seven

eight,
nine,

Eight
2. These because 3. The its
use

nine

characters

are

called

significant figures,
for

they denote
character is
to
zero. a

something. 0, called naught,stands


orders. The

nothing;
called

fill vacant

0 is also

cipherand
4. When the

figurestands
a

alone

or

in the

first
or more

place at
units

rightof
of

number,
the

it

represents one
are

order. of the first 5. Units and the first order occupy called the

place they

is called

simply units; units' place.

UNITS

OF

THE

SECOND

ORDER,
more

OR

TENS.

4.
Iso

1. Nine is

units

and

one

are

called but the

ten;
one

it is

represented by

the

figure 1

NOTATION.

11

made

to

occupy
0

the

second

writing a
2. One Two Three

in the
ten tens tens tens tens tens tens tens tens

units'

place phice.
thus
. .

from

the

right by

is written
are are are are are are are are

10. 20. 30. 40. 50. 60. 70. 80. 90. stands
or

twenty^ written

thirty,
forty,

'^

Four
Five Six Seven

"

fifty, sixty,
seventy,

''

"

''

Eight
Nine
3. When

eighty, ninety,
a

"

^'

figure in

number

in the
more

second units

placefrom
the second 4. Units

the order. of

it represents one right,

of
the

the

second

order the

are

called

tens; and

placethey

occupy

is called

tens'

place.

TENS

AND

UNITS.
,

5.

1. The

numbers

between

10

and

20,
tens

20

and

30,
units

etc.,are
of which

the expressed by representing they are composed.


ten ten ten ten ten ten ten ten ten tens
tens

and

2. One One One One One One One One

and and and and

one

unit units units units units units units

are are are are

two

three four five six


seven

eleven, twelve, thirteen, fourteen, fifteen, sixteen,


seventeeti,

written
"

11. 12. 13. 14. 15. 16. 17. 18. 19. 21.
22.

^'

"

and
and and and and and and

are
are are are are are are

"

"

"

eight units
nine
one

eighteen,
nineteen, twenty -one, -two, twenty

"

One Two
Two

units unit units

'"

''

two

''

12

KAY'S

NEW

PK

ACTIO

AL

AKITHMETIC.

]S^UMBERS

TO

BE

WRITTEN.

Twenty-three; twenty-four; twenty-five;twentysix; twenty-seven; twenty -eight ; twenty- nine. 2. Thirty-seven enty-three ; fifty-six ; sixty-nine ; sev; forty-tw^o ; ninety-four. ; eighty-seven ty-six: thir3. Eighty-three; fifty-one; ninety-nine; forty-five; -two. sixty seventy-eight; -three ; eighty-one 4. Fifty-five ; ; sixty-seven ; ninety forty-nine ; thirty-eight. ; seventy-four -two ; 5. Seventy-six fifty-seven ; ; forty-four ; eighty thirty-five ; ninety-one ; sixty-three.
1.
NUMBERS TO
BE

READ.

97. 92. 95. 64. 48.

UNITS

OF

THE
are

THIRD
one

ORDER,

OR

HUNDREDS.

6.
the
tens

1. Ten

tens

hundred;
the third filled with

it is

figure 1
and units

written

in

representedby order,the orders of


a

being each

cipher.

NOTATION.

13

2. Units

of

the

third

order

are

called

hundreds;

and

the

placethey

occuj^y

is called

the

hundreds'

place.

HUNDREDS,

TENS, between

AND

UNITS.

7.

1. The

numbers

100

and

200,

200

and

300, etc.,are
tens, and
2. One written One written One and One written One and

expressed by representing the hundreds, units of which they are composed.


and
one

hundred 101. hundred 110. hundred

unit

are

one

hundred

and

one,

and

one

ten

are

one

hundred

and

ten,
dred hun-

and

one

ten

and

one

unit

are

one

written eleven, and


two

111.
tens
are one

hundred
120.

hundred

and

twenty,
hundred

hundred,

two

tens, and
125.

five units

are

one

written twenty-five,

NUMBERS

TO

BE

WRITTEN.

1. One
one

hundred and

and

thirty;one
hundred and hundred

hundred and

and
one

forty;
dred hun-

hundred and 2. One

fifty ; one
one

seventy;
hundred

sixty; eighty.

and hundred

fifty-six ; seven
and

forty-seven ; two hundred and sixty -nine.


3. One

and twenty-three ; four hundred hundred and eighty-nine ; one hundred and fifty-eight ; three

liundred

and

two;

three

hundred

and

forty-

and five ; six hundred five hundred thirty-four; and

and hundred seventy-eight ; two and sixty-seven; eighthundred

ninety.
hundred and -three fifty
;
seven

4. Four

hundred hundred and

and and

and twelve; two nine hundred eighty-six; and fifty thirty ; six hundred ; four hundred

seventy.

14

RAY'S

NEW

PRACTICAL

ARITHMETIC.

four hundred .and fifty-three; dred and hundred twenty-nine;one huneighty-six;seven and hundred and three; four hundred six; seven
5. One

hundred

and

and

nine.
NUMBERS
TO BE READ.

UNITS

OF

HIGHER
are one

ORDERS.

8.

1. Ten

hundreds

by
2. Ten

1 in the

fourth form
a

thousand; order; thus, 1000.


unit of the

it

is represented

thousands hundred

fifth order ;

thus,

10000

one

thousands,a
units

unit of the sixth

order ;

thus, 100000, etc.


3.
next
ten Invariably, higherorder.
names

of any

order

make

unit

of

the

4. The

of

the

first nine

orders

may

be learned

from

the

followinoTable

of

Orders.
4th. 3d. 2d. Ist

9th.

8th.

7th.

6th.

5th.

t
o

'T3
O

T3
o

^ a
o

1^

NOTATION.

15

DEFINITIONS

AND

PRINCIPLES.

9.
nine

1. The
"

first nine

numbers

are

represented by

the

figures,1, 2, 3, 4, 5, 6, 7, 8, 9. 2. All other numbers are representedby combinations of the ten figures,1, 2, 3, 4, 5, 6, 7, 8, of two or more
"

J,

0. 3. The numbers numbers. numbers that end with

that

end

with

2, 4, 6, 8, 1, 3, 5, 7,

or

are

called

even

4. The

or

are

called

odd

numbers.
value of
a

5. The

is figure

the number

of units

it

presses. ex-

6. The

value

of the

depends upon
Rem.
"

figure is always local; that is,it in a number. placeit occuj)ies


a

The

the Arabic existed.

of local principle System of Notation

value from

is Avhat

peculiarly distinguishes
systems that have

all other

w^hen it stands figure expresses units' placeis called its simple value. 8. The value of a figure is increased ing tenfold by removit one placeto the left. 9. The value of a figure is decreased ing tenfold by removit one place to the right. number
a

7. The

in

GROUPING

OF

ORDERS

INTO

PERIODS.

10.
three
Rem.

1. For the

convenience

in
are

writing and

diflPerent orders each.


is

bers, reading numgrouped into periodsof

orders
"

number

of pointed offinto periods

three

each figures

by

commas.

2. The

first three
or first,

orderp units,tens,
"

hundreds

"

stitute con-

the

unit

period.

16

RAY'S

NEW

PRACTICAL

ARITHMETIC.

orders of three thousands^ten group thousands constitutes the second,or thousands, hundred
3. The

second

"

"

thousand
4. The

period.
third million group of three orders constitutes

the

third,or
5. The may be

period. periodsfrom the first to the the following learned from


Periods.

twelfth

inclusive

Table

of

6. The

in the

grouping following

of

the

orders

into

periodsis

shown

Table. 4.
Billion.

3.
Million

2.
Thousand.

1.
Unit.

TU

fl
o

't:!

9
o

a? ^

'-^

IS

en

S B H pq

mII

O 3

3
H

NOTATION.

17

7. It
lens J and

is

plain that
of

each that

hundreds

period is composed of unitSj iwriod.

To

Write

Xumhers
six hundred
-one

in

the
and

Arabic

System,
trillion three and

11.
hundred
seven

1. Write and million and

-four fifty hundred

twenty
six

billion nine and

eightythree

hundred

thousand fiftj-four

hundred

twenty-one.
-d
^c3

g o

G
o

4,

1,

1) 8

7,

4,

^ G

": c

MhP

WhP
each

WhP

Rule.

number
vacant

and write Begin at the left, composed of himdreds, tens, and orders with ciphers.
"

period as a units filling the


"

Rem. hundreds

"

In the left hand and


tens
are

however, period,
vacant

when orders

the hundreds
are

or

the

wanting, the

not

filled with

ciphers.
NUMBERS TO
BE

WRITTEN.

2. Two

thousand; thirtythousand,
million ;

four

hundred

sand. thou-

3. Five
4.

sixtymillion ; seven Eight billion; ninety billion;one

hundred
hundred

million. billion.

18

KAY'S

NEW

PRACTICAL

ARITHMETIC.

5. One

thousand

two

hundred

; two

thousand

one

dred. hun-

G. Three

thousand hundred thousand and

four and

hundred

and

fifty ;

six thou-

sand

seven

eighty-nine.
hundred and

7. Tw^elve 8. Six and


9. One

three

forty-five.
nine dred hun-

hundred twelve. milHon hundred

thousand seventy-eight

three and

hundred

and

sand thoufil'ty-seven

nine 10.

twenty-four.
one

million vSixty-eight
seven

hundred

and

forty-three
and

thousand 11. One


one

hundred and hundred.


one

and
one;

ninety-two.
one

thousand
one

thousand

ten;

thousand
12.

One

thousand and

hundred
one

and

one one

one

thousand and

one

hundred

ten ;

thousand

hundred

eleven. 13. Two 14. 15. 16. thousand and

three;

four

thousand

and

fifty.

and tw^enty-six. Forty-five thousand hundred and two one. Eighty thousand Ninety thousand and one. hundred and
ten

17. Four five. 18. One 19. Three nine.

thousand

two

hundred

and

hundred million

thousand

and

ten.

seventy

thousand

five hundred

and

Forty-five twenty-six.
20.

million

-three eighty

thousand

and

21. Nine 22. Seven 23.

hundred
hundred

and

nine

million
ten

ninety thousand.
and and
tw^o.

million

thousand thousand

Forty

billion two hundred


two

hundred

five.

24. Seven
une

and hundred
seven

million twenty-six billion fifty and

thousand 25.

forty-three.
and

Eighty

billion and

hundred

three

million

five hundred

four.

NOTATIOJ^.

19

12. Numeration

is the
a

reading of

numbers

when

pressed ex-

accordingto
To 1. Eead

system

of notation.

Read

JYutnbers

in

the

Arabic

Sr/stein.

654321987654321.

I
HP

"
1^
H

4,

1,

7,

4,

1.

OD

+J

a,

W^p
Rule.
"

W^P
1,

W^P

H^P

wIp

and point offthe number info Begin at the right, each. of three figures periods 2. Beginat the left, and read each jjeriod a number as posed comof hundreds,tens,and units, of the givingthe name period.
Rem.
two

1.
"

The

left hand

periodwill

sometimes

contain

hut

one

oi

figures.
Rem. 2.
"

It is

customarj^to omit the

name

of the unit

period.

NUMBERS

TO

BE

READ.

2. 3. 4. 5.
6.

4923176358. 41582; 763491; 2519834; 375486921; 37584216974; 432685729145; 6253971438267. 1300; 2540; 6070; 8009; 13200; 1005. 682300; 8600050; 3040; 50004; 704208. 7085; 62001; 400009; 2102102; 9001003.

20

RAY'S

NEW

PRACTICAL

ARITHMETIC.

7. 8. 9. 10.

130670921; 6900702003; 23004090701; 9420163070. 570000010326049;


800820020802008.

200103478511992485.

45763000020108000507.

THE

ROMAN

SYSTEM
DEFINITIONS.

OF

NOTATION.

13.

1. To
seven

express

numbers,

the

Roman

Notation

ploys em-

letters; namely, I, V, X, L, C, D, M.
Roman

Rem.

"

The of
to

the method is
now

used

System of Notation is so called because it was It used by the ancientRomans. expressingnumbers of clocks, mark the chaptersof books, the dial plates

etc.

2. In four

the

Koman

Notation,numbers
seven

arc

expressedin
numher,
as

ways. 1st. Each

of

the

letters expresses

lows: fol-

V, five; X, ten; L, fifty;C, one hundred ; D, fivehundred ; M, one thousand. 2d. Seven numbers ters are of the letby repetitions expressed C. 7, JT, and Thus, II represent two; Hi, three; XX, twenty; XXX, thirty;CC, two hundred; CCC, three I, one;
hundred
;

CCCC, four
numbers follows:

hundred.
are

3d. Four
as

nation, expressedby a subtractive combiIV, four; IX, nine; XL, forty;XC,

ninety.
4th. All other numbers
are

of
smaller number.

two

or

more

of

the

tions formed by additive combinanumbers, the eighteen preceding


on

being always

situated

the

right of

the

larger

example,YI is six; XYII, seventeen; LXXYIII, hundred and one -eight seventy ; CLXXXIX, eighty-nine \ hundred, and seventy MDCCCLXXYII, -seven. eighteen

For

NOTATION.

21

Write
1. The 2. The numbers numbers

in

the
one

Koman
to

J^otation,

from from

twenty.
to

twenty

thirty.

THE

FUNDAMENTAL
DEFINITIONS.

RULES.

14.

1. An

integer
are

is

whole abstract is
a

number.
or

2. Numbers

either number

concrete.

3. An

abstract

number

simply, as applied to
one

5,

12,
or

20.
concrete

4. A
more

number
1

is

number

as objects;
name

5. The

denomination.

apple,5 pounds, 12 men. number of a concrete is of the object Thus, in 5 pounds, the denomination
are

its is

pounds.
6. Numbers 7. A
or

either

simple
as

number

concrete;
8. A

3,

compound. is a single number, either abstract 1 pint. dollars,


is made up of
two ;
as or more

simpleor

compound
numbers
1

number

concrete

of different

denominations

pecks

quarts
9. There

pint.
are

primary operationsof Arithmetic; and Division; Multiplicatio7i, namely. Addition^Subtraction,


four
"

these

are

called

the

Fundamental

Rules.

15.
many

1. If will you

you

have

cents

and

find

3 Ans.

cents, 5 5 for

how

then Because
12 cents

have? 2 for
cents
a

cents. cents.
a

Why?
2. I
:

and

cents 5

are

spent
how

slate, and
I

cents

book copy-

many

cents

did

spend
Ans.

? 17 7 many 22
cents. cents cents cents. 10

Why?
for did cils, penall

3. John and cost?


4.
a

gave
9 cents

cents

for
a

an
:

orange,
how A71S.

for

ball

Why?
cents

Joseph
for much

gave paper,

5 25 of he

cents
cents

for

daily
a

paper,

for
30
:

weekly
a

for and

monthly
40
cents

magazine,
for
a

cents

book did

poems,

novel
cents.

how

spend?
in these

110

16.

1. The

operation
Addition

examples
of

is termed

Ador

dition; hence,
more

is the number.

process

uniting
is the

two

numbers
2.

into

one

The

number

obtained

by
to

addition

Sum

or

Amount. 3. When the numbers Addition Addition


two 4

be

added

are

simple,

the

operation
4.

is called

of Simple Numbers.
called (-|-),
means j)lus,

The

sign
added

of

more;

when
are

placed
to

between
;

numbers,
+
2
means

it

shows
4

that
2
are

they
to

be

thus,

that

and

be

added

together.
(22)

ADDITION

OF

SIMPLE

NUMBERS.

23

5. The

8ign
which
6
means

of

denotes (==:) equality


are

that

the

between
4

it sstands that

equal; thus, the


sum

quantities expression
:

the

of

and

2 is 6

it is

read, 4 plus 2 equals6.


Addition Table.

17.
not

When

the

sum

of

the

in figures the

eokimn

does

exceed

9, it is written

under

column

added.

24

KAY'S

NEW

PRACTICAL

AKITHMKTIC.

Examples. 1. I the
acres

own

tractH

of land the

the first contains

240 how

acres

second,
in all?

132

acres;

third,25

acres:

many

SoLUTiOiV.
write units

"

Since units oi
same

orders different

of the

order in
most

to be added

Begin
which
are

may at the in

be

in the

together, the same column, so that the figures convenient ])ositio7i.


can

not be added

write

right,and say units' place; 2


write in tens' write in

and
3

2
are

are

7 units,

2 4 0 132 2 5 39 7

acres, acres,

and

5, and
2
are

4 3

9 tens, which

hundreds, which

place; 1 and hundreds' place. $210, another


of my debts?

acres,
acres.

2. I

owe

one

man sum

S142,

and

another

$35:

what

is the the the


sum sum

$387.
8695. 53758.

3. Find
4. Find

of of

4321, 1254, 3120. 50230, 3105, 423.


of the

18. When
9, two
or

the
more

sum

are figures

figures in a requiredto

column

ceeds ex-

exi)re8S it.

Example. 1. Add the numbers

3415, 503, 1870, and


same

922.

Solution. column. which


are

"

Write say

units of the
2

order
5

in the 10

same

Then
no

and

are

5, and
in
are

are

units,
1 1

3415
503 18 70 922 6 710

(0

) units, written
tens;
are

the units'

and place,

ten, carried
are

to the

1 and

3, and

are

10, and

11 tens, which

1 ten, written

in the tens' 9
are

and place,

1
are

hundred, carried to the hundreds; 1 and 18, and 5


2 and
are

10, and 8
are

23, and 4 3, and


3

are

27

hundreds, which
2

hundreds,
sands' thou-

written

in the hundreds' 1
are

and place,
are

thousands, carried
thousands, written

to the thousands;

in the

place.

ADDITION

OF

SIMPLE

NUMBERS.

25

CaiTying the
to

dreds simply adding tens to tens,hunthe principle that only units hundreds, etc., on
tens

is

of the For

same

order

can

be

added.

convenience,the
with of the the units

addition of the

column,
the
can sum

beginsat the righthand lowest order,so that, if


column
next

in figures
sum

any

exceeds

9, the

tens

be

carried to the

of the

higherorder.
of

Rem. column

"

To

illustnitc;the

greater convenience

adding

the

units'

take the above first,

example.
the addition with the thou3415

Solution. sands'

"

Commencing
sum

adding the hundreds, the sum is 26 hundreds, which and 6 equal 2 thousands is 10 tens, equal hundreds; next adding the tens, the sum to 1 hundred; and is 10 finally adding the units,the sum have also to be units,equal to 1 ten. As these sums
added, this much
extra

column, the

is 4; next

^^^
f."^o

4 10

work

must

be

done

in order

to

__1_9
6710

complete 19.

the

solution.

Rule.

"

1.
same

Write
order

the may

mnnhers stand and

to he in the

added,
same

so

that

figures of
2.

the

coluinn. column each rately. sepacohwm

Begin at
Place and it,

the the

righthand,
units

add

each

obtained

by adding

under down

carry the tens


sum

to the next

order. higher

Write

the entire

of

the last column.

Proof. the

"

Add

the

columns

downward, commencing

with

column

of units.

1. Find
In

the

sum

of
cokimns

3745, 2831, 5983, and


of it figures, This
may

7665.
3 745 2831 5 9 83
7"ak
9 0 9 2 4

adding long
the
them
as

is necessary

to retain

numbers in

carried.

be

done

by

placing
columns,

smaller

3, 2, 1, in the

figures under margin.

their

proper

321

26

liAY'tS

NEW

PRACTICAL

AKITHMETIC.

Examples.

(2)
184
216 135

(3)
204
302

(4)
103 405 764 573 127
205

(5)
495 207 .^-85 825
403

(6)
384 438 348 843 483 834

(7)
1065 6317 5183 7102 3251 6044

401
311 109 43

320
413

101

325

13. 14. 15. 16. 17. 18. how 19.

11 23 45 51

44 + 55 + 22 + 33 -(+ 41 -I 74 + 83 + 16

how how how how how 9386

man}^ many

? ?

165. 237. 225.

+ + +
?

19 48

+ 32 + 74 + 55== 4 + 76 + 85 +
=

many?
many many ? ?

264. 843. 3456


are

263

104

+ 321 +

155

94753 many

+ 2847
has

+ 93688
31

258

204388.

January April,30 ; and


these 20. June has

days ; February,28 ; March, 31 ; May, 31 : how days are there in many


151.

five months? 30

days;July,31; August, 31; September, 153. 30; October, 3l : how many days in all? have 151 days, the next 5 have first 5 months 21. The has 153 30, and December, 31 : how days, November 365. days in the whole year? many

ADDITION

OF

SIMPLE

NUMBEES.

27

of muslin : the first contained 50 bought 4 pieces yards,the second, 65, the third,42, the fourth,89 : how 246 yd. yards in all? many 22. I 23. I
a owe one
:

man

S245,
much do

another I
owe

$325,
? A. die?

third

$187,
$853.
and

fourth

$96

how

24. General lived 67

Washington

was

born did A. he D.

D.

1732,
to to

years: in what the Great 25. Alfred

year
died
was

1799. the the the

signing of Magna Charta


American American 26. A

314

901 ; thence years ; thence year

Eevolution, 560
Eevolution

years:

in what

did

has
lambs

flocks
; in

begin? of sheep;
second,
how 243

1775. in the first


are

65

sheep
lambs
;

and

43

the

187

sheep and
lambs
; in

105 the

in the 416

third,370
95

sheep and
lambs:

fourth,

sheep and
man

many
1038

has he?
27. A

bought
$144,
many he 23

30

barrels
for did

sheep and lambs sheep,and 486 lambs. of pork for $285, 18


and 34 barrels for

barrels

for how did

barrels

$235,
he

$408

barrels

buy,
105

and

how

many

dollars

pay?
numbers
as fourth,

and bbl.,

$1072.

28. The

first of four
; and

the
what

841 third, is their

the

is 287 ; "^^^he second, 596 ; much the first three : as 3448.

sum?

built 1700 Pyramids of Egypt were years before the founding of Carthage ; Carthage was founded 47 years before and was destroyed607 years after the foundingof Eome, or 146 years before the Christian era. How the Christ before were Pyramids years many

29. The

built?
30. Add six hundred
seven

2500. three and

thousand

and

thousand five; fort^^-two


; three

thousand
thousand 275432

twenty-seven ; 105 and four; 80079;


six hundred.
;

hundred hundred

and and

three

twenty

753420. and
two

31. Add

four

hundred

thousand

and

28

KAY'S

NEW

PRACTICAL

ARITHMETIC.

thirty ;
million

three
two

hundred thousand

thouBand
three

and

five ; 872026 and


-seven. forty

four

hundred

5851840. 32. Add and eighty million dred eighthundred eighthunand eighty-nine ; seventy-seven million ; 2002002 and thirty-six and hundred thousand; two hundred five thousand and
two

four
six

million hundred

hundred

and

seven

49003;
nine

nine

ninety million
has
an

nineteen

thousand

hundred

and

nineteen. America
area

2155513020. of 8955752

33. North

square
and the
area

miles;
West of the 34. A

South

America,
continent?

6917246

square miles:

miles;
is 15967521

Indies, 94523
entire
man

square

what

the

sq. mi.

pays
for for

$600

for

$1325 lot,

for

building
work, stone-

materials, $30
$250 $120
did for

digging

the

cellar,$120
for for

for

brick-work, $140
$115
him?
to

carpenter-work,
much

and plastering, and lot cost

painting: how
wife,$3275

his house
man

$2700.
his
to each

35. A of his two what

bequeaths
sons,

$7850

and

$2650
of his

to each

of his three

: daughters

is the amount merchant

bequest?
for dress

$22350.

36. A for and

spent $8785

sheetings. He sold the dress of $1250: the sheetings at a profit


merchant

goods,and $12789 of $878, goods at a profit


for how much did

he sell the whole? 37. A w^orth worth

$23702.

began
stock
one

business worth he

with

$7000

cash,goods
other
:

$12875, bank
$4785.
In
at its close?

$5600, and

stocks
w^as

year

gained $3500

what

he worth 38. A

$33760.

each requiring 30 yards parlors, of carpet; four bed-rooms, each requiring 25 yards; a each requiring 20 yards: dining-room and sitting-room, how yards are required to carpet the entire many
two

house

has

house?

200

yd.

ADDITION

OF

SIMPLE

NUMBERS.

29
readiness The lowing fol-

20.
and

An

excellent is to

in practice, add
two

order

to

secure once.

accuracy,

columns method

at
:

example illustrates

the

(!)"
Beginning
77 ; then with

47, add

the 3 tens

above, which
7 tens

equal

7892
6 7 79 4865

the 4 units, the


9

making

81 ; then the
9

the 6 tens above,

141; and
and the

5 units,

146; then
the Put down

above, 216;
the
3 to

units, 225^ then


2 units, 317.
to

tens

above, 315, and


carry and
are

6234 9347 35117

the finally
3 hundreds

the 17, and Then


93

the

hundreds' 60
are

column.
are

carry and 8
are

are are

96, and

156, and 2
are

158, and 40
7
are

198,
70
are

206, and

60

266, and

273, and

343, and

351, wnk'h

write in its proper

place.

27744

35249

30314

3611570

3223341

(7)

(8)

(9)

(10)

30

RAY'S

NEW

PRACTICAL

ARITHMETIC.

62833

43702

97812

2802803

6685899

21.
many

1, will

If
you

you

have have
4

9 left?

apples,
from after

and

give apples

4 Ans.

away, 5 5 how

how

apples.
apj^les.
many

Why?
2.
were

Because
had 15

apples
cents ;

are

Frank left?

spending
Aiis.
8
are

7,

cents.

Why?
Ans.
a

3
4.

If If

you

take 25

from

13, how
and I have

many 10 of

left? for 15

5.

I have

cents, will

spend
left ?

them Ans.

lead-

how l^encil, 5-

much from

cents.

Twelve

twenty

leaves

how

many?

A71S.

8.

22.
termed

1. The

operation
;

in

the

preceding
is the numbers. the left

examples
process

is of

Subtraction the

hence.

Subtraction
two

finding
2.

difference

between
is the

The

larger
;

number
and

called number

Minuend;
after

the

less,

the the 3.

Subtrahend

subtraction,
the

.Difference or
When the

Remainder.

given

numbers

are

simple,

operation

is called

Subtraction

of Simple Numbers.
of

23.

The

sign
when the

Subtraction

(
"

;
two

is

called

minus,
notes it defrom 5 3. is

meaning
that the
to
one

less;

placed
on

between

numbers,
be taken that

number the

the 8
"

right
5
8
=

is to 3

on

left; thus,
8, and
is

means

be

taken

from

read,

minus

equcds
(31)

32

RAY'S

NEW

PRACTICAL

ARITHMETIC.

Subtraction

Table.

24.
than

When
the

of the subtrahend is figure corresponding figureof the minuend.

each

not

greater

Examples.
1. A
man

having $135, spent $112:

how

much

had

he

left?

SUBTRACTION

OF

SIMPLE

NUMBERS.

33

Solution.

"

Since

the difference units

between of the which

units
same

of the

same

order
same

only
made

can so

be

found, write
the
in the most

order

in

the

column,

that be

between figures
convenient

the

subtraction

is to be

may
at

position.
5 3 1

Begin
3, which 2, which

the
in

and right, units'


tens'
no

say 2 from from from the

leaves leaves leaves ^^^ minuend.

put put

place; I

in

place;1

il^
^^

subtrahend.
remamder.

0, and, there being

figureson

left of

the placeis vacant. this,

2. A

farmer he

having
left?

245

sheep,sold
for

123:

how

many 122.

sheep had
3. A
man

bought
much did

farm

$751, and

sold

it for

$875:

how

he

gain?

$124.

What

is the

difference 734 8752 79484 49528 and and and and 531? 3421? 25163?
*

4. Between

203. 5331. 54321. 33113.

5. Between
6. Between

7. Between

16415?

25.
than

When the

the

lower
a

upper,

figurein any order which we arises, difficulty

is

greater
now

will

explain.
Examples. 1. James
cents

had he

13

cents; after spending 5, how

manv

had

left?
1 3

can

not

be subtracted from

3, but it can

be from

13;

^
"qT

6 from

13 leaves 8.

2. From

73

subtract

45.

Prac. 3.

34

KAY'S

NEW

PRACTICAL

ARITHMETIC.

Solution. Take
10 1

"

units

can

not

be

taken add

from this 1
13

units. 7 3
4 5 2 8

(ten)from
to

the 7
3

and (tens),

(ten)or

units

the

units, which
1

makes

subtract the 5

and there will remain units,


ten is taken

units; then, 8 units, to be


from
tens

put in units' place. Since


tens, there
remain but
6

the from

7 6 tens

tens.

Subtract

and

pui

the remainder, 2 tens, in tens' Rem.


1.
"

place.

The

difference is 28.

Instead the 3

adding it to

actually taking 1 ten from the 7 tens, and is perfi.rmed units,the operation mentally; thus,
of
13 leaves cases, 1
in

6 from Rem.
2.
"

8, and 4 from
value

6 leaves 2.

In

such the

the which

of the

changed,since
added Rem.
3.
"

ten

is taken

number is not upper from the order of tens is

to the number

the order of units. unit of


a

higher order and adding it to the beneath be subtracted units of the next lower, so that the figure may is called borrowing ten. from the sum, the units by 10, instead of considering 4. After increasing Rem. of the up}"er number diminished the next as figure by 1, the result of the lower number be increased will be the same, if the next figure by 1; thus, in the previous example, instead of diminishingthe 7 makes tens by 1, add 1 to the 4 tens, which 5; thus,5 from 13 leaves Taking
a
"

8, and 5 from
Rem.
1 from
or

7 leaves 2.

5.

"

This process
we

depends upon

the fact that

having

borrowed 4 tens,

the 7 tens,

have

to subtract from

it lx)th 1 ten

and

their sum,

5 tens.

3. Find

the

difference between
Method.

805

and

637.

Solution
under
same

"

1st

"

Writing

the

less number

8 05
6 3 7 16 8

with units of the same order in the greater, column, it is requiredto subtract the 7 units from

the

5 units.

The because which


10

five

can

not

be

increased

by borrowing from
1

the

next

figure,
makes it to

it is leaves

0; therefore,borrow
7 hundreds
in
ten

hundred

from
1

the 8

hundreds,

hundreds' from the

place;this
the
10

hundred

tens; then, borrowing 1 the 5 units,9 tens will be


units'

tens, and
15

adding

in

tens'

and place,

units in the

place.

SUBTRACTION

OP

SIMrLE

NUMBERS.

35

Subtracting 7 from 15, 8 units are left,to be written in units* 9 tens, there are left 6 tens,to 3 tens from place; next, subtracting 6 hundreds from 7 hundreds, be written in tens' place;lastly, subtracting there remains 1 hundred, to be written in hundreds' place. If the 5 units be increased 2d Method. 10, by say 7 from the 3 by 1, say 4 from 0 can not be 15 leaves 8; then, increasing 6 by 1, sa^- 7 from 8 taken, but 4 from 10 leaves 6; then, increasing
"

leaves 1. Rem.
1.
"

The

second

method

is

and contains Rem.

less liable to error,

venient, congenerally used; it is more when the upper number especially

ciphers. 2. "Begin

at

the the

right to subtract, so
upper,
1 may

that

figureis greater than


order. Rem.
3.
"

be borrowed

if any from a

lower

higher
the less

If the
sum

difference

of two the

numbers

be

added

to

number, the
from

will be

equal to

greater. Thus, if 5 subtracted

8 leave 3, then

3 added

to 5 will

equal 8.
under
same

26, Rule.

"

1. Write the the

the

less number order in the

the

greater,

of figures placing 2. Beginning at the one over directly


3. upper
next

same

column. each

hand, subtract right

from figure
beneath.
ten
one

and it,

write the remainder the upper, add

If

the loiver

figureexceeds
the lower take
one

to

the

subtract figure, lower figure, or


"

and from it, from the next


to

carry upper

to the

figure.
;

Proof.
sum

Add

the the

remainder

the work

subtrahend

if the

is

equal to

minuend,

the

is correct.

36

RAY'S

NEW

PRACTICAL

ARITHMETIC.

5. Take 6. Take 7. Take

1234567
15161718 34992884
"

from from from

4444444. 91516171. 63046571.

8209877. 76354453.
28053687. 99579930. 89998999.

8. 153425178 9. 100000000 10. Take 11. A


more

53845248==? 10001001==?
from 63
cents.
a

"

17

cents

46

cents.

carriagecost $137,
the
tree

and

horse

S65

how

much

than 12. A

horse 75

did

the

carriagecost?
broken
was

$72.
the

fell

was

37

feet

high was long: how high


discovered

feet

part that
38 ft. 1492
:

the

stump?
in

13. America how

was

by

Columbus

years had elapsedin 1837? many in the bank 14. I deposited $1840, and
many
man

345.

drew

out

$475
$1365.

how

dollars has of

had

1 left? worth

15. A
to

property
$7426
:

$10104, and
his

owes

debts

the amount

when

debts

are

paid,how
$2678.
:

much

will be left?
man

16. A had he

having $100000, gave


19019 the from
excess

away

$11

how

many

left? 20010. of 50082. nine

$99989.
991.

17. Subtract
18.

Eequired
and
4004

hundred

and

twelve
861928.

thousand
19. Take

ten, above
from 1009006 and

four

million.
two

3995996.
million

20. Subtract nine 21.

from

sand twenty thou1011924.

hundred

thirty.
hundred and five

Subtract from

four

thousand

and

twenty-two
22. What hundred 23. A the and
man

2000687.

1595665.

is the difference
one

between

thirteen million two


4101901.

and

17102102? in business

invested he found much

$30,000; at
assets

the end

of
to

first year

that
had

all his he lost?

amounted

only $26,967; how


24.

$3,033.
8887687. 9991810891.

Take

9238715 9909090009

from

18126402. 19900900900.

25. Take

from

ADDITION

AND

SUBTRAQTION.

37

Examples 1. 275
"

in

Addition

and

Subtraction.
183.

+ 381 + 625"1098==?
479
"

2. 6723 3. In amount

347"

228

5669.

January, 1876, a
of

merchant

bought goods to
and in of

the

$2675;

in
one

February, S4375;
payment
he
sum

March,
another

$1897 ; after
of

making
much three

$3000, and
is $1300 what

$4947, how
4. I
owe

did

still owe?
"

$1000.
one

notes, whose
another

note

being for $250, and


of the third 5. Mr. note?

for $650:

is the amount

$400.

on $450 in bank Monday; on deposited Tuesday, $725; oji Wednesday, $1235; on Thursday, $4675; and on Friday,$1727. On Saturdaymorning he drew out $5935, and Saturday afternoon,^877 : how

Jones

much

money
the

had
end

he of

left in bank?
one were

$2000.
I had

6. At

year

I found

spent $2300.
ing, for cloththe remainder did the
two

Of

this amount,

$350
of

paid for board, $125


for and incidentals,

$375 for books, $150


for
acres

two

acres

ground :

how

much

cost?

$1300.
For than the first he for the first ; afterward sold them

7. A

gave and for the third he gave all for $20838: how much 8. A
man are owns

speculator bought three houses. $4875 ; for the second,$2250 more


$3725.
did he He

gain?
at

$5113.

$16785
has

in

j^ropertyvalued personal property,


was

$49570,
$24937
bank
:

of

which in real much

and

estate ; the

remainder

in deposited

how

he

in bank? merchant

$7848.
a

goods for $7895, and He sold the $3 for drayage. paid $175 for did he gain? $2020. goods for $10093: how much farmer 10. A in land, invested $10000, as follows: $5750; in horses, $1575; in hogs,$675; $925; in cattle,
9. A

bought and freight,

bill of

38

RAY'S

NEW

PRACTICAL

ARITHMETIC.

and did 11.

the

remainder
invest in

in

im^jlements
and

and tools?

tools

how

much

he
A

implements
on

81075. $4625
;
on

speculator
;
on

Monday
he he much

gained
lost

on

day, Tues-

$3785
he lost he entire
12.

Wednesday
on

$6955

Thursday
on

$895;
lost week? The

Friday
how

gained
did

$985,
he

and

day Saturthe

$1375:

gain

during

$170.

following
First for for

18

Mr.

Brown's received

private
$50
for

account

for
and

two

weeks:

week,
$7

salary,
for

spent
and

$25 $5

clothing,
sundries.

for

board, week,

$2
received

washing,
$50
for

Second
to

salary,
for

loaned and
at

$35 $8
the

Tom
for

Jones,
sundries.
of

paid
How

$7

for

board,
did

$2
Mr.

washing,
have

much

Brown

end

the

two

weeks?

$9.

MULTIPLICATION

OP

SIMPLE

NUMBERS.

43

When
22. What
Analysis. 5

the

Multiplier Exceeds
43 X
2 tens

12.

is the
Since

product of
25 is
"

25 ?
and and the
6

"

equal to

4 3
2 5 21 8G 5
=

units

"

that

is,20 -f 5,
the

multiplyby
hundreds
1 ten.

write
2

the

product,215;
set

then

multiply by
and

4 8 X

tens,and

8 product, and

tens, under

the 2 hundreds

1075

=43X20 43X25
5

Multiplyingby 5 units gives 5 and multiplying by 2 tens gives 20


times 43 and 20 times 43

times
times

43, 43; add

them, because

equal 25 times 43. and write Hence, multiplyby the units' figureof the multiplier, the product so that the right-handfigure will fall in units' place; then multiply by the tens' figure, and write the right-handfigure of the product in the tens' place.

44

KAY'S

NEW

PRACTICAL

ARITHMETIC.

Therefore, in multiplyingby
of figure the

figureof
order
as

product in

the

same

any order,write the multiplier.

the

last

Note. of the

by productsof the multiplicand called pariial are producU. multiplier


"

The

the

separate figures

General

Rule.

"

1. Write

the

under multiplier
order in
a

the miilti'

of plicand,placing figures
2.

the

same

column.

plier by each figure of the multiMultiplythe midtiplicand with units, the in succession, always setting beginning of each product under that figureof the right hand figure which producesit. multiplier will be the the partial 3. Add : their sum productstogether productsought.

Multiplythe multiplier by obtained be should the product thus first product.


"

Proof.

the the

multiplicand:
same as

the

23.

Multiply2345
SOLUTION.

by

123.

PROOr.

123 2 345
12 3

multiplier.
6
=

multiplicand.
3

234 615 492 869 246 288435

multiplicand.
12 3 X 5 40 =123X 300 =123X =123X2000 123X2345

multiplier. 7035=^2345X
469

^2345X

20 00

2345__ =2
288435
=

345X1 2345X123

24.

Multiply327 by
"

203.
is

cipher in the multiplier, leave it, and multiplyby the other figures, being careful to place the right-hand figure of each partialproduct under the multiplyingfigure. 6 5
a

Remark.

"When

there

82 7
20 981 4 3

66381

MULTIPLICATION

OF

SIMPLE

NUMBERS.

45

Examples. 25.
26.

235
34()

X13 X19 X29 X34. X37 X49. X57. X62: X76:

3055. 6574. 12325. 17612. 10323. 42581. 16758. 26598. 36860.

34.

624 X 976
342

85: 97
=

53040. 94672.

35. 36.

27. 28. 29. 30. 31. 32. 33.

425 518 279 869 294 429 485

X364: X526 X536. X215: X276 X365: X635:


= =

124488. 197776. 255136. 470205. 962964. 600790. 5354320.

37.
38.

376 476

39. 2187 40. 3489 41. 1646 42. 8432

43.
44.

45. 46. 47. 48.

Multiply 6874 Multiply 2873 Multiply 4786 Multiply 87603 Multiply83457 Multiply 31624
will
126

by by by by by by

829. 1823. 3497. 9865. 6835. 7138.

5698546.

5237479.
16736642. 864203595. 570428595.

225732112.
of flour

49. What

barrels

cost, at $6

rel? bar-

$756.
50. What will 823 barrels of

pork cost, at $12


cheese

rel? bar-

$9876.
51. What
a

will

675

pounds
bushels

of

cost, at cost, at
how
24

13

cents
cents.

pound?
52. What will 496 of
oats

8775

cents

bushel?
53. If will he
a man

11904 travel in 152


are

cents.

28

miles

day,
one

many
4256

miles miles. many

travel

days?

54. There

1760 in
24 209

yards
miles? in hours

in

mile

how

yards

are

there
are

367840
a an

55. There if year: sail in a


a

day,

and how

365

yards. days in a
miles.

ship sail

miles

hour,

far will she 70080

year?

46

RAY'S

NEW

PRACTICAL

ARITHMETIC.

56.

Multiply two
and
seven.

thousand

and

twenty-nine by
hundred and

one

thousand 57.

2048203. thousand
seven.

Multiply eighty and by sixty thousand


58.
two

four

one

4824622807. and
one

Multiplyone
by
20001. grocer

hundred

thousand

and

thirty-

2020741032.

59. A 215

bought
8
cents

2
a

barrels

pounds, for
sugar?
grocer

pound
barrel
a

of sugar, each weighing did he pay much : how


3440
cents.

for the 60. A 36

bought

of

molasses, containing
"

for 45 gallons, cents a gallon:how 61. A for


a

cents

much merchant

gallon ; and did he gain?


sold 2650

sold it for 55
360
cents.

commission
at

bushels

of wheat
him 2

farmer,
a

95 for

cents

bushel, and
much

charged
money
Was

cents

bushel

how selling:

duo
cents.

the

farmer? farmer and


5

246450

62. A

bought
horses all for

horses

of

one

man

for 75

lars dol-

each,
and sold did 63. A he

of 150

another
dollars

for each
:

125

dollars each, many for


lars dol-

them

how

gain?

$575.

merchant
more

bought
for 325

one

box

of

goods
so as

250
more

two dollars,

dollars
sold did 24

each, and
all sell them? 5

three
to

for

175

dollars

each;

he

them he

gain
$1781.

356

dollars:

for how

much

64. A 36
a

farmer 14

bought
dollars he sold
a

sheep,at

dollars

head;

hogs, at
head
:

when did 75 X 69 X

head; and 9 cows, at 45 dollars them he lost 275 dollars : for all,
$754.

how

much

he

sell them? 85 X 48 X

65. To 66. To

37 add 53 add

54, and 27, and

subtract subtract

5284. 4279.

2081. 674.

67. I
in it

bought 50 bags of coffee, averaging 63 poupds a did bag, paying 34 cents a pound : how much cost? 10719" cents.

MULTIPLICATION

OF

SIMPLE

NUMBERS.

47

CONTRACTIONS

IN

MULTIPLICATION.

CASE

I.

32.

When

the will 15

can multiplier

be

into separated
cents

factors.

1. What
Analysis.

oranges
15 will is
cost

cost, at 8
3 3

each?
8 ct. 5

"

Since oranges
as

Cost

of

1 orange,

times times

5, 15
as

oranges. Therefore, instead of multiply8

much

Cost

of

5 oranges,

4 0 3

ct.

Cost of 15 by 15, first find the cost of 6 oranges, by multiplying8 cents by 5; then take 3 times that product for the

ing

oranges,

120

ct.

cost

of

15 oranges.

the multiplier into tico or more factors. Separate and 2. Multiply the multiplicand by one of the factors, till every factor is used; the this product by another factor, last productwill be the one required.
Rule.
"

1.

Kem.

"

Do

not

confound be

into which

it may

while whose

the
sum

parts into

factorsof a number separated. Thus, the factors of 15 may be separatedare which


the
as, 7 and

with
15
are

the ;?rtrts
5 and

3,

any

numbers

equals 15:

8;

or,

2, 9, and

4.

Examples. 2. What

will 24

acres

of land

cost,at $124
in 56

an

acre?

$2976.
3. How of 1512
4. How

far will miles many per

ship sail

weeks^
there

at the

rate

week?
are

84672 in 54

miles.

each 5.
6.

pounds of iron weighing 2873 pounds? Multiply 2874 by 72. Multiply8074 by 108.

loads,

155142

pounds.
206928. 871992.

48

KAY'S

NEW

PRACTICAL

ARITHMETIC.

CASE

IT.

33.
as

When

the

is multiplier

1 with

ciphersannexed;
a

10, 100, 1000, etc.


1.

Placing one 3) changes the


and
so

cipher on
units into

the

right of
tens

number

(8,

tens, the

into

hundreds,

on,

thus, annex
2.

the number and, therefore, multiplies one cipherto 25, and it becomes
two

by ten;
250.

ciphers changes units into hundreds, the number tens into thousands, etc.,and multiplies by one two hundred; thus, annex ciphersto 25, and it becomes Annexing
2500.

to the multiplicand, as ciphers there are and the nwnber thus ciphersin the 7nultiplier, formed will be the productrequired. Rule.
"

Annex

as

many

1. 2. 3.
4.

5. 6.

Multiply 245 Multiply 138 Multiply 428 Multiply 872 Multiply 9642 Multiply10045

by by by by by by

100. 1000. 10000. 100000. 1000000. 1000000.

24500.

138000.
4280000.

87200000. 9642000000. 10045000000.

^CASE

III.

34.
both

When of the

there factors.

are

ciphers at
625

the

right of

one

or

1. Find

the

product of

by
be

500.

Analysis.

"

The

multipliermay

considered

as

composed of two factors: 5 and 100, Multiplyingby the product is 8125; and the product of this number "5, by 100 is 812500, which is the same as annexing two to the first product. "'iphcrs

6^5
500 312 5 00

MULTIPLICATION

OF

SIMPLE

NUMBERS.

49

2.

Find

the

product
number
two

of

2300

170.
2300

Analysis.

"

The

2300 23

may

be

regarded
and

as

17
161 2 3 391000

composed
the
two

of factors

the 17

factors
10.

and

100;

170,

of

and

The

product
23

of

2300

by

170

will

be

found

by
and

plying multithe the


to

by

17,

and

this
10

product
that

by

100,

resulting product product


the both of 23
as

by
17,
there

(33);
then
3

is, by
3

finding ciphers

by

and
tire

annexing

product,
factors.

ciphers

at

the

right

of

Rule.

"

Multiply
;

without
annex

regarding
to

the

ciphers
as

on

the

right

of
as

the
are

factors
at

then

the

product

many

ciphers

the

right of

both

factors.

..gmm^

^i^ m

35.
how

1.

If

you will

divide each

apples equally
have?

between

boys,

many

boy
2

Analysis. each which

"

It

will
as

require
many

apples
as

to 2

give

each

boy

1. times

Hence,
in

boy
are

will
3.

have

apples

is contained

(i,

How

many

times

in

6?

Ans.

3. 6.

Why?
2. how 3. The solved If many How you divide will many process is called each times 8

Because

times

2 2

are

peaches
have?
2

equally
Ans.
4

between

boys, Why?
Why?
arc

peaches.
Ans. 5.

in

10? the

by

which

preceding

examples

Division.

DEFINITIONS.

36.
times
2.

1.
one

Division

is is

the

process in

of

finding

how

many

number
divisor

contained number
to

another. which
;

The

is the number

by

to

divide

; the

dividend the

is

the

be

divided

the the

quotient
divisor

is is

number
in

denoting
the

how

many

times

contained

dividend.

DIVISION

OF

SIMPLE

NUMBERS.

51

Thus,

3 is contained

in 12, 4

times; here, 3

is the

12 the divisor,

dividend, and 4 the quotie7it.


3. Since
are

is

contained
divisor

in and

12

four

times, 4

times

12;
the

that

is,the
and
4

quotient multiplied produce


the
the

dividend. 3
are

4. Since

factors of

product 12, the


factors plication in multi-

divisor

and

quotient correspondto
dividend, to
process
the other the

; the

insion

is the

product. Therefore,Biof the factors of a of finding one


factor is
how known.

product,when
37.
at

A
cents

boy

has

8 cents

many

lemons

can

he

buy,

each?

Analysis.

"

He

can

buy 4, because
cents.

4 lem1st

8 cents,

ons, at 2 cents

each, will cost 8

lemon,

2 cents.
G cents,

The and

boy

would

give
cents

2 cents

for 1

lemon,

Left,
2d

then

have

6 cents

left. for the 2d

lemon, 2 cents.
4 cents, 2 cents. 2 cents, 2 cents. 0 cents,

After

giving 2
have

lemon, he

Left,
3d

would

4 cents

left. for the

lemon, lemon,

Then, giving 2
have 2 cents left.

cents

3d, he would
for the

Left,
4th

after giving 2 cents Lastly, would have nothing left.

4th, he

Left,

The

natural

method

of

is by performingthis operation it is known how of many times


2

subtraction; but, when


can

be

subtracted

from

times, say

2 in 8 four

8, instead times, and 4


may be the

subtracting2
2
are

four

times
a

8.

Division Therefore,

termed
same

short

method

of

making many
The
the the made. divisor number

subtractions is the which how

of
the

number.
; the

number

subtracted subtraction

dividend,
made
;

from

has

been

quotient shows

many

subtractions

have

been

52

KAY'S

NEW

PKACTICAL

ARITHMETIC.

38.
1st. 2d. 3d.
2. In

1. Division

is indicated
means

in three
12

ways:

3)12,
12

wliieb which which the


a

that that that

is to be divided is to be divided is to be divided the divisor

by by by

3. 3. 3.

^
D

means

12
12

12-^3,

means

using
12,
line draw

first line

sign wiien
under the divisor

does

not

exceed the

dividend, and
exceeds

write
a

quotientbeneath;
on

if the

12, draw

rightof the dividend, and the rightof this. on quotient 3. The sign (-^) is read divided by.

curved

the

place the

Examples.

2)8
4

15)45(3
45

"-"
Table.

21-^3=7.

Division

DIVISION

OF

SIMPLE

NUMBERS.

53

39.
3

If 7 cents

be divided would

as

equallyas possible among


2

boys,each boy
1 cent

receive

cents, and

there would

be

or left, remainingundivided.

The
raainder.
Rem.

number

left

after

dividing,is

called

the

re-

"

1. Since

the remainder

is

part of the dividend, it must


will he

be

of the

same

denomination.

If the dividend remainder

the remainder he dollars,

will he

if pounds,the dollars;

pounds.

54

RAY'IS

NEW

PRACTICAL

ARITHMETIC.

Rem.
were once

2." The

remainder

is

always lens

than

the be

divisor;for,if it
contained
at

the or equal to it, greatei*


more

divisor would

least

in the dividend. 3.
"

Rem.

If

the

dividend

and

divisor

are

simple numbers,

the

operationis

called Division

of Simple

Numbers.

Short

Division.

40.

When the

the result

division

is

and performed mentallj^,


is termed divisor

merely
Short
12.

written, it
when

Short
does

Division.
exceed

Division

is used

the

not

1. How

many

times

is 2 contained

in 468?

Here, the dividend


tens, and
8

is

composed

of three

numbers; 4 hundreds, 6

units; that is.of 400, 60, and 8.


Divisor. Dividend.

Quotient.
is contained
'' ''

Now,

2
2 2

in in
in

400

200 30
4 234

times. times. times. times.

60 8 468

^'

"

Hence,
The
same

in

is contained

result

can

be into

obtained

without

actually

separatingthe
Thus,
in 2 in 4

dividend

parts:

hundreds' write

which

times, which
2. How 3. How 4. How

2 times,which write (hundreds), Dividend, 3 times. Divisor, 2)468 place;then, 2 in 6 (tens), in tens' place;then, 2 in 8 (units), 4 Quotient, 2 34 write in units' place.

many many many many

times

3 in 693?

231. 212. 2341. 2102.

times 4 in 848?
times times
2 4

in 4682? in 8408?

5. How

DIVISION

OF

SIMPLE

NUMBERS.

55

6. How 7. How

many many

times times

3 2

in 3693G? in 88468?

12312. 44234.

41.

1. How

many

times

is 3 contained

in

129?

Solution.

"

Here, 3 is not
12

contained

in 1 ; but write in

is

3)129
4 3

contained

in
in 9

(tens),4 times, which

tens'

place; 3

3 times,which (units),

write in units'

place.

2. How

many

times

is 3 contained

in

735?

Solution. 2

"

Here, 3 is contained
1

in
1

seven

(hundreds),

times, and
times and

hundred 13

over;

the

hundred, united
3

3)735
245
5

with
4

the 3 tens, makes 1 ten

tens, in which

is contained with the

left; this 1 ten, united


3 is contained

makes units,

15 units,in which

5 times.

3. How

many

times

is 3 contained

in

618?

SoLUTiON.-r-Here, 3 is contained

in

times; as the
IS

1 in ten's

placed in ten's
units'

place will place;the 1 ten


units,and

not

2 (hundreds), contain 3, a cipher 6

3)618
206

is then

added

to the 8

units, making

18

the

quotient figure6

is

placed in
4. How

place.
times is 3 contained in 609?

many
solution

Here, the

is the

same

as

in the

above

ex-

3)609
0.
20 3

ample;

there

being no
many

tens, their order is indicated

by
743?

5. How

times

is 3 contained

in

the division of which there is 2 left, dividing, the is merely indicated by placing the divisor under remainder; thus, f. The quotient is written thus, 247f; read, 247, and two divided by three; or, 247, After with
a

3)743
24

7"

re7nainder, two.

56

KAY'S

NEW

PRACTICAL

ARITHMETIC.

6. How 7. How
8. How

many many
many

times times times

3 5 4

in 462? in 1170? in 948?

154. 234. 237.

Rule.

"

1. Wiite curved

the

divisor at

the

leftof

the

dividend,

tvith the each

them^ and draw a line beneath dividend. Begin at the left hand, divide successively and write the result figureof the dividend by the divisor,
a

line between

quotient. remainder 2. If there is a after dividingany figure, lower order,and divide it to the figurein the next 'prefix
in the
same as

order in the

before.
3.

If the number in any order does not contain the divisor, order in the quotient, the prefix place a cipherin the same number and divide as to the figurein the next lower order,
before. 4. If there
is
a

remainder and it,

the after dividing


annex

placethe
Proof.
"

divisor under

it to the

figure, quotient.
add is

last

and Multiplythe quotientby the divisor, the remainder, if any, to the product: if the work will be equal to the dividend. correct,the sum

Rem. that
a

"

This

method is
a

dividend

proof depends on the principle(36, 4] product,of which the divisor and quotientare

of

factors.

9. Divide

653

cents

by

3.

SOLUTION.

TROOF.

217

Dividend.

3 (151=
cents
=

Divisor,

3)653
2 1 7

divided.

Quotient,

"
"53

remainder.

dividend.

DIVISION

OF

SIMPLE

NUMBEKS.

57

PARTS

OF

NUMBERS. into two

Note.

"

When

any

number

is divided

equal parts,one

of

the parts is called

of that number. one-half If divided into three equal parts, one of the parts is called 07ieif into live equal parts, third; if into four equal parts,one-fourth; and so on. one-fifth; of a number, divide by 2; to find one-third^ Hence, to find one-half divide by 3; one-fourth, divide by 4; one-ffth, by 5, etc.

4326.

If. 1687601
.

13541

196855. 4311

7^.

1234753f
754065.
1003634.

1830023-V
54841. 3472834. 24. If
cost oranges for 894 cents? 3 cents

each,
140

how

many

can

be
298..

bought
that
26.
a

25. If 4 bushels

of

applescost
a

cents, how
barrels

much

is

bushel?
If flour cost for $812?

35 ct.

84

barrel,how

many

can

be 203.

bought

58
27. A liow much

liAY'S

NEW

FKAGTICAL

ARITHMETIC.

carpenter
is that
are a

receives month? months

$423

for

months'

work

$47.
in 1 year: how many years 45.

28. There
are

12

there
29.

in 540
are

months?
4

There

are

there 30. At

quarts in 1 gallon: how many gallons 80319. in 321276 quarts? barrels of flour can be how $S a barrel, many
217. weeks 104938. the i)ro45. is 3582
: one
"

bought
31.

for $1736?
are

days in one wxek : how many there in 734566 are days? has been multij^lied number 32. A by 11, and
There 7
.

duct
33.

is 495 The

what

is the
two

number numbers

? of

product of
is 9
:

the 398. 28.

numbers

what

is the

other ?

34. Find 35. Find


36.

one-half

of 56. of 3725. of 147. of 500. of 1945. of 4476. of 2513.


.^ *
'

one-half one-third
one-fourth one-fifth one-sixth one-seventh

1862^.
49. 125. 389.

Find

37. Find 38. Find


39.

Find

746. 359. 749.


977.
r

40. Find 41. Find 42. Find 43. Find


44.

one-eighth
one-ninth one-tenth one-eleventh one-twelfth
144

of 5992. of 8793. of 1090. of 4125. of 5556.

109.

Find

375.
463.
4

45. Find
46.

divided gave

apples equally among


of his share

boys;
:

the

eldest number

boy

one-third

to his sister

what 12.

did the

sister receive? 195

47. James fifth of them:

found Daniel

cents, and
one-third she is did 275

gave of

to

Daniel
to

one-

gave

his share

his 13.

sister: how

many 48. One-eleventh 192?

cents

receive? how much

of

greater than
1.

one-eighth of

DIVISION

OF

SIMPLE

NUMBERS.

59

Long

Division.
work of the division is written

42,

When
it

the termed

entire

down,

is

Long
the

Division.
divisor

Long
12.

Division

is

commonly
1. Divide

used

when

exceeds

3465

dollars

equallyamong
contained
in 3

15

men.

Solution.

"

Fifteen

is not

there will be no thou(thousands) ; therefore, sands in the quotient. Take 34 (himdreds) as is contained in 34, 2 Si partial dividend; 15 15 men have 200 dollars each, times; that ig, which in all 15 X 2 30 hundreds of requires
=

15)3465(231
3 0 hund. 4 6 tens, 4 5 15 1 5

units,

dollars.
Subtract
4 46 30

hundreds

from

34

hundreds, and
down

hundreds
for (tens) 46

remain;
a

to which

bring

the 6 tens, and

you

have

dividend. second partial contains 15, 3 times; that is, each


^
=^

man

has 30 dollars more,

and

all

require15 X

45 tens

of dollars.

Subtract
a

45, and

bring down
more.

the 5 units, which

third

dividend; partial
man

in this the divisor

for gives15 (units) is contained giving once,

to each

1 dollar

Hence,

each
1

man

receives

hundred dollars.

3 dollars,

ten

and dollars,

dollar;that is,231

By
ing

this process, the dividend is separated into parts, each part containdivisor
a

Divisor.
15

Parts.
3000 4 5 0

Quotients
200 3 0 1 231

the

certain

number

of

times. The first part, 30


2

tains the divisor

hundreds, contimes; the second


it 3

15 3 4 65

part, 45 tens, contains


The

times; the
it 1 time. the

third part, 15 units,contains several

parts togetherequal the given dividend, and


up the entire

several

partialquotientsmake

quotient.

60

RAY'S

NEW

PRACTICAL

ARITHMETIC.

2. In

147095

days, how

many

years, each

of 365

days?
years,

Solution."

sands)for the
we

Taking 147 (thoufirst partial dividend,


not

865

)1

4 7005 60

(403
5

14

find

it will
use

contain

the

di-

109

109 5 figures. tracting, Again, after multiplyingand subas in the precedingexample, and bringing down the 9 tens, will not contain the divisor; the partial dividend, 109 (tens), hence, and bring down the 5 units; write a cipher (no tens)in the quotient, which contains the divisor the last partial dividend is 1095 (units),

visor; hence

four

three times.

3. Divide

4056

by

13.

312.

Rule. draw
a

"

1. Flace

the divisor

on

the

leftof
on

the

dividend^ of right
in the

curved

line between

them, and

another

the

the dividend. 2. Find, how many times the divisor is contained the dividend number in that the

fewest lefthand
the and divisor^

of figures place this


divisor

will contain the

at quotient

right.
3.

the Multiply that

by

this

place the quotient figure;


from
which it
was

^product under obtained.


4. Subtract

part of the dividend

this

productfrom
the next all the

remainder divide
as

bring down
until before,

above it; to the figures figureof the dividend,and figuresof the dividend are

the

broughtdown. 5. If at any time,afterbringing down


'

thus

formed
the
as

is too

smcdl

to

contain

cipher in
which

and quotient,

down bring

the number figure, the divisor, place a another figure, after


a

divide

before.
in

Proof.

"

Same

as

Short

Division.

DIVISION

OF

SIMPLE

NUMBERS.

61

Rem. dividend

"

1. The from

product

must

never

be

gr eater

than
so,

the the

which

it is to be

subtracted; when
must

partial quotient
be less

and must be diminished. is too large, figure the remainder 2. After Rem. subtracting,
"

always
the

than last

the divisor; when

the remainder

is not

less than

the divisor,

and must is too S7nall, be increased. quotient figure order of each quotient figure is the same 3. The Rem. as dividend from which it was obtained. lowest order in the partial
"

the

4. Divide

78994

by

319.

62

KAY'S

NEW

PRACTICAL

ARITHMETIC.

21. Divide
22.

48905952 4049160 552160000

Divide

23. Divide

by by by
how
.

9876. 12345. 973.

4952. 328.

567482^VV
acres

24. At

$15

an

acre,

many

of

land 231

can

be

bought
25. If will he 26. If many

for $3465?
a man

acres.

travel

26

miles

day, in

how

many 14

travel 364

miles? divided have?

days days.
how

$1083

be

equallyamong
bushels
acre?
63

19

men,

dollars
man was

will each raised that


on

$57.
of
corn on

27. A how
28.

9523
one

107

acres:

much In
1

89
: gallons

bu.

hogshead there are hogsheadsin 14868 gallons?


29.

how

many 236.

The much The much

President is that
a

receives

$50000

how
30.

day?
from
a

year $136 and

(365 days):
$360
over.
:

how

31. The the

yearly income is that a day? product of two


what million
one

railroad
=:=

is $379600

(365
is the
two

da.

yr.)
;

$1040.
one

numbers

is 6571435

of 5321.

factors is 1235: Divide four

other? hundred and

32.

forty-seven
3080.
one

thousand

hundred
10

by
six.
men

405. hundred and

33. Divide

million and of
acres

four

thousand

by

one

thousand A

34.

colony
109440 each farmer
to

684
:

if

bought a tract equally divided,to


bushels
many of

10338yyg?_ of land,containing
how 160 many
acres.

acres

was

man

entitled? 8288 how


corn,

35. A 56 bushels

raised
acre:

averaging
ho

the

acres

did

plant?
acres.

148 36. The and each of capital into


a

joint-stock company
shares
:

is

$262275,
value of

is divided share?

269

what

is the

$975.

37. The

earth, at

the

equator, is about

24899

miles* in

DIVISION

OF

SIMPLE

NUMBERS.

63

circumference,and
how many miles railroad
cost
an

turns

on

its axis it turn?

once

in 24

hours

hour

does

1037^^.
$3731840:
what

38. A
was

238

miles

long, cost
from before the

the 39. A

per mile? feet distant fort is 27048


w^as seen

$15680.
a

city; the
the sound sound

flash
was

of

cannon
:

24
a

seconds second

heard

how

many

feet

did

travel ? 1127 feet. ute min-

40.
:

Light travels
how many
to reach

at

the

rate

of 11520000 it

miles the

minutes the

does

require for

lightof
miles

the

sun

earth,the

sun

being 92160000

distant?

8 minutes.

Examples

for

TIkvtew. and
divide

41. Subtract

86247

from

94231

the

mainder re-

by
42. 43.

16.

499.
sum

Divide Divide the

the the

of 46712 of 497

and X

6848 583

by

104.

515.
4081.

product

by
and

71. 987 add

44. To and divide

difference
sum

between 87.

2832

678,
29.

the

by

45.

the Multipl}^ divide the

diflPerence between

4896

and

2384

by

49, and
46.

product by

112.

1099.

of 228 + 786 by 95, and Multiplythe sum the product by 114. 47. Multiplythe sum of 478 and 296 by their and divide the product by 387. 48. A
a

divide 845. ence, differ364. sold

horse-dealer of them many

received

S7560

for

horses ; he
rest

part

for S3885 ; if he horses did he

sold

the

for $175 time?


21

how apiece,

sell the

second

horses.

expended at one time $7350 for land,and each time : how at another, $4655, paying $49 an acre did he buy in both purchases? 245 acr^. acres many

49. A

farmer

64

KAY'S

NEW

PRACTICAL

ARITHMETIC.

50. A
a

refiner

bought

58

hogsheads of

sugar,

at

$77

for $5742: how hogshead,and afterward sold them each hogshead? much did he gain on $22. of land,at $26 an 51. A man bought 240 acres acre, giving in payment a house valued at $2820, and horses horses did he give? valued at $180 apiece:how many 19 52. A and for

horses.

of land for $10625, bought 25 acres speculator after dividingit into 125 village lots,sold each lot did he gain on the whole? On each $250: how^ much On each lot?

acre?

$20625.

$825.

$165.

CONTRACTIONS

IN

DIVISION.

CASE

I.

43.
1. A
was

When
man

the divisor

can

be
15

into separated
acres

factors.
much

paid $255
acre?
15

for

of land:

how

that per
"

Solution.

acres

are

Dollars,

times 5 acres;
3

dividing$255 by
value
of 6 5

3)255

the value of 15 the value the value of of


6

acres. acres, acre.

gives $85, the dividing$85


of 1

6)85
1 7

"

acres;

by

gives
instead of
may

$17, the value


The whose factors

acre.

solution shows
are

that

3 and

5, we
obtained

dividingby the number 15, first divide by one factor,then

divide the

thus quotient

by

the other factor.

2, Find
Solution. is 18
twos

the

of 37, divided by quotient

14.

"

and
the

ing by 7,
mainder then

Dividingby 2, the quotient 1 unit remaining. Divid2)37 is with re2, a quotient 7)18
tioos; the whole
remainder

and 1
and

over,

of 4

^^

4 twos

left.

is 4 ttcos

plus 1, or

9.

DIVISION

OF

simplp:

numbers.

65

Rule.

"

1. Divide

the dividend divide the

the divisor; then


other
2.

by one of the factors of thus obtained by the quotient

factor.

divisor ; to the Multiplythe last remainder by the first ivillbe the true remainder ; the amount productadd the first

remainder.
Rem.

"

When

the

divisor

can

be

resolved

into

more

than

two

factors, you
will be

The true remainder successively. may divide by them found by multiplyingeach remainder by all the preceding

divisors, except that which


from firstdivisor.

produced

it.

To

their

sum

add

the

mainder re-

3. Divide
4. Divide

2583 6976 2744 6145 19008 7840 14771 10206 81344 98272

5. Divide
6. Divide

7. Divide 8. Divide
9. Divide

10. Divide 11. Divide 12. Divide

by by by by by by by by by by

63.
32. 28. 42.

132.
64.

72.
81.

121. 108.

CASE

II.

44.

To

divide

bj^1

with

ciphersannexed;

as

10, 100,

iOOO, etc.
To the
one thus, 60. On multiply6 by 10, annex cipher, of multiplicathat division is the reverse tion, principle to divide 60 by 10, cut offa cipher.

Had
manner
sam-;

the dividend
as

will

in might have been separated the cipher;6 being the quotient, 5 the remainder. apply when the divisor is 100, 1000, etc. been

65, the 5

like The

Prac-

66

KAY'S

NEW

PRACTICAL

ARITHMETIC.

Rule."

dividend

offwill

from the right of the figures in the divisor ; the figures there are cut as ciphers the quotient. the other figures, be the remainder,
Cut

off

as

viany

1. Divide

34872

by

100.

CASE

ITT.
are on ciphers

45.
the

To

divide
on

when the

there

the

rightof

or divisor,

rightof
800.

the divisor

and

dividend.

1. Divide
Solution. of the divide
In
two
as

4072

by

OPERATION.
"

Regard
factors

800

as

posed com-

I|00)40i72
8^ 40
5

100

and

8, and
the
mainder, re-

margin. dividing by 800, separate right hand figuresfor the then divide by 8.
77939
Since

in the

Quo...72 Rem.

8100)40172
5

72 Rem. Quo...

2. Divide

by

2400.
operation.

Solution.
24

"

X 100,

cut

off the
as

the figures, then divide

same

equals two right hand dividingby 100;

2400

24100)7

79139(32 1^^1
72

48 by 24. is Dividing by 100, the remainder 39; dividingby 24, the remainder is 11. To find the true remainder, multiply 11 by 100, and add 39 the product (Art. 43, Rule); this is the same as annexing to the last remainder. figurescut oflf,

to

the

DIVISION

OF

SIMPLE

NUMBEKS.

67

3. Divide

62700

by
same

2500.
OPERATION.

25iOO)627|00(25//o"^
Solution.
"

The

as

for

the

50

example

above.

T2T
125
2

Rule.

"

1. Cut

and

in remainingfigures in the divisor. remainingfigures


3. Annex

many 2. Divide the

as

at the light of the divisor^ off the ciphers from the right figures of the dividend.

the

dividend

by

the

the

cut off to figures

the

remainder^which gives

the

triie remainder.

1844"4"A

98.

223jV^. 432^H^. 306/A^V\\687^Wo.

GENERAL

PRINCIPLES

OF

DIVISION.

46.

The
values

value of

of

the

quotient depends
and dividend.

on

the may
:

tive relabe

divisor

These

and changed by Multiplication 1st. The dividend may be

by Division,thus the or multiplied,


the divisor

divisor

divided.
2d. The dividend may

be divided,or

plied. multi-

3d. Both both

dividend

and
same

divisor time.

may

be

or multiplied,

divided, at the

68

RAY'S

NEAV

PRACTICAL

ARITHMETIC.

Illustrations. Let is 4. 24 be
=

dividend,and
4.

6 the

the quotient divisor;

24-^6

If the

will be multiplied dividend,24, be multiplied by 2, the quotient 48; and 48-v-6=:8, which is the former by 2; for, 24X2
=

4, multiplied by quotient, Now,


if the

2.

will be multiplied divisor, 6, be divided by 2, the quotient


=

3; by 2; for, 6 --2 4, multiplied by 2. quotient, Principle be I.


"

and

24-f-3

8, which

is the

former

the divided,

or If the dividend be multiplied, will be multiplied. quotient

the divisor

47.
If the

Take

the

same

example,24

-f- 6

4.
be divided tient, quo-

dividend,24, be divided
24 -=-2
=

will by 2, the quotient


=

by 2;

for

12; and
2.

12-h6

2, which

is the

former

4, divided by

And, if the divisor, 6, be nudtipliedby 2, the quotientwill be divided by 2; for,6 X 2 2, which is the former 12; and 24 h- 12
=

4, divided quotient, Prin. II.


"

by

2.

If

the dividend

be

divided,or

the divisor

be

the quotient will mxdtiplied,,

be divided.

48.

Take

the

same

example,24

-f- 6

4.

If the dividend, 24, and will not be

divisor,6, be multiplied by 2, the quotient


=

the former
And

48 changed; for,24 X 2 4, unchanged. quotient,

; and

6X2

12; 48^-12

4;

if the will not

dividend,24, and
be

divisor,6, be
=

divided,

by 2, the
=

tient quo-

-=4; the
Prin. divided

former

changed; for,24 -f- 2 12; 4, unchanged. quotient,

and

6-f-2

3; 12^3

III.^
"

If
same

both dividend

and

divisor be

by

the

will number, the quotient

or jmdtiplied not be changed.

DIVISION

OF

SIMPLE

NUMBERS.

69

Promiscuous

Examples.
in
:

49.

1. In

bags
the

are

second, $120

; in

$500; third, $55

the what

first, $96;
sum

in the 4th

in

the

bag?
2. Four the second the
men

$229.

paid $1265
more

for land
the did

the

first

paid $243
$79

$61
second:

than much

first;the
the fourth

third
man

less

than

how

pay?
$493.

3. 1 have

five

apple trees:
and 186
are

the

first bears

157

apples;
the

the second, 264; the


123
are :

third,305;

the

fourth,97;
:

fifth,

sell

428,
army

stolen

how

many

apples
332.

left?
4. In
an

of 4794 how

57068
are

men,

9503

are

killed; 586
wounds;

join the
850
are

enemy; drowned: the

prisoners;1234

die of

5. On

first of the

return? 40101. man}the is worth a speculator year he

$12307

during
; in

year

gains $8706
in each

; in

January he
ing remainhe
at

$237 sj^ends
ten

months of the Bible

the

end

February,$301 ; he spends $538: year?


has 31173
verses: verses a

of the had

how

much

$15095.
in how many

6. The

I read 7. I much

it, by reading 86

day?
3

days can 362|^.


for how

bought
each
must

28

horses

for
rest

$1400:
to incur

died;
no

I sell the
times 63
can

loss?

$56.

8. How 5

many

I fill a

15

gallon cask, from


21 times. the 345.

hogsheadsof
9. A

gallonseach?
is

certain 70:

dividend what

73900;
divisor?

the

quotient214;
56

remainder,
10.
ence

is the

of 148 Multiplythe sum ; divide the product by 23. much


8

and

by

their

difier816. it take
at

11. How
to

woolen
at

pay each?

for

horses

cloth,at $6 a yard, will 14 cows $60 each, and

$45

185

yd.

70

RAY'S

NEW

PRACTICAL

ARITHMETIC.

12. 70
an acres

Two
at
:

men

paid
an

$6000
tlie in
a

for other

farm: the

one

man

took
at

S30

acre,
acres

remainder

$25
226.

acre

how income

many is

all ? year. If I

13.

My
for for
can

$1800 $300 $90

spend
for in

$360

year

provisions,
books,
I
man save

for for

rent,

$150

clothing,
how
many
13.

$100
years
14.

and

incidentals,

$10400?

bought
acres

40

acres

of
acre.

ground
He
an

at

$15
90
acres

an

acre, for much

and

80 and he

at

$25

an

sold
acre
:

$4500,
did

the sell

remainder the

at

$60

for did

how
he

whole

land?

How

much

gain?
15. A he
at

$6300.
merchant sold 250
a

$3700.
at

bought yards
:

275
at

3^ards
$5
much
a

of

cloth and

$4

yard;

yard,
did of he stock

the

mainder re-

$6

yard
buys

how 125

gain?
for

$300.

16.

A and

broker 75 stock

shares
a

$85

share,
in he other

shares
at

at
a

$115
share

share.
:

He many

invests
shares

it all does 110.

$175

how

get
17. A

by

the farmer be the

last

purchase?
to
a

sends sold. mules farmer of much the The

dealer
sells

20 the

horses

and

15

mules

to

dealer for

horses

for

$125
for

each,

and
The

$150

each,
50 and head

charging
of cattle the

$95
at

selling.
each,
in with

then money, does

buys

$45

part
how

deposits
in

remainder

bank:

he

deposit

bank?

$2405.

To

Teachers. Whole arithmetic


him

"

While is

placing

Fractions

immediately appropriate
of

after
in
a

Simple higher
convinces

Numbers

philosophical, and pupils, the


for young

for advanced

experience

the

author bers Num-

that,

in

book

learners, Compound
of after, Fractions
"

should

be
reasons:

introduced

before, instead

for the

following
1st. The Division in

operations
of

of

Addition,
are

Subtraction,

Multiplication,and
the
same

compound
and

numbers
serve

analogous
the

to

operations
the damental Fun-

simple numbe/s,
Rules. 2d. The

to

illustrate

principlesof
and

subject
most

of

Fractions

is

important
more

difficult.

Before is of

studying it,
furnished

pupils require elementary rules;

mental is

discipline than
the

by

the

this

acquired by study, do
other

study

Compound
3d.
a

Numbers.

The

general principlesinvolved
of Fractions.
as

in

their

not

require
are

knowledge
are

The

examples
should

involving

fractions exercises

few, and
that

introduced,

they

be, with

in

subject. prefer it,can they


have direct their

Teachers, who
Numbers until

pupils to
as

defer page

Compound
159.

studied

Fractions

far

as

DEFINITIONS.

50.
more

compound
numbers
1

number

is

made

up

of

two ; as,

or

concrete

of

different

denominations

pecks
Rem.
must

quarts
The
to

pint.
denominations of
in the
a

1.

"

different the
same

compound example (71)

number

belong

table;

thus,

given,

the

72

RAY\S

NEW

PRACTICAL

ARITHMETIC.

pecks
are

may

be reduced
of
u

to

quarts

or

and pints, would

the
not

j)arU
Rem. 2.

peck.

pecks

7 dollars

pints and quarts be a compound

number.
in tho resemble simple numbers Compound numbers the denominations of compound numbers respond corfollowingparticulars: certain number of simple numbers, and of to the orders a denomination make unit of the next one units of a lower higher
"

denomination. Rem. in
-5.
"

Most units

compound
of

numbers lower

differ from

this; ten
one

each

denomination

simple numbers do not uniformly System


of

make Rem.

unit of the next


4.
"

In

United

higher denomination. States Money and the


ten

Metric lower

Weights
do make

and
one

Measures, however,
imit of the next

units

of

denomination

higher denomination.

51.

1. The

operationswith

compound

numbers

are

and Bediictlon, Addition, Subtraction, Multiplication,

vision. Di-

2. Reduction

is the number

process

of Thus,
5

without

changing the aUering its value.

of

ination denom-

yards may be changed to feet; for,in 1 yard there are feet;then, in 5 yards there are 5 times 3 feet,which are 15 feet.
3. Eeduction takes

place in
to
a

two

higher denomination
to
a

lower.

ways: 2d. From

1st. From
a

lower

nominati de-

higher.
UNITED STATES MONEY.

5"2. United
States

States

money

is the

money

of the United

of America.

Table.
10
10 10 10

mills, m.,
cents

make
"

1 1 1 1

cent,

marked
"

ct.

dimes dollars

^'

dime, dollar,

d.

"

%.
E.

"

eagle,

"

UNITED

STATES

MONEY.

73

Rem.
in

1.

"

United
The
was

States
first in

money money
The
were

was

established, by
coins first made Gold
silver
are

act

of

gress, Conthe

1786.

coined, by the
made. and

authority of
were

United
cents.

States,
In 1794

1793.

silver dollars
1849.
was

eagleswere
now

copper made

in 1795; standard.
Rem.

gold dollars,in
The
2.
"

Gold

both

legally
bronze,

trade coins

dollar of

minted United
name,

for Asiatic

commerce.

The

the

States

are

classed

as

nickel,silver,and
of each coin
are

gold.

The

value, composition,and

weight

shown

in the following

Table.

Rem. allowed
a

3.

"

deviation

by law
Half

grain in

weight of | a grain to each piece,is in the coinage of Double Eagles and Eagles; of \ of Eagles and the other gold pieces:of 1^ grains in all

in

74

KAY'S

NEW

PRACTICAL

ARITHMETIC.

silver the

of 3 grains in pieces; three-cent pieceand one 4.


"

the
cent.

iivc-cent

piece; and

of 2

grainsin

Rem.

The

mill

is not

coined.

It is used

only in

calculations.

53.
the

1. A

sum

of

money in

is

expressed as

dollars

and

cents, and, when

written

is always preceded figures,

by

dollar

sign ($).
are

Rem.

"

Calculations

sometimes

carried out

to

iness mills,but, in buscent.

transactions,the final'result is always taken 2. A

to the nearest

period (.), called


dollars read and
as

the

decimal

is point,

used

to

separate the
3.

cents. tens

Eagles are
as

of

and dollars,

dimes

are

read

tens

of cents. is read
6

Thus,
4

$24.56
5

24

dollars

56

cents;
is read

not 16

eagles
dollars

dollars
cents 4.

dimes

cents.

$16,375

37

5 mills. the
to figures

Hence,
a,

the

leftof

the decimal the

press point ex-

number

the

decimal

of dollars; the two figuresto number a of cents; and the point,

of right third, figure

to the

mills. right,
"

Rem.

If the number

of cents

is less than

10, a

must cipher

be put

in the tens'

place.

EXAMPT^ES

TO

BE

^VRITTEX.

1. Twelve 2. Six

dollars

seventeen

cents

eight mills.
^

dollars

six cents
seven
,

six mills. mills.


cents

3. Seven
4.

dollars

Forty

dollars

-three fifty
cents.

five mills.

5. Two
6.

dollars three
dollars
two

Twenty

cents

two
cents.

mills.

7. One 8. Two 9. Four

hundred hundred hundred

dollars ten
dollars dollars
two
one

cents.
cent

eicfht mills.

UNITED

STATES

MONEY.

75

EXAMPLES

TO

BE

READ.

$18,625 320.324
$79.05

$ 70.015
$100.28 $150.05

^6.12 $3.06

$ 29.00

$100.03
$ 20.05 $ 40.125

$4.31 $5.43

$46.00

$100.00

REDUCTION

OF

IT. S.

MONEY.

54.

1. As

there there reduce

are are

10

mills times
to

in
as
"

cent, in any
mills
as

ber num-

of cents Therefore
Rule.
annex

10
cents

many

cents

to

mills

"

one

Multiply the cipher.

number

of

cents

by

ten ;

that

is^

2.

to Conversely,

reduce

mills to cents

"

Rule.
one

"

Divide

the iiumber the

of

mills

by

ten ; thcit is^ cut

off

from figure
3. As there

right.
10 10 X
cents

are are

in 1 dime
100 cents
are

and
in 1
100

10

dimes

in

dollar,there
any
as

10

dollar;then,
as

in

number dollars.

of

dollars

there
to

times

many
cents
"

cents

Therefore,

reduce

dollars

to

Multiply the number two ciphers. that is,annex


Rule.
"

of

dollars

by

one

hundred;

4.

to Conversely,

reduce

cents

to

dollars-^

Rule.

"

Divide

the number the

of

cents

by

07ie

hundred

; that

is, cut

offtiro figures from


there
are are

right.
and
in

5. As

10 100 X

mills

in 1 cent

100
1

cents

in 1

there dollar,
in

lOirrrlOOO

mills

dollar;then,
as

1000 times number of dollars there are any dollars mills as dollars. Therefore, to reduce

to

many mitls
"

76
Rule.

RAY'S

NEW

PRACTICAL

ARITHMETIC.

that

Multiply the number three ciphers. is,annex


"

of

dollars

by

one

thousand;

6.

to Conversely,

reduce

mills

to

dollars.

Rule.

"

Divide

the number

of
the

from is,cut offthree figures


55.
made The reduction the

by right.
or

mills

one

thousand

that

of mills decimal

cents

to

dollars may

be

simply with
sum

point. Thus,
"

1st. If the between 2d.

is mills.

Rule.

Put

the decimal the

point

the third and If the


sum

fourth figures from


Rule.
"

right.

is cents. and third

Put

the decimal the

point

betiveen the second

from figures

right.
170 280
m m m m. m.

1. Eeduce
2. Ileduce 3. Keduce 4. Reduce

17 28
43

ct. to ct. to
ct. and

mills.

mills.
6 6
m. m.

to
to

mills.
mills. 10

436 706
ct.

70 106 490
9 14

ct. and
m.

5. Reduce
6. Reduce

to

cents.
to cents.

mills

49 900 1400 10400 6013 4005

ct. ct.
ct.

7. Reduce
8. Reduce 9. Reduce 10. 11. 12.

dollars to cents.
dollars dollars and and 13
to to
cents. cents. cents. cents.

104

ct.
ct. ct.

Reduce Reduce Reduce

$60 $40
375 9004
4

ct. to

5 ct. to

ct. to
ct. to

dollars. dollars.

83.75 S90.04
4000
m. m.

13. Reduce
14. Reduce 15. Reduce 16. Reduce 17. Reduce 18.

dollars to mills.

S14
2465 3007 3187 10375

and'*2 ct.
mills
to

to

mills.

14020

dollars. dollars. dollars. dollars.

S2.46 $3.00

5. 7.

mills to
cents to
to

Reduce

$31.87. $10,375.

19. Reduce

mills

UNITED

STATES

MONEY.

77

ADDITION

OF

U.

S.

MONEY.

56.
dollars 5

1. Add
6 cents

together 4
2

dollars dollars

12

cents

mills;

mills;
87

20

43

cents; 10

dollars

mills; 16
Rule.
"

dollars

cents

5 mills.

1. Write

the

numbers

and

add

as

orERATiox.

in

simple numbers.
2. Place the decimal

$" ct. m.

point in

the

sum

under
7

o fi 9

the decimal Proof.

pointsabove.
The
same as

2 0

!4

3 0

"

in Addition

of Sim-

10.005 16.875

pie Numbers.
2. What 43 is the
sum

$58749T
of 17
19

dollars
8

15

cents

; 23

dollars

cents; 7 dollars
31

cents;
cents cents

dollars

37

dollars 3. Add 7

cents? 18 dollars dollars


4

cents; and 12 S68.45.

mills; 100
8

50

mill; 16 dollars 31 cents 3 mills; and 87 dollars 33


S222.199.
collection: much

cents

mills. had the bills for following and

4. William

$43.75;
was

$29.18; $17.63; $268.95;


be collected? 5. of

$718.07: how
watch

to

$1077.58.
a

Bought

gig

for $200 ;

for $56.93 ; a hat the amount? $2.31: w^hat was clothes


6. A

for $43.87 ; a suit for $8.50 ; and a whip for


a

$311.61.
five hundred and and ^yq four lars dol-

person

six

him, cents; $420.19; one


; $304 ; $888.47
:

has

due

hundred what

dollars
amount

fiftycents
due him? 7. Add
cents

is the

whole

$2222.22.
five dollars mills ;
one seven seven

cents; thirty dollars twenty


dollars five mills ;
one

three
two

hundred hundred

sixty
one

dollars

cents;

dollars

cent

mill; $1000.10; forty dollars

four

mills; and

$64.58

7.

$2000.

78

HAY'S

NEW

PRACTICAL

ARITHMETIC.

SUBTRACTION

OF

U.

S.

MONEY.

57.

From

one

hundred

dollars
cents

five

cents
seven

threo milla

tuke mills,
Rule.
as

eighty dollars twenty


1.

and

"

Write

the

numbers

and

subtract
operatiox.

in

Simple

Numbers.

2. Flace

the decimal

point in the remainder

$.

et. m.

under

the decimal The

pointsabove.
same as

100.053 80.207

Proof.

"

in

Subtraction

of

$19,846

Simple

Numbers. take take take


3

2. From

$29,342 $46.28

817.265. $17.75. $5.50.

$12,077. $28.53.
$14.55. $2.97. $9,999.

3. From
4. From

$20.05

5. From
6. From

7. From 8. From
cent

$3, take $10, take $50, take


one
one

et.

1 mill. 50
ct. 5

mills.
one

$49,495.
dollar
one

thousand

dollars,take
43

and

mill.
1000

$998,989.
dollars

9. B

owes

cents; if he pay
much
""

nine

dred hunstill

dollars owe?

cents, how sixty-eight

=^^%e

$99.75.
MULTIPLICATION
OF

U.

S.

MONEY.

58.
5 mills

1. What each?

will 13

cows

cost,at 47 dollars 12

cents

in Simple Numbers. as Multiply 2. Put the decimal point in the same place in the product^ as it is in the multiplicand.
Rule.
"

1.

operation.

$47,125

1_3
141375 47125

Proof.

"

The

same

as

in

of Multiplication

Simple Numbers.

$612,6

2 5

UNITED

STATES

MONEY.

79

2.

3.
4.

5.
6.

7. 8. 9.

MultiplyS7.835 by 8. Multiply $12, 9 et. 3 m. by Multiply $23, 1 ct. 8 m. by Multiply $35, 14 ct. by 53. Multiply $125, 2 ct. by 62. Multiply $40, 4 ct. by 102. Multiply 12 ct. 5 m. by 17. Multiply$3.28 by 38.
cost

$62.68
9.

$108,837, $368,288 $1862.42, $7751.24. $4084.08, $2,125, $124.64

16.

10. What
a

338

barrels of cider, at 1 dollar 6 cents

barrel?
11.

$358.2B.
38 cords did
a

Sold
to

of

wood,

at

dollars

75

cents

cord:

what 7 ct.

it amount?
cost

$218.50.
465

12. At

pound, what
of

pounds

of sugar

multiplying7 cents l)y465, mulproduct,Art. 30. tiply465 by 7, which givesthe same remember that 7 cents But, to placethe decimal point,
Note.
"

Instead

operation

4 6

.0 ?

is the true

multiplicand.
cost

$ 'S^2.61

13. What

89

yards yards

of

at sheeting,

34

ct.

yard?
$30.26.

14. What

will 24

of cloth

cost,at $5.67
$2.69 each
:

yard?
is

$136.08.
15. I the value have 169

sheep, valued
bushels to? of

at

what

of the

whole?

$454.61.

16. If I

sell 691

wheat,

at

$1.25

bushel,
$863.75.

what

will it amount sold 73

hogsheads of molasses, of 63 gallons $2529.45. each, at 55 ct. a gallon:what is the sum? 18. What cost 4 barrels of sugar, of 281 pounds each, $73.06. at 6 cents 5 mills a pound? 19. Bought 35 bolts of tape, of 10 yards each, at 1 $3.50. did it cost ? cent a yard : what 11 hours a day, 20. If I earn 13 ct. an hour, and work
17. I how much
will

earn

in

312

days?

$446.16.

80

KAY'S

NEW

PEACTICAL

AKITHMETIC.

21. I

sold

18

bags
what 150

of

wheat, of
amount?
of

bushels

each,

at

$1.25
22.

bushel:
cost

is the
acres

$67.50.
10

What

land, at
51

dollars

1 mill

per

acre?
cost

81500.15.
17 per of bags of coffee, pound?

23. What 24
cents

pounds each, at
$214.14
9.

7 mills

DIVISION

OF

U.

S.

MONEY.

59.
money 1. How

Case

I.
"

To

find in

how

many

times

one

sum

of

is contained much

another. 7 cents
a

at cloth,

yard,will $1.75 buy?


OPEllATIQN.

Solution."
times in 175

Aij many yards as 7 cents 25. are cents, which

is contained

7)175
2 5

Rule.

"

1. Beduce

both

sums

of

money

to the same

denom-

ination. 2. Divide 2. How for 72


as

in

SimpleNumbers.
rice,at
9 cents
a

much

pound, can
and

be

bought
8

cents? many

lb.

3. How
can

towels,at
for $6?

37

cents

5 mills

apiece,
16.

be

bought

4. How be

yards of many bought for $2.80?


many

calico,

at

cents

5. How
can

yards

of

ribbon, at

25

cents

purchased for $3? how 6. At $8.05 a barrel, many $161 purchase? 7. At 7 cents 5 mills each, how bought for $1.20?
8. At
can

be

can 3^ard, 35 yd. a yard, 12 yd.

barrels

of

flour will
20

bl. be 16.

many

oranges
"

can

$1,125 per

bushel, how
$234?

many

bushels

of wheat 208

be

purchased for

bu.

UNITED

STATES

MONEY.

81

Case number

II. of

"

To

divide

sum

of

money

into

given

equal parts.
worked
3

1, A

man

days

for

$3.75, what

were

his

daily wages?
OPERATION

Solution.

"

His

daily wages

were

$3.75

-^

r=:

$1.25.

) 3.7 5 $T25
much

2. A
a

farmer

sold 6 bushels

of wheat

for $d

how

bushel

did he

get?
got for eadi
a

Solution. vided

"

He

bushel with

$9"6.
a

$9 di-

operation.

by
300

gives

quotient $1,
cents

remainder quo-

6)9.00
$1.50

$3

cents.

300

divided

by

gives a

tient 50 cents.

Rule.

"

1. Divide the decimal

as

2. Put
as

Simple Nvmbers, in point in the same j^lace


in

the

quotient

it is in, the dividend,


Kem. 1." If
two
a

the

dividend the

is

dollars,and

the division for cents;

not

exact,
if
essary, nec-

annex

ciphers after

decimal

point

and,

third 2.
"

cipherfor
there

mills.
be
a

Rem. it may

Should

remainder

after

be indicated

by

the

sign -f placed after

obtainingtlie mills, the quotient. S8.125.


of wheat:

3. Divide 4. A

65

dollars
received that

equallyamong
$29.61
bushel? for 23

8 persons.

farmer
was acres

bushels

how

much

per

$1,287 H-.
much is that
an

5. If 4 acre?
6. Make

of land

cost

$92.25, how

$23,062+.
an

equal division
received

of $57.50 among

8 persons.

$7,187+.
7. A much
was man

$25.76

for

16

days' work:

how

that

day?

$1,61.

82

RAY'S

NEW

PRACTICAL

ARITHMETIC.

bought 755 bushels of applesfor $328,425: what $0,435. did they cost a bushel? much is that a day, 9. My salaryis $800 a year : how there being 313 working days in the year? $2.555-|-.
8. I

10. Divide
men:

ten

thousand man's

dollars
share? farm and of

equally among
154
acres,

133

what
man

is each

$75.187 -f.
for two
1 cent:

11. A

purchased a
hundred

thousand
what

seven

five

dollars

and

did it cost
15

per acre?

$17,565.
25

12. I sold

kegs
Avas

of

of butter,
a

pounds each, for


16 ct. 235

$60:
13.

how I

much

that

pound?
of cost?

bought

barrels did
1

of sugar,

pounds each,
$0,065.

for $122.20: what

pound

Promiscuous

Examples.

60.
2. A

1.

oAve sum

A do
;

$47.50; B, $38.45; C, $15.47; D,


I owe?

$19.43: what
owes more:

$120.85.
as

$35.25
what
me

B,

$23.75 ; C,
amount?

much

as

and

B,

and

$1

is the

$119.
twice
as

il A

paid
B:

$18.38;B, $81.62; C,
much market did I receive? w4th

much

as

and

how
to

$300.
75

4. I went

$5 ; I spent for butter


left?
a

cents, for eggs


$1.50: how
5. A for
a

35 cents, for money $20; she


was

50 cents, vegetables

for flour

much

$1.90.
dress 25 for

lady had $5.65,eight yards


for $4: what

bought
was

$8.10,shoes
a

of delaine
sum

at

cents

yard,and
25
ct.

shawl
6. I

left?

get $50
will I have farmer

month,
his

and

spend

$30.50

of

it: how

much 7. A sugar

left in 6 months?

$117.
for $21.75

sold for

marketing
$1.25, for
he left?

$3.85,
how

tea

coffee

paid for $2.50,for spices


:

he

SI .50:

much

had

$12.65.

UNITED

STATES

MONEY.

83

8. T and do E I
9.

S37.06; B, $200.85; C, S400; D, $236.75, $124.34; my property is worth $889.25 : how much
owe

owe

more

than

am

worth? of
was

$109.75.
23 cents
a

after

Bought 143 pounds paying $12.60, what


owed
a me

at coffee,

pound :
$20.29.
of corn,

due? 435

10. A at

$400:

he

paid me
sum

bushels

45

cents

bushel:

what

is due?

$204.25.

spend 65 cents a day, how much will he save in 365 days, his income $162.75. being $400? of 3 bushels 12. Bought 21 barrels of apples, each, at did they cost? 35 cents a bushel: what $22.05. 13. What cost four piecesof cambric, each containing 19 yards,at 23 cents a yard? $17.48. 14. If 25 men perform a piece of Avork for $2000, and spend, while doing it,$163.75,what will be each man's share of the profits? $73.45. receive $516 for 43 15. If 16 men days' work, how much does each man 75 ct. earn a day? 16. C earned $90 in 40 days,working 10 hours a day:
how much did he
earn an

11. If B

hour?

22

ct. 5

m.

17. A

merchant

$500
how

in much

cash, to

be

has failing, goods worth equallydivided among receive?

$1000,
22

and

creditors:

will each

$68.18-|-.

MERCHANTS'

BILL8.

Bill
or

or

Account,

is

written

statement

of

articles

bought
18.

sold,with
9
4

their

and prices,

entire cost.

Bought

45

17 "What

at pounds Coffee, pounds Tea, pounds Sugar pounds Cheese


" " "

$0.32 per lb.


1.25 .09 .20 do.

do. do.

is the amount

of my

bill?

$15.33

84 19.

RAY'S

NEW

PRACTICAL

AKITHMETIC.

Bought

22

18 25
6

yards Silk, yards Muslin, yards Linen, yards Gingham,

at
''

$1.75 per
.15 .65 .18

yd. S

do. do.

'-

"

do.

21.

Bought

43

yards Muslin, 28"^5iiaTd8 Calico, 23 yards Alpaca,


whole
amount?

at
'-

$0.13 per
.09 .23

yd.

do. do.

''

What

is the

$13.40

REDUCTION

OF

COMPOUND

NUMBERS.

DRY

MEASURE.

61.

Dry

Measure

is used

in

measuring grain,vegetables,

fruit, coal,etc.
Table.
2 8
4

make pints(pt.)

quart, marked

qt.

quarts

"

1
1

pecks
The

''

peck, bushel,
Measure

''

pk.
bu.

'"

Rem.

1.
"

standa7'd iiniiof

Dry

is the

bushel; it is a
tains con-

measure cylindrical

18^
inches.

inches

in

diameter, 8 inches deep,and

21501

cubic

REDUCTION

OF

COMPOUND

NUMBERS.

85

Rem. sold

2. "When

articles

usuallymeasured
is taken bushel
as

by

the The

above

table

are

the bushel by weight, givesthe legalweight of a

the unit.

of various

followingtable articles in avoirdupois

pounds:

To

Teachers.
to the

"

Numerous

should questions

be asked

on

each

table

similar

following:

1. How 8? In
2. How

many 10? many many 17? In

pints in
quarts
in

quarts?
3
9

In

4?

In

6?

In

pk.?
bii.?

In In

5? 11?

In

7? In

In 13?

9?

3. How 15? In

pecks
19?

in

In

4. How

18?

In

many In 25? many 48? many many In

quarts
56?

in

10

bu.?

In

12?

In

14?

In

5. How 40? In

pecks
64? bushels

in

16

qt.?
32

In

24?

In

32?

In

6. How

in

qt.?
In

In
2 ?

64?

In 5?

96?

7. How

pints in

1 bu. ?

In

St)

KAY

NEW

PRACTICAL

ARITHMETIC.

62. The
To

precedingexamples show
quarts
number
to

that

"

reduce the

quarts by
To
in the

multiplythe pints, of pintsin a quart.


bushels
to

number

of

reduce

pecks to quarts, or
same

ply pocks,multi-

manner.

Hence, to reduce multiply by the


of the

from

higher to

lower

tion, denominaone

number

of units that make

unit

denomination. required They also show that of pints To reduce pints to quarts,divide the number of pintsin a quart. by the number To reduce divide quarts to pecks,or pecks to bushels,
"

in the

same

manner.

Hence,
of the

to

reduce

from

lower

to

tion, higher denominamake


one

divide

of units by the number requiredhigher denomination.


3

that

unit

1. Eeduce
Solution.

bushels
reduce
4

to

pints.
OPERATION.

"

To
are

bu. to in
1

pk. multiplyby 4,
bu.,
or

^ bu.

because
many

there

pk.
8

times

as

pk. as

bu.

To
are

reduce

8, because
to 1

there

pk, to qt.multiplyby qt. in 1 pk. To reduce qt.


there
are

f2pk.

^
2

pt. multiplyby 2, because qt. 2. Eeduce


Solution.

pt. in

foTpt.
192
To 2

pints to bushels.
OPERATION.

"

reduce

pt. to

qt. divide
To reduce

by

2, beTo
4

2)192

pt.

cause

there

are

pt. in 1 qt.

qt. to pk.

divide reduce

by 8, because there are 8 qt. in 1 pk. pk. to bu. divide by 4, because there are
1 bu.

8)96 qt. 4)12 pk.


3 bu.

pk. in
The
a

two

precedingexamples show
a

that

reduction
a

from
a

higherto

lower

denomination^and from
each other.

loicer to

denomination, higher prove

REDUCTION

OF

COMPOUND

NUMBERS.

87

3. Eeduco

7 bu. 3

6 qt. 1 pt. j)k.

to

joints.
OPERATION.

bu.

pk. qt. pt.


3 6 1

Multiply the bu. by 4, making 28 pk.,and add the 3 pk. Then multiply the 31 pk. by 8 and add the 6 qt.; multiplythe 254 qt. by 2 and add the
Solution.
"

_4
3 1
z.

pk. in qt.in
pt. in

7 bu. 3

pk

pt.; the

result

is 509

pt.

2 5 4 2 5 0 9

31

pk. 6 qt.
whole.

the

4. Eediice

509

pt. to

bushels.

Solution.
2, and
the there remainder

"

To

reduce

pt. to qt. divide


the dividend reduce is

by
pt.

operation.

is 1
must

left;as
be

pt. To

qt. to

2)509 8)254
4)31

pk. divide by 8, and 6 qt. are left. To reduce pk. to bu. divide by 4, and 3 pk. are left. The therefore,7 bu. 3 pk. 6 qt. 1 pt. answer is,

qt. 1 pt. pk. 6 qt.


7 bu. 3

pk.

03,
I.
FROM A

RULES

FOR

REDUCTION.

HIGHER

TO

LOWER

DENOMINATION.

1. her

denomination Multiplythe highest the next


to

of

lower

which

makes the

by that num given, unit of the higher.


the lower

2. Add

the

product
like
manner

number, if any, of
icith the result thus

denomination.
3. Proceed

in

obtained,

tdl the u'hole is reduced

to the

denomination. required

II.

FROM

LOWER

TO

HIGHER

DENOMINATION.

1. Divide denomination
2. Proceed

the

given quantity by that which makes a unit of the


in like
manner

number
next

of

its

own

tillthe whole

is reduced

to

higher. thus obtained, with the quotient denomination. the required

88

KAY'S

NEW

PKACTlCAl^

AKITHMETIC.

3.

The

last

with quotient,
answer.

the several

remainders^ if any,

annexed, will be the


Proof.
answer
"

Eevcrsc the

I he

operation: that
from
as

is, reduce
it
was

the

to

denomination
same

which

derived.

Jf this result is the is correct. 5. lieduce


6. Eeduce 4

the

the quantitygiven,

work

bu.

7 bu. 3 3 bu. 1 384 47 95


508

7. Reduce 8. Reduce 9. Reduce 10. Reduce 11. Reduce

pk. 1 qt. to pints. pk. 7 qt. 1 pint to pints. pt. to pints.


2

290
511

pt. pt.
pt.

193

pt. to bushels. pt. to pecks. pt. to bushels.


pt. to bushels.
LIQUID
MEASURE.

(" bu.

pk. 7 qt. 1 pt. bu. 1 pk. 7 qt. 1 pt. 7 bu. 3 pk. G qt.

64.

Liquid

Measure

is used

for

measuring all liquids.

Table.
4 2 4
Kem. contains
The 231

(gi.)make gills pints


^*

1 1 1

marked pint,

quart,

'"

pt. qt.

quarts
standard cubic
unit

"

gallon,

"

gal.
is the

"

of

liquidmeasure

gallon,which

inches.

136 pt gal.to pints. 13 gal.to gills. 2. Reduce 416 gi. 3. Reduce 126 gal.to pints. 1008 pt 4. Reduce 1260 gal.to gills. 40320 gi. 5. Reduce 1120 gi. to gallons. 35 gal. in 1848 cubic inches? 6. How 8 gal. gallons many 7. How gallons in a vessel containing138138 many cubic inches? 598 gal. 17

1. Reduce

REDUCTION

OF

COMPOUND

NUMBEKS.

89

AVOIRDUPOIS

WEIGHT.

65.

Avoirdupois
articles.

Weight

is

used

for

weighing

all

ordinary

Table.
16 100 20

ounces

(oz.)
or

make
"

1 1 1

pounds
cwt.,
2000

pound, hundred-weight/'
ton,
" ^

''

lb.
cwt.

lb.,"

T.

Rem.
is

1.

"

The

standard

determined Rem.
2.
"

from
At
a

the
ton.

avoirdupoispound of the United States the Troy pound, and contains 7000 grainsTroy. 2240 pounds House Custom trades) (and in some

are

considered

1. Reduce
2. Eeduce

2 3 1 3 4

3. Keduce
4. Eeduce

5. Eeduce
6. Eeduce

5
2 2

pounds. cwt. 75 lb. to pounds. T. 2 cwt. to pounds. T. 75 lb. to pounds. 44 lb. to pounds. cwt. T. 90 lb. to pounds.
cwt.
to

200. 375. 2200. 6075.


444.

10090.
4444.

7. Eeduce 8. Eeduce 9. Eeduce 10. Eeduce 11. Eeduce 12. Eeduce 13. Eeduce 14. Eeduce 15. Eeduce 16. What
18

cwt. cwt.

77 17

lb. 12 lb. 3
4

oz. oz.

to to
oz.

ounces. ounces.

3475. 41666.
48 11 cwt. 3

1 T. 6 cwt.

lb. 2

to

ounces.

4803 22400

lb. to cwt. lb. to


oz. oz. oz. oz.

lb.

tons.

T. 4 cwt.
64

2048000 64546 97203 544272


is the

to

tons.

T.
oz. oz.

to

cwt.

40

cwt. 3

34

lb. 2 lb. 3 T. 17

to tons.
to tons.

T. 75 17

lb.

total

weight
of

of 52

each parcels,
9 cwt.

taining con-

lb.? is the 75

36

lb.

17. What

weight

180

iron

each castings,
6

weiffhin^

lb.?

T. 15

cwt.

90

KAY'S

NEW

PRACTICAL

A KIT

IIMET

I C.

LONG

MEASURE.

66.
or

Long

Measure

is used

in

measuring distances,

in lengtli,

any

direction.

Table. 12
3

inches
feet

(in.)

make
'"

1 1 1 1

foot,

marked
'-

ft.

yard,
rod,
mile,
the

yd.
rd. mi.

5^ yards,or
320
Rem.
"

16^ feet,

'*

''

rods
The the standard United is unit States of is

"

"

lengthis

yard.

The

standard A
""("))"

yard

for

of this standard 1. Reduce

kept

at each

preserved at Washington. state capital.


in. to inches.
to

2 7
12

2. Reduce 3. Reduce
4. Reduce

yd. 2 ft. 7 yd. 11 in.

103 263 3840 2480


2911

in. in. rd. rd. rd.

inches,

mi. to rods. rd. to rods. rd. to rods.

7 mi. 240
9

5. Reduce 6. Reduce 7. Reduce


8

mi. 31 in. to in. to

133 181 2240 2200

yards.

yd.
5

ft. 1 in.

yards.
miles. miles.

yd.
280 1760

1 in. 7

Reduce

rd. to rd. to
to to

mi. rd.

9. Reduce

6 mi.

10. Reduce 11. Reduce

1 mi. 1 mi.

yards.
feet.

y(\.
fL

5280

SQUARE

MEASURE.

67.
which

Square
has both

Measure

is used

in

measuring
; that

any

thing
dimen-

length and

breadth

is,two

figurehaving

equal sides

and

rightanglesis

square.

REDUCTION

OF

COMPOUND

NUMBERS.

91

A in A A

square

inch

is

square,

each

side of which

is 1 inch

length.
square

foot is

square,
a

each

side of which each side of

is 1 foot. which is 1

square

yard

is

square,

yard (3 feet).
Suppose the figureto represent square yard. It will then be 3
each and 9 contain way, foot will he 12 feet. Each each way, and contain
144

feet square inches square The any

inches. number of small squares in

is, therefore, large square number the of units in equal to side multipliedby itself. one

Rem. which squares in area

"

By
each

feet
foot
a

square 9 square

is

meant

square
3

each figure,

side of
3

is 3

feet,or
one

feet; but by
one

square

feet

is meant

long and

foot

the difference wide; therefore, and


one

between square

figureZfeet square

containing 3

square

is 6 feet,

feet.

Table. 144
Q 9

square
finnar.^

inches
fonf feet

make
"i

1 square
1 1
Qmior^o

foot,marked
a7"it1 yard,
"

sq. ft
en

square square square


acres

square

sq.

yd.
^',

30J
'^" 160

yards
rods

1 square 1 acre, 1 square

rod, mile,
'"

sq. rd. A. sq. mi sq. in. sq. rd. sq. rd. sq. in. sq. rd.
6

640

1. Eeduce 2. Reduce 3. Eeduce


4. Reduce

sq. 4 A.

yd.
to

inches. square rods. square


to
to

10368 640 102400 3024 900

1 sq. mi. 2 sq. 5

square

rods.

yd.
100

3 sq. ft. to sq. in.

5. Eeduce
6. Eeduce

A.

960

sq. rd. to sq. rd. sq. rd. to acres.

A.

92

KAY'S

NEW

PRACTICAL

ARITHMETIC.

7. Eeduce 8. Eeduce
9. Reduce

sq. in. to square 20000 sq. rd. to acres. 3888 515280 sq. rd. to square

yards.
miles. A.

sq. 125

yd.
A.

5 sq. mi. 20

80

10. Eeduce

4176

sq. in. to is See

sq.

yd.

3 sq.

yd.

sq. rd. 2 sq. ft.

68.
four The side is The number

Rectangle

right angles.
unit
a

the

figurehaving figurebelow.
is surfaces,
a

four

sides and

of

measure

for
a

square

whose

linear
or

unit;as

square

inch,a
of
a

square

etc. foot,

Area

contents Superficial

is figure,

tlie

of times

it contains

its unit

of

measure.

1. How

and

square many 3 inches wide?

inches

in

board

inches

long

Explanation." sides into 4

Dividing each

of

the

longer
o

equal parts, the shorter sides into equal parts, and joining the oppositedivisions by straightlines,the surface is divided into
squares.
In

each

of the

longerrows
as

there inches
are

are

4 squares,

that
as

is,as many

there
as

are

in the

longerside;and
is

there

are

such rows many The whole number

there

inches

in the shorter side.

Hence,

of squares

in tbe board

obtained

by multiplying together the 4 X 3 12. length and breadth; that is,


=

numbers

equal to the product representingthe

Rule

for

Finding

the

Area

of

Beetangle.
be the

"

Multiply
area.

the

lengthby the
"

breadth ; the

product uill
not

Kem.

Both

the

length and
made

breadth, if
so

in units

of

the

same

denomination, should, be
2. In

before

multiplying.
feet

floor 16

feet

long

and

12

wide,

how

many 192.

square

feet?

REDUCTION

OF

COMPOUND

NUMBERS.

93

3. How
room

many

yards
many
18
one

square long and square feet 15

will yards of carpeting 4 yards wide?

cover

20.

4. How rooms,

will cover two yards of carpeting long and 12 feet wide, the other 21 ? 59. in
a

feet

long and
14

feet,wide square

5. How and

many feet wide?


a

yards long and


cost to

18 ceiling

feet

long
28.

6. In acres? 7. How 18 feet

field 35 rods

32

rods

wide, how^

many 7.

much

will it 15 feet

per
and

long and yard? square


will 18 feet

each carpet two rooms, wide, if the carpet costs $1.25

$75.
21 a plaster ceiling cents yard? per square to

8. What

it cost 17

feet

long

wide, at
Area of
a

S7.14.
the

69.
of of
two

The the

Eectanglebeing equal to
the

length by

breadth, and

numbers,

divided

by

either

of

product the product as them, gives the

other

(36, 4); therefore,


"

Rule.

If the

area

of

he rectangle

divided

by either side,

will he the other side. the quotient

Illustration.

Example 1, 68, if the area the quotient3 is the width; or, 4 is the length. quotient
Dividing the area the number really dividing
"

In

12

be

divided
12

by 4,
the

divide

by 3,

Rkm.

of

by rectangle

one

of its sides,is
ber num-

of squares

on

one

of squares in the of its sides.


latter is not

the rectangle"by

In

12 by 4, the dividing inches in


a

4 linear in.

inches,but the
1 in. wide.

ber num-

of square

rectangle4

long and

See

Art. 68. figure,

94

RAY'S

NEW

PRACTICAL

ARITHMETIC.

1. A what 2 what

floor is its

containing132

square

feet,is
30

11 feet wide: 12 il.

length?
and long, contains square

A. floor is 18 feet
is its width? field

yards:
15 ft.

3. A what
4. A

containing9
rods

acres,

is 45

rods

in

length:
32 rd. is its
96

is its width? field 35

wide, contains

21

acres

what

length?

rd.

SOLID

OR

CUBIC

MEASURE.

70.

Solid

or

Cubic

Measure

is used and

things having length,breadth,


three dimensions.

measuring thickness; that is,

in

Cube

is

solid, having

equal faces,which

are

squares.

Rem.

"

If each

side of

cube is 1 inch

it is called a cubic inch; if each side long, in is 3 feet (1 yard) long, as represented the figure,. it is a cubic or solid yard. The base of a cube, being 1 square yard,contains 3 X '^ 9 square feet;and 1 foot high on this base,contains 9 solid 18 solid feet;2 feet high contains 9 X ^ 27 solid feet;3 feet high contains 9X8
= = =

feet.
or

Also, it may

be shown
12 X

that
12

1 solid
=

cubic foot contains

12 X

1728 solid

or

cubic inches.

Hence, the number of small cubes in any largecube,is and thickness, equalto the length, breadth, multiplied together.
Rem.
"

whose Any solid,


and

corners

resemble

cube,is

rectangular

boxes solid;

cellars are

of generally

this form.

KEBUCTION

or

COMPOUND

NUMBERS.

95

The the

solid are found, as in rectangular the length, breadth, and together cube, by multiplying solid contents

of

thickness.

Table.
1728 cubic inches
feet

(cu.in.)
=

make
"

1 cubic 1 cubic
,

marked foot,

cu.
cu.

ft.

27 cubic 128

j^ard,

"'

yd.

cubic feet
4

8X4X4 ft. wide, and


=

8 ft. long,")^

4 ft. high, make)

^^1cord,

^^ "

C.

Hem.
a

1.

"

cord

foot is 1 foot in feet

length of

the

cord.

It is 4 feet wide, 4
16 cubic 2.
"

high, and
mass

which makes pile foot long; hence, it

contains Rem.

and feet,

8 cord

feet make
a

1 cord.

perch

of stone

is

16J

ft. long,1^ ft. wide, and

1 ft. high,and

contains

24J

cu.

ft.

1. Reduce 2. Reduce 3. Reduce 4. Reduce 5. Reduce 6. How


'

cu.

yd. to

cubic inches.
to
to
cu. cu.

93312 ft. in. 3584 7520256 221184

cu. cu. cu. cu.

in. ft. in. in. ft. ft. ft.

28
34

cords cords

of wood of wood
to

1 cord

of wood
cu.

cu.

in.

63936 many

in. to

cu. a

cubic

feet in

yd. 1 cu. yd. 10 cu. solid, 8 rectangular


160
cu.

long,5
7. How

ft. wide, 4 ft. thick? many cubic


2

yards

of excavation

in

cellar 8
cu.

wide, yd. long,5 3^d.


8. How feet many feet cubic

yd. deep? 18 yards in a cellar,


40

80 feet

yd.

wide, 7
a

deep?
wood feet

long,15 70 cu. 3'd.


8 C. feet
30

9. In

pileof

long,12
a

feet wnde, and

feet

high, how
feet

10. What

many will be

cords? the
4 cost

of

pile of

wood
per

80 cord?

long,8

high,and
will be the

feet

thick,at S5.50
of

$110.
11. What
cost

long,15
or

ft. wide, -and

excavating a cellar 24 ft. ft. deep, at $1.25 per cubic yard


$100.

load?

96

RAY'S

NEW

PRACTICAL

ARITHMETIC.

TIME

MEASURE.

71,

Time

Measure

is used Table.

in

measuring time.

60 60 24 365 100

seconds
minutes

make (sec.)
''

1 1
1

minute, marked hour,


''

min. hr.
da. yr.

hours

''

day,
century week,

''

days,6
years 7

hours

'^

1 year,

^'

1 1

''

cen.

Also,

days
months

make

marked

wk.
mon.

4 weeks

1 month
1 year,

(nearly),
'^ ''

12 calendar 365 366


Rem.
365

yr.

days days
"

1
1

common

year.

leapyear.
moan

1.

The

exact

lengthof

the

solar,or

tropical year,

is

days,6 hours, 48 minutes, 46 seconds. To correct the error of considering365 days as the lengthof the rule has been adopted: year, the following is not divisible by 4 consists of 365 Every year whose number days. is divisible by 100, but not by 400, Every year whose number consists of 365 days.
whose number is divisible centuries, Every year, except the even centuries divisible by 400 consist of 366 days. by 4, and the even The year containing 366 days is called Leap year, and the extra day is added to February, givingit 29, instead of 28 days. Rem. 2. Among nearlyall civilized nations the year is divided
"

into 12 calendar

months, and numbered,

in their

order, as follows:
7th month,
8th
**

January, February,
March,

1st

month, 81 days.
"

July,
August,
October,
December,

31
31 30 31 30 31

days.
" "

2d 8d 4th 5th 6th

28
31

"

"

"

September, 9th
lOth

"

April, May,
June,

"

30
31 30

'"

"

"

"'

''

November, 11th
12th

"

"

"

"

"

"

REDUCTION

OF

COMPOUND

NUMBERS.

97

1. Eeduce 2. Eeduce 3. Reduce

hr. to seconds.

7200 10080
3
sec.

sec.

7 da. to minutes.
1

min.

da. 3 hr. 44

min.

to

seconds. 99843
sec.

4. Reduce

wk.

da.

10

hr. 40

min.

to

minutes. 100000 min.

5. Reduce

mon.

3 da. 4

min.

to minutes. 44644

min. 3 hr. 5 da.

6. Reduce 7. Reduce 8. Reduce


9. Reduce

10800
432000

seconds seconds seconds minutes minutes min.


to

to to

hours.

days.
2 hr. 2 min.

7322 4323 20280

to hours. to to

sec.

days.
weeks. 1
mo.

3 da. 3 min. 2 wk. 2 hr.

10. Reduce 11. Reduce

41761

months.

1 da. 1 min.

MISCELLANEOUS

TABLES.

I. MEASURES

OF

WEIGHT.

72.
and 24
20 12
The

Troy

Weight

is used

in

weighing gold, silver,


pwt.
oz.

jewels.
grains (gr.)
make
"

pennyweight, marked
'^

pennyweights
ounces

1 ounce, 1

"

pound,
in

'^

lb.

Standard

Unit

of all

pomid, containing5760 Apothecaries


medicines.
20

weight grains.
is

the United

States is the

Troy

"Weight

used

only

in

compounding

grains (gr.)make scruples


'' '^

1
1

marked scruple,

9.
3. 5.

dram,

"

8 drams
12
ounces

1 ounce, 1

''

"

pound,

"

lb.

98

RAY'S

NEW

PKACTICAL

AKITHMETIC.

The

followingare

also used

: by apothecaries

60 minims

make (ordrops)rT\^.
"

1 fluid

drachm, marked
"

f. 3. f. ". O. cong.

8 fluid drachms 10 fluid


ounces

1 fluid ounce, 1 1

'*

pt. (octarius)

"

pints

''

gal.(congius)

"

II. MEASURES

OF

LENGTH.

The
of them

measures following are

are

often

mentioned

and

most

still used

in

special professions:
3 6 3

12 3 4 9

lines

=1
=

inch.
1 inch 1 hand. 1 span.

feet feet miles

1 pace. fathom.
1

barleycorns
inches inches
= =

=1
=

69^ miles (nearly) 1


=

league. degree.

'

Surveyors
links of

use

chain

four

rods

long,divided

into

Ty^^ inches each. Engineers divide the foot into tenths and hundredths. The in estimating duties yard is also divided similarly
at the custom

houses. is divided into 60 nautical


or

A miles. A

degree
nautical miles.

geographic
mon com-

mile

or

knot

is,therefore, nearly1^

Circular

Measure

is used

in

measuring circles.
minute,
marked
"

60 60

seconds minutes

C)

make
^'

1 1

'. ".

degree,

360

degrees
circumference

"

1 circle.

Rem.

"

The

is also

divided

into

quadrants of

90"

each, and

into siqjisof

80" each.

KEDUCTION

OF

COMPOUND

NUMBERS.

99

III.

MISCELLANEOUS
1 1 1

TABLE.

12 12 12 20 100 196

things make
dozen gross
" "

dozen, marked
gross,
"

doz.
gr.

great gross.

things pounds pounds pounds pounds


sheets

"

1 score. 1

of nails, make of flour of


"

keg.

1 barrel. 1 barrel. 1

200 240

pork

or

beef make
"

of lime

cask.

24 20 2 A sheet
2

of paper

make
"

quire.
ream.

quires
reams

"

1 bundle.

folded leaves
'"

in is called
'' "

a a an a a

folio.

4 8 12 16

quarto, octavo,
16mo.

or or

4to. 8vo. 12mo.

"

''

''

''

"

"

duodecimo, or

"

"

'^

Examples

in

Miscellaneous

Tables.

73.

1. Reduce

2. Eeduce

3. Eeduce
4. Eeduce

5. Eeduce 6. Eeduce 7. Eeduce 8. Eeduce


9. Eeduce

10. Eeduce 11. Eeduce

64. Troy to ounces. 5 pwt. to pwt. 9 lb. 3 oz. 2225. 8 lb. 9 oz. 13 pwt. 17 gr. to gr. 50729. 805 pwt. to pounds. 3 lb. 4 oz. 5 pwt. 12530 gr. to pounds. 2 lb. 2 oz. 2 pwt. 2 gr. 4 lb. 5 g 2 gr. to grains. 25442. 41300. 7 lb. 2 " 1 9 to grains. 4 lb. 5 5 7 5. 431 3 to pounds. 3 lb. 4 " 5 5. 975 9 to pounds. 1 lb. 1 g 1 3 1 9 1 gr. 6321 gr. to pounds.
oz.
"

5 lb. 4

cong.

7 f

to

fluid drams.

4152.

100

KAY'S

NEW

PRACTICAL

AKITHMETIC.

12.

Eediice

5 O. !H69

6 f.

"
to

f. 3

to

minims.

41460.

13. Keduce

f. 5

gallons.
2 cong.
3

O. 4 f. 3

14. Reduce 15. How 16. What

yd. to barleycorns.

5 f. 3. 324. 216.

lines in 1 foot 6 inches? many is the height of a horse of 16^ hands?

5 ft. 6 in.

17. A

field

measures

24

chains

in

lengthand

15 chains 36.

in breadth: 18. A

how

many
contains

acres

in it?

cistern

267

cubic feet 624

cubic

inches 2000. 31305. 223168. 15"

how

does it hold? (Art.64, gallons many 8" 41' 45" to seconds. 19. Reduce 61" 59' 28" to seconds.

Eem.).

20. Reduce 21. Reduce 22. Reduce 23. What

915' to

3661"
cost

degrees. to degrees.
gross of
screws

15'.

1" 1' 1".


at

5 cents

dozen?

$3.60.
24. A 25. At cost? 26. How
a man

is 4
certs

score a

and

10:

how

old is he?

90 yr. of paper

18

quire,what

will 3 bundles

$21.60.
sheets of paper many book of 336 pages?
octavo

will be

requiredfor
512 pages

12mo. 27. An

14. has 512 used in and

work

in 5 volumes 528 in Vol.

Vol.
496

1, 528
in Yol.

in Yol. 5: how

2,

3,
was

in Vol. for
one

4,

much

paper

of the whole

work?

copy 6 quires17 sheets.

Promiscuous

Examples.

74.

1. What

cost

2 bu.

of

plums, at
of

5 ct.

pint?
$6.40.
50
ct.
a

2. What

cost

bu.

pk.

peaches, at

l"eck?

$7.

KEDUCTION

OF

COMPOUND

NUMBERS.

101

3. What

cost

pk.

qt. of

'ai baVl^y,

3* ct.

pint?

4. At

15

ct.

peck, how
2 ct.
a

many

bashels

of

apples can
5 bii. l)e

be

bought
SI.66?

for 83?

5. If salt cost with

pint,how
into
were

much
1 bu. 1

can

bought

6. I

put
:

91 how

bu.

of wheat

pk.

each

many
both

bags

pk. 1 qt. 1 pt. bags containing3 bu. 2 ? 26. required


then divide.

Rem.

"

Reduce

to pecks,and quantities

7. How

many

weighing spikes,
of

oz.

each, are
;

in

parcelweighing 15 lb. 12 oz. ? 52 lb. cwt. 8. I bought 44 weighed 9 lb. 15 oz. : how many
9. How many

63.

cheese

each
I

cheese

cheeses

did

buy?
448. from 17. be 63.

kegs, of 84 lb. each, can be filled hogshead of sugar weighing 14 cwt. 28 lb.? 10. How boxes, containing 12 lb. each, can many
7 cwt. 56

filled from 11. If


a

lb. of tobacco?
3 lb. 13
oz.

family use
6 cwt.

of sugar

week,
160

how wk.

long will
12. What 20
cents
a

10
2

lb. last them?


acres

will

125

square

rods

of land

13. A into make? 14. How 2 ft.

rod? square has a field of farmer 41

cost,at $89.
divide

16
:

A. how

53

sq. rd. to many

lots of 1 A.

sq. rd. each

lots will it 13.

many

cu.

in. in

block

of marble

ft. long, 13824.

high, 2
cu.

ft. wide? ft. of ft.


water

15. One

weighs
a

1000

oz.

avoirdupois:
312

what

do

cu.

16. What the space

is the

weigh? weight of
of

lb. 8

oz.

quantityof
each cubic

water

ing occupy4

of 1 cord
ounces

wood,

foot of water

weighing 1000

avoirdupois?

T.

102

KAY'S

NEW

PRACTICAL

ARITHMETIC.

17' jV

cufei6' fo("tI af! oak


'

weighs

950

oz.

avoirdupois:
7 T.
at 1:^ cwt.
cents
a

Ay.bat "l5c""3 oak'\\:eigh? ;Coi\lw,*of;


*'

18*

\^ihd'the *eo"t^of' 63' gallonsof wine,


the
cost

20

pint.
19. Find
31

$100.80.
of
5

barrels

of

each molasses,

taining con-

$63. gal.2 qt.,at 10 cents a quart. 20. At 5 cents a pint,what quantity of molasses can 5 gal. be bought for $2? each dozen bottle holding 3 21. How bottles, many be filled from 63 gal.of cider? 6 doz. qt. 1 pt.,can 22. How kegs,of 4 gal.3 qt. 1 pt. each, can be many filled from 58 gal.2 qt.? 12. heart beat 70 times a 23. If a human minute, how 100800. times will it beat in a da}^? many seconds of February, in the month 24. How many
1876? 25. If
a

2505600

sec.

ship sail
is fed

miles

an

hour, how
oats

many 4440

miles
mi.
cost
a

will it sail in 3 wk. 26. A


44 cents

2 da. 3 hr. ?
1

horse
a

peck
much

of

daily. If
to

oats

bushel, how

will it cost

feed

him

year

of 365

days? bought
40

$40.15.
barrels
ct.
a

27. A
ct.
a

flour dealer

of

flour
how

for much

did

pound, and he gain?

sold it for 5

pound:

$156.80.

ADDITION

OP

COMPOUND
to

NUMBERS.

75.
the

When

the

numbers

be added

are

compound,

operationis
farmer
25
1

called Addition

of Compound Numbers.
wheat:

1. A

sold bu. 3

three

lots of
the

the bu. 2

first lot

contained

pk. ;
how

second, 14
did he

pk.;

the

third, 32 bu.

pk.:

much

sell?

ADDITION

OP

COMPOUND

NUMBERS.

103

Solution. the and


same

"

Place

units of the

same

denomination with
to

in

column
sum

(Art. 17).
is 6, which number

Beginning
is reduced

pecks,
bushels

adding,the

by dividingby 4, the
there

of

pecks in

bushel,and
the column
of

write the 2 under being 2 pecks left, to the of pecks, and carry the 1 bushel adding this to the bushels,the sum bushels; write under the column
of bushels.

column

is 72, which

72

Rule.

"

1. Write

the

numbers in the

to
same

be

added, placingunits
the

of
and

the 2.

same

denomination the
stim
a

column.

Begin with
divide their

lowest

add denomination,

numbers,

by

the number

ination of units of this denom-

ivhich make 3. Write carry the the

unit

of

the next the

higher.
column

remainder

under

added, and
colurnns
to

to quotient

the next
same

column. with
sum.

3. Proceed, in the under last,

the

manner

all the

which

write its entire

Proof.

"

The

same

as

in Addition

of

Simple Numbers.

nominatio writing compound numbers, if any intermediate deis wanting,supply its placewith a cipher. Rem. for every ten, 2. In adding simple numbers we carr}^ one of the next because order always make ten units of a lower one higher; but, in compound numbers, the scale varies, and we carry
1.
"

Rem.

In

"

one

for the

number

of the lower

order, which

makes

one

of the next

higher.

104

RAY'S

NEW

PKACTICAL

AKITHMETIC.

LIQUID

MEASURE.

AVOIRDUPOIS

WEIGHT.

(8)
T. 45
14 cwt. 3 14

(9)
lb.
oz.

cwt.

lb.
85 90 74

oz.

53 75 18

10 15 13

16 15 18

14 13 12

19

17

ADDITION

OF

COMPOUND

NUMBERS.

105

SQUARE

MEASURE.

CUBIC

MEASURE.

(14)
C.
13 15
20
cu.

(15)
cu.

ft. 28 90 67

in.

cu.

yd.
50 45 46

cu.

ft. 18 17 20

cu.

in.

390 874 983

900 828 990

TIME

MEASURE.

(16)
da.
16 13 19

(17)
sec.
mo.

min. Ill*, 18
15

wk. 0

da. lir. min. 0


4 23 51

sec.

28 49 53

47 59
42

3 12 3

40

19 13

30 27

37 18

16

18. Five
14

loads 23

of wheat bu. 2

measured
18 bu.

thus
1

21
22

bu. 3 bu. 1
100

bu. 1 many A

pk. ;

pk. ;
of 400

pk.;
bu.
3

pk.; pk.:
bu.

how 19. 143

bushels farmer
1

in all? raised
oats

200
3

bu. how

pk.;
much

corn,

bu.

pk.; barley, pk. ; wheat, 255 bu. 1


1000 sugar
cwt.
:

pk.:

in all? sold 5

bu. first

hogsheads of 36 lb. ; the second, 4 weighed third,5 cwt. 19 lb.; the fourth,7 cwt. 7 cwt. 84 lb. : what did all weigh?
grocer 8 cwt. 21. Add 13 lb. 11
oz.

20. A

the

64

lb. ; the

75

lb. ; the
33 cwt.

fifth,
78

lb.
; 16
oz.

oz.;

17

lb. 13
oz.

oz.

; 14

lb. 14
99

oz.

lb. ; 19

lb. 7

; and

17

lb. 9

lb. 6

106

RAY'8

NEW

PRACTICAL

ARITHMETIC.

22. Two 104


west:

men

depart from
due
are

the

same

place:
mi.

one

travels rd. due


200

mi.

50

rd. far

east; the other, 95

270

how
man

they apart?
3

mi.
134

23. A

has

farms:

in the A. 17

first

are

186

A.

sq. rd. ; in the second, 286 much 113 A. 89 sq. rd. : how 24. Add sq. in. ; 23 in. 25. A the the has
4

sq. rd. ; in the third, in all? 586 A. 80 sq. rd. sq.

17 sq. sq.

yd.

sq. ft. 119

in.;

18

sq.

yd.

141 sq. in. ft.;

yd. 7

sq. ft.; 29

sq.

of piles C. 24 C. 10
cu. cu.

wood

88 sq. the : in

yd. 5 sq. ft. 116 yd. 8 sq. ft. 88 sq.


cu. cu.

second, 16 fourth,29

ft.;the
ft.: how

7 C. 78 first, third,35 C. 127 in all? 88 C. Ill

ft.;
ft.

much

cu.

gal.3 qt. 1 pt. of wine to A ; 945 gal. 1707 gal.1 pt. to C; 10206 to B; gal.1 qt. to D: how hogsheadsof 63 gal.each did I sell? many 277 hogsheads 50 gal.1 qt.

26. I sold 4642

SUBTRACTION

OF

COMPOUND

NUMBERS.

76.

When

two

given

numbers

are

compound,, the
Subtraction

operationof findingtheir difference


of Compound Nximhers.
1. I have after 67 bu. 2

is called

pk. of
bu. 3

wheat:

how

much

will

main re-

34 selling

pk.?
number under the in
2

Solution.

"

Write

the

less
same

greater,placing units of the


the
same

denomination be taken from

column.

but
and

1 bu.

being

taken

added

to the 2

pk., from 67 bu. reduced to pk., pk.,gives6 pk. 3 pk. from 6 pk.
not

pk. can

leaves 3

pk.; 34 bu.

from

66

bu. leaves

32

bu.

The

"32

3~

difference is,therefore,32 bu. 3

pk.

^_

SUBTRACTION

OF

COMPOUND

NUMBERS.

107
will done

Rem.
the
same

"

Instead
to

of

diminishing the
the lower

67

bu.
34

increase

number

by 1, the result bu. by 1, as is

be in

subtraction

of

simple numbers.

(2)
bu.

(3)
pt.
0 bu.

pk. qt.
0 1

pk. qt.
0 0 3 0

pt.
0

From

12

5
1 3

Take

J
3

2^
1

1 7 1

0_ 1^
7
1

Rule.

"

1. Write the less number the


same

under
same

the

placing greater,
column.

units
2.

of

denomination the lowest

in the

Begin with
But, if the
the upper,

denomination, and, if possible.


the
one

take the lower 3. than

number

from

above

it. be many

lower number increase


as

of any

denomination

greater
units tract subthe

of

that denomination
as

the upper make one


one

number

by

as

of

the next

higher ;

and before,

carry

to

the lower

number

of

next

higherdenomination.
in the
same

4. Proceed

manner

with

each

denomination.

Proof.

"

The

same

as

in Subtraction

of

bers. NumSimj^le

Rem.

"

The

resemblance
is the

between
same as

subtraction
in Addition

of

simple, and
Rem.
2.

^^f

compound numbers,

75,

Examples,

liquid

measure.

gal. qt. pt.


From Take
17
Hi 2 3

gal. qt.
43 23 1 3

pt.
1 1

gi.
2
3

0,

108

RAY'S

NEW

PRACTICAL

ARITHMETIC.

AVOIRDUPOIS

WEIGHT.

(6)
T. From Take
14 10
cwt.

lb.
50 75

T. cwt.
16 5 7 6

lb.
18 75

oz.

12 13

14 15

LONG

MEASURE.

(8)
mi.
rd.

(9) yd.
4

ft.
1 1

in. 10 11

From Take

18
11

198 236

SQUARE

MEASURE.

SUBTKACTION

OF

COMPOUND

NUMBEKS.

109

16. If 2 bu. 1
4

bushels

of

pk. 1 qt. be taken from a bag containing hickory nuts, what quantity will remain ? 1 bu. 2 pk. 7 qt.
bu. take
24

17. From

100

bu. 1

pt.
75 bu. 3

pk. 7 qt. 1 pt.


lb. 8
oz.

18. I oz., how


19. A

bought
much

46

lb. 4

oz.

of rice

after

19 selling

remained? loaded with

26 lb. 12

wagon

hay weighs

32

cwt.

66

lb. ;

the wagon alone of the hay? 20. It has is 24899 100

weighs 8
miles mi.
41

cwt.

67 lb. : what

is the
23 cwt.

weight
99
a man

lb.

round rd. what

the

earth

after will

traveled

distance 24798

remain? rd.

mi. 279

21. I had

a son

farm

146 containing
94

A.

I gave

my

86 A.

sq. rd.

how

80 sq. rd. of land. much left? was 59 A. 146


cu.

22. From taken: 23. A how cask

8 C. 50 much

cu.

ft. of

wood,

C. 75
4

sq. rd. ft. are


cu.

is left?

C. 103

ft.

of wine

63 gal.le^J^ed; containing only 51


:

gal.1 qt. 2 gi. remained


24. From 13 min.
29

how

much
11

was

lost ?

gal.2 qt. 1 pt. 2 gi.


sec.

5 da. 10 hr. 27
sec.

min.

15

take

da. 4 hr.
46
sec.

3 da. 6 hr. 13 min.

77.

In
30

the time finding days 1 month, and

between
12

two sider dates, conany months 1 year.

1. A

note, dated
:

April 14, 1875, was


time between these

paid February
dates.

12, 1877
Solution. that and
12

find the

"

In

writing

the

dates, observe
of the year
mo. re-

operation.

February is the 2d month April the 4th; then, from


1875
9
mo.

da. subtract
is 1
vr.

mainder

yr. 4 mo. 28 da.

1877 yr. 2 14 da. The

yr. 187 7 18 7 5
19

mon.

da.
12

2 4

14
2 8

110 2. The

KAY'S

NEW

PKACTICAL

ARITHMETIC.

Independenceof the United States was declared July 4, 1776 : what length of time had elapsedon the 100 yr. 1 mo. Ist of September,1876? 27 da. first crusade ended 3. The July 15, 1099; the third tween crusade, July 12, 1191 : find the difference of time bethese 4. dates.
was

91 yr.

11

mo.

27 da.
;

Magna Charta was Queen of Scots,


difference of time

signed June 15, 1215 beheaded February 8, 1587 :


between these dates. yr. 7 fought Oct.
at

Mary,

find the

371

mo.

23 da.

5. The

battle of
Prince
w^as

William,
1688:
two

of

Hastings was Orange, landed


the difference

Tor time

14, 1066; Bay Nov. 5,


the da.
yr. 21

what events?

of

between
622

6. The

battle battle of time.

of of

Austerlitz

was

fought
18,

December
find
mo.

2,
the

1805;

the

Waterloo, June

1815:

difference

9 yr. 6

16 da.

78.

To

fiijdthe time

between

two

dates in

days.
Oct. 21.

1. Find

the

number

of

days

from

May

10

to

Solution. 21
31

10 May, there remains 31 days; there are 30 days in June, 31 in July, in August, 30 in September, and 21 in October; of days from then the number May 10 to
" "

Of

==

October 164.

21, is 21 -f 30 +

31

31

30

+ 21

==

2. Find 12.

the number

of

days from

March

17 to

ber Septem179.

MULTIPLICATION

OF

COMPOUND

NUMBEKS.

HI

3. A

note

dated

April 18, 1877, is


1877, is
from

due

June

20, 1877:
63.

how
how

days does it run? many dated 4. A note Sept. 5,


many 5. Find

due

Dec.

7,

1877

days
the 1878. the

does

it run? of

93. Oct.

number

days

12, 1877,

to

May
March

25,

225. number

6. Find

of

days

from

Aug. 20, 1875, to


201.

8, 1876.

MULTIPLICATION

OF

COMPOUND

NUMBERS.

79.
the

is a compound number, multiplicand is called Multiplication of Compound Numbers. operation farmer


2

When

the

1. A

takes
3

to

mill

bags
had

of he

wheat,
in all ?

each

taining con-

bu.

pk. :

how

much

for Begin at the lowest denomination convenience. Multiply the 3 pk. by 5, making 15 pk., which, reduced, gives 3 bu. and 3 pk.; write the 3 pk. under the pecks,and carry the 3 bu. Then, multiply the 2 bu. by 5, add to the product the 3 bu., and write Solution.
"

bu.
2

pk.
3 5

13

the 13 bu. under

the bushels.

under the lowest denomination multiplier of the multiplicand. 2. Multiplythe loivest denomination and divide the first, ivhich productby the number of units of this denomination make unit of the next higher, write the remainder under a the denomination to the and multiplied, carry the quotient product of the next higherdenomination.
Rule.
"

1. Write

the

3. Proceed the writing

in entire

like

manner

with

all the

denominations,

product
as

at

the last.

Proof.

"

The

same

in

Simple Multiplication.

112

llA^'S

NEW

PRACTICAL

ARITHMETIC.

Rem. and of

"

There

are

two

differences 1. In

between

of simple multiplication

compound
to
use

numbers

it is

simple numbers
with the table.

it is more venient consimple numbers at a time; in compound one figureof the multiplier the eritire tnultiplier each time. better to use 2. In the scale is miiforvi; in compound numbers it varies

numbers:

Examples.
2.

Multiply2 Multiply2
bu.
3

bii. 1

pk. pk.
qt. 1
of

qt. 1 pt. by
13

6.

bu. 3

pk.

qt.

3.

bu.

4. If 4

pk.

qt. by 9. pt. of wheat


bl.? 58

23 bu. 2 qt. make 1 bl. of bu. 1 of

how iiour, 5. Find

much the
cwt.

will make

12
9

pk. 2 qt.
each 58 10 lb.

weight
62

hogsheads

sugar,

weighing 8
6. How

lb.
in 7

3 T. 17 cwt.

much

hay

loads,each weighing
3

cwt.

89

lb.? 7. If
a

T.

16 cwt.

23

lb.

ship sail 208 mi. 176 rd. a day, how far will it sail in 15 days? 3128 mi. 80 rd. 8. Multiply23 cu. yd. 9 cu. ft. 228 cu. in. by 12. 280 cu. yd. 1 cu. ft. 1008 cu. in. 99 T. 12 cwt. 6 lb. 9. Multiply16 cwt. 74 lb. by 119. 10. Multiply47 gal.3 qt. 1 pt. by 59. 2824 gal.2 qt. 1 pt. travels 27 mi. 155 rd. in 1 day: how 11. A far will month of 31 days? he travel in one 852 mi. 5 rd. of wood, each pile containing7 C. 98 12. In 17 piles is the quantity of wood? ft.: what 182 C. 2 cu. ft. cu. 39 13. Multiply 2 wk. 4 da. 13 hr. 48 min. sec. by
75.
14. 49
mo.

3 wk.

hr. 48

min.

45

sec.

plantersold 75 hogsheads of sugar, each weighing 6 for 10 cwt. 84 lb., to a refiner, ct. a pound. The refiner sold the sugar for 8 ct. a pound : how much did he gain? $1626.
A

DIVISION

OF

COMPOUND

NUMBEES.

113

15. A

cotton-factor
4 cwt. 85

sold for lb.,


:

425
13

bales
ct.
a

of

cotton, each
He

weighing
$24735

for the

cotton

how

much

did

pound. he gain?

paid

$2061.25.
DIVISION OF

COMPOUND

NUMBERS.

80.

When

the called

dividend Division be

is operation

compound number, of Compound Numbers.


a a

is

the

The Number.
First.

divisor This

mvij

either
to

Simple or
cases:

Compound

gives rise
find how

two

"

To

often

one

Compound
Number.

Number

is

contained
This

in another

Compound

is done

denomination

by reducing both divisor and dividend to the before dividing(Examples 6 and 8, Art. 74).
To divide
a

.^ame

Second. number

"

of

into a given Compound Number vision. equal parts. This is properly Compound Di-

1. Divide
Solution. first, so duced
to it.

14 bu. 2
Divide the
a

pk.

qt. by

3.
operation.

"

highest denomination

that,if there be
to the next

remainder, it may be relower denomination, and added


4

bu.

pk. qt.
2 3 1 3

8)14
4

3 in 14

is contained
4

times, and

bu.

are

write the left;


the

under

the

bushels,and

reduce

remaining 2 bu. to pk.,to which add the 2 pk.,making 10 pk. This, divided by 3, gives a quotientof 3 pk.,with 1 pk. remaining; which, reduced to qt.,and 1 qt.added, gives9 qt. This, divided by 3, wdiich is written under the quarts. 3, givesa quotient

(2)
bu.

(3)
da. hr.
min.
sec.

pk. qt.
2 3 6 2

7)33
4
PPAC. 8.

5)17
3

12 12

56
11

15
15

114

KAY'S

NEW

riiACTICAL

ARITHMETIC.

Rule.

of

its

quantityto be divided in the order place the denominations, beginningwith the highest;
"

1. Write

the

divisor 2. ber

on

the

left,
each
num-

divide denomination, Begin ivith the highest beneath. and write the quotient separately,

3.

If

remainder
lower

the next

reduce afterany division, add denomination,and, beforedividing,


occurs

it to
to it

the number Proof.


Rem."

of
The

its denomination. in

"

same

as

Simple Division.
is of

Each

partial quotient
from

the

same

denomination

as

that part of the dividend

which

it is derived.

4. Divide

67

bu.

pk.

qt. 1 pt. by 5.
13

bu. 2
ewt.

pk. 2 qt. 1 pt.


44

5. Eleven what is the

casks average

of sugar

weigh 35 weight of each?


rd. in 7 hr.

lb. 12

oz.

3
t). I

cwt.

22

lb. 4
rate

oz.

traveled

39 mi. 288

at what

per rd.

hour 7. 8.
9. 10.

did

I travel? 69 A. bu.
2

5 mi. 64
4

224

Divide
490

sq. rd.

by
100.

16.
4

A.
3

54

sq. rd. 5

pk.
oz.

qt.-^

bu.

pk.

qt.
oz.

265
45

lb. 10 T. 18

-^50.

5 lb. 5 2 T. 14 18
2
sec.

cwt.-^17.
hr. 45 min.

cwt.

11.

114

da. 22

-1-54.
17
sec. oz.

d"a. 3 hr. 5 min.


44

12.
13.

10

cwt.

27

lb. 13

oz.

-f-23.

lb. 11

309 127 788 A

bu. 2

pk.

14. 15.
16

gal. 3 qt. 1
mi. 169 has rd.
two

qt.-^78. pt. 3 gi.


"

3 bu. 3 63.
2 2

pk. 7 qt. gal.1 gi.


151 sq.

-^319."
farms, one
He
of 104
40
reserves

mi. 117

rd.

farmer

A.

rd.;

the and
what

other,87
divides is the

A. 78 sq. rd. the remainder share of each

equallyamong
son?

A. 40 sq. rd., his 3 sons : 105 sq. rd.

50 A,

LONGITUDE

AND

TIME.

115

17. A
one

farmer's

crop

consisted

of 5000

year, and 7245 bu. 2 pk. the year bu. 1 pk. and B022 placed the remainder crib

pk. of following.He
in many 8
:

bu. 3

corn

sold

cribs,
bu.

each

containingan

equal amount

how

bushels
528

in each 18. A each into did

crib?

speculator bought 6 adjoiningpieces of land, containing4 A. 80 sq. rd. He divided the whole
and lots, sold them
at

54

$5

sq. rd.

how

much

he

get

for each 35

lot?
oz..

S400.
lb. 14
;

19. Add

lb. 9
186

75

oz., 85

lb. 15

oz.;

from

the

sum

take the

lb. 14

oz.

multiplythe
what

remainder

by
oz.

8;

divide

productby

64:

is the result? 1 lb. 5

LONGITUDE

AND

TIME.

81.
laces. 1^

Difference of

and longitude

time

between

different

The
divided The

circumference
into
sun

360

is earth,like other circles, equal parts, called degrees of longitude. of

the

360", once
over

round the earth, to pass entirely appears in 24 hours,one day; and in 1 hour it passes
.:=

(360" -f- 24 15"). As 15" equal 900', and 1 hour equals 60 minutes of the sun in 1 minute of time passes over time, therefore, l^' of Si degree, (900'~60=r=: 15'). As 15' equal 900", and 1 minute of time equals 60 seconds of time,therefore, in 1 second of time the sun 15" of a degree. (900"^ 60 15"). passes over
=

15".

Table

for

Comparing of

Longitttde
-=.-

and

Time.

15"

15' of
15" of

longitude longitude longitude


^^^ =

1 hour 1 min. 1
sec.

of time. of time. of time.

116

KAY'S

NEW

PRACTICAL

ARITHMETIC.

1. How 18" 25' :W

many of

hr. min.

and

sec.

of time

correspondto
min.
42
sec.

longitude?
the

1 hr. 13

Analysis.

"

of By inspection
"

it is evident that, table, of time. of time. of time.

divided by 15, give hours Degrees ( ) of longitude, divided by 15, give minutes Minutes (^ ) of longitude, divided by 15, give seconds of longitude, Seconds (^^)

Hence, if 18" 25^ 30^^ of Ion. be divided


be the time in hr. min. and
sec.

by 15, the quotientwill to that longitude. corresponding


difference of

To

find

the

time

to corresponding

any

: longitude

Rule.

"

Divide

for Division
hr. min.

to the rule longitude by 15, according the quotient and mark of Compound Numbers
,

the

sec, instead

of
find

"

'

".

Conversely: To
any difference

the

to longitude corresponding

of time.

to the Multiplythe time by 15, according for Multiplication of Compound Numbers^ and mark instead of hr. rain. sec. product

Rule.

"

rule
the

"

'

"

2. The

difference of

between longitude
of time?

two

is 30*^ : places
2

what

is their

difference

hr.

8. The 4': what


4. The

difference of

betw^een longitude
of time?

two

is 71" places min. 16


sec.

is the difference

4 hr. 44

difference of
is 10"

between longitude
is the

New

York

and

Cincinnati

35': what

difference of time?
42

min. 20 and

sec.

5. The

difPerence of time
is 37 min. 20
sec.

between
:

Cincinnati
is the

adelphia Phil-

what

difference of
9"

longitude?

20'.

LONGITUDE

AlNl)

TIME.

117

6. The Louis

difference

of

time 56
sec.

between
:

!N^ew York
is the

and

St. of 14^

is 1 hr. 4 min.

what

difterence 16"

longitude?
7. The difference of time
4
sec.

between
:

London is the

and

ington Washof

is 5 hr. 8 min.

what

difference

longitude?
DlB^FERENCE
IN

77"

I'o

TiME.
any when i)l^ce the

82.
sun

It
on

is the
sun

noon

at (12 o'clock),

is As

meridian appears

of that
to
at

place.
from the is
east

the

travel any

toward

the
east

west, when
of that

it is

noon

it place,

after noon

and beforenoon place, Hence, a place has later or according as it is east or west

icest of that

earlier of it.

place. tirne than another, Therefore,

When

if EAST if WEST,

the time at another, place is given, their difference of this,is found by adding of time; their difference by SUBTRACTING of time.
at
one

the time

8. When

it is

noon

at

what Cincinnati, 37 min.


20

is the
sec.

time
noon.

at

Philadelphia?
9. When the time in it is 11

past
1

o'clock A.
east

M.

at

New York?

York,

what

is

30" longitude 12 o'clock

of New
at

P. M.
is

10. When the time


at

(noon)
A.

what Philadelphia, min.


40
sec.

Cincinnati?
it is 11 o'clock

11 hr. 22 M.

A. M. what A. is

11. When the time


at

at New

York,
4
sec.

St. Louis?

9 hr. 55

min.

M.

12. the when


at

Wheeling, W.
of the it is 1 o'clock mouth
of

Ya.,
P. M.

is

mouth

Columbia

longitude80" 42' west: in longitude 124" west: river, is the time at Wheeling, what
river? 10 hr. 6 min. 48
sec.

in

the

Columbia

A. M.

DEFINITIONS.

83.

1. Factors the

of

number

are

two

or

more

bers, num-

product of (Art.28, 2).


Thus, 2 and
3
are

which

equals the given

number

factors of 6, because
2

2X3

6; 2, 3, and

are

factors of 30, because Kem. number. Kem. factors.


2.
"

X 3 X ^

30itself are
not

1.
"

One

and

the number

considered

factors of

number

may

be the
=

productof

more

than
=

one

set of

Thus, 2 X 6== 12, 3 X 4

12, and 2X2X^ is


a

12.

2. A

multiple
is
a

of

number

productof

which

the

number

factor.

of 3; 30 is a multipleof 5. Thus, 6 is a multiple

3. INTumbers

are

divided

into

two

classes, prime

and

composiie.
4. A

prime

number
are

has

no

factors.

Thus, 5, 11, 17
5. A

prime

numbers.

composite
are

number

has

two

or

more

factors.

Thus, 6, 12, 30
6.
p

compositenumbers.
factor

prime

is

factor

which

is

prime

umber. Thus,
3 is
a

prime
(118)

factor of 12.

FACTORING.

119

7. A it is
a

factor is factor
3 is

common

to

two

or

more

numbers

when

of each

of them.
15.

Thus,
Rem. the
"

common

factor of 12 and the smallest


6 is
a

Sometimes factor.

of
common

two

or

more

numbers

may
18.

be

common

Thus,

factor of 6, 12, and

8. Two

or no

more common

numbers

are

prime

to each

other
^

w^hen

they have
Thus,
9. A

factor.

9 and

10

are

prime
divisor

to each

other.

common common

(C. D.)

of

two

or

more

bers num-

is any

factor.
each divisor of 12 and
18.

Thus, 2, 3, and
10. The
or more

are

common

greatest
numbers

common

divisor

(G.

C.

D.)

of two

is the

greatest common
divisor of 12 and

factor.
18.

Thus,
11.

6 is the

greatestcommon

common

multiple
of multiple

(C. M.)

of tw^o

or

more

bers num-

is any

all of them.

Thus, 6, 12, 18, etc.,are


12. The
more

common

multiplesof multiple

2 and

3.

least

common

(L. C. M.)

of two

or

numbers

is the

least

multiple of
2 and

all of them.
3.

Thus,

6 is the least

common

multipleof
process of

13. Factoring

is the

resolving composite

numbers

into

their

factors.

To

Find

the

Prime

Jfiniihers.

84.

All

the

prime

numbers

except

are

odd

bers. num-

120

RAYS

NEW

PKACTICAL

ARITHMETIC.

Rule.
etc. 1),

"

1. Write

the

odd

tuanUrs

in

series 1, ;5, 5,

7,

2.

After 3
number
;

'M

eixtse

every
erase

bth

after7
y

number; after 5 erase every 7th number; after 11


that remain
are

every
erase

every

llth number 2 and

etc.

3. Then numbers.

the

numbers

the

prime

KxERCiSE.

"

Find

the

prime

numbers

from

to

100.

85.

The

operationsof Factoring depend

upon

the

following
PRINTIPI.ES.

1. A

factor of
is
a

number

divides exactly
is contained

it.

Tlius,5 2. A

factor

of

30 and

in it G times.

of multiple
30 is
a

number

contains exactly
contains it G times.

it.

Thus, 3. A that

multipleof
a

5 and

factor of

number

is

factor of

any

of midtiple

number.

Thus,

being

factor of 6 is a factor of 12, 18, 24, etc.

4. A

compositenumber

is

equal to

the

product of

all its

prime factors.
Thus, the prime factors of
30
are

2, 3, and 5; 2 X

30.

86.
will be

In

resolvingnumbers
found convenient
to

into
to

their

prime
the

factors it

remember numbers

following
5.

facts in reference

the

prime

2, 3, and

FACTORING.

121

1. Two

is

factorof

every

even

number.

Thus, 2 is
2.

factor

of 4, 6, 8, 10, etc.

Three
3
or

is

factor of

number
3.
2

when

the

sum

of

its

is digits

some

of multiple
of

Thus, 3 is
times
3.

factor

2457; for

4 + 5 + -[-

18, which

is 6

3. Five 0
or

is

factor of

every

number

ichose unit

figureis

5.
factor of 10, 15, 20, 25, etc. the

Thus, 5 is
Rem. number
"

Whether
or

prime

numbers

7, 11, 13, etc.,are


trial.

factors

of

not

is best ascertained

by

To

Resolve

J^umher
30 into

into its

its Prime factors.

Factors.

87.

1. Resolve

prime

viding (Art. 86, 1). Di3 being a factor of is 15. 30 by 2, the quotient 15 (Art. 86, 2) is also a factor of 30 (Art.85, Prin. 3). Dividing 15 by 3 the quotientis 5, a prime number. Then, 2, 3 and 5 are the prime factors of 30.

Solution."

2 is

factor of 30

operation.

2)30 3)15
5

Rule.

"

1. Divide

the

given number

by
manner

any

prime number
;

divide it. exactly 2. Divide the quotient in the same is to divide, until a quotient prime number. 3. The several divisors and the the

that unll

and

so

tinue con-

obtained

which

is

last

will quotient

be the

prime factorsof
Rem.
smallest
"

given number.
convenient
to

It

will

be

most

divide

each

time

by

the

prime

number.

122

RAY'S

NEW

PRACTICAL

ARITHMETIC.

Resolve

the

into following

their

prime

factors:

88.
more

To

find

the

prime

factors

common

to

two

or

numbers. 1. What

prime
"

factors
the

are

common

to

30

and

42?

Solution.

Write

numbers

in

line.

2 is

prime factor of both 30 and 42 (Art.86, 1). Di15 and 21. 3 is a are viding by 2, the quotients prime factor of both 15 and 21 (Art.86, 2); and of both 30 and 42 (Art. consequently 86, Prin. 3). 5 and 7 are Dividing by 3, the quotients prime to the common each other (Art.83, 8). Then 2 and 3 are

operation.

2)30
3)15
5

4 2

Tl
7

factors.

FACTORING.

123

givenmimbers in a line. 2. Divide by any prime number that will exactly divide all in the same manner oj them; divide the quotients ; and so continue to divide until two or more are of the quotients prime to each other.
Rule." 3.

1. Wiite the

Then

the several divisors will be the

common

factors.

What

prime

factors

arc

common

to

2, 3,
9
"

5.
9
-J.

9 -J,

2, 2, 3. 2, 3, 3.

3, 3, 3. 2, 2, 5, 2, 3, 7. 2, 2. 2, 3. 3, 3. 2, 5. 3, 5. 5, 5. 2, 7.
11. 13. 17. 19. 23.

Finding the G. C. D. depends upon the following


Principle.
all the other
"

89.

of

two

or

more

numbers

The

G.

C. D.

of

two

or

more

numbers

tains conno

prime factors common

to the

numbers, and

factor.

124

RAY'S

NEW

PRACTICAL

ARITHMETIC.

Thus, the G. C. D. of 12 and 3; it must greatestC. D.; it can


both 12 and 18.
2 and

18 is

6; it contains

the

common

tors facbe the

contain contain

both
no

of them, else it would

not not

other factor,else it would

divide

1. Find

the G.

C. D.

of 80

and

42.

First
Solution.
42
are

Method.
OPERATION.

"

The
3

2 and

the G. C. D. of

to 30 and prime factors common (Art.88); their product is 6; then 30 and 42 is 6 (Prin.).

2)30 3)15
5

42 21 7

Rule.

"

1.

F'uid

the

prime factorscommon

to the

given

numbers.
2.

3.

Multiplythem together. The product will be the greatest common


Second Method.
re-^ rere-

divisor.

Solution. mainder mainder mainder


30

is is

Dividing 42 by 30, the 12; dividing 30 by 12, the 6; dividing 12 by 6, the


"

operation.

30)42(1
3 0 6

is 0. 42.

Then

6
r=

is the
6

G. C. D. 42
=

of

and

For, 30

X 5 a"d
7
are

T^)

3 0

(2

7; then, because
other, 6
common

5 and contain
30

each

must to

all

prime to the prime

2 4

0)12(2
12

factors

and

thereis, 42; it-

their G. C. D. (Prin.). fore,

the by the less, greater number the divisor by the remainder, and so on, always dividing last divisor by the last remainder, remains. until nothing divisor. 2. The last divisor nill be the greatest common
Rule.
"

1.

Divide

the

Rem.

"

To

find the

G. C. D. of of them, then and

more

than

two
common

numbers, first find


divisor and
one

the G. C. D. of two of the


common

of that
on

remaining numbers,

so

for all the

numbers;

the last

divisor will be the G. C. D. of all the numbers.

FACTORING.

125
divisor of the

Find numbers

the
:

greatest

common

following

12. 18. 20. 27. 30. 16. 24. 36. 31. 26. 23. 19. 17.
39.

.227. 12. 5. 8.

Finding the L. C. M. depends upon the following


90. Principle."
tains all the

of
*

two

or

more

innnbers

TAe

L.

C. M.

of

two

or

more

numbers
and
no

con-

of prime factor's

each

number

other

factor.
its primefactors Thus, the L. C. M. of 12 and 18 is J^6;
and

arc

% % 3,
contain
not

8; it
the

must

contain

all these factors, else it would contain


no

not

both

numbers;

it must

other factor,else it would

he the least CM.

126

EAY'S

NEW

PRACTICAL

ARITHMETIC.

1. Find

the

L. C. M.

of 4, 6, 9 and

12.

Solution.
2 and and

"

The

prime
are

factors of 4
2

are

OPERATION.

2; those of 6
factors of the
no

and
3.

3; of 9, 3 Then, the
are

4 6 9

2X2 2X3

3; and
and

of 12, 2, 2, and

prime
o,

L. C. M.

2, 2, 3,

other

factor

( Prin.). Hence,

12

3X3 2X2X3
=

36

is the L. C, M.

2X2X3X3

36

OPERATION.

The the
much form

process of

and factoring
the

selecting

2)4
2 12

6 3

9 9 9

12
6 3

prime

factors for the

hy simplified
of Short

L. C. M. is very in the operation

3)3

Division, as shown.
2 X 2 X
3

X 3

36.

Rule.

"

1. Wii'tethe

by any two or more of them. and 3. Write the quotients


beneath,
4. Divide

2. Divide

givennumbers m prime munher that


undivided

line,
will

divide exactly
in line

numbers

these numbers

in the

same

manner^

and

so

tinue con-

until a line is reached in which operation numbers all prime to each other. are 5. Then the product of the divisors and the numbers the last line will be the least common multiple.

the

the

in

Rkm.

"

"When

the

is 1 quotient

it need not

be written.

Find

the least

common

of multiple 24. 36. 40. 30. 72.

CANCELLATION.

127

7.
8. 9. 10. 11.

12.
13.

14. 15. 16. 17. 18. 19. 20. 21.


22. 23.

10, 12, 15 and 20. 9, 15, 18 and 30. 12, 18, 27 and 36. 15, 25, 30 and 50. 14, 21, 30 and 35. 15, 20, 21 and 28. 20, 24, 28 and 30. 45, 30, 35 and 42. 36, 40, 45 and 50. 42, 56 and 63. 78, 104 and 117. 125, 150 and 200. 10, 24, 25, 32 and 45. 2, 3, 4, 5, 6, 7, 8 and 16, 27, 42, and 108. 13, 29, 52, and 87. 120, 360, 144, 720, and

60. 90. 108. 150. 210. 420. 840. 630.

1800.
504. 936. 3000. 7200. 9. 2520.

3024.
4524. 72.

720.

CANCELLATION.

91.
for them did

1. I

bought

oranges
at 3

at

cents
:

each, and paid


how many

with

pears

cents

each

pears

it take?
OPERATION,

Solution

I." 5

cents

by multiplied
15

are

15
3

5 3

cents, the

priceof

the oranges.

divided

by

is 5, the number

of pears.

3)15
5

From
we

consideration

of this

example

and

its solution

have

the

following
A

Principle."
and then

mimber

the dividing

it changedby multiplying productby the mtdtipUer.

is not

128 For
solution

RAY'S

NEW

PRACTICAL

ARITHMETIC.

the and

example, then, : operation


"

we

may

offer

the

following

Solution and the

II.

Indicate

the
erase

tion multiplicaor

division; then,

cancel
3

OPERATION.

multiplier3 and drawing a line across the result, equal to 5.


Rem. The

the

divisor

hy

them; and

write

"

product 5 X

forms

dividend

of which

is the

divisor.

2. If 1 with

buy

10 pears
at

at 3 cents

each, and
many

pay

for them will it

oranges

5 cents

each

how

oranges

take ?

Solution.
=

"

5 is

factor of 10, for 10


the divisor
5 and
OPERATION.

5X2;

then, cancel
factor
5 'in 10

by canceling 10 and writing the remaining factor 2 above it. The product of the remaining
factors is 6.

also the

2 "==6

3. Divide

15 X

21

by

14 x

10.

Solution.

"

5 is a

common

factor of 15
OPERATION.
a

and

10; then, cancel

15, writing 3 above


below
it.

it, and
common

10, writing 2
factor of 14 and

7 is

21; then, cancel


21, writing
factors
3

14, writing2 below


above it. The

it,and

--f=-2i
2 2

product of the

maining re-

in the dividend

is 9, and

of those

remainingin
of 9 divided

the divisor is 4; the

quotient

by

4 is 2\.

Therefore,

Cancellation

is

process
of the

of abbreviation
dividend and

by omitting

the

common

factors

divisor.

CANCELLATION.

129

Rule. and

"

1
.

Cancel

the

factorscommon

to both the dividend

divisor. the the

2. Divide

dividend divisor.
3.

by

product of product of

the the

factors remaining in

the

in the factors reynainijig

The

result will be the

required. quotient
a

4. How

pay 5.

barrels of molasses,at S13 many at $4 a barrel ? for 13 barrels of flour,

will barrel, 4. 51. 30. 12. 111.

Multiply17 by 18, and


15 24

divide

by

6.

6. In

times times times

8, how
4, how

many many

times times

4? 8? 5? the

7. In 8. In
9.

37

Multiply 36 by 8. multiplied
10. In 11. 36 times

times 15, how many by 40, and divide

product by

30
6.

5, how Multiply42, 25, and


21 X 15.

man}^ 18

times

15? divide

12. the
60.

and together,

productby
at $5

at $10 each, and ^ 12. I sold 23 sheep,

was

paid in hogs,
46.

each

how

many

did I receive ?

13. How

will pay 14. What


V

many for 15 is

yards of flannel,at 35 cents a yard, 6 yd. yards of calico,at 14 cents? the quotientof 21 X H X 6 X 26, divided
33.
of
a

by 13X3X14X2? ^ 15. The factors


and 17 ; the

dividend
22

are

21, 15, 33, 8, 14,


27
:

divisors, 20, 34,

and

requiredthe
49.

quotient.
V

bought 21 kegs of nails of cents a pound ; paid for them with 35 yards each, at 9 cents a yard : muslin did I give?
17. What
26 X 30 X

16. I

95

pounds each, at 6 piecesof muslin of how piecesof many


38.
39

is the
42 ?
9.

quotient of

35 X

-10 divided

by If.

Prac.

130

KAY'S

NEW

PKACTICAL

ARITHMETIC.

18.

What

is

the

quotient
-

of

26

^3

35

divided

by 33^'.

4X9X25?
19. What is the

quotient

of

15

""

21

divided

by

4X6X10X14?
20.

S^^.
is the

What X

quotient

of

21

24

28

35

divided

by

14

18X20X25?

3||.

"33?^"Jt'/^]f^

FRACTIONS.
'

"%!'"*

92.
1st. An

unit

may
may

be
be

divided

into

equal parts ; thus,


two

apple

divided

equally between equally among


may be divided

boys, by boys, by

ting cut-

it into two 2d. An

equal parts. apple may be divided equal jpartfi.


an

three

ting cut-

it into three 3d.


any In

like manner,

apple

six, or into/o?/r,^ve,

number

These

of equal parts. equal parts into which

unit

may

be

divided

are

called

fractions.
DEFINITIONS.

93.
unit.

1.

fraction

is

one

or

more

equal parts

of

2. To

express unit

fractions
is divided is called
are

by

words

and

figures.

When

into

two

equal parts,
written
"

Each Both

part parts
unit

one-half,
iifo

i.

called

-halves,

f.

When

Each Two All

When

Each Two

equal parts. written ^. part is called one-third, called two-thirds, f. parts are the parts are called three-thirds, f. unit is divided into four equal parts. \. part is called one-fourth, written
is divided into three
" "

parts
the

are are are

called called

two-fourths,

"

Three All

parts

parts

three-fourths, called four -fourths,

"

"

f. f |.
.

(131)

132

KAY'S

NEW

PKACTICAL

AK1THMP:T1C.

equal parts, Each part is called one-fifth, written \, Two parts are called two-fifths, J. Three parts are called threefifths, |. Four parts are called /owr-//f As, ^. All the parts are called five-fifths, |. unit is divided into six, seven, When a etc., eight, equal parts,each part is called one-sixth, ^, one-seventh, \, etc. |, one-eighth,
a
"

When

unit

is divided

into

five

"

"

"

94.

1. A

fraction is

expressedin

words

by

two

bers; num-

the first numbers


the first number

the parts,the second names them; is called the numerator, the second is

called the
2. A
numerator

denominator. is

fraction above

expressed in figures, by writing the


denominator with
a

the

line between

them.
3. The
terms numerator

and

denominator

are

styled the

of the

fraction.

4. The

denominator is
are

shows the

into

the the

unit

divided, and
taken.

equal parts many of numerator, how many

how

parts

95.
of each

When

unit

is divided the

into

equal parts, the


of the
into two

size

part depends upon

number

parts.
equal parts,

Thus, if apples of equal size be divided,one


another into three be

half will
Hence,

equal parts,a third mio four equal parts, etc.,a larger than a third, a third largerthan a fourth, etc.

1st. The is

less the

number

of

parts

into

which

unit

divided,the greater the size of each part. of parts into 2d. The greater the number unit is divided,the less the size of each part.

which

COMMON

FKACTIONS.

I33

96.
one or

1. A
more

fraction units.

may

also be

regarded as

part of

Thus:

or

three boys. applesmay be divided equallyamong Each boy will receive,either one-third of each of the two apples, of the apples; therefore, ^ of 2 is ". Hence, " two-thirds of one
1st. Two

be considered either as two-thirds or as one-third of two. may 2d. Two applesmay be divided equallybetween two boys. of each of the two apples, or one-half is of the two one apples;therefore, ^ of 2 |,or 1. Hence, | may be considered either as two halves or as one-half of two. be divided equallybetween 3d. Three applesmay two boys. Each boy will receive, either one-halfof each of the three apples, one or apple and one-halfof another; therefore, J of 3 is |, or 1^. Each

boy

will receive, either

Hence, | may
three.

be considered

either

as

three

halves

or

as

one-halfof

2. A

fraction

is

part

of

one

or

more

units. of units. of each


to

3. The
4. The

numerator

expresses

the

ntimber

denominator

expresses

the

part

be

taken.

97.

1. A

fraction

may

also

be

regarded
numerator

as

an

pression ex-

of idend and

in division,

which the

the

is the

dw-

the

denominator

divisor.

Thus:

f is 2 divided by 3; here, the division can only be indicated, 2d. I is 4 divided by 2; in this case, the division can be performed exactly, giving a quotient2. 3d. f is 5 divided by 2; in this case, the division can be pernot formed the quotientbeing 2^. exactly,
1st.

2. A is the

fraction

is and

an

indicated the

division.

The

numerator

dividend

denominator

is the

divisor.

134

KAY'S

NEW

PRACTICAL

ARITHMETIC.

3. A
a

whole

number

may
the

be

expressed in
for the

tli('form

of

fraction, by writing
1 for the

number

umiierator

and

denominator.

Thus,

may

be written be written

|; 98.

4 may

|; for f, etc.
a

2 divided

by

1 is

2;

may

be written

The

value

of

fraction

is its relation the 1.

to

unit.

1. When the value

the of the

numerator

is less than is less than

denominatoi-,

fraction

Thus, ^,J,", etc.,arc


2. When the value the of the

less tlian 1.

numerator

is is

fraction

equal to equal to 1.

the

denominator,

Thus, f,|,f, etc.,equal 1.


3. When the the
numerator

is greater than
is

the

tor, denomina1.

value

of the

fraction

givater than

Thus, |,|,f,etc.,are
4. A 5. An
to
or

greater than is

1.

proper

fraction

one

whose is
one

value whose

is less th'an 1. value is

improper
mixed

fraction
1.

equal

greater than

6. A

number

is

whole

number

and

fraction.

99.

1. A

fraction

may
been

be

divided

into

equal ])ai'ts.

equal parts,each half may be divided into two equal parts; the whole apple will then be divided into four equal parts; therefore, ^ of ^ is \. Such expressionsas ^ of ^, ^ of ^, etc., are termed compoimd
an

Thys, after

apple

has

divided

into two

fractions.
2. A

compound

fraction is

fraction

of

fraction.

COMMON

FRACTIONS.

135

100.

1.

Fractions

sometimes
or

occur

in

whicli

the

numerator, the
Thus,
fractions.

denominator

botli

are

fractional.

-r"

oT"

^"
are

are

such

expressions; they
hy 4, etc.
in which

are

called

complex

They simple

read

3^ divided
is
one

2. A entire. 3. A the

fraction

both

terms

are

complex
are

fraction fractional.

is

one

in

whi^h

one

or

both

of

terms

101.

The

with operations

fractions

depend

upon

the

following
'

Principles.
numerator.

1. A

the fraction is multiplied by multiplying


numerator
are

Thus, if the

of of

be
same

multipliedbj'3, the
size
as

result will be there


are

f;

in

^
as

the many.

parts

the

in

|,but

three

times

2. A

fractionis
numerator
are

the divm^dhy dividing


of

numerator.
result will be
are

Thus, if the
in

|
same

be divided size
I
as

the
as

parts
many.

of

the

hy in f

3. the

but

there

^; only one-

third

3. A

fractionis

divided
of

the hy multiplying

denominator.
result will
the

Thus, if the denominator


be
are

be

|; in I there are only one-third


4. A

the
as

same

number

multipliedby 3, the of parts as in |, but

parts

large.

the denominator. fraction is multiplied hy dividing


if the denominator

Thus,

of

be

divided of

by 3, the
in

result will be
the

|;

in

there
as

are

the

same

number

parts as

|, but

parts are

three

times

larsre.

136

RAY'S

NEW

PRACTICAL

ARITHMETIC.

5.

Multiplying
not

both

terms

of

fraction by

the

same

ber num-

does

change
terms
as

its value.

Thus,
there
are

if both twice

of

be

multiplied by 2,
as

the
are

result

is

y"^;in ^
as

many

parts

in

J, but they

only one-half

large.
G. does

Dividing
not

both

terms

of

fraction by

the

same

number

change
if both

its value.

Thus,

terms

of

/^

be many

divided

by 2,
as

the

result

will
are

be

|;

in
as

I there large.

are

only one-half as

parts

in

but they 3^5,

twice

These
follows
:

six

j)rinciple8 may
^

be

stated

more

as briefly,

I.

A 1st.

fraction

is

multiplied,
numerator.

By multiplying the By dividingthe


fraction

2(1.

denominator.

II.

is divided,

1st. 2d.

By dividing the numerator. the denominator. By 7nultiplying


The value of
a

III.
1st.

fraction
terms

is not

changed,
the
same
same

By multiplying both By dividing both

by
the

number. number.

2d.

terms

by

The

operations

with

fractions

are

Beduction, Addition,

Subtraction, Multiplicationand

Division.

COMMON

FRACTIONS.

137

REDUCTION

OF

FRACTIONS.

102.
witiiout

Reduction

of

Fractions

is

changing their
are

form

their value. altering;

There

six

cases.

CASE

I.

103.

To
a

reduce

an

integerto

an

improper fraction,

having
1. In

given denominator.
3

how apples,

many

halves?
OPERATION.

Solution.
3

"

In
are

apple there
3X2

are
=

2 6

halves;then, in
halves.

|X

apples there Rule."l.

halves

under

the

Multiplythe integer by the given denominator product write the denominator.


how apples, how apples, how apples, how apples, many many many many

2. In 3. In 4. In 5. In
6. In

4 2 3 4 6 8

halves? thirds? fourths?


fifths? ?

7. In

tenths inches,how many twelfths? how feet, many


4
to

^-ffj ff
t^

8. Eeduce
9. Eeduce

sevenths.
ninths.

8 to
19
to

10. Eeduce
11. 12.

thirteenths.
twentieths.

Eeduce Eeduce

25 37

to to

twenty-thirds.

V' W ^ W

CASE

II.

104r.
fraction.

To

reduce

mixed

number

to

an

improper

138
1. In

RAY'S

NEW

PRACTICAL

ARITHMETIC.

31

how apples,
In
are

many
2

halves?
OPERATION.

Solution.
3

"

1 3

apple there
X
^ halves

are
=

halves; then, in
6 halves

|X3=| 1+ 1
=

applesthere
1 half
are

6 halves.

and

7 halves.

Rule.

"

1.

fraction;to
sum

by the denominator of the Multiplythe integer the numerator, and under the the product add

write the denominator.

2. In 3. In 4. In 5. In

4^ 2\ 2| 51

6. Reduce 7. Reduce
8. Reduce 9. Reduce

It). Reduce 11. Reduce 12. Reduce 13. Reduce 14. Reduce 15. Reduce

how apples, man}^ halves? how thirds? apples, many how thirds? apples, many how fourths? dollars, many 8J to an improper fraction. 12f to an improper fraction. 15f to an improper fraction. to an improper fraction. 26^ to an improper fraction. 3^ to an improper fraction. 46| 21^i| to an improper fraction. lyVA ^^ ^'^ improper fraction. 14^^ to an improper fraction. lOy^ to an improper fraction.

f J | ^^^-^ ^ ^^^~^^-p ^"Ml^ |^^| "Ti^ tVt

CASE

ITT.

105.
mixed

To

reduce

an

improper fraction

to

an

or integer

number.

1. In

of

an

apple, how
are

many

apples?
OPERATION.

Solution."
6

There
are

2 halves

in 1

apple; then, in

2)6
~3

halves, there

6^2=3

apples.

COMMON

FRACTIONS.

139

2. In

f
"

of

dollar,how
are

many
in 1

dollars?
OPERATION.

Solution.

There
are

4
-=-

fourths
4
=;

dollar;then,

)9
2^

in 9 fourths,there

2^

dollars.

Rule.

"

1. Divide be the

the

numerator

by

the denominator number.

; the

will quotient
3. In 4. In

or integer

the mixed

5. In
6. In

7. In 8. In 9. In 10. In

f of an apple,how -1^of an apple,how ^-^of a dollar,how y^ of- a dollar,how ^ of a bushel,how

many many many many many many many

apples? apples?
dollars? dollars? bushels? dollars? ounces?

2.

3.

^ of -2^of -5^of

a an a

dollar,how
ounce,

$3|. S3f. 2^ bu. ^^y^ij^.


8^
oz.

how

dollar,how
to
a a a an an a a a a an a

11. Eeduce

^^

dollars? many number. mixed mixed mixed number. number.

$131

12. Reduce 13. Reduce 14. Reduce 15. Reduce 16. Reduce 17. Reduce
18. 19.

Reduce Reduce

20. Reduce 21. Reduce

^\^ to ^^ to ^^^ to ^^U^ to ^j^- to ^^ to to ^-^-^ ^-f^ to ^^^- to -^^ to

18|. 15|. 25|^i.


40. 31.

integer. integer.
mixed mixed mixed mixed number. number. number.

number.

l^y^T* 46-j^. 2im. 6^T%199.

integer.
mixed

number.

I^tot*

CASE

IV.

106.
A both

To

reduce

fraction

to

higher
terms

terms.

fraction
terms

is reduced

to

higher

its value

number. by the same (Art. 101, Prin. 5).

This

by multiplying does not change

140

RAYS

NEW

PRACTICAL

ARITHMETIC.

1. Eeduee

|
"

to

thirtieths.
OPERATION.

Solution.

30

divided

by

5 is 6.

Multiplying

3 0-t-5=:

both terms

of

| by 6, the

result is

ff

6X4

24

Kule.

"

1. Divide the

the

denominator required

by

the denominator

of
2. the

givenfraction.
terms

Multiplyboth
result will be

the

fractionby fraction. required


of

the

the

quotient;

COMMON

FRACTIONS.

141

CASE

V.

107.

To

reduce

fraction

to

its lowest

terms.

1. A both

fraction

is reduced

to

lower

terms

terms

its value. 2. A

number. by the same (Art.101, Prin. 6).


is in its lowest
are

This

does

by dividing not change


the
ator numer-

fraction

terms
to

when each

and

denominator

prime

other.

(Art.

83, 8).
1. Eeduce

|^

to

its lowest

terms.

First
Solution.
2

Method.
factor of
24

"

is

common

and

30

operation.

(86, 1 ). Dividing both terms of |^ by 2, tbe result is -^f. 3 is a common 15 (86, factor of 12 and of ^| by 3, the result is 2). Dividing both terms f 4 and 5 are prime to each other.
.

2 4
_

1 2

^~30~T5'
12
o\
~~

1 5

Rule.
common

"

1. Divide' both terms

of

the

givenfractionby any
the
same

factor.
the

2. Divide 3. So
terms
are

fractionin resulting
to divide until to each
a

manner.

continue

fractionis

obtained

whose

prime

other.

Second

Method.
OPERATION.

24)30(1
Solution.
24 and
30
"

The

greatestcommon
terms

divisor of of

2 4 2 4(4 ~6~) 2 4

is 6.

6, the result

Dividing both is -|-.

|J by

'30

142

RAY'S

NEW

PRACTICAL

ARITHMETIC.

Rule.

"

1. Divide

both terms

of

the

givenfractionby their

common greatest

divisor. be in its lowest terms.

2.

The

fractionwill resulting

108.
common

To

reduce

two

or

more

fractions

to

their least

denominator.

1. Two when 2. A is
a

or

more

fractions the
same

have

common

denominator

they
common commx)n

have

denominator. of two their


or more

denominator

fractions

multiple of

denominators

(83, 11).

COMMON

FRACTIONS.

143 of

3. The fractions

least

common

denominator
common

two

or

more

is the

least

of multiple

their

inators denom-

(83, 12).
1. Eeduce

I, and
,

to

their least

common

de-

nominator.
OPERATION.

2)4

6 3

9 9 9

12

2)2
Solution.
"

The

least

common

3)3 4,

multipleof
6, 9, and
to
5

the denominators is 36
must

12

(90).
be

Each

fraction,then,
3 0
"

reduced

thirty-sixths (106).
8 3 2
"

i
"

U*

-6

3^'

"

^S'

nnc\ ^^^^

3 3

T2

3T-

Rule.

"

1. Find

the L. least

C. M.

of

the

denominators

of

the

fractions for their


2. Reduce

common

denominator. another

each

fraction to

having this

ator. denomin-

Kem.

1.

"

Integersmust
Before
to

be

reduced

to the

common

denominator

by

Art.

103,Rule.
2.
"

Rem.

be reduced Rem. Rem.


3.
"

mixed commencing the operation, improper fractions (104). fraction must


or more same

numbers

must

Each

be in its lowest may be

terms

(107 ).
to any
common

4." Two

fractions way.

reduced

denominator

in the

Eeduce
2. 3. 4.

to

their least

common

denominator:

144

RAY'S

NEW

PKACTICAL

AKITHMETIC.

17.
18-

2i, 3i, 4i, 5. A, ii, ii, if. If


TTTfJ
TT""

Trf

"

TTT'

ITT*

!""

T'

TTT'

TT"

A" TffTtVtT' 126"I5"' 12


85"

6 0'

TTslj'' T^VlTTJB^TF'
^17'

2^-

fj

iV' A'

ii' if' ff^T)T7' 71)^17' "llir' ^TTTTj

luiT'

ADDITION

OF

FRACTIONS.

109.
the
two
sum

Addition

of
or

Fractions
more

is the

of

two

fractional

process of finding numbers. There are

cases.

CASE

I.

110.
1. Add

When

the fractions

have

common

denominator.

I
The

and

f.
OPERATION.

Solution."
is fifths,

sum

2 fifths, and of 1 fifth,

2 _[_^

6
r=

6 fifths,

are

equal to 1^ (Art. 106).

l
=

COMMON

FRACTIONS.

145
the same,
1

Explanation. express and in


one

"

Since

the denominators
same

are

the

ators numer-

parts of the

size;therefore,add
3

2 fifths, fifth,
sum,

as fifths, you would add 1 cent, 2 cents, and in Ihe other,6 cents. case, being 6 fifths,

cents; the

Rule."
common

1. Add

the numerators;

under

the

sum

write

the

denominator.
1

Rem.

"

The
a

if an result,

or integer,

mixed The

number result must

must improper fraction, (Art.105).

be reduced

to

an

Rem.

2.
"

be

reduced to its lowest

terms

(Art.

107).

3. Add 4. Add

I,
4,
-3_

I
4,
_7_

3^
i,
_8_

f
I.
iil.
9

If
22.
6

5. Add
fi

Add

7. Add 8. Add
QAHH-JL1113 V. ^UU

-i-V, T%, AtV" a. H,


2 0?

Hif17

2t^ 2f
91 ^"^^

"20"?

2U?

"2"0^-

10. Add

if, il

if, ||.
CASE II.

2f

111.

When

the

fractions

have

not

common

inator. denom-

1. Add
Solution.
to
a common
-=

I, I, and
"

|i.
the fractions
operation.

Reducing
denominator
=

ff^f fi and of f^, ff,and sum equalto 2 f|. I


=

(Art.108), H !l; then, the |f is |f || are


.

| ^^ff f ff ih ff + f| + H U^^U
=

II

||

Explanation.
do
can

"

Since

the denominators

are

the different,

ators numer-

not

express parts of the same be added till they are reduced


not Prac. 10.

size;therefore,the fractions
to
a common

denominator.

146

RAYS

NEW

PRACTICAL

ARITHMETIC.

Bule.

"

1. Reduce the

the

to fractions

common

denominator.
write the
mon com-

2. Add

numerators^and

under
,

the

sum

denominator.

Rem. their
sums

1.
"

Integers and
then

fractions

may

bo

added

and separately

united.
the fractional

Rem. may

2.

"

The

be added

and integral and separately

parts of mixed

numbers

their

sums

then united.

A-

Solution. write the the

"

The

sum

of column of

\ and

is

|; JrrrlJ;
carry of 1, 3,
"

operation.

under column

the

of fractions and
sum

1 to the

The integers.

2\ 3^
6^ A7y.s.

and

2 is 6.

9. Add 10. Add 11. Add 12. Add 13. Add 14. Add 15. Add 16. Add 17. Add 18. Add 19. Add 20. Add 21. Add

^'

"'

2\.

2A\. 10f|.

1.

21H9f|.

COMMON

I'KACTIONS.

U7

SUBTRACTION

OF

FRACTIONS.

112.

Subtraction

of

Fractions two

is

finding the
There
are

difference
cases.

between

of process fractional numbers.

the

two

CASE

I.

113.

When

the

fractions

have

common

ator. denomin-

1. From

f
"

subtract

^.
OPERATION.

Solution. sevenths.

2 'sevenths

from

sevenths

leaves

"

Explanation.
erators

"

Since

the

denominators
same

are

the

same,

the

num.

express

parts of the
as

size; therefore,subtract
subtract
2 cents

2 sevenths
mainder, re-

from

5 sevenths

you

would

from

cents; the

in

one

case,

being 3 sevenths,in
the

the other 3 cents.

Kule.

"

1. From

greaternumerator
common

subtract

the

less;

under

the remainder

write the

denominator.

2. From 3. From
4. From

5. From
6. From

| subtract \. |. | subtract | subtract |. subtract f\. -^-^ 3^ subtract If.


operation.
"

\.

\. \. \.

Solution.
1 from

3.

1 2

f can not equalsf ; f


from
8

be taken and

from

are

J; so borrow f ; | from f leaves

3^

lf_
l^
Ans.

I; |

"

J.

leaves 1.

7. From 8. From
9. From

4^

subtract

8-I subtract

23^

subtract

2|. 3f 17^^

H4|. ^.

UB

KAY'S

NEW

PRACTICAL

ARITHMETIC.

CASK

II,

114.
inator.

When

the

fractions

have

not

common

denom-

1. From
Solution."
common

yV

subtract

|.
fractions
=

the Ri^diicing

to

OPKRATION.

denominator

(Art. 108), f
from

and

T%

H;

then, f^

|-Jleaves

^J ^%

Explanation. do fraction
to
a

"

Since express he

the

denominators
the the
satne

are

not not

parts of
from

can

subtracted

other

the numerators different, size; therefore, one till they are reduced

common

denominator.

Bule.

"

1.

Beduce

the

fractions to

common

ator. denomin-

2. Fro7n

the

greaternumerator
icrite the
common

subtract

the

and less,

under

the remainder

denominator.

.8
15*

fi-

t's
tt-

Solution. be taken

"

from
arc

f; "

fi^ndI

\ equals |, and | equals |. | can Tiot f ; so borrow 1 from the 3. 1 equals |; | from | leaves ". 2 from 3 leaves 1,

OPHUATloN^.

3|

If

COMMON

FKAOTIo/s.

149

If

17. From 18. From 19. From 20. From

4^i^subtract
56^ subtract subtract

3^.
421

2H4|
14_i__.

60| 97|

subtract

41^^. 48|.

19f 48|.

MULTIPLICATION

OF

FRACTIONS.

115.

Multiplication

of
two

Fractions
or more

is the fractional will 3

process numbers.

of

the product of finding 1. If 1

apple cost I

of

cent, what

applescost?
OPERATION.

Solution.
=

-L2_

of

of a cent They will cost 3 times -| cent (Art. 101, Prin. 1). -i/equals
"

|X f y-

"' 2f

Explanation.

hence,

3 times

apples ^-=y,^-.
"

will

cost

| -j| -f |

of ^^z.

cent;

2. At cost?

12

ct.

yard, what

will

of

yard

of

ribbon

Solution.

then, I

of

^t. \ of a yard will cost \ of 12;=rij2 will 2 ^ cost times -V" ^ ^^- ( ^^^- )' yard
"

operation.

"

44.

V" X f "".2^4^44
=

3. What pe**

will

of

yard

of cloth

cost,at | of

dollar

yard?
will cost
4

dollar;then, ^ of
a

yard

times

^3^ i|
=

of

|X

t=

35

dollar.

15a

RAY'S

NEA^
"

PHACTICAL

ARITHMETIC.

Explanation.

of

I of

dollar

is

-^^of
a

dollar

then, }

of

of

dollar is 3 times

-^^
=

/^ of

dollar

(Art. 99); (Ex. 1).

4.

MuUiply f by f
"

Solution.

J
4

by multiplied pliedhy ^ of

(Art.96). f is f (Art. 101, Prin. 1); then, f multiis J of 1 ^5 (Ex. 3, Explanation).


same
as

is the

of 4

operation.

f n^ |_

^8^

the numerators of the given Multiplytogether fractions for the numerator of the product. the denominators 2. Multiply of the given fractions together for the denominator of the product.
Rule.
"

1.

Rem. Rem. Sometimes and

1.

"

in Express integers
"

the form
to

of fractions

2.

Reduce

mixed

numbers

improper fractions
to

convenient be more it may fractional parts separately.


3.
"

(Art.97, 3). (Art. 104). the by multiply integral


Rule for Cancellation

Rem.

Indicate

wherever

and apply the operation it is practicable (Art. 91, Rule). the

5. 6. 7. 8. 9. 10.

Multiplyf Multiply8 MultiplyI MultiplyI Multiply5 Multiplyf


"

by by by by by by

3.

|. f.
4.

f |.
.

2\ ^ ^, 2|, 3|
operation.

Solution.

plying

the

Indicatingthe operationand apRule for Cancellation (Art. 91),the

|X f
^

"

"

result is ".

11. 12.
13. 14.

Multiply I 20 Multiply Multiply^\ Multiply I

by 6. by f. by if by 10.

4. 15.

H6.

COMMON

FRACTIONS.

151

15. 16. 17. 18. 19.

Multiply 12 by Multiplyj\ by Multiply f by

|. f.
6.

2JL
91-

Multiply 7 by Multiply21 by
"

|.
3i.

To.

proper Reducing 2 J and SJ to imfractions (Art.104), they are | and |. Multiplyingtogether | and |, the result is Solution.
6-3
^:^

OPERATION.

71^

--n
8.
OPERATION.

20.

Multiply18f by

144

150.

144

150

21. 22.
23. 24. 25. 26.

27. 28.

Multiply 8 by Multiply 2^ by Multiply10| by Multiply 25 by Multiply -^%by Multii:)ly lOf by Multiply 64 by Multiply 8f by

3|. 2|.
7.

29i. H7^.
215.

8|. 17^.
9.,

15^. 97|.
568.

8f. f

3f.

: Multiplytogether

29. 30. 31. 32. 33. 34. 35.

TJ'

16'

2Ali *94091.

-A2-1"^16'
IT'

6|, 2|, 21. 2h 3|, 4f, If 2i,2A, 3i,.lT^.


i
8'
1

49if
22.

_3_
10? 9 4

8.
9? 7

^
3'
.5

^'
9^'
4'

T*
2 3'

fi
""

4'

T'

-^J

152

KAY'S

NEW

PRACTICAL

AKITHMETlC.

36.

37.

f f, If, h h h h 20. 2i, 6|, ^, ^3, 2, f.


Fractioiuil

24.

116.

parts

of

integersare

obtained

by

multiplication.
1. What is

of 2?
OPERATION.

Solution." 2 times

J of 2 is f (Art.96); then, f i. i^^l

of 2 is

i=n

3f. 2f
8. 10.

12f 18f Hi.

Compound fractions (Art. 99) simple fractions by multiplication.


117.
1. Reduce

are

reduced

to

of

to

simple fraction.
operation.

Solution. the

"

result is

Multiplying | by f (Art.115, Rule), j%.

2y^*

j\

2. Reduce 3. Reduce 4. Reduce

^ f
i

of
of of of of of

5. Reduce
6. Reduce

7. Reduce

^^ f I

f to a simple fraction. |^to a simple fraction. | of 2f to a simple fraction. f to a simple fraction. f to a simple fraction. to a simple fraction. 4 of 1|^

3 3
14

3^
15

"3T'

COMMON

FRACTIONS.

153

8. Eeduce 9. Reduce 10. Bediice 11. Reduce

f
^

of of of of

f f
^
^

f of i to a simple fraction. | of f to a simple fraction. 2T of fraction. |- |^to a simple | of y7_of |f to a simple fraction.

12. Reduce 13. Reduce 14. Reduce 15. Reduce 16. Reduce 17. Reduce 18. Reduce fraction.

of of of of of of of

f f ^3 ^ ^

simple fraction. "^, | of 1^ to a simple fraction. 1 ^ of l\l to an integer. 2, 2f of If to an integer. to a simple fraction. -/^of 1|^ 4 of | of 5 to a simple fraction. \, ple | of f of | of f of | of j\ to a simiV*
| |
to
a

of

Miscellaneous

Examples.

118.
1. Of 2. Of

What

will be

the
at

cost

2^
3

lb. of

meat,

'

Of 5 yd.? Of yd. linen,at Of 61 yd.? 5fyd.? 3. Of 3^ lb. of rice, at 4|-ct. a lb. ? 16 ct. 4. Of 3| tons of iron, at $18f per T.? $60. 5. Of If yd. of muslin, at $^\ per yd.? ${. 6. Of 21 lb. of tea, at $f per lb.? $2. 7. Of 5| cords of wood, at $1| per C? $6|. 8. At the rate of b^ miles an hour, how far will a man travel in 7f hours ? 42| mi. 9. I own I of a steamboat, and sell f of my share : what f part of the boat do I sell? what 10. At cost | of a pieceof cloth $6| per 3^ard, containing5^ yards? $8^. what? 11. f of I of 161- X I of I- of 15 34f. 12. What is the sum of f + J and | X I"? l^'
' .

l^ ct. a $| a yd.?

lb.?

30f ct. 7 yd.? S3f

=:

154

RAY'S

NEW

PRACTICAL

ARITHMETIC.

DIVISION

OF

FRACTIONS.

119.
the

Division

of two

Fractions

is the

process

of

finding

of quotient

fractional numbers. ribbon


cost

1. If 3

yards of

of

what dollar,

will 1

yard

cost?
OPERATION.

Solution.

"

yard will

cost

of

f =f

of

dollar
^

(Art.117).
Explanation.
"

?Xi-f
Each

will be

^ is to be divided into 3 equal parts. f -f-^-f ^. (Art.101, Prin. 2); for f


:==

part

2. At
can

2 dollars

be

bought
"

for

yard, what part f of a dollar?


\
a

of

yard

of

cloth

Solution. and for

For
a

1 dollar

of

dollar of
a

117); then, | yard can be bought.


Explanation.
a
"

for

yard can be bought, \ of \^^^^ of a yard (Art. dollar 3 times -^^^-^^ of a

operation.

|X|

Were

it

requiredto
for $6, then
a

find how
6

yard, could
to

be the To

bought
part of
divide

would

yards,at $2 many be divided by 2;

hence,
divided

find 2.

by

yard that $| will pay for,\ must be \ by 2, multiply the denominator (Art.

101,

Prin.

3).

3. At

of

cent

for 1

how apple,

many

can

be

bought

for 4

cents?

Solution.

"

For for

bought, and
^=

J of a cent |, or 1 cent,
can

\
3

apple can times | | of


an
=

be
an

operation.
"

apple; then, for 6 apples. I


4. At

4 cents, there

be

bought

4 times

fX J

be

f bought

of for

cent

for 1
a

apple,how

many

apples can

of

cent?

COMMON

FRACTIONS.

155
be

of

cent

an

apple
i
= t=r

can

operation.

bought, and for |, or 1 cent, 3 times | of an apple; then, for \ of a cent J of f | of an apple be bought (Art. 117),and for | of a cent 3 times can 1-1-apples. f
=

|X 1 f 1 1}
= =

5. Divide

| by
"

4.

Solution.

f
4

is the is

same

as

of

(Art.96).

operation.
=

divided

by
i

divided

by

of

j\ (Art. 101, Prin. 3); then, f sy^^ 4Js 5 times j\ Ex. (Art. 115, i^ 1).
=

\^

Rule.

"

the Mulflply

dividend

by

the divisor with

its terms

inverted.

Rem. Rem. Rem.

1,
"

in Express integers

the form
to

of fractions

improper Indicate the operation and apply the 3. whenever it is practicable (Art, 91, Rule).
" "

2.

Reduce

mixed

numbers

fractions Rule

(Art.97, 3). (Art.104).


lation for Cancel-

6. If

yards
a

of

muslin

cost

of

what dollar,

will

yard
7. At

cost?
i

cent

each,

how

many

applescan
many

be

Sf bought
6.
lin mus-

for 3 cents? 8. At
can

^
be

of

dollar per for


cost

how 3^ard,

yards of
of
an

bought

Sy% ?
3

^'
part
many orange

9. If could be

1 orange

cents, what

10. At
can

purchased for ^ a cent? f of a dollar per yard, how

\. yards of
cloth

8. buy for 6 dollars? 11. At yards of \ of a dollar per yard, how many ribbon be purchased for f of a dollar? can 3f. what will 1 12. If 7 pounds of rice cost i|.of a dollar, pound cost? $^. you

156

KAY'S

NEW

PRACTICAL

ARITHMETIC.

13. Divide
Solution.

4^-by If
OPERATION.

Keducing 4^ and (Art.104), we have | and


"

tions IJ to improperfrac|. Dividing| by J,

li=J
Y
=

the result is 31.

3|

f4. 40.

fIfIf
8.

^1 I

HA25.

lOf Aoff
Explanation.
"

Invert
case

the terms

of both

operation.

^ as I simple fraction.
31. Divide 32. Divide 33. Divide 34. Divide 35. Divide

and

in

the

of the divisor

being

jXfXIXi"

f
i

of

f by f

of

by f Of 17^. j\ of | of 12f^ by ^ of 8|. f of | by f of ^ of 5. by i of 4^^ j\ of f of 12^3^


part
one

of 5^

5 6*

iof 20.

120.

What

number

is of

another

is found

by

division.

COMMON

FRACTIONS.

157

1.

1 is what
1

part
is
i

of 2?
2

Solution.

"

of

2; for | of

is

|,or

(Art.

OPERATION.

98, 2d).
2.
2

:Xi
is what
1 is

part
^
of

of 3?
2

Solution. of 3.

"

3; then,

is 2 times

1=1

OPERATION.
2 1

1
3

^3

/\

3.

is what
1 is
i

part
of

of 3?
OPERATION.
i

Solution.

"

3; then,
of

is

J of -J- i

of 3.

4.

is what

part
is
i i

f?
|, or
|
of

Solution.

"

of is
i

t-l =n:| of
times
imoQ

4 J

f; then, 8 /-"f 3 of f I
.

f, and of |
=

1, is

times

OPERATION.
2 3 \/

|, and

|-is 2

4 3

8
"

"

5. 6. 7. 8.
9.

is what
is what

10. 11. 12.

f ^ I 3| I 8| fi

is what is what is what is what is what is what

part part part part part part part part

of 4 ? of 5? of i?
2-

of

|?
f?

5^3 1 5 16-

of 5 ? of

of 11? of

I.
9-

|f?
are

_9

Complex fractions,(Art. 100) simple fractions by division.


1. Eeduce

121.

reduced

to

-|to

simplefraction.
OPERATION.

Solution."

Reducing IJ and
and

2i

to

tions improperfrac91
^3
"
"

(Art.104), we have { (Art.119),the result is if.

|. Dividing | by J

^
-i

158

RAY'S

NEW

PRACTICAL

ARITHMETIC.

2. Eeduce

-^
2

to

simple fraction. simple fraction. simple fraction. simple fraction. simple fraction. simple fraction.
mixed number.

^. y2^. ^. ^J|.
J|.

3. Keduce

-^
2
"

to

4. Keduce

to

31

5. Reduce

j| to
21

6. Eeduce

-^ to
"

7. Eeduce

to

|.
44.

97

8. Eeduce

-r^ to
8J
^^
75

9. Eeduce

^^

to

mixed

number.

If. |^.

10. Eeduce

to

simple fraction.

Miscellaneous

Examples.

122.
silk
can

1. At

be

^ a bought
a

dollar per for $3^? per

yard, how

many

yards pounds

of

6^.
of

pound, how many be purchased for ^2^? tea can 3. At 3| dollars per yard for cloth,how be purchasedwith can $42^? must 4. By what that | be multiplied
2. At

of

dollar

many
the

may

be

10?

5. Divide
6. Divide

3f by f of If ^ of 271 by ^

3|. yards 11^. product 26|. 5f


I29.

of

21f

7.

Multiply

li by i.

A-

COMMON

FRACTIONS.

159

8.

Multiply
li

JAof 5^ by ^.
'T2 ^T^
6

|".
2
32-

2i
.

9. Divide

^2 by -f
"

10

Divide

bv

"

^.

11.

FRACTIONAL

COMPOUND

NUMBERS.

$16j\', $9-1; $53-V; $2j|. SSB^-V for paper, ^4^7^; 2. I paid for books for a slate, $9|-; did I expend? $|; for pens, $lf; what amount S15^. 3. Having $50^, I paid a bill of $27-^^:how much had I left? S23Jg. 4. From $32.31^take $15.12i. $17.18f. 5. From ^4.621 $5.81^take $1.18f.
Find 6. 7. 8. the 9
21 cost

128.

1. Add

of 121
ct.
a

yd. of muslin, at
lb. of sugar, at at yd. of cloth,

yd.

6^

ct.

lb.

S1.12f $1.31f

$47,811 $3.18fper yd. 51 yd. of linen,at $0,621 per yd. 9. $3.43|. 121 10. yd. of ribbon, at 18| ct. per yd. $2.34|. 11. 131 yd. of calico, at 16| ct. per yd. $2.25. 12. 101 yd. of cloth, at $3,371 a yard. $34.59f. 13. $66.25. 17| dozen books, at $3.75 per dozen. 14. At 18| ct. per yard,how yards of muslin can many be purchased for $2.25? 12 yd. 15. At 371 ct. per bushel, how many bushels of barley can you buy for $5.81|? 15^ bu. 16. If 5 yards of cloth cost $11.56^, what cost one yard? $2.31^. 17. Seven share $31.06^ equally: what is the men share of each man? $4.43f.
,

15

IHO

KAY'S

NEW

PKACTR

AL

AKITHMETIC.

18. Eeduce 19. Keduee


20. 21.

5 mi.
2 20

to

inches.
feet.
^

316800 10595 3 rd.


3 mi.

in. ft.

mi.

2 rd. 2 ft. to rods. miles.

Reduce Ecduce

yd. to

3| yd.
in.

15875
U2634

ft. to

2 rd. 2 ft.

22. Reduce 23. How take in

in. to miles.

2 mi. 80 rd. 2

yd. 2
a

many

steps, of
2 miles?

ft. 8 in.

each, will

man

walking

3900.
a

will revolutions many circumference,make in running 65 24. How 25. Reduce


1 A.

wheel,
miles?

of 9 ft. 2 in. 37440.

136

sq. rd. 25 sq.

yd. to

26. Reduce 27. Reduce 28. How wide?


29.

7506

1 A. 88 sq. yd. to A. 5 chains 15 links to in.


acres

many
4

in

field

40^

rd.

yards. 8979 sq. yd. sq. rd. 4 sq. yd. 4078| in. long and 32 rd.
square 8 A. 16 sq. rd. hr. 35064 da. 4 hr,

Reduce

years

to

hours.
cen.

30. Reduce 31. In the


moon,

914092 time the

hr. to will of
a

cen.

4 yr. 101

what
at

body
31

move

fi'om the

earth to
distance

rate

miles per

day, the
21

being 238545
124.
A

miles?

yr.

24| da.

fraction

is reduced Rule

to

lower

denomination

(Art.63, by multiplication
1. Reduce

I).
of
a

J^
"

of

peck

to

the fraction
the The

i)int.

Solution. fraction of result is


u

of

.}^of a jx'ckto pint,multiplyby 8 unci by 2. a pint.

To

reduce

opkration.

^
^

X I X

=-

2. Reduce 3. Reduce
4. Reduce

5. Reduce
6. Reduce

bu. to the fraction of a quart. -^^ -^^lb. to the fraction of an ounce. yL.lb. Troy to the fraction of an ounce. ^ rd. to the fraction of a foot. "^' ^^ ^^^^ fraction of a square rod. y^Vo"

|. ^. |. |^. |.

COMMON

FK

ACTIONS.

161

7. Reduce
8. Eeduce 9. Reduce

$3!^ to the fraction of a cent. y^^g^da. to the fraction of a minute. -^^ bu. to the fraction of a pint.
reducing a
result is of the of
a a

f. |^. |.

125.
when the

In the

fraction mixed

to

lower

denomination,
with ing is called find-

number, proceedonly
in

reduction the value

fractional part. This

fraction

integers. day
to

1. Find
Solution.

the
To

value
reduce

of

f
a

of

in

integers.

"

| of

day

tiply hours, mulof


an

by 24;
hour

min.

9| hr. To reduce | to minutes, multiply by 60; the result I of a day, then, is 9 hr. 36 min.
the result is

is 30

2. Find 3. Find 4. Find

the the the the

value value value value

of 4 of of of

mi.

5. Find

in $|f mi. ^ lb.

integers. integers. in integers. Troy in integers.


9
oz.

in

12

pwt.
75 lb.

6. Find

the the the

value value value

of of of

7. Find 8. Find

-^^T. in integers. | A. in integers. of -J of 63 gallons

8 cwt. 100

wine
55

sq. rd. in integers.

gal.1 pt.

12G.

fraction

is reduced Rule

to

higher

denomination

by

division

(Art.63,

IT).
the fraction of
a

1. Reduce
Solution." tion of is
a a

|
To

of

pint to
| of
2 and
a

peck.
operation.

reduce

2V of

peck,divide by peck.
4 4
11.

pint to the fracby 8. The result

| X ^X i

2?

2. Reduce 3. Reduce luce


Prac.

qt. to

the

fraction fraction

of of
a

bushel. rod.
y

^.

ft. to the

162

RAY'S

NEW

PKACTICAL

ARITHMETIC.

4. Eeduce 5. Keduce 6. Eeduce

7. Eeduce

^\ oz. to the fraction of a pound. rAirlb. of the fraction ton. to a ^ t^Vttf pt.to the fraction of a bushel. -^^. ^ oz. to the fraction of a hundred-weight.
2800-

8. Eeduce 9. Eeduce 10. Eeduce

f | min. yf^ ^^-

in. to the
to

fraction

of

rod.
a a

^J^

the the

fraction fraction

of of

to

day. rwruhundred-weight.

127.

To

find what them


120.

part
to

one same

compound

number and

is of

another, reduce
as

the

denomination

ceed pro-

in Art.

1.

2 ft. 3 in. is what

part

of

yard?
OPERATION.

equals27 in. I yd. equals36 in. 27 in. are |J of 36 in. fj then, is } of a yard. equalsj. 2 ft. 3 in.,
Solution.
"

2 ft. 3 in.

2 ft. 3 in. = 1

27 in.

yd.=r=36in.

ii

2. 3.

ft. 6 in. is what

pk. 4 qt. is
part is part of part part
has sq. of of

what 2
a a a

part of 6 ft. 8 in.? part of a bushel ?


of 8

4. What 5. What
6. What

yd. 9 in. day is 13


mile

yd.

ft. 3 in. ?

| | ^

hr. 30 rd.?

min. ?

^
fj
"

is 145

7. What 8.
9. A

yard
of

is 2 ft. 8 in.?

15 mi. 123
man

rd. is what
a

part
168

of 35 mi. 287 A. 28

rd.?

f.

farm

37

A.

94

rd.,what
a

part

of his

sq. rd. ; if he sell farm will he dispose

of?
10. What 11. 2

A\part of

pound

is

7^

oz.

?
1

^.
qt. If pt.?
1 6

qt. lipt. is

Avhat

part

of

1 bu.

12. 1
? in 1^"

yd.

1 ft.

1^

in. is what

part

of

yd. 2

ft.

8f

1 91 " 5 4T3'

COMMON

FKACTIONS.

163

128.

To

add

and

subtract of the

fractional in

compound

bers, num-

find the value

fractions

proceed
Numbers.

as

in Addition

and

and then integers of Compound Subtraction

1. Add

I yd. and
"

ft.
OPERATION.

Solution.

-|yd. equals 2
2

ft. 3

in.; | ft.
10

yd. :rrr2
ft.
=

ft. 3 in.
10

of equals10 in.;the sum in. is 3 ft. 1 in. (Art.75).

ft. 3 in. and

in.

3 ft. 1 in.

2. From

|
"

da. subtract

hr.
OPERATION.

Solution.

da.

equals5
50

hr. 20 min.; subtracted hr. 30 min.

hr.

equals 50 min.;
5

min.
4

I I

da. ==5 hr.


=

hr. 20 50
4

min. min.

from

hr. 20

min.

leaves

hr. 30 min.

(Art.76).
3. Add
4. Add

5. Add

I \ I

da. and wk. wk.


i

f
da.

hr.

16

hr. 45 da. 15

min

da. and

hr.

min

hr. and

min.
5

da. 6 hr. 40
3

sec

6. Add

7. From 8. From
9. From

\^ gal.and ^2 ^t. ^ da. subtract Jg hr.


H
subtract

qt. 1 pt. 2 gi.


40
sec.

18 hr. 36 min.

SA.

55 ct.

10. From

| lb. subtract ^ oz. | da. subtract ^ hr.

2 hr. 34

min.

^oz. 17| sec.

129.
1. Reduce
2

Promiscuous
to its lowest
^2

Examples.

y g|j
-^ 8

terms.

a-

Arid

91

3. From 4. From

34^ subtract

3| f
of

subtract

5. Add

j\

and

1^. ^ of 3^. | of -j^.

16^

RAY'S

NEW

PRACTICAL

xVRITIlMETIC.

G. Add

1| -^

and 2.1-

-^ 3i

2f|.
will

7. What

niiinber

divided

by f

give
will

10

for

tient? quo6.

8. What

number

multiplied by |
is

give

10

for

product?
9. What the itself, 10. What number remainder

that,from
16?

which

if you

take

16|. ^ of
28.

will be is

number
sum

that, to
a

which

if you

add

j^ of
14.

the itself,
11. A sells

will be is worth
:
*

20?

boat

S900 ;
what

merchant has he

owns

of

and it,
is it

of his share

part

and left,

what

worth?
12. I
own

j^
part

of

$1944|: what
13. What 14. What 15. From

is the of

ship, and sell whole ship worth? 3 cents is | of 2 cents?


a

worth $375. 3^ left, ^ of my share for

810000.

part of 368

is 170? the
sum

|. i|.
1 007

|^

subtract

of

and Jj^y. ^, y^y,

of ^^ of 4yV y\. -^^ 17. From 2%| ^ f subtract | -^ |f rods in y^^of an 18. If I ride 2044 hour, at that rate how far will I ride in 1|4 hr. ? 8468 rd. 19. What part of 1^ feet are 3^ inches? |. 20. Two men bought a barrel of flour ; one paid S3^, and the other $3| : what part of it should each have ? One ^^^,the other y^^^. 21. A has $2400 ; | of his money, + $500, is | of B's : 16. From
1 subtract
"

what

sum

has

B? divided his
estate

$1600.
among Avith
was

22. John

Jones

sons

and

daughters,the
The share
estate
:

latter
son

sharing equally
$2200,
share which
was

each

other. of the

younger of the

received

-^
the

elder, w^hose
the share

^f

of

whole

find

of each

daughter.

$1356^.

130.

An

aliquot part is Aliquot

an

exact

divisor

of

number.

Parts

of

100.

The used
;

of aliquot following multiples parts of 100 are often 184=^3^, 37i=a, 40=f, 60=f, 624=t, 75=f, 87*=^ will 24

1. What

yd.

of muslin

cost

at

25

ct.

yd.?
OPERATION.

Solution. be

"

Since
dollars

25 ct. is
as

\ of
are

dollar,the

cost

will

4)24
$ 6

as

many

there

yards. \
at

of $24 is $6.

2. I

spent $1,121

for

muslin

12|

ct.

yd. :
OPERATION.

how

many

yd. did
"

? bii}
V2h
ct. is
as

Solution. he
8 times
=

Since
many

of

a are

dollar,there will
dollars. 8 times

1"^
8

as

yards

there

9yd.
8. What
cost

9" yd.

12^yd. of

ribbon

at

]8| ct.

yd?

S2.34g.

166

KAY'S

NEW

PKACTICAL

ARITHMETIC.

4. Paid did I

$2.25 for muslin

at

18J ct.
cost

yd.

how

many
12

buy?
will 5^

yd. yd.

5. What

yd. of

linen

at

S0.62i

yd.? $3.43f.
how many

6. Paid

$66.25
did will I

for books

at

$3.75

dozen

doz. books
7. What

buy? 80 gal. of
of
men

wine

cost

at

17| doz. $2.37^a gal.?


$190.
that each
one ceives re-

8. A

number

divide

$39

so

$4.87^:how
9. What ? 10. How

will 36

there? men are many barrels of flour cost at

8.

$8.33^a
yd.
at
can

rel bar-

$300.

bought for
11. What
an

yd. $246. 66|?


many will
4

of

cloth

at

$1.33^ a
land
cost

be

185 60

yd.

A.

sq. rd. of

$16.50

acre?

Solution. 4 A.
cost
=

"

Since

A.

costs

$1G.50,
160 The A.

operation.

$16.50X4:= $66.
1

Since

$16.50

sq. rd.
cost

A., 40

sq. rd. =

^
66.00
J

of 40

$4,121.
of the total

sq. rd. will be J of $16.50= The cost of 20 sq. rd. will be


of 40 sq.

cost cost

rd.,

or

$2.06^. The $2.06}=

J J

of $16.50 of

4.12^

4.12^

2mI
$72.18J

is

$66 -f $4.121 -f

$72.18|.
12. At for

$18.33^per
will acre?
a

acre,

how

much

land

can

be

bought
mA.

$229,162?
11

13. What

A.

120

sq.

rd.

of

land

cost,
much

at

$125.60
14. At land
can

per

$1475.80.
X 150 how ft.,

$250
be

50 lot, containing for $10000? A. bu. 141


3

bought
6

15. What

will 83

sq. rd. 28 sq. jd. 108 pk. 2 qt. of grass seed

sq. in.

cost,at

$6.20

bu.?

$519.63|.

PKACTICE.

167

16.

At for What At

$0.75 S167.50?
will

bushel,

how
223

raany
bu.
1

bushels

can

be
li

bought
17. 18. for 19.

pk. yard?
can

qt.

pt.

3|
a

yd.

cost,
how

at

$1.75

$6.12f
be

SI.

50

yard,

much

cloth

bought 4f
yd.

S7.12I-?
What will 45 lb.
12
oz.

of

butter

cost,

at

$0,371
$17.15|.

per 20.

pound?
At for What

$0,121

per ?
cost

pound,

how

much

sugar

can

be
lb.
ct.

bought
21.

$2.93f
is the

23-i

of

T.

cwt.

of

wool

at

37^

pound?
22.

$1837.50.
is the
cost

What

of

100

readers

at

$3.90

dozen?

$32.50.
23. dozen?
24. A

What

is

the

cost

of

3f

dozen

knives

at

$5.40

$20.25.
farmer lb. sold
6^

doz.

chickens,
per in

at

$0,331
he
at

apiece,
$36
per

and in

37^
money,

butter,
and the

at

$0.37^
remainder

pound
sugar,
did

received

$0.12^
32i

pound:

how

many

pounds

of

sugar

he

get?

lb.

131.

The
a

orders

of

integers decrease

from

left to

rightin

ratio. tenfold
mmibcr

Thus, in the
hundred

1111, 1 thousand
1 ten

is 10

times

hundred,!

is 10 times

1 ten. and

is 10 times

1 unit.

ORDERS

OP

DECIMALS.

132.
units

1. The

orders

may

be continued
the be
same

from

the

order

toward

the

rightby
units
a

law

of decrease. the order

2. Let that 3.

tlie order

separated from

follows

by

point (.).
number

Then,

in the

1.111,
point is
1

Ist. Since of the

the 1 to the left of the

unit,the

1 to the

riu;ht
order

point is 1
the unit

tenth;

for 1 unit is 10 times from the unit is 1


is 10

J^.
tenth,the second
times

2d. Since from 3d. order 4th, the the


etc.

first order
is 1

the

hundredth; for ^^
order
from

yi^. yoVoto

Since
from In

the

second

the unit is 1 hundredth, the third


10 times

the unit is 1 like


manner

thousandth; for yi^ is


it may be shown

that 1 in the fourth "n"der to


in the fifth order

right from the unit is 1 ten-thousandth; 1 is 1 hundred-thousandth; 1 in the sixth riglit

order

is 1

millionth,

Rem.
as

"

A for

number
the

of figures other consisting

than

1, might be used

well

purpose

of illustration,

(1G8)

DECIMAL

FRACTIONS.

169

4.

The
to

position of
the unit

the

integral and
in the

decimal

phices

relative

ia exhibited

Ibllowinii;

DTACiRAM.

1
\A

1.

^^y^.

/-

.^c)V^

5. The first order order


on

first order
on

on

the

left of
unit the

the is

unit

is

tens,the
the

the

rightof

the

tenths; the second


order
on

the

leftis hundreds;

second

etc. rightis hundredths,

DEFINITIONS.

133,
of

1. A

decimal

fraction,

or

is decimal,

one

or

more

written tenths, hundredths, thousandths, etc.,

like the orders

integers.
2. A
decimal to

point
orders

( )
.

is

placed
from
same

before

the

order

tenths

the distinguish decimal from

fraction. increase and rightto left,


as

3. The

decrease

left to

right
orders

the

the

orders

of

integers.
4. The
names

of

the

of decimals of

are

similar

to

the

names

of the

correspondingorders
of the
common

integers. "^, j^-^,

134.
iiro"?

Conversion

fractions

^^ decimals. ^^^'"""

170

RAY'S

NEW

PRACTICAL

ARITHMETIC.

1.

yi^ j\
4

is

written written

.1
.2

are

TIT
5

.4

TIT

.5

Hence,
order.

i^*/ienthe

denominator

is

10, fAere

is

one

decimal

2.

Yw^

^^ wi-itten in the

.01; there
order.

being no

a tenths, cipher

is written

vacant

.Hence,when
orders.

the denominator

is

100, there

are

mal tiro deci-

3.
no

is yo^oTj

written

.001 ; there written

being
in the

no

tenths

and

are hundredths, ciphers

vacant

orders. .006 .007 .008


.009

lOOlF
3

Att
.003
.004
T7T0"
8 9

^i*e

written

Tirxro

.005

TOGO

Hence,
decimal
4. In

when

the

denominator

is

1000,

there

are

three

orders. like

manner;

OUTFO

is

written

.0001
.00001 .000001

TTr"77"00^

DECIMAL

FRACTIONS.

171

Hence,
the the
same

the
as

number

the number

of orders in the decimal is always of of ciphersin the denominator

common

fractio7i.
and

5.

j\
1

j^-^are
1 1

jW
1
f\V(^

written

.11

1111

'^

1111

Hence,
;

tenths

and

hundredths

are

read
are

as

hundredths;
as

tenths, hundredths,and

thousandths

read

sandths thou-

tenths,hundredths, thousandths, and etc. read as ten-thousandths, are


numerator

sandths ten-thou-

6. The

of

decimal

is the

number

it

presses ex-

the disregarding 7. If there


are

decimal orders

point.
before the

vacant

numerator,
name

are ciphers

written
name

in them. the

8. The the

of

right hand

order

is the

of

decimal. To

Wj^ite
hundred
Number

Decimals.
and thousandths. sixty-five
.265.

135.

1. Write

two

Written.

Explanation.
The 6

"

First, write
stand
in

the

numerator,

265,

as

an

integer.

figure5
be

must

must

hundredths

and
the

is phicedbefore therefore,

(134, 8); then, be tenths; the decimal 2 must point, 2 (133, 2). figure
the order thousandths

2. Write

two

hundred

and

millionths. sixty-five
Written.
.000265.

Number

Explanation.
5 figure
must

"

Write

the

numerator, millionths

265,

as

an

integer. The

(134, 8); then, 6 must be be ten-thousandths, and ciphersmust hundred-thousandths, 2 must be written in the orders thousandths, hundredths, and tenths (134, 7 ); the decimal pointis placedbefore 0 tenths (133, 2 ).
stand
in the order

172

KAY'8

NEW

PRACTICAL

ARITHMETIC.

3. Write

two

huiidrod

and
NuMJJKR

hundredths. Hixty-fivc
Written. 2.65.
as an

Explanation.
5 figure must

"

Write

the

numerator,

2G5,

integer.The

the decimal

hundredths; then, 0 must be tenths; 2 and 6. therefore,is placed between the figures point,
four huiidred and and ninety-eight
two

stand

in the order

4. Write

dred hun-

and

miUlonths. sixty-five
NuMBKR

Written.
the

498.000265.
as

Explanation. the

"

First write

decimal

in

Ex.

2; then write

integer, placingit at
1. Write

the left of the decimal

point.

Rule.

"

the numerator

as

an

integer.
name

2. Place hand
Note. orders

the decimal shall be the


should them

point so
same as

that the the


name

of

the

right

order

of

the decimal.
the from decimal loft to

"

Pupils
as

be

rendered

familiar

with

so

to

name

in supcession, both readily,

and right, Rem. necessary Rem.


1.
"

from When

to right

left. decimal is
a

the

proper

fraction

it is sometimes

to

to prefix ciphers

the numerator is
an

( Ex. 2 ).

improper fraction, the decimal of the numerator point is placed between two of the figures ( Ex. 3). In a mixed 3. Rem. number, the decimal point is placedafter the units order of the integer( Ex. 4 ).
"

2.

When

the

decimal

"

Write 5.
G.

the

decimal following

numbers:

7.

Twenty-six hundredths. himdredths. Thirty-five hundredths. Eighty-seven


hundred and nineteen

8. Four
9. Five

hundredths.

thousandths.

10.

thousandths. Fifty-four hundred and four

11. Three

thousandths.

DECIMAL

FK

ACTIONS.

173

12. Seven thousandths.


13. 14. 15.

thousand

two

hundred

and

-three ninety

Twenty-five and
Tw^o Four hundred thousand and

-seven forty

thousandths.

five ten-thousandths. hundred and

one

twenty -five

ten-

thousandths.
16. Mne

hundred hundred hundred

-thousandths. thousandths. and five hundred-thousandths. three hundred and four hundred-

17. Nine 18. Six 19.

Twenty

thousand

thousandths.
20. Seven 21. Two 22. Three 23. 24.
25. 26.

millionths. hundred hundred and three millionths. and four

thousand

millionths.

Twenty-four ten-millionths. and six ten-millionths. Eighty thousand


Two Two hundred millionths. hundred-millionths. hundred million and
seven

27. Nine 28.

hundred-millionths.
thousand and three

Twenty
One

twenty

dred-milli hun-

29.

million

ten

thousand

and

one

hundred

ionths. mill-

30. One

million

ten

thousand

and

one

ionths. hundred-mill-

31. One 32. One 33. Two 34. Two 35. Two millionths. 36. Two nine

hundred thousand hundred hundred thousand

and and and

six and
one

thousandths. thirty-seven

thousandth.

twenty-fivethousandths. thousandths. units and twenty-five hundred nine and twenty-nine


nine hundred units and

thousand

twenty-

millionths. million and five hillionths.

37. One

174

HAY'S

NEW

PRACTICAL

ARITHMETIC.

38. Two 39. Two 40.

hundred hundred

and

two

ten-biUionths.
two

units and

ten-hillionths. and five inilUonths.

and Sixty-five the

six thousand

Change
4.1
3

followingcommon
9

fractions

to

decimals:

17

2 3

4 1

5 3

'^^^ tW'
A*:{
3

tVh' iWtt' tWttj AVtf' tVV^10


1 5 3 5 0 3

To

Read

Decimals.

136.
Number

1. Read

.2G5.

Read.

"

Two

hundred

and

thousandths. sixty-five

is Disregardingthe decimal point,the number hundred and sixty-five; this is the numerator of the decimal two (134, 6). The righthand order of the decimal is thousandths;this is the name of the decimal (134, 8).

Explanation.

"

2. Read

.000265.

Number

Read.

"

Two

hundred

and

millionths. sixty-five
decimal the

Explanation.
two

"

hundred

and

Disregarding the this is sixty-five;


is

point,the
name

number

is

numerator

of the

decimal.

The

right hand

order

millionths;this is the

of the decimal.

3. Read
Number and

2.65.
Read. Two and

"

hundredths, or sixty-five

two

hundred

sixtyfive
1.

hundredths.

Rule.
as

"

an

2. Give

the decimal pointyread Disregarding integer. the name of the righthand order.

the

ber num-

DECIMAL

FRACTIONS.

175

Note.

"

Before

commencing

to read

the

decimal, the
under

name

of the

righthand
Rem.
"

order should
A

be ascertained be may fraction

(135, Note,
either
as

Rule).
a

mixed
as
an

number

read

an

and integer

or fraction,

improper

(Ex. 3).
numbers
:

Eead
4.

the

decimal following

5. 6. 7. 8. 9. 10. 11. 12. 13. 14. 15.

.341; 2.327; 50.005; 184.173. .0003; .0625; .2374; .2006; .0104. 3.0205; 810.2406; 10720.0905. .00004; .00137; .02376; .01007. .001768; .040035; 70.360004. .1010101; .00040005; .00100304. .31456; .000133; 60.04; 45.1003. 357.75; .4928; 5.945; 681.0002. 70.1200764; 954.203; 38.027. 1007.3154; 7496.35491768. .00715; 3.00005; 28.10065701.
13.0008241094710947.

.028;

Change
16. 17. 18. 19. 20.

the

decimals following

to

common

fractions,

.9; .13; .19; .29; .37; .73. .91; .347; .513; .691; .851; .007; .0207; .00079; .001007. 1.36; .3421; .03401; .0900. .001; .5302; 8.01; .000053.
The

.917.

137,

operations with

decimals

are

dition, Reduction^Ad-

and Subtraction, Multiplication

Division.

REDUCTION

OF

DECIMALS.

138.
without

Keduction

of

Decimals

is There

changing
are

their form
cases.

their altering

value.

four

176

KAY'S

NEW

PRACTICAL

ARITHMETIC.

CASE

I.

139.

1.

Annexing decimal
vdlue.

to ciphers

an

does integer

not

change its

Thus, 7.00 is the

sume

as

7; for 7.00 is 7 and

no

hundredths

(Art.

136, Rule).
2. decimal : from Conversely Omitting ciphers does not change its value. integer
1 of this
case

the

right

of

an

Number and

to evidently corresponds

Case

I, Art. 103,

2 to Case

III, Art. 105.

CASE

II.

140.
its value.

1.

to Annexing ciphers

a.

decimal

does

not

change

Thus, .70 is the 2.

same

as

.7; for y^

"

y^o^.
the

Conversely: Omittingciphersfrom decimal does not change its value.


Number and
1 of this
case

right of

to evidently corresponds

case

IV, Art. 106,

2 to Case

V, Art. 107.

CASE

III.

141.

To

reduce

decimal

to

common

fraction.

1. Eeduce
Solution. fraction is

.75 to
75

common

fraction.
written
operatiox.

"

hundredths

as

common

reduced j^j^^. -^^^

to its lowest

terms

(Art.

.75

-^^-^

107),isf.
Bule.
"

i%=l
1. Write the decimal
as
a common

fraction.

2. Reduce

the fractionto its lowest terms.

DECIMAL

FKACTIONS.

177

2. Reduce
8. Reduce 4. Reduce 5. Reduce 6. Reduce

.6 to

a a a a a

common
common

fraction. fraction. fraction. fraction. fraction. fraction. fraction.


common

.25 to .375 to .035 to .5625 .34375 .1484375 4.02


as

common
common

|. \. f. 2-^^.
T6"-

to to

common
a common a common

7. Reduce
8. Reduce 9.

\
JL9_
12 8-

to
an

Express

integerand integerand

fraction
4
^5U-

10.

Express 8.415

as

an

common

fraction

CASE

IV.

142.

To

reduce

common

fraction

to

decimal.

1. Reduce

to

decimal.

Annexing a decimal cipherto 3, it is 3.0; 30 tenths divided by 4 is 7 tenths, and 2 tenths remaining. Annexing a cipher to .2 it is .20; 20 hundredths divided The by 4 is 5 hundredths.
Solution.
"

operation.

4 ) 3.00 .7 5

result

is .75.

Explanation. decimal
a

"

is 3

divided

by

(Art. 97 ). Annexing

cipherto 3 does not change its value (Art.139). Annexing cipherto .2 does not change its value (Art.140).

B.ule.

"

1. Annex

decimal

to ciphers

the numerator.

2. Divide 3.

there

by the denominator. Point off as many decimal decimal ciphers annexed are

orders

iyi the

as quotient

to the numerator.

2. Reduce

I
2V
12.

to

a a ^

decimal.
decimal. decimal.

.8 .625 .28

3. Reduce
4. Reduce
Prac.

f to
^^

178

KAY'S

NEW

PRACTICAL

AKITHMETIC.

5. Eeduce 6. Eeduce

to -^jj ^| to
yrsir

a a *^ a a a a a

decimal. decimal.
decimal. decimal. decimal. decimal. decimal. decimal.

.075 .9375 .0008 .0225 .00390625 .83

7. Eeduce
8. Eeduce
9. Eeduce

*^

to -^^j^ ^l^ to

10. Eeduce
11.

Eeduce

12. Eeduce

|^to ^ to ^\ to
ADDITION

-\-|-

.09 + .12

OF

DECIMALS.

143.
the
sum

Addition

of
or

Decimals

is the

process

of

finding

of two

more

decimal

numbers.

1. Add

375.83; 49.627; 5842.1963;


"

813.9762.

Solution. four decimal

Write

the

numbers be
in
a

so

that

the

operation.

points may

column, the
the left,

3 7 5.83 4 9.62 5 842.196

units 5, 9, 2, 3 in the first cohimn tenths

to the

7
3

to the right, 8, 6, 1, 9 in the first column etc.;then, adding as in simple numbers, placethe

813.9762

decimal

point in

the

sum

between

1 and

6 under

7081.6295

the column

of decimal

points.

Rule.

"

1.

Write the

the
same

ninnhers order

so

that the decimal stand in the

points
same

and

figuresof
as

may

cohunn.

simple numbers. 3. Place the decimal point in of decimal points.


2. Add
3. Add

2. Add

in

the

sum

under

the column

37.1065; 432.07; 4.20733;


4

11.706. 28 and

485.08983 35 sandths; thou-

and 7

ten-thousandths;
and

8 and

hundredths;

9404

sandths. hundred-thou40.19944

DECIMAL

FEACTIOISIS.

179

4. Find

the
4

sum

of

units

and

25

hundredths;
6913.3477 5.1233. 7231.0967

units

and

5. Add
6. Add

tenths; and 35 hundredths. 21.611; 6888.32; 3.4167. 6.61; 636.1; 6516.14; 67.1234; and
4

10.

7. Add 19

and

8 tenths ; 43 11

and

31 hundredths

; 74

and

thousandths;
8. Add
45
;

and 19

204

thousandths. 7 and 71
; 6

133.333 hundredand 401

and
93

thousandths;
4327

thousandths

and

ten-thousandths

ten-tiiousandths.
9. Add 432

151.49251 and 432

thousandths;
1000

100

and

thousandths; 61 and 793 ten7794 hundred-thousandths; 6.009;


1599.69834
9

and

1001 16

ten-thousandths. and 41

10. Add 33

thousandths;
8 and
969

and

94

millionths and

and

27

hundredths;
204

thousandths; 32
103 and
9

719906
11.

milUonths. Add and


9

100.

ten-thousandths;
and
9099

hun99

42 dred-millionths;

millionths;430

and

hundredths
12. Add

; 220.0000009.

999.99999999

35

ten-thousandths; .00035; 35 millionths, and


.0038885

35

ten-milHonths.

SUBTRACTION

OF

DECIMALS.

144.

Subtraction

of

Decimals two

is the

ing

the

difference 729.835
Write

between

decimal

process numbers.

of find

1. From
Solution. decimal and and

subtract
the be numbers in
a

461.5738.
so

"

that the

two

points may
in the

column, the
the the

units

9 8

operation.

1 in the 5

first column first column

to

the tenths left,

7 2 9.835
4 61.5 2 6 8.26 7 38 12

as subtracting

in

etc.; then, right, simple numbers, place the decito

mal under

point
the

in

the

remainder
of

between

and

column

decimal

points.

180

RAY\S

NEW

PRACTICAL

ARITHMETIC.

Rem.
as

"

The

ten-thousandth
a

occupiedby

phicein cipher(Art.140).

the minuend

may

be

regarded

Rule.

"

1. Write the

the
same

numbers order

so

that

the decimal in the

points
same

and

figuresof
as

may

stand

column.

simplenumbers. the decimal point in the 3. Place column of decimal points.


in 2. From 3. From
4. From

2. Subtract

remainder

under

the

97.5168 20.014 5.03

subtract subtract

38.25942. 7.0021.

59.25738 13.0119 2.915 10.3029 102.00169 .0092 .0476 5.463 2.99997 9.9999 4.875

subtract

2.115. 13.7013. 68.00181.

5. From
6. From

24.0042 170.0035 .0142

subtract subtract

7. From 8. From 9. From 10. From 11. From 12. From 13. From

subtract

.005.

.05 subtract .0024. 13.5


3

subtract

8.037.

subtract

.00003. 19.003.

29.0029

subtract
.125.

5 subtract 1 thousand

subtract

1 ten-thousandth.

999.9999 14. From 15. From


1

subtract

1 millionth.
take 25 millionths.

.999999

25

thousandths

.024975

MULTIPLICATION

OF

DECIMALS.

145.

Multiplication

of

Decimals

is the

process

of

the product of finding

numbers

decimals. involving

146.

Placing the decimal upon the following

point in

the

pends product de-

DECIMAL

FRACTIONS.

181

Principle. The the nvmber

of

decimal

orders in

in the both

product
the

is

equal to

number

of

decimal

orders

factors.
of decimal
=

Thus, let the factors be .2 and


orders in the
the

.03; then, the number product will be three. For, .2=z^q and .0o
.2
"

product of
rX
Too

by
To^o"

.03 will be the

same

But,

and

T^oo

:.006.

y3_. then, of ^^ by y^^ as the product Therefore, .2X-03- .006,

in which

there

are

three decimal

orders.

Examples.

147.

1.

Multiply2.149 by
"

6.34.
OPERATION.

Solution.

Multiply as
decimal

in

simple numbers,

2140

2.1 4 0 6.3 4

by

634.
are

There decimal decimal the

three in

orders in 2.140, and


must

two

"85
6447 1 2804

0 6

orders orders

6.34; hence, there


13.62466.

be

five

in the

product (Art.146).

fore, There-

product is

] 3.6 2 4 6 6

2.

Multiply.0276 by
"

.035.

Solution. 276
are

Multiply the
35;
in

numerator

(Art.134, 6)
There decimal decimal three
seven

by

the

numerator

the result is 0660.

four in

decimal

orders

.0276, and
must

orders orders
must

.035; hence, there

be

in the

be

cipheron
3.

then, product (Art.146); three ciphers, Therefore, omitting the prefixedto 0660. the right(Art.140, 2) the product is .000066. 100.

.0000660

Multiply2.075 by
"

Solution.

Write
7 and

2075

point between
than

5, two

place the placesfarther to

and

decimal the

operation.

right

2 0 7.5

it is in 2.075.

182

RAY'S

NEW

PRACTICAL

ARITHMETIC.

Rem. decimal

"

multiply 207.5 by 100, annex point two placesto the right.


1.

To

cipher and

mcve

the

Kule,
as

"

in

the Multiply together SimpleNumbers.

numerators

of
the

the

mals deci-

2. Point there
Rem.
numerators
are

off as
decimal
When

many

decimal

orders

in

as jiroduct

orders in both
the number the number of

factors.
figuresin
the

1.
"

is less than

of decimal

orders

product of the required, prefix

ciphers. (Ex.2.) point,omit ciphers at the rightof the decimal part of the product. (Ex.2.) To multiply a decimal Rem. 3. by 10, 100, 1000, etc., remove to the right as there are ciphers the decimal pointas many in places If there be not the multiplier. enough figuresannex ciphers.
2.
" "

Rem.

After

placing the

decimal

4. 5. 6. 7. 8.
0, 10

11. 12. 13. 14. 15 IG, 17, 18. 19. 20.


21. 22.

Multiply33.21 by 4.41. 32.16 by 22.5. Multiply .125 by 9. Multiply .35 by 7. Multiply .2 by .8. Multiply .02 by .4. Multiply .15 by .7. Multiply 125.015 by .001. Multiply .135 by .005. Multiply Multiply1.035 by 17. Multiply19 by .125. 4.5 by 4. Multiply Multiply.625 by 64. 61.76 by .0071. Multiply 1.325 by .0716. Multiply 4.87 by 10. Multiply 5.3 by 100. Multiply 17.62 by 100. Multiply 1.01 bv 10. Multiply

146.4561 723.6 1.125 2.45 .16 .008 .105 .125015 .000675 17.595 2.375 18.
40.

.438496 .09487 48.7 530. 1762.


10.1

DECIMAL

FKACTIONS.

183

23.
24. 25. 26.

Multiply.0001 by 100. Multiply1 tenth by 1 hundredth. Multiply1 hundred by 1 ten-thousandth. Multiply43 thousandths by 21 ten-thousandths. Multiply40000 by Multiply.09375 by
DIVISION

.01
.001 .01

.000090a 27. 28. 1 millionth. 1.064. .04

.09975

OF

DECIMALS.

148.
the

Division

of
two

Decimals

is the

process

of

finding

quotientof

numbers

involving decimals.

149.

Placing the
upon
the

decimal

point

in

the

pends quotientde-

following
Principle.

The

number

of

decimal

orders

in the

is equal to quotient the


ber num-

the number

of

decimal

orders

in the

dividend^less

in the divisor.
of decimal by .03; then, the number For .006=:y^%^ and .0?)=:j|}o; orders in the quotient will be one. of the quotient as then, the quotientof .006 by .03 will be the same fore, ; ^^^^^l -^,= :2. ThereI'o To%o divided by ^f^. But, \o%o ifo

Thus, let .006 be

divided

.006

-^-

.03

.2, in which

there

is

one

decimal

order.

Examples.

150.
Solution. There decimal orders
are

1. Divide
Divide

2.125
in

by

.5.
2125

"

as

simple numbers
orders
in
must

by
one

5.

operation.

three

decimal

2.125, and
be two

.5)2.12 5
4.2 5

order in
in

.5;hence, there

decimal the

the

quotient (Art. 149). Therefore,

is 4.25. quotient

184

KAY'S

NEW

PRACTICAL

ARITHMETIC.

2. Divide

.048

by

.006.

Solution.

"

Divide
6.

the numerator There decimal


are

(Art.134, 6)
decimal
in

48

operation.

by
in

the numerator

three

orders

.006).0

4 8

.048, and
will be

three
no

orders

.006; hence,

there

decimal the

orders

in the

quotient(Art.

149).

Therefore

quotientis 8.
.004.
OPERATION.

3. Divide

.3

by

Solution.
as

"

Annex

two

ciphersto .3; then

solve

.004

).800
75"

in Ex. 2.

4. Divide

83.1

by

4.

Solution.

"

Annex order

two

ciphers to
the division in Ex. 1.
as

the may

decimal be per-

operation.

(Art. 140, 1) in
formed

that

)8

3.1 00

then exactly;

solve

20.7 7 5

5. Divide

2.11

by
one

3.

Solution.

"

Annex
in

or

more

ciphersto
carry the
1.

the decas

operation.

imal
far
as

(Art.140, 1)
is

order

to
as

division

3)2.1 10
,7 03-|-

wanted; then solve


475.(1

in Ex.

6. Divide

by
4756

100.

Solution.

"

Write
4 and

point between
than

7, two

place the placesfarther to

and

decimal the left

operation.

4.7 5 6

it is in 475.6. To divide 4.756

Rem.

"

by

100

a cipher; thus, .04756. prefix

Rule.
numerator

"

1.

Divide

the

numerator
as

2. Point

the in

number the

of the divisor off as many of orders in

in

decimal

of the dividend by the simple numbers. orders in the quotient as


exceeds the number

the dividend

divisor.

DECIMAL

FKACTIONS.

185

Eem. the
same

1.

"

When
as

the number

number
in the

of decimal

orders in the

dividend
an

is

the

divisor, the quotientis


orders in the
in

integer
is

(Ex. 2).
Rem. less than make Rem.
2.
"

When

the
in

number the

of decimal

dividend

the number the


"

for convenience divisor, the

them
8.

same

by annexing ciphersto
the division is not

pointing off, dividend (Ex. 3).


be continued
to

When

exact, it may

of decimal number any required 4. To divide a decimal Rem.


"

decimal

point as

divisor.

many If there be not

places(Ex. 5). the by 10, 100, 1000, etc., remove in the placesto the left as there are ciphers ciphers(Ex. 6, Rem). prefix enough figures,

186

liAY'S

NEW

PRACTICAL

ARITHMETIC.

30. Divide 31. Divide 32. Divide 33. Divide

.08 1.5

by

80. .2142857 .3388278 .00383177

.001

by 7. 11.1 by 32.76. .0123 by 3.21.


DECIMAL COMPOUND

+
+

NUMBERS.

151.

decimal

is reduced

to

lower

denomination

(Art. 63, by multiplication


1. Reduce .05

Rule

I).
of
a

gal.to

the

decimal

pint.
orKRATION.

.06 Solution.
"

To

reduce
4 and

.05

gal.to
The

the

decimal

of

4 .20

multiplyby pint,

by

2.

result is .4

pint.

2_
.4

2. Reduce 3. Reduce
4. Reduce

.035 .0075
.005

pk. to
yd.
to

the decimal

of
of of

a a an

pint.
quart.
inch. of
a

.56
.24

pt.
qt.

bu. to the decimal the A. decimal


to

.18 in.

5. Reduce rod.

.00546875

the

decimal

square .875 sq. rd.

152.

To

find the

value

of

decimal

in

(Art. integers

125).
1. Find the value of .3125

bu. in

integers.
OPERATION.

Solution.

"

To

reduce is 1.25

.3125

bu. to To

by 4;

the

result

pk.

pecks,multiply reduce .25 pk. to

.8 12 5 4 1.2 5 00

quarts,multiplyby 8; the result is 2 qt. Therefore,


.3125 bu.

equals1 pk. 2 qt.

2:00 2. Find 3. Find the value the value of .75 of

yd. in integers. .3375 A. in integers.

2 54

fl. 3 in. sq. rd.

DECIMAL

FRACTIONS.

187
in

4. Find

the value

of .7 lb.

Troy

integers.
8
oz.

pwt.
2

5. Find

the value

of .8125

bii. in

integers.
3

pk.

qt.
in.

6. Find

the value

of .44 mi. in

7. Find

the value

of .33625

integers. 140 rd. 4 yd. 1 ft. 2.4 cwt. in integers.


33

lb. 10

oz.

153.

decimal

is reduced Rule

to

higher denomination

by

division

(Art. 63.
.64

II).
decimal
of
a

1. Reduce

pt. to

the

gallon.
OPERATION.

Solution.

"

To

reduce
2 and

.64 4.

pt. to the decimal


The result is .08

of

2).6
4

divide by gallon,

by

gal.

).32

708
2. Reduce .72

qt. to

the

decimal

of

bushel. .0225

bu.

3. Reduce

.77

yd.

to

the

decimal

of

mile. .0004375 mi.

4. Reduce

.25 pt. to

the

decimal

of

gallon.
.03125

gal.
bu.

5. Reduce

.6

pt. to

the

decimal

of

bushel. .009375

6. Reduce

.7 rd. to the

decimal

of

mile. .0021875 mi.

Promiscuous

Examples.
cost

154.
per
2. What

1. What

is the 12

of 9

yard,

and

yard, and

yd., at S0.75 per is the cost of 2.3 yd. of ribbon, at $0.45 per 1.5 yd., at $0,375 per yard? $1.5975.

at $0.40 yd. flannel, S12.60. yard?

188

KAYS

NEW

PKACTICAL

ARITHMETIC.

3. What

is the

cost

of

16.25

yd.
much

of

cloth, tit $2.6875


$43.671875.
be

per
4.

yard?
At

$0.75

per

bushel, how

wheat

can

boui^ht
47 bii.

for

$35.25?
$2.5625
per

5. At for

yard, how
50

much

cloth

can

be

$98.40 ?
will
6
cwt.

bought 38.4 yd.


per $21,125.

6. What

lb. of

hops
4

cost

at

$3.25

hundred-weight?
7. What will bushel? will
13

14

bu.

pk.
sq.

qt.

of

corn

cost, at
$9.296875.

$0,625

per

8. What

A.

115

rd.

of land

cost, at

$17.28

per

acre?

$237.06. $0.3125
per

9. At

bushel, how
how much

much

corn

can

be
3

bought
10. for At

for

$9.296875?
per
acre,

29 land 13
can

bu.

$4.32

be
115

pk. bought
sq. rd. cost?

$59,265?
If 63

A.
464

11.

gal.of
.34

wine

cost

$49,

what

will

gal

$360.88
12. 13. 14. 15. Add Add From From 8.92 in. 1.07 ft. and 3^d., .75 qt. .625 gal. and 1.53 yd. subtract 2 ft. 3.08 .05 yr. 2 ft. 10 3 in. 2

+
in.

qt.

.5

pt.

ft. 4 in.

subtract

.5 hr.
18

da.

hr.

48

min.

16.

From

.41

da.

subtract

.16

hr.
9

hr.

40

min.

48

sec.

17. Find

the

value

of .3 yr.

in

integers.
109

da. ft. 3

13

hr. 48

min.

18.
at

What per

is

the

cost

of

343

3^d. 2

in. of

tubing,
$55.
a

$0.16
19. At

yard?
per

$690.35
rd.

mile, what

is the

cost

of

road

17

mi.

135

long?

$12027.19140625.

THEjMETRreiSYSTEM.

DEFINITIONS.

155.

1. The unit

Metric

System
the

is

so

called

from

the

meter, the
Eem.
at the

upon

which

system is based.
of
use

"

The close

French of
the

this system originated last The

weights and
in

measures

centurj^,and
metric

its

France

became

obligatoryin

legal in nearly all system is now civilized countries, and, in several, it is making its way rapidlyinto
1841.
in the United In 1866, its use was States,by generaluse. legalized, in act of Congress. It is general use by scientific men throughout

the world.

2. All
a

the

units

of from

the the

other
meter.

measures

are

derived

in

simple manner
1st. The
Meter

Thus,

Length. It is the base of the Metric System, and is very ionth nearly one ten-mill(.0000001) part of the quadrant extendingthrough Paris from the equator to the pole.
is the unit of 2d. The
Ar

is

the

unit

of

Land

Measure.

It is

side is 10 square whose 3d. The is the Liter wdiose which 4th.
contents
are

meters.

unit

of

capacity. It
a

is

vessel of

equivalent to
is the unit
water

cube

the

edge
is

is .1 meter. The of
a

Gram

weight

cube

of pure

Weight. whose edge is

of

It
.01

the

meter.

(189)

190

KAY'S

NEW

rilACTlCAL

ARITHMETIC.

3. The

name

of the

each unit

denomination
oi' the
measure.

indicates

at

once

its relation

to

27
are

Thus: formed

1st. The

luinies

uf

the

louver dononiiiiutions

of the unit the by prefixingto the natiie Latin and dcci (.1). mdli ctntl (.01), numerals (.001), of a is one thousandth For example, a miUimeter

meter;
a

is centigi-a^n
one

one
a

hundredth liter.

of

gram;

and

deciliter is 2d. The

tenth of
of the

names

drnoiiiinatidiis liiffhrr

arc

formed

miiiuTal.- (Iil;ii by prefix! Hi;- to tin- unii tin- a ml; heldo (100), For /.v/o i l("n(" mimI inur'in i KMHXh. (10), i- I'^n nirt.T.-; hrl;ii_,r,frr i" ..nc a example, a dekameter
.

hundred
a

is one a liters; kilocjrain

tliousand

grams;

and

is ten thousand "ni/ria7neter

meters.

4. Since

in the of
a

Metric
lower

System 10, 100.


denomination
it follows

1000,
a

etc., units
unit of
a

make

higher denomination,
number is reduced
in
"i

thai.

Ist. A

i.owf.h d.s j/oi/it

d,

ination nom-

by removing the places to the multiplier.


2d. ination A
right
as

drci/jud there
are

innny the

in ciphers

number

is reduced

to

iiKiriEu

drncnn-

by removing the
the
left as

deciniai there
are

jxiinfa.^

iiKinij the

places to
divisor.

in ciphers

MEASURES

OF

LENGTH.

156.

The

Meter

is the

unit

of

length ;

it is

legalin

the United

States at 39.37

inches.

Rem.
3 ft. 3|

1." Its which in.,

length is
may

also

littleless than
as

1.1

or nearly yards,

be remembered

the rule

of ike three threes.

THE

METKIC

SYSTEM.

If
are

Rem.

2.
"

The the
"

decimeter

and

its divisions

shown

in the

graving en-

on

oppositepage.
meters

Rem.

3.

Standard

have furnished

l)een

provided by

the

United

have States,and copies

been

to the several

states.

Table.
10 }0 JO 10 10 10 10

marked millimeters, centimeters decimeters


meters

mm.,

are

centimeter, marked

cm.

dekameters hektometers kilometers The

Rem. The

and kilometer. chieflyused are the meter tances; meter, like the yard, is used in measuring cloth and short disthe kilometer is used in measuring long distances.
"

measures

1. Eeduce
Solution.
"

5.638
To and

m.

to

centimeters.
to

reduce

meters

centimeters, multiply by

100.
two

"Write

5638

orders farther to

3 and 8, place the decimal point between the right than it is in 5.638 (Art. 155, 4, 1st).

A71S, 563.8

cm.

2. Eeduee
Solution.

3642.9
To and

m.

to

kilometers.
to

"

reduce

meters

kilometers, divide

by

1000.

"Write
orders

36429

place the

decimal
it is in

farther to the left than

point between 3 and 6, three 3642.9 (Art. 155, 4, 2d).


Ans. 3.6429

Km.

3. Reduce
Solution.
10
"

4.27
To

Dm.

to centimeters.
dekameters 427 and
annex

reduce "Write

to
a

X 100

1000.

centimeters,multiply by cipher (Ex. 1 ).


A71S. 4270
cm.

4. Reduce
Solution.
100
=

5.6 dm.
To reduce

to hektometers.

"

decimeters

to

hektometers, divide

by

10

1000.

Write them

56, prefixtwo

and ciphers,

place the

decimal Hm.

point before

(Ex. 2).

A71S. .0056

J92

RAY'S

NEW

PRACTICAL

ARITHMETIC.

5. Eeduce 6. Eeduce 7. Reduce 8. Reduce


9. Reduce

30.75

m.

to to

centimeters.
meters. meters.

3075
4500 .075
2.5

cm.

4.5 Km. 75 .025 36.5 .4875


mm.

m. m.

to to to

Dm. dm. Km.

decimeters,

dm.

dekameters.
centimeters.

.365 48750

Dm.
cm.

10. Reduce

to

LAND

OR

SQUARE
unit

MEASURE.

157. The
at

Ar

is the

of Lund

Measure;

it is

legal

119.6

square

yards.
Table.

100 centars, marked


100
ars

ca., are
"

1 ar,

marked
"

a.

1 hcktar,

Ha.

Rem. is very Rkm.

1.

"

An

ar

is 100

square

meters, marked

m^,

Xhe

hektar

nearly 2|
2."

acres.

For
are

measuring
used.

other

surfaces,squares

of the

meter

and

its subdivisions

1. Reduce 2. Reduce 3. Reduce


4. Reduce

2.625

a.

to to
to

centars.

262.5 3.978 .25 380

ca.

397.8
2500 3.8
a.

a. ca.

hektars. hektars.
meters.

Ha. ITji.
m^.

to

square

MEASURES

OF

CAPACITY.

158. The
1.0567

Liter

is the

unit

of

Capacity: it

is

legalat

quarts,Liquid

measure.

Table.
10
10

marked centiliters,
deciliters

are cl.,
"*

1
1 1

marked deciliter,

dl.
1.

liter,

"

10 liters

"

10 dekaliters

"

dekaliter, 1 hektoliter,

""

Dl. HI.

"

THE

METRIC

SYSTEM.

103

Rem. The

1." The

measures

commonly

used

are

the liter and

hektoliter.

liter is very

etc.,in
is used Rem.
to
are
use

nearlya quart; it is used in measuring milk, wine, The hektoliter is about 2 bu. 3i pk.;it moderate quantities. in measuring grain, fruit, roots, etc.,in largequantities.
2.
"

Instead

of the milliliter and and

the

it is customary kiloliter,

the cubic centimeter

the cubic meter

(marked m^), which


meter.

their Rem.

equivalents. For measuring wood 3.


"

the ster is

used; it is a cubic

1. Eeduce 2. Reduce

2.456 873.5
1.83

1. to

centiliters.

245.6 8.735 1830


2.4

cl. HI. dl. Dl.

1. to hektolitem HI. to deciliters.

3. Eeduce 4. Eeduce 5. Eeduce

2400 1400

cl. to dekaliters.
1. to cubic
meters.

1.4 m^

MEASURES

OF

WEIGHT.

159. The
15.432

Gram

is the

unit

of

Weight;

it is

legalat

grains.
Table.

kilogram,and The gram is used in mixing medicines, in weighing the metric ton. preciousmetals, and in all cases where great exactness is required. is the usual The kilogram or, as it is commonly called,the "kilo"" articles generally; it is very nearly and coarse weight for groceries 2i pounds Av. The metric ton is used for weighing hay and other than our ton. it is about 204 lb. more heavy articles;
Rem.
"

The

weights commonly

used

are

the

gram,

"

Prac.

13.

194

KAY'S

NEW

PRACTICAL

ARITHMETIC.

1. Eediice 2. Eeduco 3. Reduce 4. lieduce 5. Reduce

1428.06 .28

g. to

kilograms.

1.42806
280

Kg.
g.

Kg. to grams. 1713.5 Kg. to metric tons. .00654 Hg. to centigrams. 192.7 dg. to dekagrams. legaland
the

1.7135

M.T.
eg.

65.4 1.927

Dg,

IGO.

The

approximate

values
are

of in

nominat those decommon

of the Metric
use are

in presented

System which following

Table

Note.

"

The

value legal

is used in

examples. solvingthe following


4
m.

1. How

many

yards,feet,etc., in
there
are

Solution. 39.37 reduced


are

"

In 4 meters
are

4 times

in. which
to

157.48

in., 157.48

in.

integers of higher yd. 1 ft. 1.48 in.

tions denomina-

THE

METRIC

SYSTEM.

I95

2. What

is the

value

of 36

lb. in

kilograms?
OPERATION.

Solution."
are are are
as

many contained 16.329

pounds kilograms as 2.2046


36

In

there

2.2 0 4 G

)3

6.0 0 0 0

( 1 G.3 2 9 +

2 2046

times
.

in 36

which

139540 1322
7 6

72640 66138
65020 44092 2 0 9 2 8 0

198414

3. What
4. How 5. What 6. What

is the many is the is the many many many .5


m.

value
hektars value value

of 20 in
160

Km.

12.4274

mi.

acres?
9

of 49 of 15 in
500

m.?

rd. 4
9

Ha. 64.75-|yd. 3.13 in. gr. HI. C.


m.

g.?
42

7. How
8. How 9. How

hektoliters cords square wide? how many


in

bu.?

pwt. 15.48 14.8+


137.95 paper 5.382
9

sters? in
a

yards

roll of

long
10.

and
32

sq.

yd.

1. are

gallons?
Examples.

8.4536

gal.

Miscellaneous

161.
and
234.5 2. What
an

1. What dl.? will

is the

sum

of

127

1563 4.87 1., dl.,

cl,
1.

56.65 be the
cost

of

45

Ha.

of

land,
S3
a

at

$3.32

ar?
3. A

S14940.
merchant
meters

how

many

paid $457.92 did he buy?


marble
:

for

cloth,at
m.

meter:
m.

152.64

4. A
m.

block
cost

of

.72

m.

long,.48
cost

wide,

and

.5 per

thick

$.864

what

is the

of the

marble

cubic

meter?

$5.'

196

RAY'S

NEW

PRACTICAL

ARITHMETIC.

5.

manufacturer how many 1.6 much


was

bought
that of
a

380 8tcr?
oats

stcrs

of

wood

for

$454.10:
6.

SI. 195
in 4685

How

hektoliters

sacks,
7496

each

containing
7. did 8. in I I

HI.?
346.75

HI. what $0.56

bought
pay per nickel

Kg.

of

coffee

for

$194.18

kilogram?
5-cent coin be
to

The

weighs
the make

g.

and of in

is

cm.

diameter:
coins

what
hiid in
a

would
row,

weight
a

enough length
250

of ? g.
a

these

meter

9.

How

much made of

lining
6.5
m.

1.85 of

m.

wide

will 1.25
m.

it

take

for

garment

cloth

wide?

4.39+
10. the 11.

m.

How distance
A does

many

kilometers
60
mm.

from

Cincinnati

to

Dayton,
Km. what

being
is it 29

miles.

96.56-|and 22.4
mm.

map

long

wide 649.6

space
12. how that

cover?

mm^.
Km.:

The
many distance

distance

between
of .715
m.

two.

towns must

is I

13.24037 take 18518


to

steps
?

each,

walk

steps.

Note.
"

To

illustrate

the of

difference
a

between

the

metric may and

system
be

and

our

common

system
8 mi. of
one

measures,

similar
in. for the

example
distance,

given,
in.

substituting
for the

72

rd.

yd.

1.7

28.15

length

step.

162.

1.

Any

per

cent

of

number

is

so

msiuy

dredths hun-

of it.

Thus,
Rem.
"

1 per cent

of

number

is

yi^

of

it,2 per
centum,

cent

is

y^^,etc.

Per

cent

i^from
per

the Latin per

by

the hundred.

2. The

sign of
%
is read

cent

is

%,

read

per cent.

Thus,

fiveper

cent.

3. In all in
two

with operations 1st. As


a

per

cent, it may

be

expressed
As
a

ways:

common

fraction; 2d.

decimal.

Thus

the

are followingexpressions

equivalent:

Rem. reduced

1.
"

Per

cent, which

is expressed as

mixed 121

number, and

may

be

to

equivalentexpressionsby
9 which

Arts.

142.
.045.

Thus,

4i

4^ ^

Tqq"

may

be reduced

to

^qqI also, i\ ^

no7)

198

RAY'S

NEW

PRACTICAL

ARITHMETIC.

Express
decimals
1. 2. 8. 4. 5. 6.
Rem.
:

the

following as

common

fractions

and

as

10%

yV
i

and

.10 .15 .20 .30 .50 .025


be reduced
=

^-^and

20%
30"^
70

and
and and and

f^
-1-

2i%

i^

2.--C()inmon

fractions
as

may

to

hundredths
ol

by

Art. 108, snul tluMi read

per cent.

Thus, ^

.165 or 16" the

How

many

per

cent

are

to equivalent

following

fractions ?

163.
with

1. Percentage

embraces

the

various

operations

per cent. 2. In Percentage three

the

Base^ (2) the


Base

considered. are quantities Rate^ and (3) the Percentage. number upon which the per

(1)
cent

3. The

is the

is estimated.
4. The
common

Rate

is
or

the
as a

per

cent

when

expressed as

fraction

decimal.

5. The
cent 6.

Percentage
base. of these There
two

is the

resujt of taking the

per

of the

Any
be

quantities being given, the


are

third

may

found.

four

cases.

PEECENTAGE.

199

CASE

I.

164.

Given

the

base

and

the

rate,

to

find

the

centage. per-

1. What

is

25%

of 32?
OPERATION.

SoLUTiox."

is i (Art. 162). 25^0

i of

32 is 8.

2b"f^ ^
=

2. What

is

7%
^^is

of 162?
OPERATION.

Solution."

.07

(Art.162). Multiplying 1G2 by

16

.07,the result is 11.34.

.0 7
1 1.3 4

Rule.

"

the Multiply

base

by

the rate; the

product will

be

the

percentage.
"

Rem.
or

Whether
as a

the

rate must

should be is
a

decimal,
best

expressedas a common of judgment. That matter


most

be

tion, fracform
in

expressionis given example.


3. What 4. What

of

which

simplestor

convenient

the

is is is is
is

5. What
6. What

7. What
8. What 9. What 10. What

is is is
is

11. What 12. Wliat


13.

is is is
is is

What

14. What 15. 16.

What What

1% 2% 3% ^% 3f% 4% 5% H% ^% H% 8% 8i% 10% 12i%

of 278? of 180? of 97? of 165? of 240? of 140? of 118?


of

2.78
3.6 2.91 5.5
9.

5.6 5.9 8. 15. 30.

150?

of 250? of 450? of 11? of 384? of 57?

,88
32. 5.7 36.5

of 292?

200

HAY'S

NKW

PRACTICAL

AUITUMETIC.

17. What 18. What


19. What

s s " 8 H

20. What 21. What


22.

What

8
s

28. What 24. What 25. What 26. What 27. What 28. What 29. What 30. What
31.

8 B " "
s

8 8 8

What

15% of 95? 17% of 53.4? of 11.2? m% 20% of 9.85 ? 25% of 43? 33J% of 0.93? 45% of 5.7 ? 50% of 38.75? i% of 456 ? 8% of 464? of 144? A% 125% of 36? 208% of 650? 450% of 12? 1000% of 24.75?

14.25 9.078
2.1

1.97 10.75 2.31 2.565 19.375 2.28 1.74


.63

45. 1352. 54. 247.5

CASE

II.

165.
rate.

Given

the

base

and

the

percentage, to

find the

1. What

per

cent

of 8 is 2?
OPERATION.

Solution."

is

of 8

(Art.120). \

is

25^^.

2. What

per

cent

of 56

is 3.5?
OPERATION.

Solution.
IS

"

Dividing 3.5 by 56, the

result

3.5

-^

5 6 "..0 6 2 5
=r

.0625.

.0625 is 6] %. One per cent times


of 56 is

.0 6 2 5

6] ^^
many per

Explanation.
cent
as

"

.56; then

3.5 is

as

.56 is contained

in 3.5.

-R^lQ,

"

1. Divide

the

percentageby

the

base;

the

quotient

will be the rate.

3. What 4. Wlaat

per per per per per per per per per per per

5. What
6. What

7. What 8. What
9. What 10. What

11. What 12. What 13. What 14. What 15. What 16. What 17. What 18. What

per
per per per per

166.
base.

Given

the

rate

and

the

percentage

to

find

the

1.

15

is

25%
25

of what

number?
OPERATION.

Solution." of
some

^,is | (Art.162).
the

Since
15
=

15 is

25^,
15X4

number,
4.93

number

is 4 X

00,

60

2.

is

17%

of what

number?
OPERATION.

Solution. number

(Art. 162). Since some multiplied ])y.17 givesthe product 4.93,


"

17

oi is .17

=.17 17/,

4.9 3 -t-.I 7

29

the number

is 4.93 divided

bv

.17

or

29.

Rule.

"

Divide

the

percentage by

the

rate; the quotient

will be the base.

202

RAY'S

NEW

PRACTICAL

ARITHMETIC.

167.
of the
1. A

Given base and

the

rate

and

the

sum

or

the

difference

percentage, to

find

the

base. what
is

number, plus 35% of itself, equals675:

the number?
OPERATION.

Solution.

"

35 % is .35.

The

number

plus
1

35

^
5

=.3

.35 of
the

itself

equals 1.35
is 675, and

of the

it; then, 1.35 of


number itself

5=:1.3 -f-.3
=

number

6 75-^-1.3

5 00

is G75

divided

by 1.35, or
minus

500.

2. A the

number,

5%

of

itself, equals 57:

w^hat is

number?
OPERATION.

is 2V- The number minus c!^ ^^^ of itself equals ^" of it; then, ig of the number
5

Solution."

^
uV

"

\%
3

"

=^

^i^ 2^
60

is 57,
3
=

2^0of

it is 3, and

the

number

is 20

times

CO.

,^ X H
"

Rule.

Divide

the

sum

by

plus the rate, or


will quotient

divide

the

difference by
3. 721 the is

1 minus

the rate; the


a

be the base what is 700.

3^ greater than

certain

number;

number?

PERCENTAGE.

203

4. 68

is

66%

less than

what

number?

200.

5. What
to

number, number,
is

increased

by 25% by 6%

of

amounts itself,

2125?
6. What

1700. diminished of is equal itself, 8.

to

7.52? 7. 8250
8. What 9. 6.6

is

37J% greater than what number? less 10% of itself, fraction, equalsf ? number? than what 20% more
for the the Four Cases the of

6000.

-:f^.
5.5

168.
Let b

Formulas

Percentage.
j^ the

represent Then,
I.
II. b X J) ^b ]) -^
r

base, r

rate, and

centage. per-

Case Case Case Case

=p.
=^r.

III. IV.

^=b.

^""==6. *-^^=
1 +
r

6.

\"r

Miscellaneous

Examples. and drew


out

169.
how much

1. I had had

S800

in bank

36%

of it:

I left?

$512.
after he had

2. A per
cent

man

had he

$300;
have withdrew

spent $225,

what

did

left?

3. A

merchant

40%
:

of his

25%. leaving deposits,


did draw? he with-

$3000

remaining
'

in the bank

what

amount

$2000.
4. A
was

grain
more

dealer than

sold

corn

for 56 him: what

ct.

bushel, which
the
cost

40%
5. A
man

it cost

was

per

bushel?
sold horse
a

40 ct. horse what for $175, which did the horse


was

12J%

less

than

the

cost:

cost?

$200.

204

KAYS

NEW

PRACTICAL

ARITHMETIC.

G. A

12^^
at

grocer bought 4 sacks of coffee of 75 pounds Ccach; what lost by waste: the remainder wortli was was per owed remains

35 cents 7. A
man

pound?
$500;
lie

$91.87^.

paid $425:

what

per

cent

of

the debt
8. A and

unpaid? invested speculator


remainder much of did he
a

the

75% of his estate in it, amounting to $5000,


invest farm in bonds? A. 86 sq. of 250

15%. bonds,
in real

estate:

how

$15000.
which rd., how A. in 112
one

9. A
was

farmer
more

owned
than

12^%
did 10. A

his

neighbor owned:
222

much

land

neighbor own? flock of IGO sheep increased


were

his

sq. rd. year:


216.

35%

how

many

then
takes what

in the

flock?

11. A

miller

for toll 6

qt. from
a

every of

5 bu. of

wheat

ground:
farmer
was

per

cent

does

he take?
tract

3i%540 how many 2000 A. the

12. A
acres,
acres

which
were

ow^ning45% of G0% of what


in the tract?

land,sold

he

owned:

there the

13. When

gold dollar

is worth

in are $371.29 in greenback dollar, how^ much $347. greenbacks w^orth? is $800 a year; he spends 18% of it for '4. A's salary and rent, 15% for clothing, 12% for 23% for provisions, sundries: how much does he save $25G. annually? answered 15. A pupil at an examination 17 of the 20 ? : what questionscorrectly per cent did he make 85^ 16. 2 bu. 3 pk. are 33J% of what number? 8 bu. 1 pk. number of pupilsattending school on 17. The a tain cer37; this w^as day was 7|% less than the number
.

7% gold

more

than

enrolled 18. A it is

how

many

w^ere

enrolled

40.

gold dollar weighs 25.8 grains Troy; 10% of tain grains of pure gold does it conalloy: how many
23.22

PERCENTAGE.

205

19. The G.
are

five-cent

piece weiglis5

grams,

of whicli is the

1.25 per

nicl":eland of A

tlie remainder

copj^er: what

cent 20.

copper?
man

75%.
a
:

sold him

horse if he would

for had

$150, which
sold the been it have

was

25%
for

more

than how
cost

it cost many

horse
more

$200,
it

per

cent

than

him?

66f%.

APPLICATIONS

OF

PERCENTAGE.

DEFINITIONS.

170.

1. The into
two

of Percentage may applications classes: (1) Those without the with the element of time.

be

vided di-

element

of

time; (2)
2. The most

those

of the first class importantapplications are (1) Mercantile Transactions, (2) Stock Transactions. 3. The most of the second class important applications are (1) Interest;(2) Discount. 4. Percentageenters less extent into to a greater or the calculations of Exchange,Insurance, Taxes, Equation of Payments,etc. 5. The principles of Percentage apply directly to application with the fol^of the first class in accordance lowing
"

pr"^
.

Jyi/
V

GENERAL

RULE.

Rule.

"

1.

Ascertain

the their

which quantities
sum or

correspondto

base, percentage, and


2. Note 3.

their

difference.

the the

Apply

required. quantities given and the quantities ample. of Percentageto the given exproper case

206

KAYS

NEW

PKACTICAL

AKITHMKTIO.

MERCANTILE

TRANSACTIONS.

DEFINITIONS.

171.

1. Mercantile

Transactions

relate

to

the

chase pur-

and 2. Price

sale of merchandise. is the value is of any

thing in

3. Merchandise retail

bought priceis

and

mone}^ sold at wholesale and

at

prices.
wholesale

4. The

the

priceof

merchandise

in

quantities.
5. The small

price is quantities.
retail "Wholesale
merchants

the

price of

merchandise

in

Rem.

buy
of

and

sell merchandise
of every

at

sale whole-

prices. Retail dealers


to

distribute

merchandise

tion, descrip-

the

users

or

consumers

it,at retail prices. Transactions

6.

The

chief

Mercantile

of Percentageare application and Loss. Discount,and (3) Pj^ofit


Rem.

involving an (1) Commission; (2) Trade

"

Wholesale

merchants

buy

and
a

sell merchandise

largely
ices; serv-

or through agents, who receive salaries, buyers at wholesale are sometimes purchases;and merchants usuallymake

commission, for their


discounts
a or profit,

allowed

upon their sufier a loss, in

their

transactions.

COMMISSION.

172.
business
Rem. called

1. of
The

An

agent

is

person

intrusted

with

the

another.
who

"

person

employs

the

agent, in reference

to

him, is

the

principal.
merchant

2. A

commission

buys

and

sells merchandise

for another.

COMMISSION.

207

Kem. his and


own

1.
"

factor is and

an

agent who

buys

and

sells merchandise the

in

name,

is intrusted

by

his

with principal

possession
sold is

control
2.
"

of it. The person


to whom

Kem. termed

merchandise

is sent

to be

the

consignee;the

while

who sends person itself is called the merchandise

it is termed
a

the

signor; con-

consignment.
an

3. The

commission

is

the

sum

paid

agent
an

for

business. transacting
4. The

charges

are

expenses

incurred

by

agent

in

business. transacting
5. The
net

proceeds
commission of the

is the and

sum

remaining
in the is the

after deducting

the
6. The

charges.
business
acted trans-

value

materials commission base

is the the
net

base; the
the

proceedsis

less the

percentage ; percentage.

and

Examples. 1. An much

agent whose
a

commission

is

5^,
to

receives

how

upon

sale of

goods amounting
received

$240?
a selling

|12.
lot of

2. An

auctioneer
to

$11.50
what

for

goods amounting
did he 3. At receives how much receive?
a

$460

per

cent

commission

2^%.
for

commission

$8.12^
per

2^^ selling25
did he

of

commission of

merchant for

barrels

molasses:

barrel

sell the with

molasses?
to

$13.

4. An

agent receives

$210

which of

buy goods :
sum

after he

deducting his expend?


are

commission

5^
on a

what

must

$200.
net

5. What

the

proceeds

sale

of

goods
his the

amounting to $180, at 4% commission? 6. A lawyer received a $11.25 for collecting


commission debt?

$172.80.
debt: of

being 5^,

what

was

the

amount

$225.

2(18

KAY'S

NEW

PKACTICAL

ARITHMETIC.

7. An and

agent

receives
at

$1323.54
:

to

cover

cost

of

goods
$98.04.

commission

8%

what

is his commission?

8. A per

commission

merchant

sells 250

bbl.

pork, at $15

bbl.; 175 at 25 feathers,


sum

bbl.
ct.

at $7 per bbl.; and flour, 1456 lb. is 3%: what per lb.;his commission

does

he

remit

the owner?

$5178.83.

TRADE

DISCOUNT.

173.
a

1. Merchandise from
an

may

be

sold

at

net

price or
no

at

discount 2. A
net

assumed
a

or list,

regular, price.
which
discount

price is

fixed

pricefrom
an

is allowed. 3. A

list,or

regular, price is
seller
as a

established which
to

sumed asprice,

by
discounts.
4.

the

basis upon

calculate

The

discount

is the deduction

from

the

or list,

ular, reg-

price.
Rem.
to
a

1.

"

In

the wholesale

trade,the

amount

of discount and purchased,

purchaserdepends upon
of payment. 2.
"

(1

) the amount

granted (2 ) the
from the the

time

Rem.

In

some

lines of

goods

the discounts from the

are

made

of price-list market.

the

dealer; in others,

of price-current

regard to time, sellingfor cash means payment as be delivered. the goods can as soon in a certhat the payments are to be made tain Time purchasesmeans time after the purchase the time varies with different lines of goods.
Rem.
3.
"

In

"

5. The
or as so

discount many

is

expressedas

so

many

per

cent

off

off.
20

Thus, 20

off,or (fc

off,means

at

discount

of 20 %

from

the

price.

TRADE

DISCOUNT.

209

6. There

may
or

be:
more

1st. A

singlediscount;
discounts.

as

5^,

or

off.

2d. Two

successive
and
5 cL off

Thus, the expression20

means,

a first,

discount

of

20^
The

and then a discount price, 25, 10, and 5 cL oif,means expression from the The off
means

of

5^

from

the remainder. discounts. fraction.


5 off.

three

successive

Rem."

per cent

is sometimes

as expressed

common

Thus,

12^ cj^off;\ and


the

5 off

means

33i

and

7. The the

priceof

seller is the

base ; and the

the

sum

of

all the

discounts
or

is the

percentage;
the base

price of

buyer

pricepaid is

less the

percentage.

Examples.
1. A

bill of

goods

amounted
was

to

$225.50 list; 20%

off

being allowed, what


2. A

paid

for the
to

goods?
he

$180.40.

bill of articles amounted 5

$725.16,the purchaser

being allowed i and 3. I paid $1430.75


3%
amount

off,what
for
a

did of the

pay?

$459.27.
was

lot of

which groceries, bill: what


Avas

discount of the bill of

from bill?

the

face

the

$1475.
cost

4. A
was

the
5. Sold

goods list price?


20

$390.45

at

25

and

5 off: what

$548.

doz.

feather

dusters,giving

the

discount

of 10,

10, and
was

10%
my

his discounts

purchaser amounting
$60.

to

$325.20, how
6.

much doz.

price per
60

dozen?
ct. per

Bought

100

at stay bindings,

dozen,
$29.97.

for

40, 10, and

7^^

off:

what

did

I pay

for them?

7. A dozen

retail dealer for $50 in he

buys

case

of

slates

containing 10

and list, usual per

for them what did

the pay

gets off 50, 10, and 10^ ; paying additional 2^ : time, he gets an
dozen
for

the

slates?

$1.98.

210

HAY'S

NEW

PRACTICAL

ARITHMETIC,

PROFIT

AND

LOSS.

174.
2. The
Rem.

1. The

cost

is the is

selling
The
cost
to

price
the

i)rice paid for goods. the price received for goods.


is the
chant; selling priceof the meris the selling price of the

"

consumer

and
wholesale

the

cost

to the

retail dealer

dealer.

3. Goods 4. The

are

sold usually

at

or profit

at

loss.
more

profit is what
is what

the

goods

sell for

than

they
cost.

cost.

5. The

loss

the

goods

sell for less than

they
is the

6. The

cost

is the the and

base; the profitor

the
sum

loss
or

percentage;and of the base

price is selling
percentage.
Examples.

the

the

ence differ-

1. A

merchant's

on profit

piece of
he

cloth which

cost

$40

is

10%

for how
cost

much
6 ct. cent
a

does

sell it?
arc

$44.
for 5
ct.
a

2. Prints

that

yard

sold

yard:
3. A

what

is the

per

of loss?

coflPee at 27 cents per by retailing 24 gains 12^% : w^hat did it cost per pound? lot of goods at a loss of 4%, the loss 4. Selling a the entire lot was $15.30: what did the goods cost? grocer,

16f%. pound,
ct.
on

$382.50.
5. To make
a

of 37^%, profit sell shawls sells


a

at

what

price must

dry -goodsmerchant
6. A

that cost $8? for 90 ct. which

$11.
cost

bookseller
:

grammar cent?

75

ct.

w^hat is his
is the

gain per
cost

7. What

of tea, which, when


5 ct. per

sold

at

20%. 6J%
80 ct.

of profit yieldsa profit

pound

PROFIT

AND

LOSS.

211

8. A

grocer

sells
:

apples at $4.75
was

per

barrel, making

of 18f% profit
9. Sold

what

the

cost?

$4.
lost it to

silk at
'

$1.35 per
would I

yard, and
sold

10^
make

at
a

what

price per yard of 16f% ?


10. A

have

profit
$1.75.
and

peddlerbought a stock of goods of 25% : how disposedof them at a profit


did he make?
a

for

$874,

much

money

$218.50.
makes 25
ct.
on an

11. If

bookseller what sold and

which atlas,

he

sells for $1.75: 12. A he much dealer

is his per cent of profit? horses for $150 each ; two


on

16f%.
on
:

one

gained 25%,
did he lose
13. A

the

other

he

lost

25%

how

by

the

transaction? the

$20.

merchant
5 ct. per cloth per from

reduced

of cloth
on

yard, and

price of a certain piece his profit thereby reduced


:

the cloth

10%

to

8%

what

was

the

cost

of

$2.50. yard? 14. A speculatorbought 10000 bushels of corn, at 60 ct. per bushel; in a few days, corn advancing in price, then he sold 7000 bushels,at 65 ct. per bushel ; corn he disposedof the remainder at 55 ct! fallingin price, of the per bushel : what per cent profitdid he get out
transaction?
15. A

the

'^i%.
real
estate

speculatorin

sold
a

house

and

lot

for the he he

$12000, which
cost; he then
was

sale afforded invested sell at


a

lose

obligedto by the two

profit of 33J% on the $12000 which in city lots, did loss of 33 J % : how much
$1000.

him

transactions?

Miscellaneous 175.
1. A

Examples.

lisher purchasesbooks from the pubat 20% off the list price;if he retail them at the list price, what wnll be his per cent of profit? 25%.
bookseller

212

KAY'S

NEW

PRACTICAL

ARITHMETIC.

2. A
at

grocer
ct. per

bought

5 luilf-chests of off for be his cash


:

tea

of 74

lb.

each,

45

12^^
8.

lb.,at 2^ advance, what will

if he

retail it at

12

of Bought 5 assorted cases pairs each, for $45 per case, 5^ at

^20.12. profit? men's boots,containing


off for cash ; I retail

them

$4.25
a case a

4. Sold

pair: what is my profit? of hats containing 3 dozen, on


discount
on

$41.25.
which 1

had

received
or

of

10^

and

made
was

of profit wholesale

12^^

374

ct.

each

hat:

what

the

$120. j)rice per case? 5. A merchant bought 100 packs of pins,of 12 papers each, for $1.00 per pack, 00, 5 and 5% off; if he retail of $23.90,lor how them much to make as a profit so a

merchant's

paper will he sell them?


6. I sent
a

5 ct. of 100 bbl., to flour,


;
a

car-load New

commission

merchant

$9.50
of

per

$17.25:
did

disposed of the flour at his commission was barrel, 2-^% with charges flour cost if the how $7.50 per barrel, me
I make?

in

York

he

much
7. A

$159.

contractor

$125

apiece;the
was

bought 80 horses for government, at was mission com$200, and the agent's freight
that
cent

such per

the
was

horses the

cost

the

government

$10450:
8. A hhd. his of

what

commission? sells
a

2^%-

commission sugar, 1500 is

merchant lb. each

2% and clears 14^ by the transaction: what did he pay 9 ct. per pound for the sugar? 9. A dealer in notions buys 60 gross shoe-strings, at 70 for 50, 10 and ^t. per gross, list, 5% off;if he sell them what will be his profit? $10.77. at 20, 10 and 5% off list, buttons for 25, 10 and 10. Bought 50 gross of rubber of the lot for $35.91,at a profit of 12^ : 5% off; disposed the list priceof the buttons per gross? what $1.00. was
commission

consignment of 50 net, at 10^ ct. per pound; signor charges $22.50; the con-

STOCK

TRANSACTIONS.

213

STOCK

TRANSACTIONS.

DEFINITIONS.

17G.
and

1. Stock

Transactions

relate

to

the

purchase
shares.

sale of

stocks,bonds,
is in capital

and

gold.
of transferable

2. Stock
Rem. other
The

the

form

of banks, of railroad, insurance,telegraph and capital companies is held in this way.


"

3. The
4. A

Stockholders

are

the

owners

of the

stock.

share

is

usually^100.
$50
or some

Rem.
are

"

share

is sometimes New York

other

number.

Stocks
to

quoted, in the

market,

as invariably

$100

the

share.

5. A certain

bond
sum

is

written
at
a

promise, under
time. specified
of the Government allowed
are

seal,to

pay

of money
Bonds
are

Rem.

1.

"

the notes
are

and

ous of the vari-

corporate bodies which


bear
a

to issue

them; usuallythey
a

given
2.
"

rate

of interest and

payable

within

time. specified

quoting United States bonds, the different issues are 1st. By the rate of interest; as distinguished, 6's,5's,4J's, 4's; 2d. By the time at which they mature; as 5-20's, which are payable in 20 be paid after 5 years. The 5-20's also distinguished are years, but may by the date of their issue,as 5-20's of 1868. Bonds of the
Funded Rem. company
"

Rem.

In

Loan
o.
"

bear

5%

and interest, of local

later

ones

4J and

4^.

Adams

of the corporationstake the name issues which them; as, "Chicago and Northwestern," Union Express," Western Telegraph,"etc.
"

The

bonds

6.

Currency
"

is the

paper

money

of the

country.
"greenbacks,"and

Rem.

It consists Bank

of

notes, legal-tender

called

National'

notes.

214

KArS

KEW

PRACTICAL

ARITHMETIC.

7. The

par

value

of

stocks

and

bonds

is the

value

given
Hem.
"

on

the

face of them.

The

quotationsfor stocks, bonds, and gold are


dollar.

all based

on

the currency

involvingan application of Percentage are (1) Brokerage^ (2) Assessments and Dividends^(3) Stock Values^and (4) Stock Investments.

8. The

chief

Stock

transactions

BROKERAGE.

177.

1. A

broker

is

an

agent

who

buys

and

sells

stocks,bonds, gold,etc.
Rem.
"

Persons

who

"operate"

in stocks

usually do
and

so

broken^;the
are

latter

authorized

buy and sell stocks by the "operator."


is the
sum

in kind

amount

through as they

2. Brokerage the 3. The

paid

the
on

broker

for transacting value. the 2"^r-

and business,

is calculated is the

the par

par

value

the brokerage hase^

centage.
Examples.
1. A and

broker
Hudson

bought
Eiver

for

me
:

75

shares

New

York

tral Cenat

stock

requiredthe brokerage
50
:

J%.
2. The and cent? 3. At
an

$18.75.

brokerage
Island

for

buying
was

shares what

of
was

Chicago
the per

Eock

stock

$6.25

1^.
\% brokerage a
in bank broker stock
:

received

$10

for

making
did he 40.

investment

how

many

shares

buy?

ASSESSMENTS

AND

DIVIDENDS.

215

4. A

broker

buys

17

shares

Milwaukee

and ?

St. Paul

at J^ preferredstock : what is his brokerage, of Vermont 95 shares 5. The brokerage on is the per cent? stock is $11.87^: what 6. A

$4.25.
Central

-J^.
for he 38. did

broker

received Pacific

$9.50,or
stock
:

brokerage of ^%,
many shares

buying Union ? purchase

how

ASSESSMENTS

AND

DIVIDENDS.
sum

178.
the

1.

An

assessment

is

of money

paid by

stockholders.
In the formation of for the transaction of any but

Kem.

"

company

the stock subscribed business,


assessments
are

is not
to

usuallyall paid for


time
as

at once;

made

from

time

the needs

of the business

require. The
2. A

stock is then

said to be

paid for
money

in installments.

dividend

is

sum

of

paid

to

the

holders. stock-

Rem.

"

The

gross

earningsof

company
net

are

its total
are

in receipts

the

transaction

of the

business; the

earnings
The

what

is left of the
are

after deducting all receipts of the net earnings.

expenses.

dividends

paid

out

Examples. 1. I
a own

35

shares of
an

of

bank

stock;
will of

if the

bank

clare de-

dividend
man

4^,

what

I receive?

$140.

2. A

pays stock:

assessment

7^%,
does
a

or

$300, on
own? of

his 40.

insurance
3. A

how

many

shares declares

he

mining
does Mr.

company Jones

dividend
owns

15^
of

what stock?
4. A

receive

who

80

shares

$1200.
man owns a

60

shares of

of

railroad

stock

if the

declare company much stock will

dividend then

5^ payable in stock,how
03

he

own?

shares.

216

KAY'S

NEW

PRACTICAL

ARITHMETIC.

5. A gross
:

gas

company
cent

has

stock capital

of

$lt)0000; its
$4500
ally annu-

earnings are
what per

$15700, and
does it pay

its expenses the

stockholders?

1%-

STOCK

VALUES.

179,
is the
Rem.

1. The

market

value

of

stocks,bunds,

and

gold

priceat
"

which

they sell.

is above pnr, or at a premium, when it sells for more it sells than the par value; stock is below jxir, or at a disconni, when for less than the par value. Stock

2. The estimated

market
at
a

value per

of

stocks, bonds,
of the

and

gold

is

certain

cent

])ar value.

gold dollar is worth 106J^ of New York tral Centhe currency dollar,or is at a premium of 6| ^. and Hudson that the stock of this railroad River, 91 J,"means sells for 91^ ol of the par value, or is at a discount of Sh 4).
Thus, "gold, 106^,"means
that the
"

3. The

par

value

is the market

the

percentage;the

base; the premium value,the amount

or

discount

is

or

difference.

Examples.
1. What will be
at

the

cost

of

150

shares

($50 each)

of

$10500. 139|, brokerage \% ? 2. Bought $8000 in gold at 110, brokerage ^^ : what did I pay for the gold in currency? $8810 broker sells 50 shares 3. My of Chicago and North $2475 : at what western, brokerage ^% ; he remits me
per
cent

Harlem,

did

the

stock the

sell?
cost

49f^.
of 25 1000-dollar ? St. 5-20 U. S.

4. What

will be

Bonds
5. I

of

1867, at 114J, brokeragei%


for Milwaukee shares

$28593.75.

paid $1560 brokerage\% : how

"
did I

Paul,

at

19i 80.

many

buy?

STOCK

INVESTMENTS.

217

6. When
a

gold is
in

at

105,

what

is the

value

in

gold

of

currency? ^^2T ^^' the value of a 7. When at 112J, what was gold was dollar in currency? 88|-ct. worth 8. In 1864, the only "greenback" doilar was the price of gold? 280. 35f ct. in gold: what wa^ age 9. A merchant paid $8946.25 for gold, at 105, broker-

dollar

\^c
"

l^")w much broker

gold
sells
a

did

he

buy?
amount

$8500.
of

10.

My
me

certain

remits

$25734.37^? His brokerage,at the price of the gold? was $15,621: what

gold,and xV%' ^^"


103.

STOCK

INVESTMENTS.

180.

1.

The

income

is the

annual

profitfrom
States bonds

the

investment.
Eem;" coin
or

The

income

from

most

of

the

United

is

in

its

equivalent.
cost

2. The is the

of

the

investment

is the

base; the income

'percentage.
Examples.

1. If I invest

$39900 $39900 $39900


man's when

in

6^

bonds,

at

par,

what

will

be

income? my 2. If I invest my
3.

$^394.
in

6% bonds, 6%

at

105,

what

will

be

income?

$2280.
in

ifI

invest

bonds, at 95, what


who 107?
owns

will be

my

income?
4. What

$2520.
is
a

income

20

1000-dollar $1284.

IT. S.

6^

bonds,
income

gold is
8. 5-20

5. What

in currency
in

would

man

receive
at

by

investing $5220

U.

6^ bonds,

116, when
^283.50.

gold is ^105?

218

RAY'S

NEW

PRACTICAL

ARITHMETIC.

6.

What

per when receive

cent

of is

income 105?

do

U.

S.

per

cents,

at

108,
7. Central income 8.

yield
If I

gold
an

4g%.
dividend of

annual
cost

G^
what

on

Michigan
cent

stock,
do What I
sum

which receive

me

but

37^,

per

of

on

my in

investment?

16^.
S's in of

invested income

U.
of

S.

1881,
currency,

at

118,
when

yielded

an

annual
at

$1921

gold
9. at

was

113? shares of
on

$40120.
stock

How

many

bought

at

95J,
will

and

sold

105,
of

brokerage
"^

\%

each

transaction,

yield
100.

profit
10. income 11.

$925?
must

What
of

be

paid

for

6^

bonds

to

realize

an

8^^?
U.
S. will

75^.
4%
be for bonds received
are

When
income
can

quoted gold
from

at

iOG,
the

what bonds

yearly
that 12.

in

be I

bought
pay
of 87

$4982?
railroad bonds
cent

$188.
that do I

If income

for

yield
get
on

an

nual an-

7^,

w^hat

per

my

vestment? in-

%%.
13.

What
income

could of

afford

to to

pay

for

bonds

yielding
so as

an

annual
realize

7^

invest

my

money

to

6%?

116f.

DEFINITIONS.

181.
Rem.

1. Interest

is money
is

paid
the

for the

use

of money.

"

The

interest

paid by
is the

borrower

to the

lender.

2. The interest
3. The

principal
is

money

for the

use

of

which

paid.
is the
sum

amount

of

the

principal and

terest. in-

4. A

promissory
sum

note
at

is
a

written

promise

to

pay

certain

of money
borrower

time. specified
the lender
his
note

Rem. money.

"

The
The

always gives
a common

for

the

following is

form:

$500.00.
One
or

Dayton,
year after date hundred I

O., June
pay

promise

to

IG, 1877. Thomas, Clu^*les


at

order, five
received.
"When
are

dollars,with

interest

8^,,for
Dean.

value
Rem. "from

James
note

Q.

"

is made

to

draw

interest

from

date, the words


"interest."

date"

frequentlyinserted
of the
note

after the word

5. The
6.

face

is the

principal.
at
a

Legal

interest

is

interest

per

cent

that

is

allowed

b}^ law.
(219)

220

KAY'S

NEW

PRACTICAL

ARITHMETIC.

Rem.
and

per cent of interest that is legalin the different States is exhibited in the following Territories,
"

The

TABLE.

NAME

OF

STATE.

NAME

OF

STATE.

Alabama
Arizona Arkansas

8^"
Any. 6% Any.
Anv.

California Colorado
Connecticut Dakota

12 fo

60/, 10/. lOf, Any. Nebraska 10/. 12/. Nevada 10/. Any. New Hampshire... 6/. New Jersey 6/. New Mexico 6/. 12/.
Montana
New York Carolina

MiBSouri

Delaware District Columbia. Florida

6^
Any. 7"foAny.

North Ohio

6/f

Georgia
Idaho Illinois Indiana
Iowa Kansas

lOfc lOfc

Oregon Pennsylvania
Rhode Island

8/. 12/.

Kentucky
Louisiana Maine
-^

Maryland
Massachusetts

Michigan
Minnesota

Mississippi

6/. Any. 7/. Any. Tennessee 10% 6/. Texas 8/. 12/. 7"/o 12/. United States 6/. 6^ Utah 10/. Any. .."T. Vermont Any. 6/. Virginia 0/. Wash. Territory... 10/. Any. 6/. 7fc lOfc West Virginia 7/. 10/. 12/, Wisconsin Wyoming 10% 12/. Anv.
South
Carolina

When
the law.

the per

cent

of interest is not

mentioned
cent cent

in the note
may

or

tract, con-

first column in stipulated column

gives the
the note,
mav
a

per per

that

be collected
as

by

If

of interest

high

as

that

in the second

be collected.

SIMPLE

INTEREST.

221

7. Usury than
Rem.

that

charging interest allowed by law.


is
be
seen

at

per

cent

greater

"

It will

from

the

table

above

that

abolished practically

in

nearly half the

States and

usury is Territories.

now

subject of Interest may be divided (2) Compound Interest^ (3) Annual Simple Interest^ (4) Partial Payments.
8. The

into

(1)
,

Inter est

SIMPLE

INTEREST.

182.

1. Simple

Interest

is interest

on

the

principal

onl}^
Simple interest is due. principal
Rem.
"

is not

due

and

can

not

be collected

till the

2. In

(1) the
the 3.

Simple Interest principjal, (2) the


three of these There

four per

arc considered, quantities and (4) cent, (3) the tirne,

interest.

Any

quantities being given,the


are

fourth

may

be found.

Jive cases.

CASE

r.

183.
to

Given

the

the principal,

time

and

the per

cent,

find the

interest.

1st.

When
the

the

time

is

one

7jear,

1. Find

interest

of $25

for 1 yr., at

6%.
OPKnATlON.

SoLtJTioN."
one

^^ is

!f)6

(Art.162, 3). Then,

since

2 5 .0 6

year

is the unit of time, the interest

for 1 yr. is

$25 X. 06 ^.$1.50.

IJO

222

KAY'S

NEW

PRACTICAL

ARITHMETIC.

2. Find

the

interest

of 818.75

for 1 yr., at

6"%.

OPKRATION.

15)18.75(1.25
15

Solution."

is ^, (Art.162, 6|^o

3, Rem.
-i-

3 7 3 0

2). Then, the interest for


15
r=

1 yr. is $18.75

$1.25.

7 5
V b

3. Find

the

amount

of S215

for 1 yr., at

G%.
OPERATION.

$2 15
Solution." U The interest of $215
amount

for 1 yr. ni 6
^

fo
2 1 5

.0 6
1 2.9 0

$12.90; then, the

is $2\ry -\$V2/J0

$227.90.

$ 2 2 7.9 0 Hule.
"

the principal Multiply by the


To find the amount

rate.

Rem.

"

add the

and principal

interest.

Find
4. Of

the

interest
V

$200
$150

5. Of
6. Of

for 1 yr., at for 1 yr., at

S%.
7%.

$16.00.
$7.50. $5.95. $450.

5^.
GJ^.

$85 for 1 yr., at


$7200
for 1 yr., at

7. Of

Find

the amount
for 1 yr., at for 1 yr., at for 1 yr., at for 1 yr., at

8. Of $28.20
9. Of

$45.50 $420

10. Of 11. Of 12. Of


13.

$857

Of

$96 for 1 yr., at $2000 for 1 yr., at $164


for 1 yr.. at

8J%. 10%. ^%. 9%. 8^%.

$30.55. $50.05.

$442.40.
$934.13. $104.16.

14. Of

U^. m%.

$2090. $184.50.

SIMPLE

INTEKEST.

223

2d.
1. Find
SonjTioN.

When
the
The

the
interest

time

is Two

or

More

Years.

of $50

for 3 yr., at

7%.
operation.

"

$3.50; then, the


$10.50.

interest of $50 for 1 yr., at 7^, is interest for 3 yr. is $3.50X3=

$^0
.0 7 3.5 0

Rem.

"

It

is sometimes and

more

convenient

first to the

multiply the per cent above example, the per


2. Find

time

together. In

$ 1 0.5 0

cent

for 3 yr. is 21.

the amount

of

$225.18 for 3 yr., at

4^%.

OPERATION.

Solution."

The

interest of $225.18

is $10.1331; A\c/ci

then, the
$255.58.

interest and the

for \ yr., at for 3 yr. is


amount

$ 2 2 5.1 8
^0

4|

$10.1331 X

$30.3993;
^

is

9 0 0 7 2

$30.3993 -I$225.18
Rem. In

112

5 9

roX3~3T
"

business,it is customary
nearest

to

take

the
exare

3 3 0.39 2 2 5.1 8 9 3

final result to the

unit. and

Thus, in the
of
a

ample,
nearest

57 58

cents cents.

9 mills

3 tenths

mill

2 5 5.5 7 9 3

Rule.

"

1. Find

the interest

for

one

year.

this 3Iultiply

by

the

givennumber
the interest

of

years.

Find
3. Of 4. Of

S65

5. Of
6. Of

7. Of Find 8. Of 9. Of

5%. $300 for 2 yr., at 6%. $275 for 3 yr., at 6%. $187.50 for 4 yr., at 5%. $233.80 for 10 yr., at 6%.
amount

for 4 yr., at

$13.

$36.
$49.50. $37.50. $140.28.

the

for 2 yr., at 8%. $80 for 4 yr., at 7%.

$45

$52.20.
$102.40.

224

KAY'S

NEW

PRACTICAL

ARITHMETIC.

10. Of

11. Of 12. Of 13. Of 14. Of 15. Of

$237.16 for 2 yr., at 3f%. $74.75 for 5 yr., at 4%.


$85.45 $325
lor 4 yr., at 6%. for 3 yr., at 5|%. for 4 yr., at

$254.95.
$89.70. $105.96. $377.65. $152. $10144.20.

$129.36
$8745

for 2 yr., at

4f%. 8%.

3d,

When
the

the

Time

is atiy

J^iunhev
nio., at

of Months. 6%.
OPBRATION.

1. Find

interest

of $24

for 9

Solution

I.

"

mo.

are

of ij

ji

year.

The

interest of
for 9

$24 for 1 yr.,


mo.

at

6%, i^ $1.44; then, the interest


is $1.08.

is

J of $1.44, which

Solution
u

II."

(Art. 130)6
\ \
of 6
mo.

mo.

arc

h of
est inter-

are year, and 3 mo. of $24 for 1 yr., at mo.

The

is $1.44; 65^,

then, the
is 72
6 3
mo.
mo.

interest for 6

is

of $1.41, whieh
mo.

ct.,and
which

the

interest for 8

is \ of 72 et.,
for 9
mo.

is 36 ct.
36
ct.

Then, the interest


=r$1.08.

is 72 ct. +

Rule.

"

1. Find this
as

the interest for


the

one

year. 7honths

Take
is

such

a
a

part of
year.

given number

of

part of

Find
2. Of 3. Of
4. Of

the interest

$300
$240

for 1 mo., for 2 mo.,

at
at at at

5. Of
6. Of

$ 50 for 5 mo., $ 86 for 3 mo.,


$ 50
for 4
mo.,

at

6%. S%. 6%. 6%. 8%.

$1.50. $3.20. $1.25. $1.29.


$1.33.

SIMPLE

INTEREST.

225

Find 7. Of 8. Of 9. Of 10. Of
11. Of 12. Of

the

amount for G mo., at at


at at at at

S150.25

S360
$204 $228

for

7 mo.,
mo., mo.,
mo.,

8%. 5%.

$156.26, $370.50. $217.09. $238.26.


$143.00. $8102.40.

for 11 for
9 8 10

7^.
6%. 6%. 8%.

$137.50 for
S759()
for

mo.,

^th.
1. Find

When

the

Time

is any

JWoinher

of Days,

the interest of $288

for 24

da., at

5^.
OPERATION.

"288

Solution of $288

I." 24

da.

are

4 of

month.

The

terest in1 2

.0 5

for 1 mo.,

at

is 5^^^

$1.20; then, the


is 90 ct.

) 1 4.4 0
5 ) 1.2 0 .2 4

interest for 24 da. is

| of $1.20, which

_4
.9li
Solution
of da.
1
a

II."

(Art.130)
are

15

da.

are

h
3

OPERATION.

month, 6 da.
are

of

month, and

288
.0 5 1 2

^
at

of 6 da.

The

interest of $288 for

mo.

5^, is $1.20;
I
of
-i

then, the interest for


terest is 60 ct.; the in-

)1 4:40

15 da. is

$1.20, which

for 6 da. is and which the interest


ct.

of $1 .20,which

is 24 ct.,
of 24

for 3 da. is

ct.,

is 12
ct.

Then, the
24
ct.

interest
ct.
"

for 24
ct.

da. is 60

-[-12

^Q

.9 6

Rule.

"

1. Pind
as

the interest for the

one

month.

Take is

such

a a

part of this
month.
Rem.
1 month. In

given number

of days

part of

"

it is customary computing interest,

to

regard 80 days as

Prac. 15.

22G

RAY'S

NEW

PRACTICAL

ARITHMETIC.

Find
2. Of 3. Of 4. Of 5. Of 6. Of

the

interest for 20 for 10 for 15 for 19 for 27 for 21 for 25


at 6%. dii.,
tit dti.,

S360 S726
$1200

$1.20,
$1.21 $3.00
76
ct

$180 $240 $320 $450

da.,at da.,at 8%.

6^. 6^.

7. Of
8. Of

da., at 7%. da., at 5%. da., at 10%.

$1.26
93 ct.

$3.13.

Find
9. Of 10. Of 11. Of 12. Of 13. Of 14. Of

the amount $100.80 $150 $360 $264 $900


for 28 for 18

for 11 for
9

da., at 5%. da., at 5%. da., at da., at 6%.

$101.19. $150.38. $360.66.


$264.40. $902.45.

6^.

for 14 for 19

da.,at 7^.
da., at 4^%.

$430

$431.02.

Sth.

When

the
or

Time Two

is Tears, Months,

and

Days,

any

of

these

Periods.

First 1. Find
at

Method.

the

interest

of

$360

for 2 yr. 7

mo.

25

da.,

8%.
OPERATION.

$360
Solution
1 yr., at

1 2 ) 2 8.8 0

I." The is 8^^,

interest of $360

for

.0 8
2 8.8 0 2 5 7.6 0 1 6.8 0 6

2.4 0

$28.80; then, for 2 yr. the interest is $57.60; for 7 mo., or /^


of 25
a

7_
$16.8
0

year, the interest is da., or f of a month,

$16.80; and
the
2

for

interest is yr. 7
=

) 2.4 0
M)
5

$2.
25

Then, the interest for


da. is $57.60 +

mo.

2.0 0

$16.80 + $2

$76.40.

$7 6.4 0

$2.0 0

simple:

INTEKE8T.

227

OPERATION.

Solution

II."

(Art. 130).

The

est interfor

$?

of $360 for 1 yr., at


2

8^^,

is

28.80, and

half of a or yr. it is $57.60; for 6 mo., year, the interest is $14.40, and for 1 mo.,
or
a

^ of 6 mo., it is $2.40; for 15 da., or ^ of month, the interest is $1.20, and for 10

7.6 0 6 1 15
mo.
mo.
=

^
I

4.4 0

da.,or
interest

of
for

month, it is 80 ct.
yr. 7
mo.

Then, the
$76.40.

2.4 0
1.2 0 .8 0

25

da. is $57.60 -f
=

da.

=:

$14.40 + $2.40 + $1.20 + $0.80

10 da.

=1

$7 6.4 0 Rule I.- -1. Fhid

the interest

for

each

and period,

add

the results. Second


2. Find
at

Method. $120
for
4 6

/^
yr.
mo.

the

interest

of

20

da.,

6%.
OPERATION.
3 0
"

S 2_0.

Solution.

"

20 of
a

da.

are

of

Q2
a

mo.;

6|
at

mo.

are

of $120

terest Then, the inyear. 20 da., for 4 yi-. 6 mo. be

6^c,will

$120 X

-06X41

$32.80.
40 .02

32.80

Rule

II.

"

1
.

Reduce

the months

and

days

to the

fraction

of

year.

the principalby the rate, and Midtiply product by the time expressedin years.
Kem.
"

2.

multiplythe

Indicate

the

operationas

far

as

is

and employ practicable,

cancellation.

3. Find

the

interest

of $150

for 4 yr.

mo.,

at

6^.
$37.50.

228

KAY'S

NEW

PRACTICAL

ARITHMETIC.

Find
4.

the interest of

S375.40 for 1 yr. 8 mo.,


$ 92.75 $500 $560 $750 $456 $216
for 3 yr. 5 mo.,
mo. mo. mo. mo. mo. mo.

fit at

5. 6. 7. 8. 9. 10. 11.

G^. 6^. 6^.


8%.

S37.54. $19.01. $34.00. $106.40.


$192.00. $79.04.

for 1 yr. 1 for 2 yr. 4 for 4 yr. 3 for 3 yr. 5 for 5 yr. 7 for 3 yr. 9 of

18

da., at

15 6 18 27
9

da.,at da.,at da.,at

6^.
10^.
5%.

$380

da., at da., at 15%.

$122.22.

$215.18.

Find
12. 13. 14. 15. 16. 17.

tlie amount

$300 for 3 yr. 8 mo., $250 for 1 yr. 7 mo.,


$205.25 $150.62
for 2 yr. 8 for 3 yr. 5
mo.

at at

6%. 6%.
15 12 20 23

$366.00. $273.75.
$238.60.

da., at 6%.

mo.
mo. mo.

da.,at

5^. $176.60.
$249.09. $64.54.

$210.25 for 2 yr. 7 $ 57.85 for 2 yr. 3


the interest
at

da., at 7%. da., at 5%.

18. Find
to

of

$150, from

January 9, 1847,
$20.50.

April 19, 1849,


Rem.
"

6%.
between
two

To

find the time

dates, see

Art. 77.

19. The

interest

of

$240,

from

February 15, 1848, to


$23.04.

April 27, 1849, at 8^.


20. The interest of

$180, from

May 14, 1843, to August


$28.84.

28, 1845, at
21. The

7^.
of

interest

$137.50,from $125.40, from $234.60, from


from

July 3, to
March

November

27, at 9^.
22. The
amount

$4.95.
of

1, to August
$130.64.
to

28, at
March
24.

Si%.
amount

23. The

of

August 2, 1847,
October

9, 1848, at
The
amount

5J^.
of $153.80,

$242.02.

25, 1846,

to

July 24, 1847, at 5%.

$159.55.

SIMPLE

INTEREST.

229 of

184.

The

twelve

per

cent

method

findinginterest.
time,
at

Ist. To

find the

interest

of SI

for any

at

12%,

Explanation. 1 ct.; for 2 mo., The cents


as

"

The

interest of $1 for 1 mo.,

is $0.01, or \2(ij,

it is 2

ct.; for
any

3 mo., number

it is 3 et.,etc.

Hence, VloL, is
1
as

ifiterest there

of $1 for
mofiths.

of months,
or

at

many

are

The it is 2
The mills
as

interest of $1 for 3

da.,at

is $0,001, 12^/^,

mill; for

da.,

mills; for
interest
there

da.,it is 3 mills,etc.
any number

Hence,

of $1 for are days.

of days,at 12^,is J

as

yna.ny

Rule.

"

Call

the months

cents, and, one-third of the days

mills.

Rem.

"

Reduce

years

to

months.

Find

the

interest

of

$1, at

12^,
$0,094
$0,046,

1. For
2. For

9
4

mo. mo. mo. mo.

12 18 12
3 4 5 3

da. da.
.

3. For
4. For

7
9

da. da.
mo. mo. mo. mo. mo.

$0,074

$0,091 $0.16,
27
21 12 15

5. For
6. For

1 yr. 1 yr. 2 yr. 3 yr. 4 2

da.
da. da. da.

$0,179,
$0,277

7. For 8. For
9. For 10. For

7
2 1

$0,434 $0,505

yr.
mo. mo. mo.

da. da. da.


mo. mo. mo.

11. For 12. For 13. For 14 For

5
10

17 13
2 9 5

1 yr. 2 3

4 20 29

da.
da. da.

yr. yr.

15. For

$0.020J $0.055f $0.104J $0.141J $0.336" $0.419"

230

RAY'S

NEW

PRACTICAL

ARITHMETIC.

2d.
per

To

find the

interest

of

$1,

for

any

time

at

any

cent.

1. Find

the interest

of

SI, for

2 yr. 5

mo.

18

da.,at

Solution. for 2 yr. 5

"

mo.

6q^ is i of 12^. The interest of $1 is $0,296; then, the in18 da.,at 12^^,

operation.

terest, at 69^, will he

^ of $0,296, which

is $0,148.

9 G ).2 ~ATs'

2. Find

the interest of

$1, for

yr. 7

mo.

20

da., at

operation.

Solution."
3 yr. 7
mo.

20

is ^ of Ufc The interest of 8^0 is $0.436";then, the da.,at 12^^, be

$1 for
inter-

3) .4 3 6^

TTSJ
2

est,at

will Sfo,

of

$0.43G",which

is $0,291^.

.29lj
Rule.

-1. this

Find
as

the interest, at

12^, and
is

take

such

fart of
Find

the

givenper
of

cent

of 12%.

the

interest 24 15 18
2

81,
$0,039.

For For
For For

7 10
11 1

mo. mo. mo.

da.,at 6%.
da., at 5%. da., at 9^.
mo. mo. mo. mo. mo.

?0.043f.
$0,087. $0.07 U. $0.1 9G.

da.. at

For For For


10. For

2
3 4

yr-

12 17
11 24

da., at

yi*- 10 yi^ yi'3

da., at da., at
da.. at

10^. 4^.
for

$0.388yV $0.299|f.
$0,226.

3d. To any per

find the
cent.

interest

of

any

sum

any

time, at

SIMPLE

INTEREST.

231

1. Find

the

interest

of

S25,

for 1 yr.

mo.

18

da., at

Solution.

"

The

interest of

$1 for 1 yr. 5

mo.

18

da., at 12t/c,is $0,176; then, at 6%, it is $0,088. $2.20. Then, the interest of $25 will be $0,088 X 25
==:

2. Find
at

the

interest

of

$134.45, for

1 yr. 7

mo.

15

da.,

S%'
OPERATION.

Solution." yr. 7
at
mo.

The

interest

of

$1

for

).l9

$13

4.45

15

da., at

is $0,195; then, 1^^^.,

.0 6 5 2 .13 0 403 1 3445

J_3
35

Sfc,

it is $0.13.

Then,

the
=

interest of

$134.45 will be $.13 X

134.45

$17.48.

$ 1 7.4 7 8 5 Rule. 1. Fhid the

"

interest

of $1, and

this by multiply

the

given sum.
"

Rem.

Take

either f^ictor for the

as multiplier

is most

convenient.

Find 3. Of
4. Of

the

interest for 6 for 8 for 10 for 11


mo. mo. mo. mo.

5. Of
6. Of

7. Of
8. Of 9. Of

$40, $50, $120, $200, $500,

21
24 12 15
mo. mo.

da., at 6%. da., at 9^. da., at 7%.


da., at 6%.
G

$1.34
$3.30 $7.28 $11.50 $19 $89.50 $5.14 $5.81

for 1 yr. 3 $750, for 1 yr. 5

27

da., at 3%. da.,at S%.


3

$48.75, for

1 yr. 1 yr.

9 10

mo. mo.

da., at
25

6%.

10. Of

$7G.32, for
the
amount

da., at

4^.

Find
11.

Of

12. Of

$600, $900,

for 2 yr. 1 for 2 yr. 4

mo. mo.

da., at 5%. 10 da., at 6%.

$663.25. $1027.50.

232

RAY'S

NEW

PRACTICAL

ARITHMETIC.

13. Of 14. Of 15. Of

S86.25,for 2 S450, for 3 $534.78, for S1200, for

yr. 7 yr.
3

mo. mo.

yr.

mo.

da., at 9%. $10G.G7. 13 da., at 8%. $565.30. 22 da., at 4%.


S609.17.
15

17

16. Of

3 yr. 11

mo.

da., at 10%.

$1675.

CASE

II.

185.

Given
to

the the

the principal, time.

jkm* cent

and

the

terest, in-

find

1. The

interest
was

of $225
time?

for

certain

time, at 4%,
OPERATION.

was

$66: what
Solution." yr., at
terest

the

The is

interest of $225

fori

$225
.0 4

9)66 7J
7 yr. 4
mo.

4^,
for

$9;

then, $66 will be the inyears is


as

times

many in 66, which

as

is contained
mo.

$9.0 0

7J,or

7 yr. 4

7J

yr.

2. In

what

time, at 10%,

will $500

amount

to

$800?

OPERATION.

800 Solution. -The interest will he

$800

"

$500=
10

^^^
^^^

$300.

The

interest of $500 will be the

then, $300
50

for 1 yr., at interest for as many is 6.

is $50; 10^^, years


as

) 5 00
^0

is contained

times

in 300, which

50)T00
6

3. In itself?

what

time,

at

8%,

will

any

principaldouble

Solution. doubled
comes

"

principal
the the be interest interest 100 f^ in

has beis
as

operation.

itself when
100

8)100
1

^.
as

Since

2^

8^
many

in

yr., it will
8

years

is contained
..r

times
6
mo.

12^

yr.

12

yr. 6

mo.

in 100, whicli

is VJ.\.

V2

vr.

SIMPLE

INTEEEST.

233

Rule.

"

1. Divide
one

the year.

given interest by

the interest

oj the

for prineipal
If

Rem.

1.
"

the

principaland
a

amount

are

given, subtract
in the

the

from principal Kem. 2.


"

the amount

to find the interest,

If there be

fractional

part of

year

duce result,re-

it to months

and

daj^s.

4. I lent

$200, at 6%,
money

and

received

$36 interest:
3

how yr.

long was
5. In

the what

lent? will $60


amount to

time, at 5%,
time, at
lent
6

$72?
4 yr.

6. In

what

%
at

will any

be principal

doubled
mo.

16 yr. 8 7. A how
man

$375,

8%,

and

received

$90 interest:
3

long was
what

it lent?

yr.

8. In

time, at 9%, time,


at

will $600

amount

to

$798?
mo.

3 yr. 8 9. In

what

10%,

will

any

double principal
10

itself? 10. How interest? 11. How

yr.

long will
long

it take

$250, at 6%,
2

to

yield $34.50
mo.

yr. 3
to

18 da.
to

will it take

$60,
any

at

6%,

amount
mo.

$73.77 ?
12. How
at

3 yr. 9

27 da.

long will
interest
on

it take

to principal

treble
33

itself,
mo.

6%? 13^ The


14.

$400,

at

7%,

was

$68.60:
2 yr. 5

yr. 4 how
mo.

long
da.

was

it loaned? In what

12

time, at 9%,
will

will $700

amount 3 yr. 6

to
mo.

$924.70?
24

da.

15. How

one-half, at

long 8% ?

it take

any

to principal

increase
mo.

6 yr. 3

16. In whut

time, at

will 10^;,

$1200
"

amount 3 yr. 11

to
mo.

$1675?
15 da.

234

KAY'S

NEW

PKACTICAL

ARITHMETIC.

CASE

III.

186.
to

Given per

the

the principal,

time

and

the

interest,

find the

cent.

1. A

merchant
mo.:

paid $30
what
was

interest for the the per cent?

use

of

S300,

for 1 yr. 8

OPKRATION.

Solution."
Since
terest

1 yr. 8

mo.

are

1|, or J

yr.

1 yr. 8

mo.

J
"

the interest for for 1 yr. is $18.

yr. is $30, the inof $300; $18 is ^3^

X | -Y'

yr. 18

^y*^ -^^
=r:

^^

are

Qfc (Art. 162).


what per
cent

i^jf
=

Qfc'

2. At in 20

will

any

double principal

itself

yr?
"

Solution.
when

principalhas
become

doubled

itself the

operation.

the interest has

100 o^. Since

100-^2

interest for 20 is

yr. is 100 o^, the interest for 1 yr.


=

^V

of 100

fc

%.
the interest the

Bule.

"

1. Find this is

for

one

year, and

find what

per

cent

of

principal.
for 2 years and

3. I borrowed

$000
did I

paid $48
use

interest:

what
4. A

per

cent

pay?
interest the for the of
was

4^.
$1000 per cent?
yr. 4
mo.

broker
mo.:

paid $200
what
of the

for 2 yr. 6 5. The

8^.
24

amount
was

$250
per

for 2

da.

was

$310: what
6. $23.40

cent?

10%.
for the
use

interest
was

was

paid
cent? will

of

$260 for 2

yr.

what

the per

per
cent

4^%any double principal itself

7. At

what

in 12 yr. 6 mo.? 8. The amount what


was

S%.
of

$175

for 3 yr. 7

mo.

was

$250.25:

the

per

cent?

12%.

SIMPLE

INTEREST.

235

9. The

interest

of

8450
per

for

yr.

mo.

12

da.

is

$61.20: what
10. At what

is the

cent? will any double principal

8%.
itself in

11 yr. 1 mo. 11. The amount

per cent 10 da.? of

9%.
$650
for
2

yr.

mo.

18

da. is

$746.20: what
12. The
was

is the of

per

cent? for 6 yr.


was

6^.
$110.40:
what

interest cent?

$640

the per

2^%CASE IV.

187.
the

Given

the

time,

per

cent

and

to interest,

find

principal.
interest for
2

1. The

3^-.,at

6%,

is $27:

what

is the

? principal
Solution."
6

^0 is -5%(Art.162, 3).

Since

operation.

for of the principal, -5^^ 2 yr. it is -^-^ of the principal. Then, ,^ of the is $9, and are $27, 2V ^^ ^^" principal principal the principal is $225.

the interest for 1 yr. is

X f J^^
^

-j3_ 2^5
225

X ^"^
=^

2. The

interest for 3 yr., at

9%,

is $21.60: what

is the

? principal
Solution.
for 1 yr. is ^oL is .09.

"

Since

the

interest yr. it is

operation.

for 3 principal, mul.27,the principal. Then, the principal, is tiplied by .27,is $21.60, and the principal

.09,the

9 oL =r.09 .09

3 =.27
=

$21. 60---. 27

80

$21. 60

"

.27

$80.

Rule.

"

interest
3. The

Multiply the rate by the product.


interest

by

the

time, and

divide

the

for 3 yr., at

5%,

is $8.25:

what

is the

? principal

$55.

236

RAY'S

NEW

PRACTICAL

ARITHMETIC.

4. The

interest

for 3

yr., at

5%,
at

is

$841.25: wliat is
$2275.

the

princi2)al?
interest for
1 yr. 4 mo.,

5. The is the 6.

6%,

is $2.20: what

principal? What at 5%, principal,


at 8%, principal,
mo.

$28.25.
will

produce a yearlyinterest
$20475.

of $1023.75? 7. What in 1 yr. 6 8. What in 12 yr. 3 9. The will

produce $30.24 produce $525.40


mo.

interest

27

da. ? will

$240.
interest

at 9%, principal,
mo.

20

da.? for 2 yr. 7 11

$474.40.

interest is the interest

da., at da., at

4%, 6%,

is

$9.41: what
10. The

principal?
for 5

$90.
mo.

yr.

24

is

$28.38: what

is the

principal?

$82.50.

CASE

V.

188.
the

Given

the

time, per cent,

and

amount

to

find

principal. principalin
5 yr., at

1. What

G%,

will

amount

to

$650?
Solution.
1 yr. is

"

6^

""f* the 5 yr. it is j\ of 3^0 the principal, and the amount is \^ of the are $650, principal. Then, }| of the principal is $50, and is of the principal the principal y^^

Since -^jj. for principal,

is

the interest for

operation.

^-^ A ,3yXf }g -f fV^if


=

(jcL

50

$500.

IMx^$='jOO
"

Rule.
amount

Multiplythe rate by by 1 -\-the j)roduct. principalin


9

the

time, and

divide

the

2. What

yr., at

5%,

will

amount

to

$435?

$300.

COMPOUND

INTP]REST.

237

3. The the

amount

for 4

yr., at

5%, 7%,
mo.,

is $571.20:

what

is

interest?
amount

S95.20.
for 6 yr., at is $532.50: what is

4. The

the

interest?
amount

$157.50.
for
2

5. The what

yr.

at

8%,

is

$285.48: $234.

is the

j)rincipal?
for 2 yr.
6 mo., at

6. The is the

amount

6%,
24

is $690:

what

interest?
amount

$90.
for is the for
3

'

7. The

yr.

mo.

da.,

at

7%, 4%,

is

$643.76: what
8. The
amount

principal?
4

$520.
mo.

yr.

27

da., at

is

$914.94: what 189, Formulas


Let
i the h

is the

interest? of Interest. the

$134.94.

for the five

cases

t the time, r represent the principal, interest. Then,

rate,and

Case Case Case Case Case

I. II. TIT. lY. Y.

b Xr
I

Xt=^i.

-f-

(6 X 0
~

"

^"
r.
*

t) i -^{rXt)
+

(i^

'

b,

^--^"^-^^^b. 1
X 0 (^INTEREST.

COMPOUND

190.

In

Compound
addition

Interest

the

is increased principal

yearlyb}^the
Rem.
1.
"

of the

interest.

Sometimes

tlie

interest

is

added

semi-amiually,or

quarterly.
Rem. 2.
"

The

way

in which
to

interest is
up the old

the end with


note.
a

of each face

year,
to

take

compounded is,at legally and give a new note one


interest

equal

both

the

principaland

of the

former

238

KAY'S

NEW

PRACTICAL

ARITHMETIC.

1. Find

the

compound
interest

interest

of

$300

for 3

3^r., at

6%.
Solution. of $300
"

The

for 1 yr., at
amount

6^, is
$18+

operation.

$18; the
$300
of is
r^

is

$3 0 0
0 6 18.0 3 00 0

$3 3 7.0 8
.0 6 2 0.2 2 4 8 3 3 7.0 8

$318.
for
1

The

interest

$318

yr., at
amount
=

6^^,
is

$19.08; th'
+
$318

$10.08
The 1

$337.08.
for

-3X8.
.0 6
19.08

357^3048
3 0 0

interest of $337.08

yr., at
amount
=

6^^,

is

$20.2248; Then,
is

$57.3048

the

is $20.2248 +

$337.08
the

$357.3048.

compound
"

interest
=

$357.3048

$300

$57.30.

Kule.

"

1.

Find
and

the

amount

of

the

firstyearj

make

it the

for givenprincipal principal for the second


second
so on

the

year,
2. Find make the amount it the

given number
3. From

of this principal for the for the third year, and principal of yearns.
subtract the be the

year^
the

for

the last amount will

givenprincipal ; the

remainder

compound

interest.

or payable half-yearly, quarterly, find the interest for a half, or a quarter year, and proceed in other respectsas when the interest is payable 3'^early. the time Rem. is years, months, and days, find the 2. When
"

Rem.

1.

When

the

interest is

"

amount

for and

months

the interest compute years, then days,and add it to the last amount.

the

on

this for the

Find 2. Of 3. Of

the

amount,

at

6%, compound interest,


S595.51.

$500, $800,

for 3 years. for 4 3^ears.

$1009.98.

ANNUAL

INTEREST.

239

Find 4. Of 5. Of
6. Of

the

compound

interest

$250, $300, $200,


the

for 3 yr., at for 4 yr., at for 2 yr., at

$47.75. 6%. $64.65. 5%. 6%, pajablesemi-annually.


$25.10.

7. Find

amount

of

$500,

for 2 yr., at

20%

pound com-

interest, payable quarterly.


8. What
mo.,

$738.73.
of

is the

compound compound 6%?


amount

interest

$300,

for 2 yr.

at

6%?
is the interest of

$47.19.

9. 8
mo.

What 15

$1000,

for 2 yr.

da., at

$171.35.
at

10. What

is the

of $620
at

compound

interest

for semi-annually, 11. What is the interest

3 yr. 6 mo.,

6%?
at

$762.52.

diiference
on

between

simple interest

and

compound

$500, for

4 yr. 8 mo.,

6%

$16.49.

ANNUAL

INTEREST.

191.
and
on

Annual

Interest

is

interest

on

the

principal,

each

annual

interest

after it is due.

Rem. Rem. reads Rem.


"

1.
"

This interest is sometimes


"

semi-annual

or

quarterly.
the note
or

2. with
3.
"

Annual

interest may

be collected when

bond,

interest The
are

payable annually."
interest is sometimes
at the
same

annual

notes; these
and draw 4.
"

given
annual

time due. bonds


are

as

by interest represented the note for the principal,


representedby
the bond and

interest if not The

paid when
interest
on

Rem.

is sometimes detached
from

interest notes, called coujwns; presentedfor payment when

these

the interest is due.

1. ]^o

interest
mo.

having
da., on

been
a

paid, find
note

the

amount

due

in 4 yr. 8
at

24

for

$400,

with

interest

6%, payable annually.

240

KAY'S

NEW

PKACTICAL

ARITHMETIC.

Solution.

"

The

interest of

24 da.,at for 4 yr. 8 mo. terest 6^,is $113.60. One annual in-

$400

of $400, at The mains first annual

is 6^c" interest

$24.
re-

unpaid

yr. 8

mo.

24
mo. mo. mo.

da.; the second, 2 yr. 8 24 da.; the third, 1 yr. 8


24 24

da., and

the

fourth, 8

da.; hence,
on

interest must

be

reckoned
mo.

$24

da.;

for 8 yr. 11 this is $12,864.

The

amount,

then, is $12,864
=

-f $113.60 -I-$100

$526.46.

Bule.

"

1. Find
no

the interest annual

of

the

during xchich
2. Find the times 3. The and each
sum

interest is
one

for principal paid.


interest

the time

the interest of annual

annual

for

the

sum

of due,

interest remains
two

unpaid.
be the amount due.

of

the

interests will be the interest

added this,

to the

will principal,

2. ^o in 3 yr.

interest
on a

having
for

been

paid,find
Interest

the amount
at

due

note

S800, with
been
a

8%, payable
S1007.36.
yr., find

annually.
3. The
amount at

interest due in 5

having
yr.
on

paid
for

for

the

note

$750, with

interest

10%, payableannually. interest having been 4. No


due in 6 yr.
on a

8997.50.

interest
at

bond

paid for 4 yr., for $10000, with

find

the

interest

$2150. 5%, payableannually. due interest having been paid,find the amount 5. No Sept.1, 1877, on a note for $500, dated June 1, 1875, with interest at 6%, payable semi-annually. $571.10.

PARTIAL

PAYMENTS.

241

6. For G.
at

[S1200.]
value

Milwaukee,

Wis.,May 12, 1873.


I

on demand, received, Morgan, or order,twelve hundred 6%, payableannually.

promise to pay John with interest dollars,


H.
was

W.

Slocum. the
amount

No due
on

interest this

having been paid, what 20, 1877? note, Sei:)tember


New

$1545.66.

7. On

[$1500.]
the first

Orleans, La., October 10, 1872.


of

I May, 1877, for value received, dred promise to pay Andrew^ Jackson, or order, fifteen hunwith interest, dollars, payable annually,at 5%.

day

George No
at

Quitman.
was

interest

having

been

what paid,

amount

due

maturity?
8. What is the difierence

$1872.75.

between

simpleand

annual

interest

on

$1000
will be

for 5 yr., at due


on

6%

$36.

9. What

six 500-dollar
at

3 yr., with interest not if the interest should 10. The interest
on

be S.

U.

ning citybonds run6%, payable semi-annually, $3580.50. paid? bonds 4 per cent is payable

in gold; granting that the income quarterly might be immediately invested,at 6%, w^hat

from would
to

them the 5 yr.,

income with

on

20 at

1000-dollar

bonds

amount

in

gold

105?

$4798.50.

PARTIAL

PAYMENTS.

192.
than the
2. The

1. A

partial payment
a

is

sum

of

money,

less

face,paid on
of receipt it
16. on a

note.

payment partial
of the
note.

is

acknowledged

by indorsing
Prac.

the back

242

KAY'S

NEW

TKACTICAL

ARITHMETIC.

3. The of the

indorsement

consists

of

the

date

and

amount

pajMuent.
rule of the
to

4. The

in reference
'^

Supreme Court Partial Payments, is

of the United
as

States,

follows

partialpayments have been made, apply the of the interest place,to the discharge payment, in the first
When
then
^'

due.

If

the payment

exceeds

the

the surplus goes interest,

toward

the subsequent the principal, and terest indischarging remuinis to be computed on the balance of principal

ing due.
"

If

the

payment
not

be less than be

the

the surplus interest, of the

but principal, tintil the period interest continues the former principal, on the interest due, exceed when the payments, taken together, then the surplus is to be applied toward and discharging the principal interest is to be computed on the balance, ; and as aforesaid.'' Kent, C. J. interest must

taken

to

augment

"

Rem.
nor

"

This

rule is founded

on

the

that principle

neither interest

payment shall draw


1. For

interest.

[SIOOO.]
value received,
or on

Boston, Mass., May 1, 1875.


demand, I promise to pa}^ to thousand dollars,with order, one
William Murdock.

Alonzo interest

Warren,
at

6%.
note

On
:

this

partial payments

were

indorsed

as

lows fol-

25, 1875, $134; March 7, 1876, $315.30; August 13, 1876, $15.60; June 1, 1877, $25; April 25, the amount due on settlement, w^as 1878, $236.20. What

November

September 10, 1878?

FAUTIAL

PAYMENTS.

243

Solution.
from

"The

time
to

May
24

1, 1875,

November
mo.

25, 1875, is

da.;

the

interest is

of

$1000

for this time

$34; the payment, $134,


exceeds
amount
"

the
is
=

interest; the

$1034; $1034
the
sec-

$134

$900,

ond

iirincijiol.
time from vember Noto
mo.

The

25,
March
12

1875,
3

7, 1876, is
for this the

da.; the

interest time

of is

$900

$15.30;

payment,
the
terest; inamount

$315.30, exceeds
the

is

$915.30; $915.30"1315.30 =r$600,


the

third

jirin-

cipal.
The time
to
mo.

from

March

7, 1876, 1876, is
interest 5

August
6 for

13,
this
ment, pay-

da.; the

of $600

time

is $15.60; the

$15.60, equals the

interest;the
$600,
The
the

amount

is

$615.60; $615.60"
^^

$15.60

fourth prinfrom June

cipal.
time
to

August
1, 1877,
for this

13, 1876,
is 9
mo.

18 of

terest da.; the in-

$600

time

is

$28.80; the payment,


to

$25, is less than


the

interest;continue

find

the

interest

on

the

fourth

princij^al.

244

KAY'S

NEW

PRACTICxVL

AKITIIMETIC.

The

time

from

June

1, 1877,
of

to

April 25, 1878, is


sum

10

mo.

24

da.;
is

the interest of $000 for this time

is $32.40; the

of the payments,
amount

$261.20, exceeds

the
"

sum

the

interests,$01.20; the

$661.20; $661.20
The time from the ";la.;

$261. 20 ==$400, the

April 25, 1878, to

ffth principal. September 10, 1878, is 4


is $9; the amount

mo. on

15
tlement set-

interest of $400 for this time is $409.

due

RULE.

I. When
1. Find

each

payment from of

equalsor
the date

exceeds

the

interest.

the time

of

the note

to

the date

of

the

first payment
the amount this amount the

2. Find 3. From

for givenj^rincipal

this time.

subtract

the payment ; the remainder


the

is the second
4. Find

principal.
time

the

from

the date

of

first payment

to

the date
5. Then anil first,

of the
so on

second

payment.
the second
as principal

proceed with

with the

to the date

of

settlement.

II. When
interest. 1. Continue until
a

one

or

more

payments

are

less than

the

to

date

is

find the interest the when reached,


the
sum

on
sum

the

same

principal
payments
the

of

the

equalsor
2.

exceeds

Then

subtract

the

of the interests. sum of the payments from


is th?. next

amount;

the remainder

principal.

Rem."

Sometimes

payment

that the it may be determined, by inspection, when this can be done, it is not is less than the interest;

but interest may necessary to find the intermediate time and interest, be found to the date when it is apparent that the sum at once of the

payments

exceeds

the interest.

PARTIAL

PAYMENTS.

245

2. For
or

[S350.]
value
on

Boston, Mass., July 1, 1875.


Edward and James

order,

received,I promise to pay demand, three hundred


at

Sargent, doHars, fifty


Gordon.

with

interest

6%.
March

Indorsements:

1, 1876, ^44;
; December

October

1, 1876,
What

SIO ;
w^as

January 1, 1877, S26


due
on

1,1877, $15. 16, 1878?

the amount

March settlement,

$306.75.
3. A
note

of $200

is dated

January 1, 1875.

ment: Indorse-

the amount due was January 1, 1876, $70. What $150.52. January 1, 1877, interest at 6% ? Indorsements: of $300 is dated July 1, 1873. 4. A note was January 1, 1874, $109; July 1, 1874, $100. What the amount due January 1, 1875, interest at 6%?

$109.18.
ments: May 10, 1870. IndorseSeptember 10, 1871, $32; September 10, 1872, the amount due November $6.80. What was 10, 1872, 5. A
note

of

$150

is dated

interest
6. A

at
note

6%
of

$132.30.

$200

is dated

March

5, 1871.

ments: Indorse-

June What
was

5, 1872, $20; December


amount

5, 1872, $50.50.
at

the

due

June

5, 1874, interest

10%

$189.18.
7. A
note

of $250

June

What

was

the

ments: January 1, 1875. Indorse1, 1875, $6; January 1, 1876, $21.50 due July 1, 1876, interest at 7%' amount is dated

$248.40,

August 1, 1874. Indorse ments: February 1, 1875, $25.40; August 1, 1875, $4.30 due July the amount was January 1, 1876, $30. What $138.54. 1, 1876, interest at 6% ? Indorse 9. A note of $400 is dated March 1, 1875. ments: September 1, 1875, $10; January 1, 1876, $30
8. A note

of

$180

is dated

24G

RAY'S

NEW

PRACTICAL

ARITHMETIC.

July 1, 1876, $11; September 1, 1876, $80.


the amount due March iuterest 1, 1877,.
at

What

was

6%?
8313.33.

ments: April 16, 1876. IndorseJanuary 1, 1877, $20; April 1, 1877, $14; July 25; 1877, $10; July4, 1878, 16, 1877, $31; December 10. A
note

of

$450

is dated

$18.
at

What
?
note

was

the

amount

due

June

1, 1879,

interest

8%

$466.50.
of

11. A

December

dorsemen InJanuary 1, 1870. May 1, 1870, $18; September 4, 1870, $20; 16, 1870, $15; April 10, 1871, $21; July 13,

$1000

is dated

1871, $118;
amount

December October

23, 1871, $324. 1, 1873, interest


at

What

was

the

due

6^?

$663.80.

193.
accounts

When

partialpaj'ments are made on running a 3'ear or less, the amount


found

notes

and is

due

commonly

by

the

MERCANTILE

RULE.

1. Find
note
to

the amount

of of of

the

from principal payment from

the date

of

the

the date

of

settlement. each its date


to the

2. Find

the amount

date

of

settlement. the amount the subtract principal the


sum

3. From

of

the amounts 1. A
note

of

the

payments.
is dated Nov. Jan.

of $320

1, 1876.
?

Indorsements: What
was

May 1, 1876, $50;


amount 2.

due An

16, 1876, $100. Jan. 1, 1877, interest at 6^


of

the

$186.45.

account

$540

was

due

March

1, 1877.
the
amount

Credits:

May 1, 1877, $90; July 1, 1877, $100; Aug. 1,


W^hat interest
was

1877, $150; Oct. 11, 1877, $180.


due
on

settlement

Jan.

1, 1878,

at

S%?

$39.

DEFINITIONS.

194.

1. Discount
are

is interest kinds of

paid

in

advance. Discount
and

2. There

two

Bank discount,

True

Discount.

BANK

DISCOUNT.

195.

1. Banks

lend

money

on

two

sorts notes.

of notes.

(1)

accommodation

notes, and
notes
are

(2)

business

Rem. and

"

These

frequentlytermed

accommodation

paper,

business

paper.

2. An bank

accommodation

note

is

made

payable

to

the

which

lends

the

money.
form

TvEM.

"

The

followingis a

common

of

an

accommodation

note:

^500.

Chicago, III., October


after

20,

1877.

Ninety days
pay
to

date, we,
National value

or

either of

the

Second

Bank received.

promise to ^ve Chicago, 111.,


O. W. S. West. B. Sharp.

of us,

hundred

dollars,for

18 /

Due

January"^

^^^

/oi
,

1878.

(247)

248

RAY'S

NEW

PRACTICAL

ARITHMETIC.

3. A
4. A

5. A sold.

payable to an individual. be nefjotiahle business note not negotiable. or may is one that be bought and can negotiable note
business note

is

Rem.

"

The

aro followinu:;

common

forms

of business

notes:

1st.

JV'ot ne^otiahlc.

$200.
On

demand,

Buffalo, N. Y., March 21, 1877. promise to pay Charles II. Peek, two
value received. G. W. Clinton.
at
once,

hundred

ibr dollars,

This and

note

is

payahle only

to

Charles

H.

Peck;

it is due

bears

interest from

datt?.

2d. $1000.
One
or

JS^egotidble,
St.
year after

Louis, Mo., May 1, 1877.


to

order, one

date, I promise to pay thousand for value dollars,


Elmer

David

King,

received. B. Archer.
David

The

words

"or

order"

make it
"

this note

negotiable. If
his
name

King

transfers it, he must


of it.

indorse
no

that is, write

across

the back

This

note

bears

interest tillafter it is due.

3d. $150.
On

JYeiotiable.
Washington,
or

D.

C, August 10, 1877.


1878. hundred I

before

the

first
or

day

of

May,
one

pay

Amos

Durand,

bearer,

promise to and fifty


received.

with dollars,

interest at

10^

from

date,for
John

value

Sherwood.
ment. indorse-

The

words
This

"

or

bearer

"

make

this note

without negotiable

note

bears interest from

date, it being so specified.

PANK

DISCOUNT.
A

249

6. A the time

note

is payable, the
or

or

nominallydue, at legally due,


three

the

end

of

in specified
note

note.

7. A the

matures,

is

days
are

after

time. specified three 8. The days days of grace.


Rem. months.
1.

after the

time specified

called

"

Banks

lend

money

only

on

short

time; rarelybeyond

To

find

ivhen the

note

matures:

1st. When

time

is

expressed in days :
the date

Rule.

"

Count

the

days from

of

the note and

add

three

days.
the time
Count the is

2d. When
Rule.

expressedin
from
the

months: date and add three

"

months

days.
Rem. the 2.
"

In

Delaware, Maryland, Pennsylvania, Missouri, and


the

District of Columbia,

day of

discou7it is the

first

day

of the

time. Rem.
"

8.
"

When

note

in bank

is not

protest that is,a written notice of this is served on the indorsers, securit^^ or by a notary public,
9.

paid at maturity, it goes to fact,made out in legalform,

The

bank

discount

is

simple

interest

taken

in

advance.
10.

The

proceeds
Bank

is the

money

received

on

the

note.

11. In

Discount

four

are quantities

considered:

(1)
and

The

/ace of the note, (2) the per cent, (3) the time^
these We

(4) the discount. 12. Any three of


may be found.

being given, quantities


will consider
two
cases.

the

fourth

250

KAY'S

NEW

PRACTICAL

ARITHMETIC.

CASE

I.

196.
the time

Given
to

the

face

of

the and

note, the
the

per

cent, and

find the

discount

proceeds.
interest.

1st.

When
the the

the
date

note

does

not

hear

1. Find of
at

when

due, bank

discount,and

ceeds pro-

followingaccommodation
Mobile, Ala.,
after date
we,
or

note, discounted

6%:
June

$700.

25, 1877.

Sixty days
pay to hundred
the

either of us,
of

promise

to

First

National
for value

Bank,

Mobile, Ala., seven


Walker.

dollars

received.
Charles

Walter

Smith.
OPERATION.

Solution.
1877
at

"

The

note

is due

August
of

/oy ^ '

nTTTF .0 1 0

(Art.78).
700
= =

The

interest of $1 for 63 the interest

days,
is
73

7 00
5 0 0

6^, is
"

$0.0105, and

$700

$0.0105 X $700

$7.35;this

is the discount ;

then,

7 0 0 0

0~

$7.35

$692.65, the proceeds.

7*3 ^
ir92ir5
the

Rule.

"

1. Find

the interest

on

face of

the note

for
the

the
2.

given time;
From the is the

this is the

hank

discount. subtract the discount ;

face of

the note

remainder

proceeds.
when

Find of 2. A and

the

date

due, bank

discount, and

ceeds pro-

note

of

$100, dated
at

June

20, payable in
^

60

days,

discounted

6%.

August

/22' SI.05, $98.95.

BANK

DISCOUNT.

251

3. A

note

of

$120,

dated
at

October

12, payable

in

30

and (lays,

discounted

8^.
November

$119.12. ^7^4, S0.88,


4

4.

note

of

$140,

dated
at

months,

and

discounted

January 15, payable in 6^. May

$2.87,$137.13. ^^jig,
in
6

5.

note

of

$180,

dated
at

months,

and

discounted

April 10, payable 4%.

October
6. A note

$3.66,$176.34. ^^/^3, 1, payable


in 5

of

$250,

dated
at

December

months,

and

discounted

8^. May
$8.50, $241.50. y^,
in 30

7. A

note

of

$375, dated
at

August 4, payable

days, and

discounted

6%. September

^g $2.06,$372.94.
,

8. A

note

of

$600,

dated

in 60

days, and

discounted

February 12, 1876, paj^able at 9%.

April
9. A in 90
note

$9.45, $590.55. ^^^^5


,

of

$1200,

dated

February 20, 1877, payable


at

days, and

discounted

10^.
May

^^24' ^^1' ^^^^^"

10. A in 90

note

of

$1780,

dated
at

January 11, 1872, payable

days,and

discounted

6^.
$27.59, $1752.41. ^^/i3,

April
Find the and date when of

proceeds

due, time of discount, bank notes: the following business

count, dis-

252

KAY'S

NEW

PRACTICAL

ARITHMETIC.

11.

[$600.]
E.

San

Francisco, Cal., Sept.15, 1876.

One
Abel

3^ear after date, I

promise to
First

pa}^ to

the

order of

of
San

Worth,

at

the hundred

National

Bank
value M.

Francisco,Cal.,six
Discounted

for dollars, George

received.

Burgess.

May 21, 1877, at 10%. Sept.


1877, 120 ^^/-^g, days, S20, $580.

Nashville, Tenn., May 8, 1877. [SIOOO.] Ninety days after date, I promise to pay Albert
12.

E.

Kirk,

or

order, one
June

thousand

for dollars,

value

received.

Jacob Discounted

Simmons.

22, 1877,

at

6%.
48 /g,

August
13. Six

days,$8, $992.

[S1500.]
months
or

Guthrie,
received.

Pittsburgh,Pa., July 10, 1877. after date, I promise to pa}'' Alex. M. bearer, fifteen hundred dollars,for value
Orlando
October

Watson.

Discounted

25, 1877,

at 81

6^.
days,$20.25,$1479.75.

January

^^3, 1878,
note

2d.

When
the

the

hears

interest. time of

1. Find

dale

when

due,
the

discount, bank
note:

discount,and
$800.
Six months

proceedsof

business following

Dayton,
after

O., January 5,
to

1877.

date, I promise
the

of Charles

at Stuart,

Dayton

to the order pay National Bank, of Dayton,

O., eight
value received.

hundred

with dollars,

interest Francis

at

6^,

for

Murphy.

BANK

DISCOUNT.

253

Discounted

April 15, 1877, at 8^.

Solution.

"

The The

note

is due
of discount,

July
is 84 for 6
The

1877. ^/g
,

time
to

from

April
The

15

July 8,
of $800

days.
mo.

amount

is $824.40. da.,at ^o]^, of $824.40 for The

bank

discount

is $15.39. days, at '^c/^, $809.01. are j^roceeds

84

Rule.

"

1. Find the bank


In has the
29

the amount discount

of
and

the note

2. Find
Eem.

for the giventime. proceeds of this amount.


that in

"

February
Find

followingexamples, remember days.


when

leap years

the and

date

due, time

of

discount,bank

count, dis-

proceedsof
of

2. A

note

$150, dated
at

months, with interest 9, 1875, at S%.


November
3. A 1 year, note

May 20, 1875, payable in 6 6%, and discounted September days,$2.58,$152.

^^/23 1875, 75
,

of

$300,

dated
at

with

interest

August 5, 1876, payable in S%, and discounted April 16,

1877, at 6%.

August
4. A note

1877, 114 ^g,


dated
at

days, $6.16,$318.04.
4, 1877, due

of

$450,

March
and

1, 1878, with interest 1877, at 10%.

6%,

discounted

January August 13,

January

^^4, 1878, 144

days,$18.90,$453.60.

254

KAY'S

NEW

Pll ACTIO

AL

ARITHMETIC.

5. A

note

of

$650,
at

1878, with
at

interest

May 16, 1876, due Sept.1, 9%, and discounted April 25, 1878,
132

dated

6%. Sept.Y4,1878,
6. A note

days,$17.26.

$767.29.

of

$840,
at

dated

in 6

months, with
20, 1875,

interest

September 1, 1875, payable discounted at 10^, and cember De-

8%.

March 7. A
note

Y4, 1876, 75
at

days, $14.71,$867.99. May 1,

of

$1400, dated July 19, 1875, due

1876, with
at

interest

6^,

and

discounted

Jan. 17, 1876,

10%. May
8. A
note

y4, 1876, 108


8^,
and

days, $44, $1422.50.


1876, due
1, July 26, 1877,
Jan.

of $2400, dated
at

Oct. 16,

1878, with
at

interest

discounted

10%.
January
9.

'^/ji^, 1878, 162

days, $118.51,$2515.09.

or

Macon, Ala., October, 15, 1877. One Moore, year after date,I promise to pay Adam hundred dollars,with interest at order,thirty-five

[$3500.]

6%,

for value

received.

Joseph

Stephens.

Discounted October
10.

May 15, 1878, at 9^.


1878, 156 ^^j^g,

days, $144.76,$3566.99.

or

Frankfort, Ky., 3Iay 10, 1875. One year after date, 1 promise to pay Henry Warren, with interest at S%. for order, six thousand dollars,

[$6000.]

value

received.

Amos

E. Burton.

Discounted

November

21, 1875, at

10^.

May

^^L,1876, 174

days, $313.39,$6170.61.

BANK

DISCOUNT.

255

CASE

II.

197e
to

Given the

find

per face of the

the

cent, the time, and


note.

the

proceeds,

1. For
note

what
a

sum

due

90

to

bank, that,when
S177.21?

days hence, must discounted at 6%,


OPERATION.

give
pra

the

ceeds

will be

3)93
Solution."
of $1 for 93

1.0 00 1

The

bank

discount

3 )"0

-0155 .9845

and

the

is $0.0155, days,at 6^^, $0.0155 proceeds $1


"
"

.0155

$0.9845.
ceeds

Then, $177.21 is the


--

pro-

.0845)177.21(180
9845
787CO

of 177.21

.9845

$180.

78760

Rule."

1. Find
cent.

the

proceedsof %l for givenproceeds.


note

the

given time

at

the

givenper 2. By this
2. The

divide the

proceeds of

discounted
what
was

at

bank
face

for GO of the

days, at
note? 3. For discounted will be

6^,
what
at

were

$197.90:

the

$200.
sum
a

must for

note

be made,

so

that

when

bank,
be bank

90

days, at 6%,
a

the

proceeds
$400.
counted dis-

$393.80?
must

4. What
at

the for

face of 5

note, that
at

when

months^
are

8^,

the

proceeds
$225.
time 4

may

be $217.35?

5. The

proceeds of
and the

note at

$352.62, the
what is the

months,

discount

(3^

face?

S360.

256

KAY'S

NEW

PRACTICAL

ARITHMETIC.

6. 1 wish

to

borrow the

$400

from

bank

for 30

days:

what
at

must

be

face of my

note, that,when

discounted

6%,
7. I

receive this amount? I may bank wish obtain from to a


sum

$402.21.

$500 for 60
at

days:

for what

must

give my
for 6
a

note,

8^

discount?

$507.10.
8. I wish
to
a use

$1500
at
note

months;
of

if I

can

obtain
what

money
sum

from
must

bank,

discount
to

10%,

for

give my
dated

realize this amount?

$1580.33.

February 19, 1876, payable January discounted ber Octowas 1, 1877, and bearing8% interest, $1055.02: what 12, 1876, at 6fc; the proceedswere
9. A
note
was

the

face of the

note?

$1000.

TRUE

DISCOUNT.

198.
money,
2. The

1. The

present
will amount
discount

worth

of

note

is

sum

of
at
a

which, being on
true

interest
to

for the the


same

given time
as

given i)er cent,


present
Rem. True Rem. Discount.

the

note.

is the
amount

difference of the
note.

between

the

worth

and

the

1.
"

and Notes, d^^bts,

running
discount

accounts

are

discounted

by
True

Discount.
2.
"

Banks

sometimes

by the

method

of

the face of the note, the time, and cent, to find the present worth and discount.

199.

Given

the per

1. Find
note

the present worth

and
mo.

at 6^, discount,

of

of

$430.50, due

in 2 yr. 5

18

da.

TRUE

DISCOUNT.

257

Solution.
2

"

The
18

amount

of is 6^o, of and

$1

for

yr. 5

mo.

da., at

$1,148. $430.50

Then,

the

present worth
=

is 430.50 --1.148

$375;
$375
=

the

count dis-

is $430.50

"

$55.50.

$5 5.5 0

2. Find note

of

present worth and discount,at 8^, of $500, due in 3 yr., and bearing interest

the

at

6%.

Solution.

"

The
at

amount

of

$500 for 3 yr.,


The at
amount

is $590. 6^^,

$1 for 3 yr., is $1.24. Then, the 8^^,


of

of

present worth
^

$590

is 590

1.24

$475.81; and
is $590
"

the
=

discount

$475.81

$114.19.

590
4 7 5.8 1 1 1 4.1 9
Prac. 17.

80

258

HAY'S

NEW

PKACTICAL

ARITHMETIC.

Kule.

"

1. Find
cent.

the amount

0/ SI for of
note

the

given time

at

the

givenper 2. By this

divide

the

amount

the

note;

this is the

present worth.
3. From

the

amount

of

the

subtract

the

present

worth;
Hem.
amount

this is the discount.

"

When
is the
same

the
as

note

does

not

bear
note.

interest,of

course

the

the

face of the

3. Find
note

the

present

worth

and

discount,at

G^,

of

of

$224, due
the

4. Find note

in 2 yr. present worth

8200, $24.
and
at 6^, discount, at

of

of

$300, due
the

in 2 yr., and

bearing interest

8^.
of
a

$310.71, $37.29.
5. Find debt of

present

worth
10

and
mo.

at 6^, discount,

$675, due
the
an

in 5 yr.

$500, $175.
discount for 5 mo.,
at

6. Find

present worth
of

and

10%,
7. A

of

account

of $368.75.
dated

note

$800,

$354, $14.75. September 10, 1876, due

January 1, 1878, and


of for the what
was

posed disbearing interest at 6%, was present worth, at 10%, July 19, 1877:
at

the

present worth

this

date

and

the

count? dis-

$825.65, $37.15.

goods amounting to is worth $775, on 4 months' credit: if money 10% to $750. him, what might he pay for the goods in cash? 8 9. Bought a bill of goods, amounting to $260, on will is worth "months' credit: if money 6%, what sum
a

8. A

merchant

bought

bill of

pay

the

debt

in cash?

$250.
a

10. A

merchant he
can

buys
have 4 is worth cash?

bill of

goods amounting

to

$2480:
cash:

months'

credit,or
to

5%
what

off,for
will he

if money

only 10%

him,

gain by paying

$45.47.

TRUE

DISCOUNT,

259

11.

Find

the one-third third in

present
to

worth,

at

5^,
yr.,

of

debt in
2

of

S956.34,
and
12.
one

be

paid

in

one-third

yr.,

yr. three the


true
at

8870.60.

Omitting
between of

the

days
discount

of

grace, and

what the bank

is

the count dis-

difference

$535,
man was

for

yr.,

7%?
$1122
for
a

S2.45.

13.

offered in much
10

house,
interest.

in

cash,
He is chose worth

or

$1221,
the

payable
latter
to
:

mo.,

without he

how

did

lose,

if

money

12%
14. for end

him? offers
to

$12.
sell in
3

man

his

farm

for

$8000

in

cash,
at

or

$10296,
of

payable
2,
be and w^orth cash?

three

equal
without what will

installments interest: be the

the

1,
to

years,

considering
gain
to

money

10^,

the

buyer
15. A

by

paying
note

$620.
dated
at

of

$2000,
interest of

July

4,
was

1876,
cancelled

due

May
October

1,

1878, 25,
was

and

bearing by
payment

8^,
present

1877,
the

the
at

worth and the

at

6^
discount?

what

present

worth,

this

date,

$2223.08,

$68.92.

Xe

HANG

EI.

200.
of money.
Rem.

1. A
one

draft,

or

bill to

of

exchange,
a

is

written
amount

order, from

person

another,for

certain

drawee;
Rem.
called
a

is called the the bill is drawn whom person upon is called the payee. the person in whose favor it is drawn it is the draft is to be paid upon 2. When presentation,
1.
"

The

"

sightdraft;when
a

it is to be

paid at

the

end

of

certain

time, it is called
2.
means

time

draft.

Exchange
of
a

is the

method bill of

or draft, are

3. There and

two

sorts

of

making a payment by exchange. exchange: domestic or Inland, place between


localities

of

foreign.

4. Domestic

exchange
country.
followingis is commonly
a

takes

in the
Rem.

same

"

The which

common

form
a

of

an

inland

bill of

change, ex-

termed

rfraftor

check:

$500.
At

sight, pay
value

for dollars,

Cincinnati, O., May 1, 1877. to John Jones, or order, five hundred of received,and charge to account
Silas Thompson.

To

Charles

Smith

"

Co., Xew

York.

(260)

EXCHANGE.

261

5. Foreign different

exchange takes place between

localities in

countries.

Rem.

"

The

following is

common

form

of

foreign bill

of

exchange:
"500. At of the
or

Cincinnati, O., May

1,

1877. third roll, Carvalue

sight of
same

this

first of and date

tenor

order, five

hundred

exchange (second and unpaid),pay to Amos for pounds sterling,


of Stanley

received,and
To

charge to
Smith "

account

Bingham.

James

Co., London.

in dupliforeignbill of exchange is usually drawn cate called a set of exchange; the different or triplicate, termed the firsts copies, respectively second,and third of riage exchange,are then sent by different mails,that miscarWhen is paid, the or one delay may be avoided. A others
are

void.

6. The

acceptance
drawee

of
to

bill of

exchange
due.

is the

ment agree-

b}' the
KEM.-Abill with

pay
the

it when

is

accepted by
across

drawee's

writing the

word

cepted," "acan

his name,

the face of the

bill;the bill

is then

acceptance.

201,

To

find

the

cost

or

face of

domestic

bill of

exchange (Art. 170, Eule).


1. What

is the

cost

of

sight draft
^

on

New

York

for

$1400, S2580,

at

^% \%

premium?
cost

$1407.
of
a

2. What
at

is the

sight

draft

on

Boston, for
$2567.10.

discount?

262

RAY'S

NEW

PRACTICAL

ARITHMETIC.

3. What which
cost

is the

face

of

$375.87, at J%
is the
cost

4. What

of

sight draft premium? sight draft on


a a

on

Wheeling,
$375.40.

Chicago,for
82778.04.
cost

$2785,

at

1%
is

discount? the face of

5. What

sight draft, which


draft
on

$1852.55, at
G. What

\\%
is the in

discount?
cost

$1876.
a

of

New

Orleans

for

$5680, payable

60

premium,
7. What

and

interest

days, exchange being at ^% $5649.08. 6% ?


a

is the cost of in 30

draft

on

IS'ew York

for

$1575,

payable

days,exchange being at ^^% premium, and interest 6%? $1578.13. face of a draft, 8. The payable in 60 days, is $2625; exchange being at \\% premium, and interest 6%, what
is the
cost

of the

draft?

$2636.69.

FOREIGN

EXCHANGE.

202,
money
Rem.
Great
"

Foreign
of the
The

bills

of

country in

exchange are which they are


of the United and Canada.

drawn
to

in

the

be

paid.
with chiefly

foreignexchange

States is

Britain, France, Germany,

ENGLISH

MONEY.

The

unit
4 12 20

English money 1 make farthings


pence
"

of

is the penny,

pound sterling.
marked
''

d.
s.

1 1

shillings
usual coins
=

''

shilling, pound,
=

"

".

Rem.

"

The

are:

gold,sovereign
crown,

", and
s.,

half

ereign; sov-

silver,crown
and

s., half

florin ==2 penny,

penny, sixshilling, penn}^ and

three-penny;

copper,

the

half

farthing.

EXCHANGE.

263

FRENCH

MONEY.

The

unit

of French
10 10

is the franc, marked money 1 decime. make centimes decimes


^'

ft\

1 franc.

Rem.

"

The

usual

coins

francs; silce?-piecesfor
5, 2, and
1 centimes.

gold piecesfor 100, 40, 20, 10, and 5 5, 2, 1, h, and \ francs; bronze piecesfor 10,
are:

GERMAN

MONEY.

The divided
Rem.silver

unit into
The

of 100
usual

German

money

is

the

mark,

which

is

pennies(pfennige).
coins and
are:

"

piecesfor 2, 1, pennies.
Canadian with The standard United
par of

gold piecesfor 20, 10, and 5 marks; ^ marks; nickel piecesfor 10, 5, and 1

money States

is in dollars and currency. is the countries.

cents, corresponding
of the

exchange
of two

comparative value

coins

Rem.
or

"

The
the

commercial par value.

below

foreignexchange may Quotations are always in gold.


value of

be

above

par commercial The par


at

The

value value value

quoted gold.
The

about

quoted varies from $4.83 to $4.90 gold. It is usually is $0,193. of the franc dollar 5 fr. 14f centimes, equal to one
of the

pound

is $4.8665.

Its

par

value

of

the

mark four

is $0,238.

The

cial commer-

quotations, always
to

for

marks,

vary

from

$0.95

$0.98.
To find the
cost
or

face of

foreign bill

of

exchange :

264

RAY'S

NEW

PRACTICAL

ARITHMETIC.

1. What
cost

will

in 'New

sightbill on London, for York, exchange being at $4.87?


a

"500

IOh.,

OPERATION.

10

S.r=:".5
5 0 0.5

Solution. If "1

"

Since

20

s.

"1,
are

10

s.

r.=

".5. 4.8 7

is worth
=z

$4.87, "500.5

worth

$4.87

3~5T35
40040 20020

X 500.5

$2437.44.

$2437.435

large a bill on London $1808.04, exchange being at $4.88?


Solution."
as

2. How

can

be

bought

for

Since
can

"1

is worth be

$4.88,
for

OPERATION.

many

pounds
as

bought

4.8 8

)1

8 0 8.0 4

(3 7 0

$1808.04
in

$4.88
It

is contained is contained Reduce

times 870 the

1464 3440 341o


244

$1808.01.
a

times, with
remainder

remainder.

to

by
"870

20. 10

4.88 times.

by multiplying shillings is contained in the product


The bill will be for

20

4.88) 4

80(
0

10

s.

10s.

488

3. What

will

bill

on

London

for

"890

8s. .cost,exchange

being at $4.86? 4. How large a bill on London $2130.12,exchange being at $4.88?


5. What will
5
a

$4327.34.
can

be

bought
"436

for lOs.

bill

on

Paris

cost to

for 1290

change exfrancs,

being
6. How

fr. 15

centimes

$1?

$250.49.

largea bill on Paris can be.bought for $1657.60, 8553 fr.22. exchange being at 5 fr. 16 centimes?
7. What will
a

bill on

Berlin

cost

for 12680

reichsmarks,
$3074.90.

exchange being $.97 per 4 reichsmarks? Frankfort 8. How can large a bill on $1470, exchange being at .98?

be

bought
6000

for
m.

DEFINITIONS.

203.

1. Insurance
a

Companies
certain of
a

of money, to pay the occurrence on 2. The company.


Eem.
"

amount

sums agree, for specified to the person insured

certain written

event. contract

policy

is the

given by

the

The
are

persons sometimes

insured

are

called

the

policyholders.

The

companies
3. The insurance.
4. Pire

the underwriters. stj^led

premium

is the

sum

paid to
for
a

the

company

for

Insurance

is fire.

indemnity
is

certain

amount

in
5

case

of loss

by

Marine
case

Insurance

in

of

loss'bythe
Insurance

indemnity for a certain dangers of navigation.


an a

amount

6. Life
sum

is
at

agreement
certain time

to

pay the

at the

death, or

in

specified of the life,


a

insured.

FIRE

AND

MARINE

INSURANCE.

204-.
a

The

premium

in fire and of the


amount

marine

insurance

is

certain

percentage

insured

(Art. 170,
(265)

Eule).

260

RAY'S

NEW

PRACTICAL

ARITHMETIC.

Rkm. value.

"

Insurance
The

companies will insurance is commonly


is the
cost

seldom upon

insure

property at its full


the value.

" or ^ of

1. What
at

f of its value, costing$1 ?

the

liouse worth a $3375, insuring and the policy premium being H^

of

Solution. house

"

of the value

of the

premium is U^;^ of $2250, which is $33.75;adding $1, is the sum the cost of the policy,
is $2250. The

$34.75; the

cost

of insurance.
3 4.7 5

2. What
at

is the

cost

f of its value,the S1.50? costing


3. A
store

house worth a $5000, insuring premium being ^^, and the policy
of

$20.25.
at

goods at S14400; " of the value of the store is insured at f^ and ^ the value of the goods at 2% ; the cost of the two policies the total cost of inis $1.25 apiece: what surance was
$209.50.
4. A
a man owns a

is

valued

$12600,

and

the

manufactory valued

at

$21000,

and

worth will it cost. to insure $7200: what dwelling-house the manufactory, at ^ of its valuiB, at IJ^, and the house, at its full value, at f^, the two policies $23G.50. costing$1.25 each? 5. A man's dwelling,valued at $5600, was burned ; it had been insured, in a certain company, 20 years, for f much of its value, at l^% : how did he receive from the than the sum total of the annual premiums? more company

$2940.
6. A
man secures a

for

$3600,

furniture

policyof insurance,on his house, for $1600, and $800; the library

INSURANCE.

267

premium
cost

is

^^,
is

mid

cost

of

policy $1.25:
of its

what

is the

of the

insurance?

$53.75.

7. A

hotel

insured, for |
and

value, at 1J%

; the
:

costs $1.25 policy at what 8. The


sum

the total cost

of insurance

is $151.25

is the hotel valued? of

$15000.
worth the

cost

insuring a
cent

house
cost

$4500, for ^

of
:

its what

value, was
was

$32.75 ; the
per

of

policywas

$1.25

the

of insurance?

^^.

9. A

and

of $1000 on his house, farmer, with an insurance $1500 on his barn, in the Yermont Mutual, pays an
assessment

annual

of $3.50

what

is the

per

cent

of the

premium?
LIFE INSURANCE.
are policies

-J-^%.

205.
kinds
2. A

1. Life

Insurance

of

two

principal
person

(1) lifepolicies, (2) endowment life policyis payable at the


endowment at
In

policies.
death of the

insured.
3. An

policy is payable
if it
occurs

at

specified

time, or
Rem. annual
"

death
life

within

this time.
is upon The

insurance

the

premium
amount,

commonly
the
tables

regular
the company

payment,
when he the

dependent, in
effects

age
of
a

of

dividual in-

his insurance.
at any

show

annual

premium,
at

age, for $1000 of insurance,

1. A

man

the

age

of

40

insures
on

his life for $5000 ;

the

company's

annual

premium

$1000,

for

life

at policy

how

this age, is $31.30; if he dies at the age of 70, much will he have paid the company? money
OPERATION.

Solution.
is

"

Since

the

annual

premium
X
^
=

on

$1000
30

$31.30
5 15

$31.30, on
amount

$5000 it

is $31.30 30

$1^6.50; then,
be

the

paid, in

yr., will

$156.50 X

6.50
8 0

--=$4695.

$r6"5.W

268

RAY'S

NEW

PRACTICAL

ARITHMETIC.

policy life insurance for $10000, payable in 10 in a company premium on $1000, at bis years ; the cost of the annual age, is $105.58 : if he lives to receive the endowment, what will be the cost of the paid-uppolicy, without interest?
out
an

2. Mr.

Harris, aged 35,

takes

endowment

$10553.
3. At the age of

50, the
on

cost

of

life policy, payable


of
on an

is $47.18 annually, payable in policy, end


a

$1000; the
years, much
more

cost

endowment
; at

20

is $60.45 will

$1000
been than

the

of 20 years,

how

have

policy of $8000 life plan?


4. At
amount

by
of

the

endowment

plan
his

paid on by the
the

$2123.20.

the
of

age

44,

man

insures his for


a

life to

$12000
at
man

in favor

of

wife;
life

annual
on

premium
:

this age, dies than

company's is $36.46 policy,


of 5 premiums, will his widow

the

$1000

if the much

after he

the

payment

how receive ? 5. At the

more

paid out,
man

$9812.40
age of

21, a

young the

takes

out

life is

policy for $5000,


$19.89
on

upon
if he

which

annual
age of

premium
75, how
much

$1000
him

lives to the
up

will it cost

to

keep
30,
10

his insurance?

$5370.30.
6. At for of the age of
to
secure an an

endowment

$1000, payable in
$104.58 ; what
at

3 ears, the

costs amount

annual
of the

policy premium
ten

will

be

ments pay-

the

end

of the

time, allowing interest


took
out
a

at

6%

$1390.91.
7. At the
on

age

of

38, a gentleman

policyfor

plan,paying annually $29.15 on $1000. After keeping up his premiums for 15 years, he suffered had much he paid out, how his policyto lapse : money $3882.78. allowing interest at 6% ?
$6000,
the life

DEFINITIONS.

206. coimtry
2. A 3. A
or

1.

tax

is

money of

paid by
government

the
or

citizens for other

of

for the

support

lic pub-

purposes.
tax

is either
tax

direct
one

or

indirect. is levied upon the son per-

direct

is

which

property
tax

of the the

citizens. person is called


a

4. A

upon

poll tax;
some

upon

property, a property tax.


5. An indirect tax

is

one

which,

in

way,

is

levied

upon

the

business of the

of the

citizens.

6. The to

taxes

United and

their

nature

ence States,considered in referof two are classes, (1) purpose, United States Revenue.

State and

Local

Taxes; (2)

STATE

AND

LOCAL

TAXES.

207.

1. The

money

for State

and

local purposes

arises

from chiefly

direct

taxation.

Kem.

"

Some

revenue

accrues

to the State

from

the rent

of school

lands,from

etc. licenses, fines,

(269)

270

KAY'S

NEW

PKACTICAL

ARITHMETIC.

2. For
as

the Estate

purposes
and is

of

taxation, property

is ([classed

Real

Personal

Property.
which is

3. Real

Estate

property
is
etc.

fixed,as
is

lands,
as

houses, etc.
4. Personal

Property

that

which

movable,
of the

furniture,merchandise,
5. The
valuation

is the estimated

worth

erty. prop-

Rem.^
"

The In

vnluation
some

the
must

tax.

is generally the basis upon which to estimate the polls tax upon states, however, the specific
a

first be

subtracted; in Massachusetts,

sixth part of the tax

is

assessed upon the polls, vidual; provided it does not exceed $2 for each indiin Vermont, the basis is what is called the Grand List,which is ascertained each the by dividing valuation

by

100

and

adding $2

for

poll.
valuation is made

6. The
assessor.

by

an

officer

called

an

Rem. contains

"

This the

official makes

out

list called
to be taxed,

an

assessment

roll; it
the valuation

names

of the persons

along with

of their

property.

208.

To

find the

rate

of taxation.

The

rate

of taxation of taxable

is

expressedas
or as

so

each

dollar

property,

many such a

mills per

on

cent

of it.

1.

The

property
; there
are

of 483

certain

town

is

valued

at

$1049905
a

persons

In to poll-tax. subject
tow^n
are

certain

year

the total taxes for

of the each

$13323.36
w^hat

the
rate

poll-taxbeing $1.50
of taxation upon the

person,

is the

property?

TAXES.

271

poll-taxis $1.50x483r=$724.50;then, the


Solution.
"

The

property
$724.50
since
=

tax

is

$13323.36"

^ ^^ ^
'

'"^

n
'

$12598.86.
tax
on

Then,
is is
1 0 4 9 9 0 5 1 '^^ 2 3.3 6 7 2 4.5 0

the

$1049905
on

$12598.86, the
$12598.86
12
--

tax

$1

1049905 1
1

$0,012,

)1
10

2 5 9 8.8 6 4 9 9 0 5

1 (.0

mills,or

^c-

2 099810

2099810

Multiply the tax on each poll by the number of polls ; the product is the poll-tax. the total amount 2. From of tax subtract the poll-tax ;
Rule.
"

1.

the remainder
3. Divide

is the the

property tax.
the valuation ; the

property tax by

quotient

is the rate

of

taxation.

Rem.
amount

"

Of

course,

where

there

is

of the tax

is to be divided

the total poll-tax, specific immediatelyby the valuation.


no

2. A to

tax
a

of

$2500

is

assessed The there

upon

certain

district

build

school-house.

property
are

of the

district is

valued

at

$618000,
the

and

28

persons will be
on

subjectto
the
rate

poll-tax:if
taxation?
3.

is $1, what poll-tax

of

4 mills

$1, or 1%.
$18409.44:
$1.
of

there

Upon a valuation of $2876475 the tax what is the rate? being no poll-tax,
total valuation of

is

6.4 mills 4. The

on

property
what

in

the

State

was Wisconsin, for j.874,

$421285359;

the tax
was

levied upon the mills


rate
on

this the

valuation hundredth

was

$656491.61:
a

to

of

mill?

1.56
.

$1.

ITl

KAY'S

NEW

PRACTICAL

AR1THMETI(\

209.
I. A

To
tax

apportion the
of

tax

among

the upon

tax-])ayer8.
a

$1373.64

is aBsessed
at

the village,

$748500 ; 57 persons pay of $1.25 each; find the rate of taxation,and a poll-tax tax table to $9000. construct a
TAX TABLE.

property of which

is valued

Rate, 1.74 mills

on

$1.

Rem. is

"

In order to facilitate the calculation


to construct

of each

tax, person's

it

customary
in any

such

table.

It is not

necessary to carry

it out

column

farther than

the nearest

mill.

1. James

Turner's
2

property
persons:

is valued

at

$7851, and
OPERATION.

he

pays

for poll-tax

what

is his tax?

Solution.

"

By

the

table, the
on on

tax

on

$7000

is
on

7851

$12.18;

on

$800, $1,392;
the
tax
=r

$50, $0,087; and


$7851

$1, $0,002; then, $0,087 + $1,392 -ftax.

is $12.18 +

$0,002

The

$13.66;this is his property is $1.25 X 2 $2.50. poll-tax


=

Then,
$16.16.

James

Turner's

tax

is

$13.66 + $2.50

at

16.16

UNITED

STATES

REVENUE.

273

Explanation.

"

It is evident

that

the

multiplying$7851 by
2. John

the rate, 1.74,and

operationis equivalent to adding the poll-tax.


at

Brown's

property

is valued

$2576,and

he

for 1 person : what is his tax ? S5.73. poll-tax 3. Henry Adams' property is valued at $9265, and he what for 3 persons: is his tax? $19.87. pays poll-tax pays
4. Amos

Clarke's

property is valued
what

at

$4759,
$8367

and

he

pays
5.

for poll-tax Emily Wood's tax?

1 person:

is his tax?
at
:

$9.53.
what

property

is valued

is her

$14.56.
tax
to

II. The

be raised
at

in

city is $64375
;

able ; its taxrate


a on

property
taxation
to

is valued

$16869758
a

find the
construct

of
tax

thousandths

of

mill,and
Eate

table to $90000.
1. William is his tax? 2. Samuel

3.816
at

mills

$1.

Mill's

property

is valued

$56875:

what

Young's property

is valued

$217.04. at $27543 : what $105.10.

is his tax? 3. Charles is his tax? 4.

O'Neil's property is valued


is valued

at

$83612 $72968 $69547

what

$319.06.
at
:

Adolph Meyer's property


Ganot's

what

is his tax? 5. Louis is his tax?

$278.45.

property

is valued

at

what

$265.39.

UNITED

STATES

REVENUE.

210.
from and

1. The

United taxation from

States
; it

Revenue of

arises

wholly
Revenue

indirect the
revenue

consists
or

Internal

Duties

Customs. the sale of

2. The

Internal
a

Revenue tax

arises from

lic pub-

lands,from
the sale of

upon

certain
etc.

manufactures, from

postage stamps,

274

KAY'S

NEW

rilACTICAL

ARITHMETIC.

3. Duties

or

Customs

are

taxes

on

goods imported

from

foreigncountries.
INTERNAL REVENUE.
are

211.
per
acre:

1. The what

public lands
will the sq. miles?

disposed of
receive
for

at
a

SI. 25
ship town-

government

containing36
2. Letter thereof:

$28800.
or half-ounce, on a

postage
what

is 3 ct. for each is the

tion frac-

postage

letter

weighing
9
ct.
or

IJ

oz.?

3. The fraction

postage
thereof:

on

books
is the

is 1 ct. for each

oz.,

what

postage

on

book

ing weigh11
ct.

1 lb. 5 oz.? 4. The


tax
on

is 70 ct. per gallon : what spirits ])roof barrel of 40 gallons? is the tax on $28.00. a 5. The tax on cigarsper 1000 is $5 : how much does the price of a single this enhance cigar? ^ ct. beer is $1 per barrel of 31 gal. Each 6. The tax on in malt wholesale dealer tax of liquors pays a special of $20 ; in a tax $50, and each retail dealer a special certain 12 wholesale retail 250 dealers, citythere are

dealers,and
bbl. : what

the is the

annual
revenue

manufacture
to
OR

of

beer

is 30000

government?
CUSTOMS.

$35600.

DUTIES

212.
valorem.
2. A

1.

Duties

are

of

two

and kinds, specific

ad

specific

duty

is levied

upon

the

quantity of

the

goods.
allowance is made duties, levying specific (1 ) for waste called draft,(2) for the weight of the box, cask,etc.^containingthe of liquors, goods, called tare. The waste imported in casks or is called leakage)that of liquors age. breakbarrels, imported in bottles, Gross weightis the weight before deductingdry ft and tare; net weight is the weight after deductingdraft and tare. Rem.
"

In

DUTIES.

'

275

3. An

ad

valorem

duty is

levied

upon

the

cost

of the

goods.
Rem.
"

The

is determined

or invoice, goods is shown by the foreign the custom-house. at by appraisement

cost of the

it

4. Duties
Kem.
cost

must

be
is

paid in

coin.
on

"

The

duty

of the article

duty is dollars, pounds, etc.


1. The

which

the

computed on the net weight and in the foreigncountry. The dutiable estimated, is always the nearest exact

the total upon of

value number

weight of a hogshead of imjiorted sugar is 1760 is the duty at lb.; allowing 12^% tare, what S26.95. If ct. per pound? 2. A manufacturer imported from Spain 40 bales of
gross

wool,
ad

of 400 what

lb.
was

each, tare
the

^%

the

cost

was

45

ct. per

pound:
3. A of the

duty, at

ct. per

pound

and

10^
$2052.
cost

valorem? merchant
ware

imported a
was

case

of

glassware ;
francs,the

the
case
:

in France

365.15

and

57.15 francs,and chargeswere the duty at 40^ in U. was franc at 19-\ ct.?
Rem.
"

the commission S. money,

5^ what reckoning the


$34.40.

The

total cost

being $85.58, the dutiable

value

is $86.00.

4. A

book-seller
was

imports a

case

of books
case was

; their cost

in

Germany
marks,
and

1317.04

marks,

and

commission
the mark

6^

what

charges 34.36 the duty at 25^


at

in U. S. money,

being estimated
cases

23.8

ct. ?

$85.25.
5. A merchant

imports six

of

woolen

weight 1500 lb. ; the cost in England was and charges "8 48. 6d., commission 2^^ : what was the in U. S. duty, at 50 ct. per To. and 35% ad valorem $1637.25. estimatingthe pound at $4.8665? money,

cloth,net "500, cases

DEFINITIONS.

213.

1. Ratio

is the

relation

of

two

numbers

pressed ex-

by

their

quotient.
0 to

Thus, the ratio of


Rem. The
makes

2 is 6

"

2^3;
in several

that

is, 6 is

times

2.

"

established custom
it advisable editions of
to

departments
of

of

ematics mathas

given in
2.

former

change the treatment Ray's Arithmetics.


numbers is indicated

ratio

The

ratio

of two

by writing

the

sign ( : )
2
:

between

them.

Thus,
3. ratio.
4.

is read

the

ratio of 2 to 6.

"

The

two

numbers

are

styled the
the

terms

of

the

The

first term the is 3,


a

is called

antecedent,
abstract

and

the

second
5. 6

term
:

consequent.
a

ratio

between
two

two

numbers. of

$6
the

S2 is 3,
same

ratio between To
:

concrete

numbers
:

denomination.
to

find

yd.

reduce ft.,

the 2
A

yd.
ratio

ft;

ft.

2 ft. is 3.

can

not

exist

between

ft. and

because

(276)

KATIO.

277
the

they can Hence,


1st. The
concrete.

not

be

reduced

to

same

denomination.

terms

of
terms

the

ratio

may

be

either

abstract

or

2d.
same

Whe7i

the

are

concrete, both
abstract

must

be

of

the

denominatioji. The ratio is

3d.

always

an

number.

6. Eatios

are

either ratio

simple or
a

compound.

7. A

simple
:

is

singleratio.

Thus, 2 8. A ratios.
2

G is

simple

ratio.

compound
6 ^
MS
a

ratio

consists

of

two

or

more

simple

"

Thus,

!Q

compound
three

ratio.

9. In antecedent of these

Eatio
J

quantitiesare

considered:

(1)

the
two

and (3) the ratio. (2) the co7isequent, being given, the third may be found. Given the

Any

214.
1. What

terms,
ratio

to

find
3?

the

ratio.

is the
-The

of 6 to

OPERATION.

Solution.

ratio of 0 to 3 is 6 divided

by 3,

G 6
--

3 3
=

equal to

2.

2. What

is the
The

ratio of
ratio of
,

"
to

to

^?
OPERATION.

Solution.
or

"

f^is|

divided

by |,

I multipliedby f equal to |.
Rule.
Rem.

I = f I "^ f

"

Divide
When
to the

the antecedent

by

the

consequent.
must

"

the terms
same

are

of different

denominations, they

be reduced

denomination.

278

KAYS

NEW

PKACTICAL

AKITIIMETIC.

What 3. 12 4. 30 5. 35 6. 56 7. 8.

is the
to to

ratio of 4,
6,

3? 5? 7? 8? 10? 21? 18? 20? 25? 15?

If

to to

5 7

I-

5 to 7 to
to to

9. 12 10. 15 11. 15 12. 25

to to

^. J. |. f f

2.

If ^"

l|
ratio of 3. 9 12 17

What

is the
to

23. S18 24. 54 25. 96 26. 221

$6?
to
to

days
men

days?
men?

6.

8.
13.

bu.

to

bu.?
3

27. 1 fl. 9 in. to 28. 5

in.? in.? the

7.
3.

yd.

1 a. to the

5 fl. 4 ratio and

215.

Given

consequent, to

find the

antecedent. 1. 7 is the
Solution.

ratio of what
The

number

to

4?
OPERATION.

"

number

is 4

multiplied by 7,

4 X

"

=^28

equal to
Rule.

28.

"

Multiply the consequent by


ratio of what number number number number number

the ratio.

is the

to to
to

13? 27 ? 52?
24 ?

52. 15. 28. 63. 7^

|-is the ratio of what -j^is the ratio of what 2|-is the ratio of what
45^ is the

to
to

ratio of what

If?

EATIO.

279

7. 3 is the
8.

ratio of what ratio of what

to

75
4
to

ct. ?

$2.25.
oz.

is the

to

lb. 8

3 lb. 15

oz.

9. 2.6

is the

ratio of what

$4.

$10.40.
find the

216.

Given

the

ratio

and

the

antecedent,to

consequent.
1. 5 is the ratio of 45
The number
is

to

what

number?
OPERATION.

Solution.

"

45

divided

by 5,

45 -=-5

equal to

9.

Rule.

"

Divide

the antecedent

by
what

the ratio.

2. 3.
4.

is the is the is the

ratio of 56 ratio of 42

to

number? number? number ? ?

14. 60. 8i

to to

what what

5.

2f 7|

ratio of

23f

is the ratio of $27.20 to what

$3.60.

217.
1

To

find the

the

value of the

of

compound
ratio

ratio.
6
:
"

Find

vahie

2)
3 f

compound

OPERATION.

product of the antecedents 6 and 9 is 54, the product of the consequents 2 and 3 is 6; then, the value of the compound ratio is 54 divided by 6, equal to 9.
Rule.

Solution."

The

6X^ 2X3=
5 4-^6=

5 4 6 9

"

Divide

the

'product

of

the antecedents

by

the

uct prod-

of
Rem.

the

consequents.
antecedents the the

Multiplyingthe evidentlyreduces together,


"

togetherand compound ratio to


ratio
6
g
" "

the
a

consequents

simple
.

one;

thus, in the above


to

example
54
:

compound

2 ^
o

"is equivalent

the

simple ratio

6.

280

RAY'S

NEW

PRACTICAL

ARITHMETIC.

Find 2. Of

the the

value

compound compound compound


compound
1

ratio

.'^

3,

3. Of

the

ratio

o^f! of [
f
8
it"

8.

4. Of

the

ratio

if[
men
j

f:
"

5. ^n Oi
-

the

.1

*" ratio

2
24

men.

,,

d 6.

6. Of

the

compound
compound

ratio

I ; f^^^^ ^^u^
5
:

7. Of

the

ratio

7:3^
9:5)

2)
10^.

218.
a

The

terms

of

ratio
to

correspondto
the

the terms
quent conse-

of

the fraction,
to

antecedent

numerator, the

the

denominator.

Thus, ill 2
numerator

3 the

ratio is

", in
3

which the

the

antecedent Hence

is the

and have

the the

consequent

denominator.

(Art.

101)

we

following
Principles.

I. A

ratio is

multiplied

1st. 2d. II. A

the antecedent. By multiplying the consequent. By dividing ratio is divided the antecedent. By dividing the consequent. By multiplying ratio is not

Ist. 2d. III. A

changed
same

1st. 2d.

both terms by the By midtiplying both terms by the same By dividing

number.

number.

KATIO.

281

219.

To

reduce

ratio

to

its lowest

terms.

1. Eeduce

16

24

to

its lowest

terms.
OPERATION.

Solution." ing both


terms

The

G.
:

C. D. of 24

16

and

24

is 2

8; divid:

8)16:24
2~:
3

of 16

by 8, it

becomes

(Art.

218, III, 2d).


Divide divisor. both
terms

Rule.
common

"

of

the

ratio

by

their greatest

25 30 51 133 125 279

to
to

its lowest its lowest its lowest its lowest its lowest its lowest

terms. terms. terms. terms. terms. terms.

5. 3. 3. 7.

1
2

to to to to

3:5. 7
:

9.

220.

To

clear

ratio

of

fractions.

1. Clear

1^

2\ of fractions.
operation.

Solution.
3 is

"

The

L. C. M. both

of

the of

denominators

2 and

U:2^
6

6; multiplying
9
:

terms

K]

2\ by

comes 6, it be-

14

(Art. 218, III, 1st).

~^T~1"4

Rule.

"

Mvltiply

both

terms

of

the

ratio the

ymdtipleof

the denominators

of

by the least fractions.

mon com-

-zz:'. z^*;"-''-O,

DEFINITIONS.

221.
of two Thus,
is

1.

Proportion

is

an

for expression

the

equality

ratios.
2 4 and 3 6

may

form

for proportion,

the ratio of each

\.
2. The

proportionis

indicated

by writing :

between

the ratios.
Thus,
2
:

is read

is to

as

is to

6.

3. A 4. In Thus,

is either simpleor compound. proportion both the ratios are a simple proj^ortion simple.
2
:

is

simple proportion.
or

5. In
are

compound

proportionone

both

the

ratios

compound.
Thus,
o

!4

"

"

." Q

's

compound

proportion.

G.

consists Every proportion first and fourth

of four terms. of
a

7. The

terms

proportion
(282)

are

called the extremes.

PROPORTION.

283

8. The called the

second
means.

and

third

terms

of

proportionare

9. The the other

last term three

is said

to

be

to fourth proportional

taken

in order.

Thus, in the
means

2 proportion

: :

G, the extremes

are

2 and

6; the
3.

are

4 and

3; and

6 is

fourth

to 2, 4, and proportional

10.

When
is two.

three said

numbers
to

form
mean

number other

be

the second proportion, between the proportional


a

Thus, in the proportion 2


between
2 and 8.

8, 4 is

mean

proportional

222.

The

operations of proportion depend

upon

the

following
Principle.
is
"

In

every the

equal to
the

proportionthe product of productof the means.


:

the

tremes ex-

Thus,

in

proportion 2

6, 2 X 6
==

4 )" 4

3;

"i

the
-^nd

proportion 3 !
the
same

4}

^|2 X

X ^ X 8

3 X

X 4 X ^i
Hence

may

be

shown

for any

other

proportion.
be

(36, 4),
one

1st.

the

product of the Kill extremes, the c/uotlent


Tf
the

means

divided

by by

of of

be the other extreme. divided


mean.
one

2d.

If

the

the means,

be product of the extremes the quotient will be the other

223.
fourth.

Given

three

terms

of

to proportion,

find the

1. What

8?

284

RAYS

NEW

PRACTICAL

ARITHMETIC.

G and 4, product of the means of the extremes, is 24; then, 24 divided by 8, one equals8, the other extreme (222, 1st).

Solution.

"

The

operation.

GX4
24--8.

24

'^

'

2.

what

: :

5|, I ?
12]
"

10

operation.

Solution.
4 X of the
means

"

tremes, product of tlie exX 12, divided by 3 X 10, one

The

4 =-8

equals 8,

the

other,

mean

(222, 2d).

Rule.
name

"

Divide the other

the

'product

of

the

terms

of

the

same

by

given term.
operation and
cancel whenever it is practicable

Rem.

"

Indicate

the

(91).

24. 14. 3. 10.


G.

18. 20.

21.

15.

A-

I3^.
9.

PKOrOKTlON.

285

when to the sohition of concrete Proportion, applied problems,has been styled The Rule of Three,'' because three terms are given to find the fourth. The of Proportion was use formerly so extensive that it was

224.

"

often

called

"

The

Golden

Bide.''

The
two

solution

of

problem by proportion consists

of

parts:
The
the proper statement; that is, into
a

1st.

arrangement

of

the

numbers The

2d.

proportion. term. of finding the required operation

Rem.

terms

in a proportion,it is customary, arranging the numbers the number or quired quantityrethough not necessary, to make to the other three; then, the first three a fourth pi'oportional of the proportionalways are given to find the fourth.
"

In

I.

SIMPLE

PROPORTION.

1. If 2

yd.

of cloth

cost

$4,

what

will

yd.

cost?

OPERATION.

Solution.
or quired,

"

Since
term

the

number

re-

6 3
"

what?

fourth

of the

the third tion,is dollars, Since the cost of 6

proporis $4. term

yd, will be greater than the cost of 2 yd., 6 yd. is the second term of the proportion, and 2 yd. the first term. Dividing the product of 6 and 4 by 2 (Art. 223, Rule), the requiredterm
is $12.

^X ^

Rem."

In

this
of

example,the

number

of dollars is in
number

direct ratio

the greaier the yards; that is, greater the number of dollars they will cost.

to the number

of

yards,the

286

KAY'S

NEW

PliACTICAL

ARITHMETIC.

2. If 3

men

can men

days

can

dig a dig it?

cellar in 10

days, in

how

many

OPERATION.

Solution.

"

Since
term

the

number

re-

1 0 2

what?

fourth or quired, tion,is days,the Since


5
men

of the propor-

third term

is 10 da.
a

3 X 5

/l^ 0

/,

will of

less number
men

dig days than


term

the cellar in
3
men,

is the second

of the 3 and

Dividing the productof


term

proportionand 5 n)en the first term. 10 by 5 (Art. the required 223, liule),

is 6 da.

Kem.
to

"

In

this

example, the number

of

days

is in

an

inverse ratio the

the number number

less the

of men, that is,the yreaicr the number of men; of days in which they will dig the cellar.

Rule.

"

1. For
same

the

third

term, write that


as

number

which

is

of

the

denomination second

the mimher

required.
of
the
two

2. For

the

term, tcrite the


the

gueat'er

remainingnumbers, when
than the

fourth
when

term

is to be

greater
is to

third;
the

and

the

less,

the

fourth term

be less than 3. Divide the the first;

the third.

of j^^oduct will be quotient

the
the

second

and

third terms number

by
quired. re-

fourth tenn,or

3. If
men

men

can

will
4. If 3

dig it in yd. cloth


oz.

dig a cellar 6 days?


cost

in

12

days,how
6
3

many
(i

$8, what

cost cost

yd.?
bl.?
cost

816. $18.

5. If 5 bl. flour cost


6. If 3 lb. 12 tea

$30, what
cost

$3.50,what

11 lb. 4 oz.?

$10.50.
7. If 2 lb. 8
oz.

of tea

cost

can $2, w^hat quantity

you
oz.

buy

for $5 ?

6 lb. 4

8. If 4

hats cost

$14, what

cost

10

hats?

$35.

PROPORTION.

287

9. If 3 caps 10. If 11. If 12. If

caps? 4 yd. cloth cost $7, what cost 9 yd. ? 12 yd.? 8 yd. cloth cost $32, what cost cost 8 yd.? 12 yd. cloth cost $48, what
cost cost

69

cents, what

11

$2.53. $15.75. $48. $32.

purchase 8 yd. of cloth,how will $48 buy? 14. If $48 purchase 12 yd. of cloth,how be bought for $32? can
13. If $32 15.' A much
16.
man

many

yards
12.

many

yards
8. how

receives he have

$152
for 4

for 19 months'

months' work?
in

work:

should If time 8
men can

$32.
24

what

17. If 60 many
men

men

will
oz.

perform a piece of work 12 men perform it? perform a piece of work perform it in 2 days?
of pepper co^ 25 ct.,what

days, in 16 days.
da., how
240.

in 8

18. If 15

cost

6 lb.?

$1.60.
19. If
6

gal. of
85

molasses

cost

$2.70, what $42.12, what


will be

cost

26

gal.?
20. If 5 cwt.
cwt.

$11.70.
lb. of sugar
cost

will 35

25 lb. cost?

$253.80.
cloth cost

21. If 11 of 11
22.

yd. of
bu.

$2.50, what
horses

the

cost

yd.?"'
If 90 bu.
6

$1,871

of oats

supply 40
a

da.,how
30

long
da.
men

will 450 23. If


can

supply them?
build wall in 15

men

da., how

many

build
24.

it in 5 da.? bu.
can

18.

If 15
corn

of be
25

corn

pay

for 30 bu.
cost

bu. of potatoes,how

much

had

for 140
"

potatoes?
$22.60, what

70 bu. will be

25. If 3 cwt. the


cost

of 16
a

lb. of sugar cwt. 25 lb.?

$113.
shadow shadow 120 ft.

26. 4

If

ft. 6

3 ft. long, cast a perpendicularstaff, whose what is the height of a steeple in., 180
a

measures

ft.?

27. If

man

perform

journey in

60

9 da.,traveling

288

KAY'S

NEW

PRACTICAL

ARITHMETIC.

lir.each

day,

in
a

how

many

days
60

can

he

perform
each

it

by
45.

12 traveling

28. A

day? merchant, failing, paid


He
owed A

hr.

ct.

on

dollar of
what B did

his debts.
each

and S22()0,

B A

$1800: S1320.
A

receive?

$1080.
B

29. A

owes merchant, having failed,

$800.30;

$250; C $375.10; D $500; F $115; $G12.12,goes to his creditors: how


on

his

property, worth
will this pay 30 ct.

much

the

dollar?
4-ccnt

30. If the what bl.,

loaf

weigh
mo.

oz.

when
a

flour is $8 bl.?
12

will it

weigh

when

flour is $6
:

oz.

31. I borrowed

$250 for G

how

long should
mi.
a

I lend 5
mo.

$300 to compensate the favor?


32. A da.
starts
on a

journey,and
and

travels 27
same

day ;
mi.

7
a

B after, in how

starts

travels the B

road
A?
a

3G

day:
he

many 33. If William's


labors
9

days will
services

overtake worth

21.
mo.,

are

$15"
to

when
for

4|^ he labors 12 hr. a $91.91^. mo., when 34. If 5 lb. of butter cost cost f lb.? $3%. 35. If 6 yd. cloth cost $5f what cost 7f yd.? $6|f cost | bu. ? 3G. If J bu. wheat cost $f what $^. If cloth what ? 37. $|. cost 2 yd. cost $2^, If yd. will $31J much how 38. If $29f buy 59^ yd. of cloth, buy? G2iyd. will be 39. If .85 of a gallonof wine cost $1.36, what the cost of .25 of a gallon? $0.40.
a
,
,

hr.

day, what

ought he day? $|, what

receive

40. If 61.3
cost

lb. of tea

cost

$44.9942, what

will

be

the

of 1.08 lb. ?

$0.79. of cloth cost

41. If

^ f

of

yard yard
has
35

$|, what

will

^V

^^

yard
42.

cost? If of
a

of velvet

yd.?
43. A

wheel

cogs ;

^^. cost $4|, what cost 17f $178.38J. smaller wheel working in

PilOFOKTlON.

289 of the

26 it,

many cogs : in how will the smaller gain 10


44.

revolutions revolutions? of
a

wheel larger 28f.


a

If

grocer, 1

instead

true

use gallon,

measure

will be the true measure what of 100 gill, of these false gallons? 96J gal. be 1142 feet per sec, and of sound 45. If the velocity in a person of pulsations 70 per min., what the number counted is the distance of a cloud, if 20 pulsations are of seeing a flash of lightning the time between and 3 mi. 22G rd. 2 yd. 2\ ft. hearing the thunder? 46. The \v^s by a measuring line, length of a w^all, deficient

by

643

ft. 8

but in.,

the 25

line

was

found

to be 25 ft. 5. 1

in.
was

long,instead of the true length

its sup2)osed feet, length: 654

wliat

of the wall?

ft. 11.17

in.

II.

COMPOUND

PROPORTION.

225.
men earn

1. If 2

men

earn

$20 in 5 da.,what

sum

can

in 10 da.?
OPERATIOIS^.

Solution." ber the

Since

the

num-

requiredor fourth term of proportion is dollars, the


term
can earn

\.i}-2 0
:

what?

third
men

is
a

$20.

Since
ber num-

greater
2
term

of dollars
men

0X|0X2O ?X^
proportionand
term

120

than

men,

is in the second

of the
a

men can

in the first

term; and since in 10 da.


than
in in 5

greater number

of dollars
of the

be earned
da.

da.,10 da.
2 and 5

the

first term.

product of
2. If 6

and 5 proportion Dividing the product of 6, 10, and 20 by is $120. term (Art.223, Rule),the required
is in the second

the

men, 2 ft. 80

in

10

da., build
how
ft.

wall

20

ft.

long, 3
15
men

ft.

high, and
build
a

thick,in
ft.

many

days

could

wall

long,2

high, and

3 ft. thick?

I'rao. 19.

290

RAY'S

NEW

PRACTICAL

ARITHMETIC.

OPERATION.

Solution. rumber
term

"

Since

the

15 2 0 3
2 94
o

of

fourth or required, is the proportion, third 15 in


a

10

what?

days, the
days.
build
a

term
men

is 10
can nun,men

Since wall

less
6

^x^^X^X
^

ber
6 term

of
men

days
is in

than the

^XX ^ P
^

, " l^

second

,^y/^f^y^jj/\

of the

and proportion, since to build than


to build

^
a
a

15

men

in the first term; of

wall

80 Tt. long will take 20 ft. long, 80

greater number
t to

days
ft.

wall

ft. is in since

second

term

of the
2

and proportion take


a

20 ft. in the first term; less number


term
a

build
a

wall
3

build and
1
..

wall

high will ft. high, 2 ft. is


first term; of of and

of of

in the second since to build


a

days than to the proportion


will

3 ft. in the
e

wall

3 ft. thick

greater number
second
term

in the

days than to build the proportion and

wall
2

2 ft. thick,3 ft.is

ft. in the

first term.

Dividing the product of 6, 80, 2, 3, and 10 by the product of 15, 20, is 16 da. 3, -nd 2 (Art. 223, Rule),the requiredterm
Rule.

"

1. For

the third term, \trite that number


as

which

is

0^ the 2.

same

denomination each

the number the numbers

Arrange
ratio
as

pair of

required. forming the


a

pound com-

if

with the third term, they formed

simple

proportion. product of the numbers in the second and tnird terms by the productof the numbers in the first term ; the quotient will be the fourth term number or required.
3. If
a a man

3. Divide

the

travel
same

24

mi.

in 2

da., by walking
be travel 240

hr.
10

day: at the da.,walking 8


4. If many
men

rate, bow
a

far will

in

br.

day?
18 72 rods of

mi.

16

men can

build build

fence in 12

days, bow
96.

rd. in 8 da. ? in 8
mo.,

5. If
men

men

spend
20

$150

bow

mucb

will

15

spend

in

mo.?

8^37.50.

PARTNERSHIP.

291

6. I travel
can

217
in 9

mi.

in

days
11

of

hr. each

how
511

far

I travel

days

of

hr. each? what


Hum

7. If $100
in 9 mo.?

gain $6
lb. be carried
12

in 12

mo.,

will

J mi. $75 gain $3.37f


far will 60 mi

8. If 100 10100 lb. be

carried

20

mi.

for 20

ct.,how

for $60.60? 75 lb. 400

9. To

carry
to

cwt. 10

mi., costs $57.12:


a

what

will it cost 10. If ft. 18

carry in

tons

75 mi.?
wall 40 20 rd.
men

$168.

men,

15

da.,build
what time

long,5
build
a

high, 4
87 rd.

ft.

thick, in
ft.

could

high,and 5 ft. thick? 58|f da. in 6 days, of 10 hr. each, dig a trench 11. If 180 men, 200 yd. long, 3 yd. wide, 2 yd. deep, in how days many 100 men, can working 8 hr. a day, dig a trench 180 yd. 24.3 long,4 yd. wide, and 3 yd. deep?
wall

long,8

PARTNERSHIP.

226.
for the
a

1. A

Partnership

is

an

association Such
an a

of

persons
is

transaction

of business. each

association

called

firm,or

house,and
or

member,
amount

partner.
of money
or

2. The

capital,

stock, is the

property contributed
3. The
assets
are

by
the

the

firm. due
a

amounts

firm, together
it.

with

the

property
liabilities
net

of all kinds of
a

belongingto
are

4. The 5. The

firm

its debts. the


assets

capital is the

between difference

and

liabilities.

engaged in trade; B's,$300; they gained $100: find


Solution" The whole

1. A

and

A's

was $200; capital share. each partner's

capitalis $200 + $300

=r

$500; of this A

292

K/iY'S

NEW

PRACTICAL

ARITHMETIC.

owns

will

hence, A's gain fgg | of the capital; |ggr=|, and B owns be f of $100:= $40, and B's gain will be ^ of $100=: $60.
"

Or, Solution."

The

whole

is capital

$200 -f $300

$500; then,

$500 $500

: :

$200 $300

:: ::

$100 $100

: :

$40, A's share; $00, B's share.

Rule.

"

Take

such

each

jmrtnefsstock
Rule.
"

part of is part of

the

whole

gain
stock.
'

or

loss,as

the whole

Or,
so

As

the whole

stock is to each

is the whole

gain or
is
a

loss to each

partnefsstocky gain or loss. partner's


requiredto
other; as
in divide
a
sum

Rem. into

"This

rule

when applicable ratio to each

parts having

given

Bankruptcy,

General

Average, 6tc.
and
B form
a

with a partnership, capitalof ^800 : A's part is S300 ; B's,$500 ; they gain $232 : what is the share of each ? A's,$87 ; B's,$145. 3. A's stock was $70; B's,$150; C's,$80; they gained
2. A

$120: what

was

each

man's

share

of it?
;

B's,$60 ; C's,$32. 4. A, B, and C traded together:A put in $200; B, $400; C, $600: they gained $427.26: find each man's share. A's,$71.21; B's,$142.42; C's,$213.63. 5. Divide 3 persons, the that $90 among so parts shall be to each other as 1, 3, and 5. $10,$30, and $50. 6. Divide $735.93 in the ratio of 2, 3, 4 men, among 5, and 7. $86.58; $129.87; $216.45; $303.03.
A's, $28
7. A person
6

left

an

estate

of

$22361

to

be

divided

in the ratio of their ages, which are children, 3, 6, 9, 11, 13, and 17 yr. : what are the shares? $1137; $2274; $3411; $4169; $4927; $6443. 8. Divide $692.23 into 3 parts,that shall be to each other as ^, f, and f $127.60; $229.68; $334.95.

among

BANKRUPTCY.

293

BANKRUPTCY.

227.
debts

Bankrupt
due.

is

one

who

has

failed to pay

his

when

Rem.
of
an

divide

bankrupt are usuallyplaced in the hands whose them into cash, and duty it is to convert assignee, the creditors. the net proceeds among
"

The

assets

of

A owes $175; B, $500; C, $600; failing, D, $210; E, $42.50; F, $20; G, $10; his property is 1. A
man,

worth

$934.50:

what

will

be

each

creditor's

share?

A's, $105;

B's,$300;
2. A is what
man

C's,$360; D's, $126;

E's, $25.50;

F's,$12.00; G's, $6.


his property

owes

$234; B, $175; C, $326:


:

worth

$492.45
creditor

what

can

he

pay
67

on

$1 ; and
on

will each

get?

ct.

$1;

3. Mr.

Smith

A, $156.78; B, $117.25;C, $218.42. failed in business, His owing $37000.


the stock much for did

assignee sold
for expenses:

$25000,
he pay

and
on

charged $4650
the dollar?

how

55%.
GENERAL

AVERAGE.

228.

General

Average
of
sea.
a

is the

method
cargo,

of

apportioning

the owners among by casualties at


1.

shipand

losses occasioned

A, B,
A

and

a freighted

ship
C
:

with
tuns

108
;

tuns

of

wine.

owned
cast

48,
45

36, and
overboard

24 how

they
of the

Were

to obliged
must

tuns

much

loss
tuns.

each

sustain?

A, 20; B, 15; C, 10

294

RAY'S

NEW

PRACTICAL

ARITHMETIC.

2. From
at

ship valued
was was

at

SI 0000, with
overboard

cargo

valued
at

$15000,
what loss of

there

thrown the

goods

valued

$1125:
the

generalaverage, and what was valued at $2150? A, whose goods were General average, 4J%; A's loss, $9G.75. %

PARTNERSHIP

WITH

TIME.

229.
at

1. A 5 the

and

built B 3

a men

wall for $82;


7

had
should

men

work

divide

days, and money?


"

days :

how

they

Solution.
or

The

work

of 4

men

5 da.

equalsthe

work

of 4 X

S"

of da.; and the work of 3 men 1 da.; it is then required to divide $82 into two 3X7, or 21 men ratio to each other as 20 to 21; hence, A's parts, having the same $42. $40; B's part is f| of $82 part is If of $82 20
men

7 da.,equals the work

:=r

2. A

put

in

trade what

$50 for 4 mo.;


w^as

B, $60
share?
$200

for 5 mo.;

they gained $24:


Solution."

each

man's

$50 for 4

mo.

equals $50X4
=

for 1 mo.;

and

$60

for 5

into two

gives A
Rule.

equals$60X5 parts having the same $9.60, and I of $24


mo.
=

$300
ratio B

for
as

mo.

Hence, divide $24


300,
or

200
=

to

2 to 3.

This

of $24

$14.40.

each partnefs stock by the time it was Multiply employed; then take such part of the gain or loss as each partner's product is part of the sum of all the products.
"

pastures 23 horses 27 da.; B, 21 horses 39 da.: Avhat will each pay? A, $23.28|;B, $30.71i. for 8 mo. ; C, 4. A put in $300 for 5 mo. ; B, $400 $500 for 3 mo.: they lost $100; find each one's loss. A's, $24.19i|; B's,$51.61^; C's,$24.19^.
a

3. A

and

hire

pasture

for $54

EQUATION
5. 6
cows

OF

PAYMENTS.

295

A, B,
30

and da. ;

hire
5

a cows

pasture
40

for $18.12: A

pastures

B,

what

shall each
men

pa}^?
formed
a

28 da. : C, 8 cows A, S5.40; B, $6; C, S6.72.

da. ;

6. Two

partnershipfor

16

mo.

put

in, at first, $300, and, at the end of 8 mo., $100 more ; B put in, at first, $600, but, at the end of 10 mo., drew out $300; they gained $442.20: find each man's share. A's, $184.80; B's, $257.40.
7. A and B
are

partners : A
sum

put
B

in $800

for 12 end

mo.,

and
7
mo.

B, $500.
to

What him

must

put

in at the

of

entitle

to

half

the

year'sprofits?

$720.

EQUATION 230.
the
mean

OF

PAYMENTS.

Equation
or

of

payments
time of times.

is the

method
two
or

of
more

finding
ments, pay-

average
at

making

due
1. A at

different B

owes

$2,
the

due

in 3 mo.,
'

and be

$4,

due

in 6

mo.

wlvdt

period can
The

both

sums

paid so
equals

that

neither

party

will be
"

loser?
on
=

Solution. the interest


on
==

interest
2

$2 for 3
6 mo.;
on

mo.

operation.

on

$1 for 3 X

the interest

2 4 6

X X

^ 6

==

^ 2 4

$4 for 6
24 mo.;

mo.

equalsthe
on

interest
on

$1 for 6 X $2 + $4
mo.
=

==

then, the interest


interest
must

$6
30 5
mo.

y^O
5

equalsthe
mo.;

$1 for 6
be
on

mo.

-j-24
^-

hence, $6
"

interest 30

Rule.

1.

Multiplyeach
due.

payment

by

the

time

to

elapse

till it becomes
2. Divide
;

the

sum

of

the be

the
When

will quotient
one

productsby the sum the equatedtime.


is due
on

of

the payments

Rem. the
sum

"

of the

payments

the

day
in

from

which

equated time
of the

is reckoned, its
must

product is 0; but,
with

findingthe

payment, this

be added

the others.

2m

KAY'S

NEW

PRACTICAL

ARITHMETIC.

2. A find the 3. A find, the 4. A in 2 time

owes

$2, ^8,
time

due

in 4 of

mo.,

and both and

$6, due
sums.

in 8

mo.;

average B owes
mean

time due of

paying
mo.,

7
due in 8
6
are

mo. mo. mo. :

in 5

$4,

payment.
of

buys $1500
$500

worth

goods ; S250
in 8
mo.:

to

be

paid
mean

mo.,

in 5 mo.,

$750

find

the

of

payment.
owes

6 1

mo.

5. A 8 mo.; time of

$300;

third in 12

due
mo.:

in 6 mo.; what

1 fourth

in

the

remainder

is the

average 9
mo.

payment?
$200
mo.

6. I

buy

worth
; the

of
rest

goods ;
in 10
mo.

1 fifth to be
:

paid

now

2 fifths in 5 time of

what

is the average
6
mo.

paying
In

all?

231.

the Average or finding


sums

of may be

several

due which

at
to

for the ment paydifixirent times, any date time the time.

Mean

tal^en from

reckon

1. A credit: average

merchant

buys goods

as

May 1st,1848, $100; time of payment?


"

June

60 follows,on 15th,$200: what

days
is the

Ju\y 30th.
1, it is
operation.

Solution.
60

Counting
time

from

May
first

days
105

to the

and the that

days to equated time is is,July 30th.

payment, that of the second; then,


90

of the

$100 X $200 X

^"0"

6000

105=^21000

days from

May 1st,

$300

) 27000
^90

2. I

bought goods

on

90

as days credit,

follows:

April
time 6th.

'2d, 1853, $200; June


of

Ist,$300:

what

is the average

payment?
3. A

Aug.

merchant

on

mo.,
on

bought goods as follows: April 6, 1876, $1250; May 17, 1876, on 4 mo., $4280; June
6 mo.,

21, 1876,

$675:

what

is the

average

time

of

payment?

Sept.12, 1876.

AVERAGE.

297

AVERAGE.

232.
average

Average

is the
a

method when

of

priceof
it,and

mixture,

or findingthe mean the ingredients composing

their

are jDrices,

known.

1. I

mix

pounds
ct.
a

of

tea, worth
what is
1

40

ct.

with lb.,
the mixture

worth lb., worth ?

50

lb. :

lb. of

OPERATION.

Solution.

"

4 lb. at 40 ct. per lb.

are

worth

4 6

X X

-40
-50

1.60 3.00

$1.60, and

lb. at
=:^

50
are

ct.

are

worth

6 then, 4 -f-

10

lb.

worth

$3.00; $4.60;hence,

rrr

10

) 4.60
"746

Rule.

"

Divide

the

whole

cost

by

the

whole
or

number
mean

of

will ingredients ; the quotient

be the average

price.

2. Mix
ct.
a

at 8 6 lb. of sugar, at 3 ct. a lb.,with 4 lb., be worth? what will 1 lb. of the mixture lb.,

5 ct. 3. Mix and


40

25

lb. sugar, 25 ct.


:

at

12

ct.

25 lb.,at lb.,

18

ct.,
ct

at lb.,

what

is 1 lb. of the mixture

worth?

19|
4. A mixes 3

gal.:what
5. I have

gal.water, w^ith 12 gal.wine, at 50 ct. a 40 ct. worth? is 1 gal.of the mixture worth 30 sheep: 10 are $3 each ; 12, $4 each ;
:

the

rest,$9 each
a

find the

average mercury A.

value. in the
at

S5.
thermometer

6. On stood A.
3
as

certain

day
from

the
6

follows:

till 10

M.,

03"^

from

10

M.

till 1 P.

M., 70"
was

; from

1 till 3

P.

M.,

till 7 P.
:

M.,
what

73" ; from the

7 P. M.
mean

till 6 A. M.

75" ; from of the next the

55" da}^,

temperature

of

da}^,
62X".

from

sunrise

to

sunrise?

DEFINITIONS.

233.
2. A
3. The 4. The

1. Involution
one or

is the times.

of multiplication

number

into itself

more

power

is the

product obtained by
is the
or

involution.

first power second

number
square,

itself is the
as a

power,

tained product ob-

by taking the
Thus, 2 X 2
5. The
==

number

twice

factor.

4, is the second

power

or

square

of 2.

third

power,

or

cube,
times

is the
as a

product obtained
factor.
of 2.

by taking the
Thus,
Rem.
square power
2

number
2

three

X
The

8 is the third power

or

cube

"

second

power
of

is called the square, because the


two

area

of

is the

product
the

equal

factors

(Art. 68).
of
a

The cube

third is the

is called three

cube, because the solid

contents

product of
6. The

equal factors (Art.70).


are

of a number higher powers the fourth power^ fifth respectively power

denominated
^

sixth power
of

etc.

Thus, 2 X 2 X 2 X 2 X 2 X
2
==

"

16, is the fourth


of

power

2; 2 X
.

2 =6

X 2 4,

32, is the fifth power is the sixth power of 2, etc.

2; 2X2X2X2X2X2

(298)

INVOLUTION.

299

7. The
which the

exponent

is

number is to be

denoting the
raised.
the square the cube

power

to

given

number
3 square,

Thus, in 3 2, read
32=9.

the 2 denotes
3

In 125.

53, reads
7* is read

cube, the

denotes

3; hence, of 5; hence,
etc.

of

6^

z=

1 fourth power j 9^, %

Jifth power,
power.

234.
1. Find
Solution.

To

raise

number
of 75.

to

any

the
75

cube

OPERATION.
"

multipliedby
square is of 75.

75

is

75 75 375 525
5625

5625

5625; this is the

5625

7_5
28125

multiplied by
cube of 75.

75

421875; this is the

39375
421875

Rule.
a

"

Obtain

productin which
as

the number

is taken

as

factor as many of the power.

times

there

are

units in the

exponent

EVOLUTION

DEFINITIONS.

235.
2. A

1. Evolution
or more a

into two
root

is the process of equal factors. number is


one

a resolving

ber num-

of

of

the

two

or

more

equal factors.
3. The
square root

of

number

is

one

of

two

equal

factors.

Thus,

3 is the square

root

of

9; for

X 3.

4. The factors.

cube

root

of

number

is

one

of

three

equal

Tlius,3 is the cube


5. The

root

of

27;

for 27

=r

3 X

3 X

3.

are higher roots of a number the fourth root, respectively fifth root,etc.

denominated

Thus,
fifth root

3 is the fourth

root
=

of
3

81;
X
3

for 81
3

3 X

3 X

^ X

3.

3 is the

of

243;

for 243

X 3.

6. The

radical

sign
is

|/
number

placed
extracted.

before

number

shows

that

its root
index

is to be
a

7. The

placedabove
root.

the

radical

sign to

show

the

number

of the

(300)

EVOLUTION.

301
omit

Rem. square

"

It

is

customary, however,

to

2, the

index

of

the

root.

Thus, VH^

is read the cube

the

square of

root

of

25; hence, l/^=r5.

^"^
read

is read the fourth

root

27; hence, f~2T"Z.

^T6

is

root

of

16; hence, Vl6:=2.

8. A

perfect

power

is

one

whose

root

can

be

tained ob-

exactly.
Thus,
16 and 25

and
are

Jg

if are perfectsquares; fourth powers. perfect


and cubes

27 and

^V are

cubes; perfect

9. The

exhibited

squares in the

of the first ten

numbers

are

following

TABLE.

Rem.

"

The

numbers

in

the

first line

are

the

correspondingnumbers
in the third line.

in the second

and line,

square roots of the the cube roots of those

10. obtained

An

imperfect

power

is

one

whose

root

can

be

only approximately.

Thus, ]/Tz=: 1.41421 +.

302

RAY'S

NEW

PRACTICAL

ARITHMETIC.

SQUARE

ROOT.

236.
root.

To

find

the

number

of

in figures

the

square

1. The 100 is 10

square

root

of

is

1, and

the

numbers
1 and

(Art. 235, 9, Table);between and consistingof one or two figures,


are

square root 1 and 100 are

of all

between

10

all numbers

of consisting

one

figure ;

fore, there-

When

number

consists

of

one

or

two

its square figures^

root consists

of

one

figure.
root

2. The

square is

of

100

is

10, and
and

the square
are

root
bers num-

of

10000

100;

between three
or

100

10000

all

of consisting and 100


are

all numbers

and between 10 figures, of two figures fore, consisting ; there-

four

When
square
3. In

number consists

consists

of three

or

its four figures^

root

of

tico

figures.
be

like

manner

it may consists

shown
six

that,
its square figures,

When
root

number

of five or

consists

of
;

three

figures.

And 1st.

so

on

therefore, pointedoff into periodsof tivo figures the number as of periodswill be the same
be

If

number

each,the number in the square root. of figures 2d. The square of the units will be found in the first the square of the tens in the second period, the period, etc. square of the hundreds in the third period,

SQUAKE
To

ROOT.

303 into

237.

point

off

number

j^eriodsof

two

each. figures 1. Point


2. Point 3. Point

off 368425. off 6.843256. off 83751.42963.

368425. 6.843256 83751.429630

Rule.

"

Place

every second
Rem.
1.
"

order

point over from units


period on

the order
to the

and then over units, leftand to the right.

integral part of the number will often contain a singlefigure. Rem. 2. the first period on When the rightof the decimal part contains but a singlefigure, to complete a ciphermust be annexed the period.
but
"

The

first

the

left of the

4. Point

off 864326

; 4.758462

; 7584.3769. ; .046827 ; .0625 ; .625.

5. Point

off 97285.46138

; 75300

238.

To

extract

the

square
root

root

of

number.

1. Extract

the

square

of 256.
OPERATION.

Solution. each figures

"

Point

off 256

by placing a (Art. 237, Rule).

periodsof two 2 6 and point over

into

2 5 1

6(16

26)156
156

(Art. 235, 9, Table) is 1; its root is 1; place the root 1 on the right and subtract the square 1 from 2; the remainder is 1, to which bring down the next period 56. Double the root 1 and place the result 2 on the left of 156 for a trial divisor. Find how times 2 is contained in 15 (making many the result is allowance for subsequentincrease of the trial divisor); 6; place 6 in the root on the rightof 1 and also on the rightof 2, the trial divisor;then 26 is the complete divisor. Multiply 26 by 6
The

largestsquare

in

and

subtract
256

the
a

product 156

from

156; the remainder


its square root

is 0.

fore, There-

is

perfect square,

and

is 16.

304

HAYS

NEW

PRACTICAL

ARITHMETIC.

GEOMETRICAL

EXPLANATION.

findingthat the sq. root will contain of the given number two places of figures (tens and that the figure and units), form a in tens' place is 1 (ten), square figure(A) 10 in. on each contains side, which (Art. 67) 100 sq. in.; taking this sum
After from the whole number of
156 sq. in. remain, squares, which ber, correspondto the num-

156, left after


above. It is obvious

subtracting
the

that to increase

A, and figure

at the

same

time

preserve

it

equally;and,
10 is doubled Now side of

length and breadth must be increased square, both it will take twice 10, that since each side is 10 in. long,
two

to encompass is,20 in.,

sides of the square A.

For

this reason,

in the numerical

operation.
of the

determine the square

the A.

breadth After

addition to be made
side
as

to

each
will

each increasing
sa.vie

it equally, each of the

requirea
B
contents

small

square
to

(D) of the
the
must

breadth

ures fig156.

and of

C,

complete
D,
are

B, C, and

entire square; be equal to

hence, the
the

superficial

remainder,

Now

their contents

obtained

their length by by multiplying

their breadth.

figurein the units' place that is,the breadth of B and be found by trial,and it will be somewhat less than the must C of times the length of B and C (20) is contained in the renumber mainder than 7 times; let us (156). 20 is contained in 156 more 27 for the whole length of B, C, and try 7: 7 added to 20 makes D, and this,multiplied by 7, gives 189 for their superficial contents; than taken too this being more 156, the breadth (7). was great. and breadth 6 the of for to 20 6 D; adding Next, try length gives and breadth of 26 the 26 for the length B, C, D; multiplying by of contents B, C, and D. (6) gives 156 for the superficial
Then the
" "

Hence,

arranged

256 sq, in. square root of 256 is 16; or, when in the form of a square, each side is 16 inches. the

are

SQUAKE
2. Extract the

KOOT.

305

8qiiare root
off
^

of 758.436.
OPERATION.

Solution.
.

"

Point
of
,"

758.43G
each 8

^ro^o^. ' ^ ""'*


.

"^0U(

A/o-rro

I .O

-4-

into

periods
over

two

ngures
over

by
then and

a placiiiijj point

and
3
4 /

7 to

to

the

left, and

j .i o

"

the

Rule),
the
root

Then
as

right (Art. 237, find the figures of


1.

^ '^^ 5

45)2943
2 7 2 5

in Ex.

The

last

re-

mainder is
an

is5351.

Therefore, 758.43G
square,

5503)21860
16 5 0 9 5 3 5 1
or

imperfect
root

and

its

square Rem.

is 27.53

-f.
more

By bringing down one the operationmight be continued places in the root.


"

to any

of decimal periods ciphers, of number decimal required

3. Extract
Solution. the square
root
"

the
The

square

root

of

||f
625

root

of root square of the denominator is

the

numerator

256

is 16, and the

is 25

(Ex. 1); then,

square

of

||f
the

if.
root
a

4.

Extract
"

square
to

of

Solution.

|
-f
.

reduced

decimal

is .375,

.375, to five decimal


of

is .61237 (Ex. 2); places,

root of square then, the square root

The

is .61237

Rule.

"

1. Point

off the given number


in the

into

periodsof
the

two

each. figures
2, Find

the
root

greatest square
on

periodon first

left ;

place its
down 3.

tract like a quotient in division;subrights the square from, the period, and to the remainder bring the next period for a dividend. Double the root found, and place it on the left of the the

dividend

for

trial divisor.

Find

trial divisor

is contained

in the

the times many dividend,exclusive of the how the root^ and also

in righthand, figure;place the quotient the right on of the trial divisor.

306

RAY'S

NEW

PRACTICAL

ARITHMETIC.

divisor by the last figure of the Multiply the complete root; subtract the product from the dividend,and to the remainder for a new dividend. bring down the next period 4. 5. Double and continue the ivhole root the

found, for
the
same

new

trial

divisor,

in operation

manner

until all the

are periods

down. brought

Rem. may
root

1.
"

When

the luiniher is
to any

an

imperfectsquare,
decimal
root

be

continued

number required

by bringingdown
2.
"

periodsof
the square

Rem. when

To
terms

extract
are

of

operation of decimal placesin the ciphers(Ex. 2). fraction: (1) a common


the square
root

the

both

perfectsquares,
of the denominator reduce

extract

of the

numerator
terms
are

and
not

then

(Ex. 3); (2)


fraction to
4
u

wh(;n

both
and

perfectsquares,
root

the

decimal

extract

the

square

of the decimal

( Ex.

).

Extract

the

square

root

of

239.

To

extract

the

square

root

of

perfectsquare

by factoring.
1. Exti-Mct

llie square

root

of 441.

SQUAKE
Solution.
-441

KOOT.

307

=3X
the
one

3X

7X

7;

hence,

v^ 44l

X 7

21.

Rule.

"

Besolve

number

into
two

its

find the product of


Extract the

of
root

each

prime factors,and equalfactors.

square

of

240,

Given

two

of

the side.

sides

of

triright-angled

to find the third ani^C^e

triangle

bounded
its sides.

by

three

plane figure called lines, straight


a

is

Thus, D E F is

triangle;its sides

are

E, E F, and

F.

2. When

one a

of the

sides is

they

form

right-angled

and right-angle, triangle.


B

to another, perpendicular the triangleis called a

A Thus, in the triangle

G, the side A
is

beingperpendicularto
a

the side B ABC

C, they form
a

at C; hence, right-angle angled triangle.

right-

3. The

side

opposite the
other
two

is right-angle
base

called
the

the

hypotenuse

; the

the sides,

and

pendicular. per-

Thus,
A C the

in

C, A

is the

hypotenuse, B C

the

base, and

perpendicular.

308

KAY'S

NEW

PRACTICAL

ARITHMETIC.

4.

Proposition.
on

"

The

square
sides.

described the

on

the

hypotenuse
the square.^

of

is equal to triangle right-angled the other two

sum

of

described
Draw with A C
a

ABC, triangle, right-angled


side B C 4

the
3

in.,and
B

the

side

in.; then, the side A


a

will be

each side of square on and divide each square into the triangle, smaller squares of 1 in. to the side. Then, 5 in. Describe the square
25

described

on

B will contain
two

square described

inches, and
on

the
A

and

squares will contain

la-j-9

=;=

25

square

inches.

5. From

this

we proposition

deduce

the

following

Rules.

"

Ist. To add

find

the

the base

the square 2d. To

the square root of the sum.

hypotenuse; To the square of and extract of the perpendicular,

find

the

base

or

the

From perpendicular;
the square root of the

the

square

subtract of the hypotenuse givenside,and extract the square 1. The

-of the other difference.

of a right-angled triangle perpendicular 30 and is the hj'potenuse? 40: what 50. are is 100, 2. The hypotenuseof a right-angled triangle is the perpendicular? and the base 60: what 80. 3. A castle 45 yd. high is surrounded by a ditch 60 side yd. wide : what length of rope will reach from the outof the ditch to the top of the castle? 75 yd. 4. A ladder 60 ft. long reaches a window 37 ft. from the ground on side of the street, one ing and, without movwill reach one it at the foot, 23 ft. high on the other side: find the width

base

and

of the street.

102.64+

ft.

CUBE

HOOT.

309

5. A island from the

tree

140 ft. in of

ft.

high
tree

is

in
a

the line

center

of

circular

100 the

diameter;
the
to

600

ft.

long reaches
:

top

the

further
on

shore each

what

is

breadth
same room

of

the

stream, the land


ft. long,16 from
one

side 533.43

being
-f-ft.

of the
6. A

level ? is 20 ft.

wide,

and

12

ft.

high :
to

what

is the

distance

of

the lower

corners

the

opposite upper
Given the

corner?

28.28

-f

ft.

241.

area

of

square

to

find

its side

(Art. 67).
Biile.
"

Extract

the square

root

of

the

area.

1. The

area

of
one

square side?

field is 6241

sq. rd.

what
79

is

the

length of

rd.

2. The

surface

of

square

table
one

contains

8 sq. ft. 4
2

sq. in.: what 3. The side of 4. A is the


9
a area

is the of
a

length of
equal area?
measures a

side? sq.

ft. 10 in. is
64

circle is 4096

yd. :
each

what

the

square

of field

yd.

square

rd.

on

side

what

lengthof
as

the side of square

square

neld

which

contains
12

times

many is the

rods?
one

rd.
taining con-

5. What

length of

side of

square

lot

1 acre?

208.71+
CUBE

ft.

ROOT.

242.
1. The is 10

To

find the
root

number of
1 is

of

in figures the
1
or

the

cube

root.

cube

1, and
between

cube and

root

of 1000
are

(Art. 235, 9, Table); numbers consistingof one,


1

1000

all

two,

three

between

and

10

are

all

numbers

and figures, of one consisting

figure ; therefore,

310

RAY'S

NEW

PRACTICAL

ARITHMETIC.

When
cube
root

number consists

consists

of

one,

two,

or

three

its figures,

of

one

figure.
1000 is

2. The

cube
is

root

of

10, and
and six

the

cube
are

root

of

1000000

100; between
are

1000

1000000

all

bers num-

of four,five, or consisting 10 and 100 all numbers

and figures, of two consisting

between

figures;

therefore,
When
cube
root
a

number

consists

or of four, five,

six

its figures,

consists like
a manner

of

two

figures.
be shown
seven,

3. In

it may consists

that.
nine

When
its cube And 1st.

number

of

root
so on

consists
;

of

three

or eight, figures.

figures^

therefore,
offinto periods of three figures pointed will be the same the number as of periods
be

If a

number

each,the number in the cube root. of figures 2d. The cube of the units will he found in the first period, the cube of the tens in the second period,the cube of the
hundreds in the third To
etc. period,

243.

point

off

number

into

periods of

three

each. figures
1. Point 2. Point 3. Point Rule.

off 876453921. off 7.356849227. off 37683.5624. Place

87^453921.

7.356849227
37683.562400 the order the and units,
to the

"

every third order


Rem. number Rem. contains the
1.
"

point over from units to


a

then

over

leftand

right.

The

first

will often 2.
"

When
one or

but

integral part of the contain but one or two figures. the first period on the rightof the decimal part two be annexed must to complete figures, ciphers

period on

the

left of the

period.
off^ 138975462;

4. Point

3.561325482;

684536.256403.

CUBE

KOOT.

311

5. Point

off 2756.56843
extract

98451.3276; .856375;
root

.0064.

244.

To

the

cube
root

of

number.

1. Extract
Solution.
13824 into
"

the
Point

cube
off

of 13824.

periods of three figures each by 4 placing a point over unci 3 (Art. 243, Rule). The largestcube in 13 (Art. 235, 9, Table) is 8; is 2; place the its root 2 on the right,and root
subtract the cube 8 from is

OPERATION.

13824(24 2X2X300=:120()
2X4X 4X4
30=
"=

i824 240 10 1456 5824

13; the
to

remainder

5,
the

which

bring
the

down
2

Square
result is

root

and how

period 824. multiply it by 300;


next

the result,1200, is the


in

trial divisor.

Find

many
root

times
on

1200

is contained 2.

5824;

the

4; place 4
the
sum

in the

the

rightof

Multiply 2 b\^ 4
IG to

and

by 30,and
is the

square

4; add the products240 and

1200;

1456

complete divisor.
5824; the
its cube

Multiply 1456 by
remainder
is 24.

4, and

subtract
13824

the
is

Therefore,

product 5824 from a perfectcube, and

is 0.

root

GEOMETRICAL

EXPLANATION.

findingthat the cube root of the given number will contain two placesof figures (tens and units),and that the figure in the tens' place is a 2, form cube, A, Fig. 1, 20 (2 tens) inches long, 20 in. wide, and 20 in. high; this cube will contain, (Art.
After

70,)
take of

20X20X20
this
sum

8000

cu.

in.;
left,
5824

from

the whole
cu.

number
are

cubes, and

5824

in.

which

in the

correspondto the number numerical operation.

312

RAY'S

NEW

PRACTICAL

ARITHMETIC.

A, and at the sarr.e timo figure each receive breadth, and heightmust preserve it a cube, the length, an equal addition. Then, since each side is 20 in. long,square 20, of square inches in each which 400, for the number gives20 X 20
It is obvious that
to

increase

the

face

of

the

cube; and
400
in

since

an

addition

is to

be

made

to

three of

sides, multiply the


square inches the

by 3, which
This

gives 1200
1200 of the

for the

number

3 sides.

is called the additions

trial

divisor;

because,by

means

the thickness of it,

is determined.

each of By examining Fig. 2 it will be seen that,after increasing 3 oblongsolids, there will be required the three sides equally, C, C, C, of the
same

length as
each the
same

each- of the
as

and sides,

whose

thickness

and

heightare
D, whose
additional

the additional and

and also a cube, thickness; each the of


same as

length,breadth,
thickness.

height are

the

Hence,

the

solid contents

the

first three

the three oblong solids, and solids, rectangular be equalto the remainder together (5824). Now find the It thickness will of the

the small

cube, must

additions.
.

always be
the number

something
of is times

less than

the

trial divisor in the

(1200)
1200

contained

dividend
find in

we (5824). By trial,

is

contained
to

4 find

times the

5824;
of solid tions, additiplying mul-

proceed
the
contents

contents

different of

solids.

The

the

first three found

B, B, B,
the the face

are

by

number the

by

of sq. in. in thickness (Art.

70);

there

are

400

face of each, and sq. in. in


one

sq. in. in the 400 X S 1200


"

Fig. 2.
4

face of the

three;then, multiplying by

ness) (thethickof the

The solid contents gives4800 cu. in. for their contents. three oblong solids, C, C, C, are found by multiplyingthe of sq. in. in the face by the thickness; there are 20 X 4 now in. in
one

number
=

face of

each,

and
4

80

240

sq. in. in

one

sq. face of the

80

multiplyingby (thethickness), gives 960 cu. in. for their contents. Lastly,find the contents of the small cube, D, by breadth, and thickness;this gives multiplyingtogetherits length, 04 cu. in. 4X4X4=-

three; then

CUBE

ROOT.

313

If the solid contents additions the will be added their number

of the

several
in

ADDITIONS.

together,as
sum,

B C

B C

Br=4800 C=:
D=

cu.

in.
"

margin,
be the

5824

cu.

in.

960
64 5 8 2 4

"

of

small

cubes

"

"

remaining after forming the


A.

first cube, in.


are

Sum,

Hence,
of 13824

when

13824

cu.
a

arranged in
root

the form is 24.

of

cube, each

side is 24

in.;that is,the cube

Kem.

"

It is obvious
same

that the way


as

additions

be

arranged in the
2. Extract

in the

margin may readily of the example. operation

in the

the

cube

root

of 413.5147.

OPERATION.

413.5 343

14

700(7.4

7X
7X 4X4

7X300r^
4X
30=
=

14700 840
16 15556

70514

62224
8290700

74X74X300
74X
5X

1642800
11100

SX
5

3 0=

25 1653925 8269625
2 10 7 5

Solution."

Point

off 413.5147 into

of periods
4 and root
as an

three

placinga point over

3, and

then

over

0 to the in Ex.

each by figures right(Art.243, 1. The


last
nv

Rule).
mainder cube

Then

find the

of figures

the

is 21075.

Therefore, 413.5147

is

imperfectcube,

and

its

root

is 7.45 -|-.
or more

By bringingdown one the operation might be continued placesin the root.


Kem.
"

to any

periodsof decimal ciphers requirednumber of decimal

3. Extract

the

cube

root

of

yVg"?-

314

KAY'S

NEW

PRACTICAL

AKITIIMETIC.

Solution. cube
root

"

The

cube

root

of the 13824

numerator

2197

is 13

and

the

of the

denominator

is

24; (Kx. 1); then, the cube

root of

tVbV? i" \lthe cube


to

4. Extract
Solution. three

root

of

^.
is .8.

"

i reduced

decimal

The

cube
cube

root

of .8 to of ^

decimal

places is .928; (Ex. 2); then,

the

root

is .928 +.

Bule.

"

1. Point

off the givennumber


the

into

of periods
the

three

each. figures
2. Find

the
root

cube in greatest
on

periodon first

left;

place its
down 3.

tract in division ; sublike a quotient right, and the cube frojn the period, to the remainder bring the next period for a dividend. it by 300 for a Square the root found, and multiply the Find how

trial divisor. in the

many

times the trial divisor is

tained, con-

in the root. and place the quotient dividend, the preceding 4. Midfiply or figures, of the root by figure, the last and add the by 30, and square the last figure; is the complete divisor. to the trial divisor ; the sum products 5. Multiplythe complete divisor by the last figure of the

root; subtract the product from the dividend, and


remainder
6. Find

to

the

bringdown
a new

the next

periodfor a
as

new

dividend. continue the

trial divisor
same manner

and before, until all the

operationin down. brought


Rem.
1.
"

the

periods are

imperfectcube, the operation be continued of decimal to any requirednumber places in the may root by bringing down periodsof decimal ciphers. (Ex. 2). fraction: (1)when Rem. 2. To extract the cube root of a common both terms are cubes, extract the cube root of the numerator perfect and then of the denominator; (Ex. 3); when both terms not perare fect
When the number is
an
"

cubes, reduce
root

the

fraction

to

decimal

and

extract

the

cubo

of the

decimal.

(Ex. 4).

CUBE

ROOT.

315

Extract

the

cube

root

of

245.
side

Given

the

solid

contents

of

cube

to

find

its

(Art. 70).
"

Rule.

Extract

the

cube

root

of

the

solid

contents.

1. The find
2.

contents

of of
one

cubical side. cubes of


are

cellar

are

1953.125

cu.

ft.: ft. of
1
a

the

length

12.5

3-incli Sixtj^-four what is the box

piled
side?

in

the

form

cube: 3. A
are

length
of the

each
512

ft.

cubical dimensions cubical


are

contains box

half-inch

cubes

what
4

the
4.

inside?
450
cu.

in. ft. : ft. ft. ft. side


as

excavation

contains

yd.

17

cu.

w^hat
5.

its dimensions? the ft. side of and


a

23

Find
216

cube
48

equal

to

mass

288
144

long,
6-

broad,
of
a

ft.

high.
is 1 foot: contain find
3

The

side

cubical vessel

vessel that

the times

of

another

cubical

shall

much.

17.306+

in.

MENSURATION.)!

I. MEASUREMENT

OF

SURFACES.

DEFINITIONS.

246.

1. A

line

has

length without
curved. or straight form meet, the}'^

breadth

or

ness. thick-

2. Lines 3. When
an

are

either lines

^^^^

~^^-

two

angle.
Rem.
"

The

point

at

which

the

lines meet

is called

the

vortex

of the

angle.
either

Angles are rightangles.


4.

acute., obtuse^or

5. When dictular to

two

each

lines are straight other, they form

perpena

right

angle.
6. An acute

angle is angle

less than

right
a

angle.
7. An
obtuse

is

greater

than

rightangle.
8. When where
9. A

lines are straight ever^^distant equally they are parallel. has surface length and breadth
tw^o

without

thickness.

(316)

MENSURATION.

317
curved.

10. Surfaces

arc

either

plane or
or

Thus, the surface of


is curved. g^lobe

table

floor is

plane; that

of

ball

or

11. A bounded
12. A

plane

figure
or more

is

portion

of

plxme

surface

by

one

lines.

polygon

is

plane figurebounded

by straight

lines.
Rem.
The lines are straight a polygon is the called
sum

"

the sides of the

polygon;the

perimeter of
13. A

of all its sides.

triangle

is

bounded

by
If

three

plane figure lines. straight


a

/1\
/
I

\^

Rem.

"

one

side be from

taken the

for the

base, the

let perpendicular the altitude

fall upon of the

the base

oppositeangle

is called

triangle. quadrilateral
three
is
a

14. A

bounded planefigure
of

by

four

lines. straight 15. There the


are

kinds
the

and trapezoid^

the : quadrilaterals gram. parallelo-

pezium^ tra-

quadrilateral v;ith no sides parallel. two / 17. A is a trapezoid quadrilateral / with only two sides parallel. 18. A is a parallelogram quadri/ its opposite sides equal lateral with / and parallel.
trapezium
a

16. A

is

Rem."
fall upon

If

one

side be
the

taken

as

the base, the

let perpendicular

the base from

side opposite

is called the altitude of the

parallelogram.

318

KAYS

NEW

PKACTICAL

ARITHMETIC.

with parallelogram and its angles not rightangles. equal, with 20. A rectangle is a parallelogram all its anglesrightangles. is a with all 21. A rectangle square its sides equal. 22. A polygon of five sides is called a an pentagon ; of six,a hexagon ; of eight, rhombus
a

19. A

is

all its sides

octagon,
23. A

etc.

diagonal

is

line

joining two

anglesnot adjacent. is a. plane figure hounded 24. A circle hy a curved line, point of which every from is equally distant a point within
called the
25. The the curved The
center.

circumference line which


diameter

of

circle is

bounds
of
a

the
is

figure.
a

26.

circle

through
the 27. The

the

center, and
of
to

line passstraight ing both Avays, by terminated,

circumference.
radius center
a

circle

from

the

the

line drawn straight circumference;it is half the


is
a

diameter.

247.

To

find the

area

of

parallelogram (Art.246,

18, 19, 20, 21).


Rule.
"

the Multiply
"

base
of of

by
a
a

the altitude.

Explanation. is

The
the

area
area

lelogram paralangle, rectsame

equal to

having an equalbase

and the

but the area of the rectangle is altitude; equal to its length multipliedby its is equal to breadth; (Art.68); hence, the area of a parallelogram
its base

multiplied by

its altitude.

MENSURATION.

319

1. How 15

many

square

feet

in

floor

17

ft.

long

and

ft. Avide? 2. How


acres

255

sq. ft.

many lengthof which


84

of

land

in the

is 120

rd.,and
in
a

the parallelogram, breadth perpendicular


a

rd. ?
3. How

63

A. of

many

acres

square

each field,
26

side 65

which

is 65 rd. ? many side between


acres

A.

4. How

in

field in the
35

form the

of

sq. rd. bus, rhom-

each distance

measuring
two

rd., and

perpendicular
3

sides

being 16
area

rd. ? A.
a

5. Find square, 6. A how and

the
two

difference others
3

in each

between

sq. rd. floor 30 ft. 80

table

is

ft. 4

50 sq. ydft. square. in. long, and 2 ft. 10 in. wide: 15

many

sq. ft. in its surface?

Solution."

ft. 4

in.r=3J

or

then, the surface of the table is

2 ft. 10 in. ft.; -i/2| ft. Qr }^oy^lJ_ ^ gq,


=
"

or

-i/ft.;

Solution.
of

"

3 ft. 4 in.:r=40

2 in.;

ft. 10 in.
sq.

34

in.;then the
--

face sur-

the

table

is

40X34^=1360

in.; 1360

144=^9

sq. ft.

64 sq.

or in.,

9|

sq. ft.

7. How

many

square

feet in

marble

slab 5 ft. 6 in,

long and
8. How

1 ft. 8 in. wide?

dj-sq.

ft.

many
21

square

yards

in

ceiling25

ft. 9 in,

long, and
9. A
room

ft. 3 in. wide? 60 sq. is 10 ft.

yd. 7
wide

long:

how

sq. ft. 27 sq. in. it be to must 8 ft.

contain

80

sq. ft.?

10. How
a

floor 18
11. How

yards of carpet,1^ yd. wide, will cover ft. long and 15 ft. wide? 20 yd.
many
many

yards
of

take

to

line 3

yd.

flannel, | yd. wide, will it 6 yd. cloth,li^ yd. wide?


of

320

KAY'S

NEW

PRACTICAL

ARITHMETIC.

12.

How
to
cover

mtiny
a

yards
floor
21

of ft. 8

carpet, IJ yd. wide,


in.

will it

lake wide?

long

and

13

ft. G in.

25iyd.
field rectangular
to

13. A

is 15

rd.

long:

what

must

be
rd.

its width

contain

1 A. ?

lOf
a

248.
Rule
"

To

find

the

area

of the

trapezoid (Art.246, 17).


of
the

Multiply half

sum

jxiralldsides by

the altitude.
Explanation.
grarn
area

"

The
same

base of

having

the

is one-half

the

sum

prirallcloaltitude and equal of the parallel


a

sides of the

trapezoid.
sides parallel

1. The 2 ft. 11

of

are trapezoid

2 ft. 2 in. and is its area?

in.; its

altitude

is 11

in.: what

2 sq. ft. 47^ sq. in.

2. A

field is in
is 25 many

the form

of the

trapezoid ;
19

one

of

the

sides parallel is 32 rd.


:

rd.,and
acres

other

rd. ; the

width

how

in the

field?
4 A. 64 sq. rd.

3. How in. wide


on

many
the

upper

side,the
To

yards in a pieceof roof 10 ft. 8 lower side,and 6 ft. 2 in. wide on the length being 12 ft.? 11 sq. yd. 2 sq. ft.
square
area

249.

find the

of

triangle.
are

Ist. When Rule. the

the base

and
base

altitude

given.
take

Multiplythe product.
"

by

the

and altitude^

half

Explanation." is one-half the


same area

The
of

area
a

of

triangle
/
/

/\
I

7
/ \^ _\^/

parallelogram
altitude.

having

the

base and

MENSUKATION.

321

2d. When

the

three

sides

are

given.
of
the
root

Rule.

"

1. From

half

the

sum

the

three

sides

take

each
2.

separately, Midtiply the half-sum


and
^

side

and

three

remainders

gether to-

extract

the

square

of

the

product.
its altitude
90 12

1. The ft.: what 2. One

base

of
area a

is triangle ?

15

ft. and

is its

side of

distance is 18 rd.
:

lot is 44 rd.,and triangular from the angle oppositeto


acres

sq. ft. the perpendicula

this side 76 sq. rd. the base

how

many
area

in the
a

lot?

2 A.

3. What

is the

of

of triangle,
16

which

is 12

ft. 6 in. and

the

altitude
11

ft. 9 in.?

sq.

yd. 5

sq. ft, 99 sq. in. sides


are

4. Find

the

area

of

whose triangle

13, 14,

and

15

ft. sides of
a are triangle

5. The is the

30, 40, and


QQ

50

84 sq. ft. ft.: what sq. ft.

area?

sq.

yd. 6

250.
or.

To

find the

area

of

trapezium (Art. 246, 16)

other

irregular figure.
"

Rule.

1. Divide the
areas

2. Find

into triangles by diagonals. figure and add. them together. of the triangles, of field in the 50 rd. and form the of

the

1. Find of which the


a

the

area

trapezium,
20 rd.

diagonalis
the

to perpendiculars

from diagonal

oppositeangles 30

rd. and 130

7 A.

sq. rd. the

251.
diameter

1. To

find the circumference

of

circle when

is

given.
diameter

Rule.

"

Multiplythe
Prac. 21,

by

3.1416.

322
2.

KAY'S

NEW

PRACTICAL

ARITHMETIC.

Conversely:
circumference

to

find

the

diameter

of

circle when

the

is

given.
circumference by
a

Rule.

"

Divide

the

3.141G.

1. The

diameter

of

circle is 48

fl.: what 150

is the

cumferen cir-

fl. 9.56 in. is the

2. The

circumference

of

circle is 15

fl.: what
4

diameter?
3. The diameter of
a

fl. 9.3 in.

wheel

is 4

fl.:

what 12

is its circumferen fl. 6.8 in. is its in.

4. If

the

girthof
is the

tree

is 12

fl. 5

in.,what

diameter? 5. What circumference of the

3 fl. 11.43

being 7912
252.
is 1. To

mi.?

eter earth,the diam24856+ mi.


the

find

the

area

of

when circle,

radius

given.
Rule.
"

the Multiply
to

square
the

of

the radius

by

3.1416.

2.
area

: Conversely is given.

find

radius

of

(drcle when

the

Rule. root

"

Divide

the

area

by 3.1416,and

extract

the square

of

the

quotient.
the
area

1. Find

of

circle whose 153 sq.

radius

is 21

fl.
sq. in.
:

yd.

8 sq. ft. 64 sq. in.

2. The
are

area

of

its diameter

circle is 6 sq. fl. 98.115 and circumference?


a

what

2 ft. 11
3. How
a

in.; 9
fasten

fl. 1.9+
a

in.
to
no

long
that

rope may

will graze

it take
over

to 1 A.

horse

post
?

so

he

of grass, and

more

7 rd. 2 ft. 3 in.

MEASUREMENT

OF

SOLIDS.

323

4. Two
same

10 circles,
:

and

16
area

ft. in of the

diameter, have

the their

center

what

is the

ring between
122

circumferences? 5. The diameter?


area

of

circle

is 1 square

sq. ft. 75 sq. in. is its foot,what 13.54 in.

II. MEASUREMENT

OF

SOLIDS.

DEFINITIONS.

253.
thickness. 2. A

1. A

solid,
is

or

body,

has

length,breadth, and
bases, parallel faces parallelograms.

prism
are

solid with with

two

which

and ^^olygons,

its

Rem.

"

A
to

cording etc., acprism is triangular, quadrangular, the shape of the base.

3. A

right prism
altitude
one

has of
a

its faces

4. The

rectangles. prism is the perpendicularlet


the of

fall from 5. The


areas

base

upon
surface

other.
a

convex

prism

is the

sum

of the

of its faces.

6. A

parallelopipedon right
six

is

prism
is

with

its bases

lelogram paral-

7. A

parallelopipedon

solid with

faces (Art. rectangular

70).
8. A square
9. A cube

is

solid

with

six

equal
base,

faces.

pyramid
is
a

is

solid with with

one

which

polygon, and

its faces

triangles.

324

RAY'S

NEW

PRACTICAL

ARITHMETIC.

10. A

right pyramid
slant

has of
a

all its faces

equal.
is of

11. The the

height
the

rightpyramid
to

distance

from

vertex

the

middle

each

side of the three

base.
round bodies
are

12. The

the

cylinder^

the cone^ and the sphere. 13. A cylinder is a solid with

two

lel paralwith
a

bases,
curved 14. The the

which

are

and circles,

surface.
axis

oi^ a

is cylinder
two

line

ing join-

centers
convex

of the

bases. of
a

15. The the


area

surface

is cylinder

of its curved
cone

surface. solid with


one a

16. A which surface 17. A curved is

is

base,
curved

with circle,and terminatingin an apex. is


a

sphere

is

solid

with

surface, every
distant the

point of
from
center.
a

which

equally
18. The

point
is its

within

called

volume

of

body

solid

contents.

254.
Rule.

1. To

find

the

convex

surface

of

rightprism.
the altitude,

"

the perimeterof Multiply find the surface

the base

by

2. To

convex

of

cjdinder.
the base

Rule. altitude.

"

Multiply the circumferenceof

by

the

3. To

find

the

entire

surface

of

prism,

or

of

cylinder.

MEASUREMENT

OF

SOLIDS.

325

Rule.

"

To

the

convex

surface add

the

areas

of the

two

bases.

1. Find

the

surface

of

cube,

each

side

being 37

in.

2. Find

the

surface

of of

yd. 3 sq. ft. 6 right prism, with a


is 4 ft.; the 73.85

6 sq.

sq. in. gular trianof ft.

the

base, each side prism is 5 ft.


the surface and
9

whicii

altitude

-f-sq.

3. Find 2 ft in.

of

box

which in.

is 3 ft. 6 in.

long,

wide,
the radius

1 ft. 10

4. Find

surface

of

high.. 42^ sq. ft. its altitude being 5 cylinder,


.

ft. and

the

of the

base, 2
of
a

ft.

87.96

sq. ft.

255.
Rule.

To

find

the volume

prism

or

of

cylinder.
altitude.

"

Multiply the
The rule for

area

of the base
volume of
a

by

the

Rem. is

"

findingthe

rightparallelopipedon

given

in Art.

70.

1. Find which the


4

the

volume
12

of

of right parallelopipedon,
width
3

length is
ft. 4 in.

the ft.,

ft. 3

in.,and
169
cu.

the ft.

SoLTJTiON."

ft. 3 of

in.

then, the volume


cu.

the

ft.; 4 ft. 4 m. ^ or -i/ft.; 3|:or -i^3 is 12 X -V" -^^^ parallelopipedon XV'^


=

ft.

2. How ft. 6 in.

many

cubic and
10

yards

in

room

24

ft.

long,18
1
cu.

wide,

ft. 7 in.

high?
yd. prism triangular
cu.

174

ft.

3. Each its altitude

side of the base is 14 ft.: what

of

is 2

ft.;

is the

volume

of the
cu.

24J
4. Find

prism? ft. nearly.


ft. nearly.

the

volume

of

cylinderwhose
150.8

altitude is 12
cu.

ft. and

the radius of the base 2 ft.

326

RAY'S

NEW

PRACTICAL

ARITHMETIC.

5. How diameter

many

cubic

inches

in

peck
the

measure,

the in. ?

of the bottom

being 9J

in. and

depth 8

537.6 -f cu. in.

256.

1.

To

find

the

convex

surface

of

right

pyramid.
Multiply the perimeterof and take half the product. height^
Rule.
"

the

base

by

the slant

2. To Rule.

find the

convex

surface

of

cone.

of Multiplythe circumference and take half the jnoduct. height^


"

the base

by the

slant

3. To
cjone.

find

the

entire

surface

of

p3'ramid or

of

Rule.

"

To

the

convex

surfaceadd
surface of
a

the

area

of the

base.
a

1. Find

the entire

right pyramid, with


is 5 in.
cone

base, each side of which triangular slant heightof the pyramid is 7 fl. 6
2. What

ft. 4 in. ; the


72.3

-f sq. ft.
the 8 ft.

is the 25

convex

surface the

of

of which

slant
6

height is

ft. and

diameter

of the

base

in.? 3. Find the entire


4

333.8 surface the of


a

cone,

sq. ft. nearly. of which the slant of the

height is
11 in.

ft. 7 in. and

diameter

base

ft.

27.6

+ sq. ft.
a cone.

257.
Rule.
"

To

find

the volume
area

of

pyramid
base

or

of the

Multiply the
of

and

take one-third the

the

of the product.
a

by

altitude^

1. Find

volume base

of

square the

pyramid,
altitude

of
21 175

which
ft.
cu.

each

side of the

is 5 ft. and

ft.

MEASUREMENT

OF

SOLIDS.

327

2. Find 15 ft. and


3.- A

the the

volume radius

of

cone, base

of which
5

the

altitude 392.7
cu.

is ft.

of the

ft.

square

pyramid

is 477

ft.

base

is 720

ft.: how

many

cubic

4. The 37 ft. 8

diameter

of the base its altitude

of

high ; each side of its yards in the pyramid ? 3052800 cu. yd. a conical, glasshouse, is
ft. 9 in.
:

in.,and

79

what

is the
cu.

space

inclosed?

29622 +

ft.

258.

To

find

the

surface

of

sphere.
the diameter

Rule.

"

Multiply the

square

of

by

3.1416.

1. What

is the

surface

of

sphere,of sphere,of
earth's

which 3.14

the

eter diamsq. ft. eter diam-

is 1ft.?
2. What

+
the

is the

surface

of

which

is 4 3. What

ft. 6 in. ? is the


area

63.6 of
a

+ sq. ft.
the in miles

the

surface,on
7912

that supposition diameter?

it be

perfect sphere

196663355.75

+ sq. mi.

259.

To

find the

volume

o-f a

sphere.
diameter

Bule.

"

Multiply
.5236.

the

cube

of

the

by

one-sixth

of 3.1416, or
1. Find the

volume

of

sphere 13
a

ft. in

diameter.

1150.3
2. Find

CU.

ft.

the

volume

of

sphere
is 1

eter. ft. 6 in. in diam8.2


cu.

ft.

nearly.
is its

3. The

volume

of

sphere

cu.

ft.: what 14.9 in.

diameter?

nearly.

328

UAY\S

NEW

PRACTICAL

AKITHMETIC.

III.

APPLICATIONS

OF

MENSURATION.

house-painting, Plastering, paving, paperby the square foot or square hanging,etc.,are meaBured yard. 2. Glazing is measured by the square foot or by the
260.
1. pane. 3. Stone
4.
'

cuttingis measured by the square foot. etc., are measured roofing, Flooring, by the
or

square

yard

by
room

the

square
is 20

of 100

sq. ft.

long,16 fl. 3 in. broad, 10 ft. 1 in. high : how in it,deducting yards of plastering many 6 ft. 3 in. by 4 ft. 2 in. ; a door 7 a fire-place ft. by 4 ft. 2 in.,and two windows, each 6 ft. b}'3. ft. 108 sq. yd. 8 sq. ft. 6 sq. in. 3 in. ? is 20 ft. long,14 ft. 6 in. broad, and 10 2. A room ft. 4 in. high: what will the papering of the walls cost, 4 ft. by at 27 ct. per square deductinga fire-place 3'ard, and two 4 ft. 4 in., windows, each 6 ft. by 3 ft. 2 in.?
1. A ft. 6 in.

SI 9.73.

court, 21 rectangular 5 ft. a foot-path, yd. long and^VlS j". broad, in which the whole wide, runs length:the path paved with flags, and the rest with at 36 ct. per square bricks, at 3'ard, 24 ct. per square ^79.80. yard ? 4. At 10 ct. a square yard, what will it cost to paint both sides of a partition15 ft. 6 in. long,12 ft. 6 in. S4.31. high?
it cost
to

3. What

will

pave

5. A tier:

house the

has

three the

tiers

of

heightof

first tier

in a window^s, seven is 6 ft. 11 in.; of the

second, 5
16

ft. 4 in. ; of the third,4 ft. 3 in. ; each window what will the glazingcost, at is 3 ft. 6 in. wide: square foot?

ct. per

$64.68.

APPLICATIONS

OF

MENSUKATION.

329

6. A

floor
to

is 36

ft. 3 in.

will it cost 7. At roof 40

lay it,at

long,16 ft. $3 a square?


what
on

in. wide

what

$17.94.
the side
cost

S3.50 jDer square,


ft.

will each

be
"

of

long,the

rafters

18

ft. 6 in.

long?
BOARD MEASURE.

$51.80.

261,
lumber 2. A and
3. 1

1.

Board

Measure

is

used

in

measuring all

which

is sawed

into

foot, board
inch thick.
to

measure,

boards, planks,etc. is 1 foot long, 1 foot wide,


of feet in
a

Hence,
the

find

the

number

board,

we

have

following
1. Find

Rule. 2.

"

Multiplythe
many

surfaceof the hoard surfaceby the thickness


feet in
an

the

in square in inches. ft.

feet.

1. How ft. 3

inch

board

16

long
12

and
20

in. wide? many


2

ft.

2. How

feet

in

two-inch

plank

ft. 6 in. 56

long and
3. How
4

ft. 3 in. wide? feet in


3
a

ft.

many in. wide, and 4. How 12 ft. many

15 piece of scantling

ft.

long,
15

in. thick? inch

ft.

feet of
2

boards
make?

will

stick of timber 576 ft.

long and
many

ft. square
an

5. How 1 ft. 3

feet in
at
one

inch

board,
11

12

ft. 6 in. wide


at

long
the fl.

in. end?

wide,

end,

and

in.

other

13i|
MASONS' AND BRICKLAYERS' WORK.

usuallymeasured perch,which is 24f or 24.75 cu. ft. (Art. 70). 2. Bricklaying is commonly measured by
262.
masonry
is

1. Stone

by
the

the

1000

bricks.

330

RAY'S

NEW

PRACTICAL

ARITHMETIC.

1. How 18

perches in a stone many ft. 3 in. high,2 ft. 3 in. thick?


is the cost of
a

wall

97 ft. 5 in.

long,
161.6

2. What ft. 6 in. 3. How ft. 6 in. cubic

stone

wall
a

53 ft. G in.

long,12
$121.59.

high,2
many

ft. thick,at ^2.25

perch?
48 ft. 4 in.

bricks

in

wall

long,16
23925.

high,1

ft. 6 in. thick,allowing20

bricks to the

ft.?

each 8 in. long,4 in. bricks, many for a wall 120 ft. in. thick, will be required
4. How

wide, 2.25

long,8

ft.

high,and
5. Find

1 ft. 6 in. thick?

34560.

the ft.

cost

of

buildinga
per

wall

240

high, 3
in.

thick,at $3.25 long,4 in. wide, and 2

1000, each

long,6 ft. brick being 9


S336.96.

ft.

in. thick.

MEASUREMENT

BY

BUSHELS

OR

GALLONS.

263.
Rule.

1. To

find the

number

of bushels

(Art.61).
divide

"

Find

the

volume

in

cubic

inches , and

by

2150.4. 2. To find the number of

gallons(Art.64).
and iiiches, divide

Rule.

"

Find

the

volume

in cubic

by

231. 1. How and


4

many

bushels

in

bin

15

ft.

long,5
ft.

ft.

wide,
+.
ft.

ft. deep?

241
a

2. How

many
4

wide, and
3. How

gallonsin ft. deep?


bushels in 8 ft. deep? of barrels,
a

trough
a

10

long,5
1496
6

+.

many
and

tub cylindincal

ft. in

diameter
4. How

in the and

many of form

31 J

gal.each, in
the

181.76+. a cistern,
is 4 ft.

of cylinder, ft.?

which

diameter

the

depth

17.9+

bl.

I. ARITHMETICAL

PROGRESSION,

264.
numbers

1. An which

Arithmetical increase
or

Progression
decrease

is
common

series of

by

a an

2. If the

if it

it is called increase, series. a decrease, decreasing 1, 8, 17, 5, 14, 7, 11,

series

difference. series; increasing

Thus,

11, etc.,is an 5, etc., is the series


a

20,
3. The

series. increasing series. decreasing

numbers

forming
are

are

called

terms; terms,
ered considmon com-

the first and the


means.

last terms

the

extremes;

the other

4. In
:

every

arithmetical

five things are series,

(1)

i\\Q first term,

(2)

the last term, of terms, and

difference^ (4) of
the terms.

the number

(3) the (5) the

sum

CASE

I.

265.

To

find the

last term, the of

when

the first term, the


of terms
are

common

and difference,

number
at

given.
first
so

1. I

bought
ct.

10

yd.

muslin,
11

ct.

for

the and

yard, 7
on
:

for the

second,

ct. for the

third,

what

did

the last A^ard cost ?

(331)

332

KAY'S

NEW

PRACTICAL

ARITHMETIC.

Solution. add
cost

"

To
to

fihd the the


cost 4

cost of the

second

yard,
.

operation.

ct.

once

of the

to find the first; to

4 3

36 3 9

of the

third, add
the

ct. twice

the cost of the add


4 ct.

-f 3 6

first;to
three times

find

cost

of

the

jourth,
so

to the cost of the

and first,
times

on;

hence, to find the


ct.,the
cost

cost 9

of the tenth times


4 ct.

yard,add
36

4 ct. nine
3 ct.

to the cost of the


=

but first; of the

are

ct.,and
of the

-f 36 ct.

39

last

yard,or

last term

progression.
of
a

2. The
mon

first term

scries decreasing number of terms

is

39, the
10
:

com-

difference

4, and

the

find the

last term.
OPERATION.

Solution.
9

"

In

this case,

4 must

he

subtracted for the last

4 X
3 9
"

9 36=

==:

3 6 8

times

from

39, which

will

give 3

term.

Rule.

"

1. less
an

Multiplythe
one.

common

difference by the number

of

terms

2.

If

term;

if a

the product to the first add series, increasing subtract the product from the series, decreasing

term. first

3. Find the

the last term


is

of

an

series increasing difference

in which the 149.

first term of terms

2, the
50.
term

common

3, and

number

4. What

is the 54th is

of

series in decreasing
difference 2? series decreasing difference ^?

which
34.

the first term 5. What

140, and 329, and

common

is the 99th is

term

of
common

in which

the first term

243J.

CASE

II.

266.

To
and

find the

the

common

number of
7:
a

of

when difference, terms are given.

the

tremes ex-

1. The number

first term
of terms

series is is the

2, the last 20, and


common

the

what

difference?

AKITHMETICAL

FROGKESSION.

333

Solution. and
terms

"

The 18

difference of the extremes divided

20

operation.

is

18;

by 6,

the

number

of

20

"

18 3

less 1, is 3, the

common

difference.

18^6=

Rule.

"

Divide less

the
one.

difference of

the extremes

by

the

ber num-

of

terms

2. The

extremes
common

are

3 and

300 ; the

number

of terms 33.

10

find the

diiferenco.

3. A

travels

from first

Boston

to

Bangor
of

in

10

da. ; he
eled trav-

goes

5 mi. the each

day, and
same
:

increases number

the distance

day by
50

the

miles; on

the

last

day

he

goes

mi.

find

the

dailyincrease.

5 mi.

CASE

III.

267.
when

To
the

find
extremes

the

sum

of all the

terms

of

the
are

series

and of
6

the number
terms

of terms the

given.
first

1. Find
term

the

sum

of

series whose

is

1, and
"

last term
series is
.

11.

Solution.
In The Since but

The order

inverted
sum

it is is
. .

of the two series is


are

12^

12^
sum

12^ 12^
12, the
of the

12;

12.

the two
sum

the same, many

their

is twice the first series;


sum

their
as

obviously as
2
=

times
sum

of the

tremes, ex-

there

are

terms; hence, the


36.

series is 6

times

12

72 divided

by

Rule.

"

the Multiply
and

sum

of terms;
2. The 24
:

take

half the
are

of the extremes product.


50 ;

by

the

number

extremes
sum

and

the

number

of

terms,
624.

find the

of the

series. does the hammer of


a

3. How

strike

in

strokes many 12 hours?

clock 78.

334

RAY'S

NEW

PRACTICAL

ARITHMETIC.

3 yd. fi-om each apples in a right line, basket: what distance will 3 yd. from a other, the first, a gathersthem singlyand placesthem boy travel who 17 mi. 69 rd. ^ yd. in the basket? 5. A weight,if not resisted by its own body falling by the air,would descend in the first second a space of 4. Place

100

16

ft. 1

in.;
times

the

next

second, 3

times

that

space;

the

third,5
that

space ; the fourth,7 times, etc. : at it fall in 1 minute? rate, through what space would that 57900

ft.

II.

OEOMETRICAL

PROGRESSION.

268.
numbers

1. A

Geometrical
a common

Progression,

is

series of

by increasing
divisor. 3, 24, 9, 12, 27,

or multiplier^ decreasing

by

common

Thus,

1, 48,

81, is 3, is

an a

6,

increasing geometric series. geometric series. decreasing


is divisor,

2. The the ratio.

common

or multiplier

common

called

Thus, in second,
2.

the

first of

the

above

the series,

ratio

is

3; in the

3. The

numbers last terms every

forming
are

the

series

are

the

terms; the

first and
4. In

extremes; the others,means.

(1) the
terms

five thingsare considered : geometricseries, last term ; (3) the number of term ; (2) the first (4) the ratio ; (5) the sum of the terms.

CASE

I.

269,

To

find the the number

last

term, when
are

the

first

term, the

ratio,and

of terms

giv^en.

GEOMETRICAL

PROGRESSION.

335

1. The
2 ; the

first term
:

of

an

is increasing geometric series, fifth term ?

ratio,3
"

what

is the
is

Solution.

The

first term

2; the

2X^X3;
3.

the
term
as

fourth,2X8X3X3;
after the times less one,
as

the third, second, 2X3; and the fifth, 2X3X3X3X

Each

consists of the first term first, is denoted

multiplied by
of the
3 taken

the ratio

many

term; then, the fifth term


times of 3.
as a

consists of 2
4

by multipliedby
a

the number

four

factor;but 3, taken

times
3^
=.

as

factor,is the 4tt\\ power

Hence, the fifthterm first term


2: ratio,
The

is 2 X

162.

2. The

of what

decreasinggeometric
is the
is

series

is

192;

the

fourth

term?
term

Solution. the third

"

first term

192; the second


the fourth

is

192-7-2;

is 192

divided
-f-

by 2X2;
24.

is 192

divided

by

2X2;

that is,192

2=* =

Rule.
one

"

1. Raise

the ratio

to

power

whose

exponent is

less than
2.

If

the

this power; 3. The

of terms. series be increasing, the firstterm by multiply divide the first if decreasing, t^m^by the power.
of
an

the number

first term of

series increasing
13:

is 2 ; the

ratio,
8192.
; the

2;

the

number

terms,
of
a

find the

last term. is 262144

4. The 4 ratio,

first term
; number

decreasingseries
:

of terms, 9 of
an

find the

last term. is

4.

5. The

first term is the

series increasing term?

10;

the

ratio, 3: what

tenth

196830.

CASE

II.

270.
series.
1. Find whose

To

find the

sum

of all the

terms

of

geometric

the

sum

of is

terms

of

the

geometricseries,

first term

4, and

ratio 3.

336

PvAY'S

NEW

PRACTICAL

ARITHMETIC.

Solution. each
term

"

Write
the

the terms
remove

of the

series

as

below;

then

multiply
the

by
:

ratio,and

the

product one

term

toward

right,thus
4

12-f36-flO
12

8 + 108 is

324 324 the

=:=^

sum sum

of the

series.

36

+
once

+ 97
sum

=r^

3.

Since three upper

the

upper the sum, be

line

of the the

and series,
sum;

the

lower
if the

times

their

difference from the


sum

is twice

hence,

line the

subtracted
will
"

lower, and
of the divided

the remainder

divided this

by 2,
the

quotient
have the
of of

be

the

series. the

Performing
quotient product
of

operation,we
sum

972

4=^968 In this series

by 2;
972

is 484, the

series. the

process,

is the

greatest term
the divisor

given

by

the
one.

ratio, 4 is the least term, and

2 is

equal to

the ratio less

Rule.

"

Multiply the greatest term


subtract the least

by
divide

the

ratio; from
remainder

the

product
the

term;

the

by

ratio less 1.
When
series is 0. is

Rem.

"

decreasing,and

the

number

of terms

finite, in-

the

last term

2. The

first term
what

is 10 ;
sum

the of

ratio, 3
the
on

the

number

of

terms, 7:
3. A he what
4.

is the gave his first

series? T^ew month

10930.

father

daughter

Year's
for

day $1;
a

doubled
sum

it the did
1

day
at

of

every

year:

she

receive?

84095.
1 ct. for

I sold the

lb. of 16

gold

the
:

first what

oz. sum

; 4

ct.

for I

second,
the
term ^tc.

ct. for

the

third,etc.

did

get?
5. Find
sum

$55924.05.
of
an

infinite

series, of
that

which
yu
~r

the
toj^

greatest + T0V0'
6. Find

is .3 and

the

ratio,10;

is,of

^.
^

the

sum
sum

of the of the

infinite series
infinite series

i, i, 2V?

^^c.

J.
1.

7. Find

the

i, ^, J, etc.

Вам также может понравиться